You are on page 1of 233

ACTIVE SITE

a
EDUTECH Coordination Compounds

Smart Booklets

09 Coordination Compounds

ACTIVESITE EDUTECH- 9844532971 56


ACTIVE SITE
a
EDUTECH Co-ordination Compounds
Terminologies of co-ordination compounds.
1. Molecular or addition compounds:
When solutions containing two or more simple stable compounds in molecular proportions are allowed to
evaporate, crystals of new substances are obtained. These substances are termed as molecular or addition
compounds. Some common examples are:
CuSO4 + 4NH3→ CuSO4. 4NH3
AgCN + KCN →KCN. AgCN
4 KCN +Fe(CN)2→ Fe(CN)2. 4KCN
The molecular or addition compounds are of two types:
Addition Compounds

Double salts Coordination compounds


(Complexes)
a. Double salts or Lattice compounds:
The addition compounds which are stable in solid state only but are broken down into individual
constituents when dissolved in water are called double salts or lattice compounds. Their solution has
the same properties as the mixture of individual compounds.
When Mohr’s salt, [FeSO4.(NH4)2SO4.6H2O], is dissolved in water, it exhibits the properties of FeSO4
and (NH4)2SO4, i.e., they produce Fe2+, NH4+ and SO4-2 ions in solution. Thus, each ion has its identity in
double salt.

Carnallite, KCl.MgCl2.6H2O; potash alum, K2SO4.Al2(SO4)3.24H2O are the examples of double salts.

K 2 SO4 + Al2 (SO4 )3 + 24H2 O →K 2 SO4 . Al2 (SO4 )3 . 24H2 O


Alum
Double Salt Responds test for the ions
Carnalite: KCl. MgCl2 6 H 2 O K + , Mg 2 + , Cl −

Potash alum : K 2 SO 4 . Al 2 (SO 4 )3 24 H 2 O K + , Al 3 + , SO 42 −

ACTIVESITE EDUTECH- 9844532971 57


ACTIVE SITE
a
EDUTECH Co-ordination Compounds
b. Co-ordination or complex compounds:
Addition compounds in which some of the constituent ions or molecules lose their identity and when
dissolved in water they do not break up completely into individual ions, are called coordination
compounds. Properties of their solutions are different than those of their constituents.
When crystals of CuSO4. 4NH3 are dissolved in water there is no evidence of the presence of Cu 2+ ions
or ammonia molecules. A new ions, [Cu(NH3)4]2+, is furnished in which ammonia molecules are directly
linked with the metal ion. Similarly, the aqueous solution of Fe(CN)2.4KCN does not give tests of Fe2+
and CN– ions but gives the test for a new ion, [Fe(CN)6]4–. The ions [Cu(NH3)4]2+ and [Fe(CN)6]4– are
called complex ions.
CuSO4.4NH3⇌ [Cu(NH3)4]2+ + SO42–

Fe(CN)2.4KCN ⇌ 4K+ + [Fe(CN)6]4–


Thus, a coordination or complex compound may be defined as a molecular compound that results from
the combination of two or more simple stable molecular compounds and retains its identity in the solid as
well as in dissolved state. The properties of such compounds are totally different than individual
constituents.
Complex compound Cation Anion
[Cu(NH 3 )4 ]SO 4 [Cu ( NH 3 )4 ]+2 SO 42 −

K 2 [PtF6 ] 2K + [PtF6 ]2 −

[Co ( NH 3 )6 ] [Cr (CN 3 )6 ] [Co ( NH 3 )6 ] 2 + [Cr (CN ) 6 ] 3 −

ACTIVESITE EDUTECH- 9844532971 58


ACTIVE SITE
a
EDUTECH Co-ordination Compounds
Solved Examples
Example 1. Explain that Fe(NH4)2(SO4)2 is a double salt but K3Fe(CN)6 is a complex.
Sol. Fe(NH4)2(SO4)2 is a double salt because its solution gives tests of Fe2+, NH4+ and SO42–. K3Fe(CN)6 is
not a double salt but it is a complex because the solution of this compound renders K+ and
[Fe(CN)6]3– ions and further [Fe(CN)6]3– does not give the tests of Fe3+ and CN–.

What is a coordinate bond?

H +
H
H N: H
+
H N H  NH4+
H + H
LEWIS
LEWIS BASE + Ammonium ion
– ACID
e pair –
donor e pair
acceptor

2. Ligand:
Any atom, ion or molecule which is capable of donating a pair of electrons to the central atom is
called a coordinating group or ligand. Ligand donates a pair of electrons to the central atom. Ligands
are attached to the central metal ion or atom through coordinate bonds or dative linkage. Ligands are
normally polar molecules like N̈H3 ; H2 Ö: or anions such as Cl¯, OH¯, CN¯ etc. which contain at least one
unshared pair of valence electrons.
Types of Ligands on the basis of number of donor atoms present in them:
(a) Mono- or unidentate ligands:
They have one donor atom, i.e., they supply only one electron pair to central atom
Examples: F¯, Cl¯, Br¯, H2O, NH3, CN¯, NO2¯, NO¯ CO, (C6H5)3 P, etc.

(b) Bidentate ligands:


Ligands which have two donor atoms and have the ability to link with central metal ion at two
positions are called bidentate ligands. Some examples are:

ACTIVESITE EDUTECH- 9844532971 59


ACTIVE SITE
a
EDUTECH Co-ordination Compounds

Note: A ligand with 2 lp ofe̅ on the same atom (H2 Ö:) is not considered as bidentate.

(c) Polydentate Ligands: Ligands with more than 2 donor atoms

ACTIVESITE EDUTECH- 9844532971 60


ACTIVE SITE
a
EDUTECH Co-ordination Compounds

(d) Chelating ligands:


Polydentate ligands whose structures permit the attachment of two or more donor sites to the same metal
ion simultaneously, thus closing one or more rings are called chelating ligands and the compounds formed
are known as chelate compounds.
A chelate may be defined as a ring structure formed by the combination of a polydentate ligand having
two or more donor atoms with a metal ion forming part of the ring. The process of formation of chelates
is called chelation.
Chelate complexes are more stable than ordinary complexes in which the ligand is a monodentate. This
increased stability of the compound due to chelation is called the chelate effect in the complex ion given
below, 5 membered rings are formed. So all these are called chelate complexes
2+
CH2—H2N NH2—CH2
Cu
CH2—H2N NH2—CH2
Generally, the chelate complexes with 5 or 6 membered rings are more stable.

(e) Flexidentate ligands:


Polydentate ligands have flexidentate character. It is not necessary that all the donor atoms present in the
polydentate donor atoms present in the polydentate ligands should form coordinate bonds with central
metal atom (or ion). For example: EDTA which is hexadentate ligand can function as pentadentate or
tetradentate ligand with certain metal ions. Sulphate ion can also act as monodentate ligand.
Out of these, 5 membered rings are very stable when they involve saturated ligands.
On the other hand, 6-membered ring structures acquire maximum stability when they involved
unsaturated ligands containing conjugate double bond. This is due to the resonance effects involving metal
d-orbitals and ligand p–orbital electrons.

ACTIVESITE EDUTECH- 9844532971 61


ACTIVE SITE
a
EDUTECH Co-ordination Compounds
(f) Ambidentate ligands:
There are certain ligands which have two or more donor atoms but during formation of complexes only
one donor atoms is attached to metal ion. Such ligands are called ambidentate ligands such as CN¯, NC¯,
NO− -
2 , ONO , etc.

ACTIVESITE EDUTECH- 9844532971 62


ACTIVE SITE
a
EDUTECH Co-ordination Compounds
3. Coordination number:
The number of atoms of the ligands that are directly bonded to the central metal atom or ion by
coordinate bonds is known as the coordination number of the metal atom or ion.
It is actually the number of coordinate covalent bonds which the ligands form with the central metal
atom or ion some common coordination numbers exhibited by metal ions are 2, 4, and 6.
The light transition metals exhibit 4 and 6 coordination numbers while heavy transition metals may
exhibit coordination number more than 6.
For example, the coordination number of Ni in the complex [Ni(NH3)4]Cl2 is 4 and that of Pt in the
complex K2[PtCl6] is 6

4. Coordination sphere and Ionization sphere:


The central metal atom and the ligands directly attached to it are collectively termed as the
coordination sphere. Coordination sphere is written inside square bracket, for example [Co(NH3)6]3+.
The central metal atom and the ligands inside the square bracket behave as a single entity. The part
outside the bracket is called ionization sphere.
The species present in the coordination sphere are nonionizable. The species present in the ionization
sphere are ionizable.

Charge on
Counter ion
Central metal ion com p lex io n Ionization sphere
3+
Co-ordination Co(NH3)6 .3Cl−
sphere

Ligand Co-ordination
n u mb er
5. Oxidation state:
It is a number which represents the electric charge on the central metal atom of a complex ion.
For example, the O.N. of Fe, Co and Ni in [Fe(CN)6]4–, [Co(NH3)6]3+ and Ni(CO)4 is + 2, + 3 and zero
respectively of the complex is the sum of the charges of the constituents parts.

ACTIVESITE EDUTECH- 9844532971 63


ACTIVE SITE
a
EDUTECH Co-ordination Compounds

6. Effective atomic number (EAN):


In order to explain the stability of the complex, Sidgwick proposed effective atomic number.
It can be defined as the resultant number of electrons with the metal atom or ion after gaining
electrons from the donor atoms of the ligands. The EAN generally coincides with the atomic number of
next noble gas in some case.,
EAN is calculated by the following relation EAN = atomic number of the metal-number of electrons lost
in ion formation + number of electrons gained from the donor atoms of the ligands
The EAN values of different metal in their respective complexes are tabulated as follows:
Complex Oxidation At. No. of Coordination Effective atomic number
Compound state of central No.
central metal metal
K4[Fe(CN)6] +2 26 6 (26 – 2) + (6 × 2) = 36[Kr]
[Cu(NH3)4]SO4 +2 29 4 (29 – 2) + (4 × 2) = 35
[Co(CH3)6]Cl3 +3 27 6 (27 – 3) + (6 × 2) = 36[Kr]
Ni(CO)4 0 28 4 (28 – 0) + (4 × 2) = 36[Kr]
K2[Ni(CN)4] +2 28 4 (28 – 2) + (4 × 2) = 34
K2[PtCl6] +4 78 6 (78 – 4) + (6 × 2) = 86[Rn]
K3[Cr(C2O4)3] +3 24 6 (24 – 3) + (6 × 2) = 33
K3[Fe(CN)6] +3 26 6 (26 – 3) + (6 × 2) = 35
K2[HgI4] +2 80 4 (80 – 2) + (4 × 2) = 86[Rn]
[Ag(NH3)2]Cl +1 47 2 (47 – 1) + (2 × 2) = 50
K2[PdCl4] +2 46 4 (46 – 2) + (4 × 2) = 52

ACTIVESITE EDUTECH- 9844532971 64


ACTIVE SITE
a
EDUTECH Co-ordination Compounds

Solved Examples
Example 1. Determine the E.A.N. of the metal atom in the following:
(a) K3[Cr(C2O4)3] (b) K4[Fe(CN)6]

Sol. (a) Cr3+ (Z = 24)  E.A.N. = 21 + 6 × 2 = 33 (Less stable)

(b) Fe2+ (Z = 26)  E.A.N. = 24 + 12 = 36 (more stable)

Example 2. In which of the following complexes, the effective atomic number of the metal atom equals
the atomic number of the next noble gas ?
(1) [Co(SCN)4]2– (2) [Cr(NH3)6]3+ (3) [V(CO)6]– (4) [Ni(Cl)4]2–
Ans. (3)
Sol. V– (Z = 23)  E.A.N. = 23 + 1 + 12 = 36 which is atomic number of Kr.

ACTIVESITE EDUTECH- 9844532971 65


ACTIVE SITE
a
EDUTECH Co-ordination Compounds
7. Co-ordination polyhedron: The spatial arrangement of the ligands attached to the central metal
atom/ion in the co-ordination sphere or complex entity is called co-ordination polyhedron. The
arrangement in fact, determines the geometry of the complex which may be tetrahedral, square
planar, pyramidal, trigonal bipyramidal, octahedral etc. depending upon the number of the ligating
groups present.

Here M stand for the metal atom/ion while L indicates a monodentate ligand.

8. Homoleptic and heteroleptic complexes:


Complexes in which the metal atom or ion is linked to only one type of ligands are called homoleptic
complexes, e.g., [Co(NH3)6]3+ and [Fe(CN)6]4- etc.
The complexes in which the metal atom or ion is linked to more than one kind of ligands are called
heteroleptic complexes, e.g., [Co(NH3)4Cl2]+ and [Cr(en)2Cl2]+ etc.

ACTIVESITE EDUTECH- 9844532971 66


ACTIVE SITE
a
EDUTECH Co-ordination Compounds

CLASS EXERCISE
1] The solution of the complex [Cu (NH3)4]SO4 in water will
a) give the tests of Cu2+ ion b) give the tests of NH3
2−
c) give the tests of SO4 ions d) not give the tests of any of the above
2] Which is incorrectly matched?
Complex Co-ordination number
a) Cuprammonium sulphate 4
b) [CO(en)3] 3+ 3
c) [Co(NH3)3Cl3] 6
d) Ni (CO)4 4
3] Which one of the following cannot act as a ligand?
a) PH3 b) NO+ c) BF3 d) Br −
4] The complex that violates EAN rule is:
a) Potassium ferrocyanide b) Potassium ferricyanide
c) Tetracarbonyl nickel d) Hexaammine cobalt (III) chloride
5] In the coordination compound, K4[Ni(CN)4], oxidation state of nickel is
a) -1 b) +1 c) 0 d) +2

HOME EXERCISE
1] Which one among the following is not a coordination compound?
a) Potash alum b) Cuprammonium sulphate
c) Potassium ferrocyanide d) Potassium ferricyanide
2] Ligands are
a) Negative ions only b) Never positive ions
c) Neutral molecules only d) Negative or positive or neutral molecules
3] Which one of the following ligands forms a chelate ring?
a) Acetate b) Oxalate c) Cyanide d) Ammonia
2+
4] The coordination number of Pt in [Pt(NH3)4Cl2] ion is
a) 2 b) 4 c) 6 d) 8
5] Which is a double salt?

ACTIVESITE EDUTECH- 9844532971 67


ACTIVE SITE
a
EDUTECH Co-ordination Compounds
a) Carnalite
b) Potassiumferrocyanide
c) Potassium ferricyanide
d) Nessler’s reagent
6] In which of the following transition metal complexes does the metal exhibit zero oxidation state
a) [Co(NH3 )6 ]Cl3 b) [Fe(H2 O6 )]SO4 c)[Ni(CO)4 ] d) [Fe(H2 O)6 ]Cl5
7] Which of the following is cationic complex
a) K 4 [Fe(CN)6 ] b) [Ni(CO)4 ] c)[Co(NH3 )3 Cl3 ] d) [Cu(NH3 )4 ]SO4
8] Central metal ion in complex compound act as
a) Lewis acid b) Lewis base c) Arrhenius acid d) Arrhenius base
9] Which one of the following acts as a lewis base in complexes
a) CO2 b) BF3 c)NH3 d) BCl3

CLASS EXERCISE
1) c 2) b 3) c 4) b 5) c
HOME EXERCISE
1) a 2) d 3) b 4) d 5)a 6) c
7)d 8) a 9) c

NOMENCLATURE OF CO-ORDINATION COMPOUNDS


Rules of Nomenclature:
1. In ionic complexes, the cation is named first and then the anion. The names of cation and anion are
separated by a space. Non-ionic complexes are given a one-word name.
2. Naming of coordination-sphere: Within a complex ion, the ligands are named by first followed by the
metal ion
3. Naming of ligands:
a) Anionic ligands: Anionic ligands ending with ide are named by replacing ‘–ide’ with suffix ‘-o’ or
replacing ‘–e’ by ‘– O’
Anion Symbol Name
Chloride Cl– Chlorido
Bromide Br– Bromido
Cyanide CN– Cyano
Oxide O2- oxo
Peroxide O2−
2 peroxo
Hydroxide OH– hydroxo
Sulphide S2– Sulphido
Amide NH2− amido
Imide NH2- imido
Nitride N3– nitrido
Phosphide P3– phosphido

ACTIVESITE EDUTECH- 9844532971 68


ACTIVE SITE
a
EDUTECH Co-ordination Compounds
Ligands whose names end with ‘– ite’ or ‘–ate’ becomes –‘ito’&‘–ato’ respectively
Carbonate CO2−
3 Carbonato
Oxalate C2 O2−
4 Oxalato
Sulphate SO2−
4 Sulphato
Nitrate NO− 3 nitrato
2−
Sulphite SO3 sulphito
Thiosulphate S2 O2−
3 thiosulphato
Acetate CH3COO– acetato
Nitrite ONO–, NO−
2 nitrito nitro
Thiocyanate SCN– thiocyanato
Isothiocyanate NCS– isothiocyanato
Glycenate gly glycenato
b) Neutral ligands: Certain neutral ligands are named as the molecules while others are named
differently.
i) Ligands which are named as molecules:
C5H5N pyridine (py)
(C6H5)3P triphenyl phosphine
H2NCSNH2 thiourea (ta)
H2N.CN2CN2NH2 ethylenediamine (en)
PH3 phosphine
ii) Ligands which are not named as molecules:

c) Organic free radicals: These are named as usual i.e., CH3 is methyl, C2H5 ethyl, C6H5 phenyl

d) Positive ligands: Positively charged ligands has suffix-ium


NO+ 2 nitronium ;
+
H2 N. NH3 hydrazinium
NO + nitrosonium
4. The ligands are named in an alphabetical order without separation by hypen
5. If there are several ligands of same type, prefixes like di, tri, tetra, etc., are used to indicate the number
of ligands of that type.
When the name of the ligand includes a number, like dipyridyl, ethylenediamine, then bis, tris, tetrakis,
etc are used

ACTIVESITE EDUTECH- 9844532971 69


ACTIVE SITE
a
EDUTECH Co-ordination Compounds
6. The oxidation number of the central metal is shown by roman numericals immediately following its
name
7. Positive & neutral complexes have no special ending but the anionic complexes end in the suffix‘-ate’
Ex: Pb: plumbate; Au: aurate; Zn: zincate
Sn: stannate; Ag: argentate; Co: cobaltate
Fe: ferrate; Cu: cuprate; Pt: platinate
8. For ligands which act as bridge between two metal atoms,’’ is written before their names

Ex:
Here amido & nitro are bridging ligands.
Tetraamminecobalt (III) -amido--nitrotetraamminecobalt(III) nitrate

Ex:
Tetraaquairon (III) -- dihydroxotetraaquairon (III) sulphate
9. The point of attachment in case of unidentate ligands with more than onecoordinating (donor) atoms
is either indicated by using different names for the ligand ( thiocyanato isothiocyanato) or by replacing
the symbol of the donor atom attached after the name of the ligand separated by a hypen.
Ex. (NH4)3 [Cr(SCN)6]
Ammonium hexathiocyanato-S-Chromate (III) or
Ammonium hexaisothiocyanatochromate (III)

Ex: (NH4)2 [Pt (NCS)6] ammonium hexathiocyanato – N- palatinate (IVX)(OR)


Ammonium hexaisothiocyanatoplatinate (IV)
10. The name of the complex should not start with a capital letter.
Complex compound NAME
1. [Co(NH3)6] Cl3 hexaamminecobalt (III) chloride
2. [Cr(H2O)4Cl2] NO3 tetraaquadichloridochromium (III) nitrate
3. [Co(NH3)4Cl(NO2)] NO3 tetraamminechloridonitrocobalt (III) nitrate.
4. K3 [Fe(C2O4)3] potassium trioxalatoferrate (III)
5. K3 [Co(CN)5(NO)] potassium pentacyanonitrosylcobaltate (II).
6. K [Pt(NH3)Cl3] potassium amminetrichloridoplatinate (II)
7. Na2 [CrF4O] sodium tetrafluoridooxochromate (IV)
8. Na2 [SiF6] sodium hexafluoridosilicate (IV)
9. [Cr(en)3] Cl3 tris (ethane-1, 2-diamine) chromium (III) chloride.
10. [CoCl2 (en)2] SO4 dichloridobis (ethane-1, 2-diamine) cobalt (IV) sulphate.

ACTIVESITE EDUTECH- 9844532971 70


ACTIVE SITE
a
EDUTECH Co-ordination Compounds
11. Na3 [Fe(C2O4)3] sodium trioxalatoferrate (III).
12. [Co(NH3)4(H2O) Br] (NO3)2 tetraammineaquabromidocobalt (III) nitrate.
13. [Cr(H2O)6] Cl3] hexaaquachromium (III) chloride
14. Hg [Co(NCS)4]* mercury tetraisothiocyanatochobaltate (II). Or
mercury tetrathiocyanato-N-cobaltate (II)
15. [CoCl(en)2(ONO)]+ chloridobis (ethane-1, 2-diamine) nitritocobalt (III) ion.
16. [Ni(CO)4] tetracarbonylnickel (0)
17. K4 [Ni(CN)4] potassium tetracyanonickelate (0)
18. [Cr(H2O)4Cl2] Cl tetraaquadichloridochromium (III) chloride
19. [Co(NH3)3(NO2)3] triamminetrinitrocobalt (III)
20. [Pt(NH3)2Cl2] diamminedichloridoplatinum (II)
21. [Co(NH3)5(CO3)] Cl pentaamminecarbonatocobalt (III) chloride
22. [CoCl2 (en)2] SO4 dichlorobis (ethane-1, 2-diamine) cobalt (IV) sulphate
23. [Pt (NH3)4 Cl2] [PtCl4] Tetraamminedichloridoplatium (IV) tetrachloridoplatinate (II)
24. Fe4 [Fe(CN)6]3 ferric hexacyanoferrate (II)
25. [Co(NH3)4 Cl2]3 [Cr(CN)6] tetraaminedichloridocobalt (III) hexacyanochromate (III)
26. Li [AlH4] lithium tetrahydridoaluminate (III)
27. [Cr(PPh3)(CO)5] pentacarbonyltriphenylphosphinechromium (0)
28. [Fe(C5H5)2] bis(cyclopentadienyl) iron (II)
29. [Ni(dmg)2] bis(dimethylglyoximato) nickel (II)
30. [Mn3(CO)12] dodecacarbonyltrimanganese (0)
31. [Co(NH3)6] ClSO4 hexaamminecobalt (III) chloride sulphate
32. [(NH3)5 Cr-OH-Cr(NH3)5 Cl5 pentaamminechromium(III)-hydroxo
pentaamminechromium (III) chloride
33. [(CO)3Fe(CO)3Fe(CO)3] tri--carbonyl-bis(tricarbonyl iron (0)) or
(di iron enneacarbonyl)
Note that the three CO groups in the middle are bridge groups.
*Co(II) forms tetrahedral as well as octahedral complexes. The large monodentate ligands
Cl -, Br-, I -, OH- SCN- commonly form tetrahedral complexes. Thus, the given tetrahedral complex
hascobalt in (+II) oxidation state.

CLASS EXERCISE
1] IUPAC name of Na3[Co (NO2)6 is
a) sodium cobaltinitrite b) sodium hexanitritocobaltate (III)
c) sodium hexanitrocobalt (III) d) sodium hexanitritrocobalt (II)
2] The IUPAC name of the compound [Cu(NH3)4](NO3)2 is:
a) cuprammoniumnitrate b) dinitratotetraaminecopper (II)
c) tetraamminecopper (II) nitrate d) tetraamminecopper (III) nitrate
3] The IUPAC name of K2[Zn(OH)4] is

ACTIVESITE EDUTECH- 9844532971 71


ACTIVE SITE
a
EDUTECH Co-ordination Compounds
a) potassium tetrahydroxyzinc (II) b) potassium tetrahydroxozincate (II)
c) potassium tetrahydroxozincate (IV) d) potassium tetrahydroxozinc (II)
4] IUPAC name of [Co(ONO)(NH3)5]Cl2 is
a) pentammine nitrocobalt (II) b) pentammine nitrosocobalt (III) chloride
c) pentammine nitritocobalt (III) chloride d) pentammine oxo-nitrocobalt (III) chloride
5] Pick the correct name of [Co(NH3)5Cl]Cl2
a) chloropentammine cobalt (III) chloride b) chloropentammine cobalt (III)
c) chloropentammine cobalt (II) chloride d) pentammine chloro cobalt (III) chloride

HOME EXERCISE
1] The IUPAC name of K2[Cr(CN)2O2(O)2(NH3) is
a) Potassiumammine dicyanodioxoperoxochromate (VI)
b) Potassiumammine cyanoperoxodioxochromatic (IV)
c) Potassiumammine cyanoperoxodioxochromium (IV)
d) Potassiumammine cyanoperoxodioxochromium (IV)
2] The IUPACname of K4[Fe(CN)6] is
a) potassium ferrocyanide b) potassium hexa cyanoferrate (I)
c) tetra potassium hexa cyanoferrate (II) d) potassium hexa cyanoferrate (II)
3] The chemical formula of diammine silver (I) chloride is
a) [Ag(NH3) Cl] b) [Ag(NH3)3]Clc) [Ag(NH4)2Cl d) [Ag(NH3)2]Cl
4] The IUPAC name of [Pt(NH3)4(NO)2Cl]SO4 is
a) chloronitro tetrammine platinum (IV) sulphate
b) tetrammine chloronitro platinum (II) sulphate
c) tetrammine chloronitro platinum (IV) sulphate
d) chlorotetrammine nitroplatinum (IV) sulphate
5] K3[Al(C2O4)3] is called
a) potassium aluminium (III) oxalate b) potassium alumino oxalate
c) potassium trioxalato aluminate (VI) d) potassium trioxalato aluminate (III)
NCERT Test Book Questions: 9.6, 9.7

CLASS EXERCISE
1)b 2) c 3) b 4) c 5) d
HOME EXERCISE
1) a 2) d 3) d 4) c 5) d

ACTIVESITE EDUTECH- 9844532971 72


ACTIVE SITE
a
EDUTECH Co-ordination Compounds
ISOMERISM IN COORDINATION COMPOUNDS
Compounds having same molecular formula but different structural formula or stereo-forms are called
isomers and the phenomenon is said to be isomerism. Isomerism in coordination compounds may be
divided into two main types:

1. Structural isomerism: It is displayed by compounds that have different ligands within their coordination
spheres. The different types of structural isomerism shown by coordination compounds are:
(i) Ionization isomerism: This type of isomerism arises when the coordination compounds give
different ions in solution.
For example, there are 2 isomers of the formula [Co(NH3)5Br]SO4. as [Co(NH3)5Br]SO4 and [Co(NH3)5SO4]Br.
[Co(NH3 )5 Br]SO4 [Co(NH3 )5 Br]2+ + SO2−
4

(Violet) Pentaamminebromocobalt(III)ion
This isomer gives a white precipitate of BaSO4 with BaCl2 solution.

[Co(NH3 )5 SO4 ] Br [Co(NH3 )5 SO4 ]+ + Br −



(Red) Pentaamminesulphatocobalt(III)ion
Above isomer gives light yellow precipitate with AgNO3 solution.
Other examples of ionization isomerism are:

d)

ACTIVESITE EDUTECH- 9844532971 73


ACTIVE SITE
a
EDUTECH Co-ordination Compounds

(ii) Hydrate isomerism: This type of isomerism arises when different number of water molecules are
present inside and outside the coordination sphere.
Ex: 𝐂𝐫𝐂𝐥𝟑 . 𝟔𝐇𝟐 𝐎
(a) [Cr(H2O)6]Cl3 - Violet. All the six water molecules are coordinated to Cr. It has three ionizable
chloride ions.
(b) [Cr(H2O)5Cl]Cl2.H2O- Green. Five water molecules are coordinated to Cr. It has two ionizable
chloride ions. One water molecule outside the coordination sphere can be easily lost.
(c) [Cr(H2O)4Cl2] Cl.2H2O - Green. Four water molecules are coordinated to Cr. It has one ionizable Cl
ions.
Other examples of hydrate isomerism are:

(iii) Coordination isomerism: This type of isomerism is observed in the coordination compounds having
both cationic and anionic complex ions. The ligands are inter changed in both the cationic and
anionic ions to form isomers. Some examples are:

a)

b)

c)

ACTIVESITE EDUTECH- 9844532971 74


ACTIVE SITE
a
EDUTECH Co-ordination Compounds

ACTIVESITE EDUTECH- 9844532971 75


ACTIVE SITE
a
EDUTECH Co-ordination Compounds
(iv) Linkage isomerism: This type of isomerism occurs in complex compounds which contain
ambidentate ligands like, NO− – – 2−
2 SCN , CN andS2 O3 and CO. These ligands have two donor atoms
but at a time only one atom is directly linked to the central metal atom of the complex. These
types of isomers are distinguished byinfrared (I.R) spectroscopy.

For example, [Co(NH3)5NO2]Cl2 and [Co(NH3)5ONO]Cl2 are linkage isomers as NO2− is linked through N or
through O.

a)

b)

(v) Polymerization isomerism: This type of isomerism exists in compounds having same Stoichiometric
composition but different molecular compositions. The molecular compositions are simple multiples
of the simplest stoichiometric arrangement. For example, in the following three compounds, the
second and third compounds are polymers of the first
[Pt(NH3 ]2 Cl2 ; [Pt(NH3 )4 ][(PtCl4 )]; [Pt(NH3 )3 Cl]2 [PtCl4 ]
(i) (ii) (iii)

ACTIVESITE EDUTECH- 9844532971 76


ACTIVE SITE
a
EDUTECH Co-ordination Compounds
(vi) Co-ordination position isomerism: This type of isomerism is exhibited by polynuclear complexes by
changing the position of ligands with respect to different metal atoms present in the complex.
Example:

2. Stereo-Isomerism: Compounds are stereo-isomers when they contain the same ligands in their
coordination spheres but differ in the way that these ligands are arranged in space. Stereo-isomerism
is of two types, viz., geometrical isomerism and optical isomerism.
(A) Geometrical Isomerism: This isomerism is due to ligands occupying different positions around the
central metal atom or ion. The ligands occupy positions either adjacent or opposite to one another.
This type of isomerism is also known as cis-trans isomerism.
Geometrical isomerism is very much common in coordination number 4 and 6 complexes.

Square planar complexes (coordination number four) exhibit geometrical isomerism.


(i) Complexes with general formula, Ma2b2: (where both ’a’ and ’b’ are monodentate)

The cis and trans isomers of [Pt(NH3)2Cl2].

ACTIVESITE EDUTECH- 9844532971 77


ACTIVE SITE
a
EDUTECH Co-ordination Compounds
(ii) Complexes with general formula Ma2bc:

(iii) Complexes with general formula, Mabcd: In these complexes, all the four ligands that are linked
with the central metal atom/ ion are different. Such compounds can exist as 3 geometrical isomers.
In writing these structures, the position of one of the ligand is kept fixed and the remaining 3 ligands
are placed in transposition one by one.

Example: [Pt (NH3)(NH2OH) (NO2) (py)]NO2

(iv) Square planar complexes having unsymmetrical bidentate ligands can also show geometrical
isomerism. For example, platinum glycinato complex, [Pt(Gly)2], exhibit geometrical isomerism.

ACTIVESITE EDUTECH- 9844532971 78


ACTIVE SITE
a
EDUTECH Co-ordination Compounds
Octahedral Complexes (coordination number six) exhibit geometrical isomerism.

Positions 1-6, 2-4 & 3-5 are trans, while all other positions are cis.
(i) Octahedral complexes of type 𝐌𝐚𝟒 𝐛𝟐 :

Ex: [𝐂𝐨(𝐍𝐇𝟑 )𝟒 𝐂𝐥𝟐 ]+

ACTIVESITE EDUTECH- 9844532971 79


ACTIVE SITE
a
EDUTECH Co-ordination Compounds

(iii) Octahedral complexes of type 𝐌(𝐚𝐚)𝟐 𝐛𝟐 :


aa = symmetrical bidentate ligand
Ex: [𝐂𝐫(𝐞𝐧)𝟐 𝐂𝐥𝟐 ]

X
X X A
A A
M
M
A A
A A A
X

Cl Cl
Cl

en Cr en Cr en

Cl en

ACTIVESITE EDUTECH- 9844532971 80


ACTIVE SITE
a
EDUTECH Co-ordination Compounds
(iv) Octahedral complexes of type 𝐌𝐚𝟑 𝐛𝟑 :
In octahedral complexes of type Ma3 b3, if similar ligands are on the same triangular face of the
octahedron, the geometrical isomer is called facial or fac-isomer.
If the ligands are on the same equatorial plane of the octahedron or around the meridian of the
octahedron, the isomer is called meridional or mer-isomer.

Ex: [Co(NH3 )3 Cl3 ]

B B
A B A B

M M
A A A B
B A
Mer Fac
[Co (NH3)3 Cl3]
Cl Cl
NH3 Cl NH3 Cl
Cr Cr
NH3 NH3 NH Cl
3
Cl NH3

ACTIVESITE EDUTECH- 9844532971 81


ACTIVE SITE
a
EDUTECH Co-ordination Compounds

(v) Octahedral complexes of type 𝐌(𝐚𝐛)𝟑:


ab = unsymmetrical bidentate ligand
Ex: [𝐂𝐫(𝐠𝐥𝐲)𝟑 ] 𝐠𝐥𝐲 = 𝐍𝐇𝟐 𝐂𝐇𝟐 𝐂𝐎𝐎−

(vi) Octahedral complexes of type Mabcdef having all 6 different ligands attached to the central metal
atom can have 15 geometrical isomers.
Ex: Only one such complex is known: [𝐏𝐭(𝐩𝐲)(𝐍𝐇𝟑 )(𝐍𝐎𝟐 )(𝐂𝐥)(𝐁𝐫)(𝐈)]

(vii)

Note:
1. Geometrical isomerism is not observed in complexes of coordination number 2 and 3.
2. Geometrical isomerism is not observed in complexes of coordination number 4 of tetrahedral
geometry.
3. Complexes of general formulae, Ma3b or Mab3, or Ma4 of square planar geometry do not show
geometrical isomerism.
4. The complexes of general formula, Ma6 and Ma5b, of octahedral geometry do not showgeometrical
isomerism.

ACTIVESITE EDUTECH- 9844532971 82


ACTIVE SITE
a
EDUTECH Co-ordination Compounds

(B) Optical isomerism:


A coordination compound which can rotate the plane of polarized light is said to be optically active. When
the coordination compounds have same molecular formula, same structural formula but differ in their
abilities to rotate directions of the plane of polarized light are said to exhibit optical isomerism and
molecules are optical isomers. The optical isomers are pair of molecules which are non-superimposable
mirror images of each other.
The essential requirement for a substance to be optically active is that the substance should not have a
plane of symmetry in its structure. The isomer which rotates the plane of polarized light to right direction
is termed dextro (d-form) while the isomer which rotates the plane of polarized light to left direction is
termed laevo (l-form).
The two optically active isomers are collectively called enantiomers. Enantiomers are non-superimposable
mirror image to each other and their physical properties are different. Optical isomerism is expected in
tetrahedral complexes of the type Mabcd but no optical isomer has been isolated until now. Optical
isomers rarely occur in square planar complexes on account of the presence of axis of symmetry. Optical
isomerism is very common in octahedral complexes.

ACTIVESITE EDUTECH- 9844532971 83


ACTIVE SITE
a
EDUTECH Co-ordination Compounds

1) Octahedral complex of type 𝐌𝐚𝟐 𝐛𝟐 𝐜𝟐 :


Ex: [Pt(py)2 (NH3)2 Cl2 ]2+

ACTIVESITE EDUTECH- 9844532971 84


ACTIVE SITE
a
EDUTECH Co-ordination Compounds
2) Octahedral complexes of type 𝐌(𝐚𝐚)𝐛𝟐 𝐜𝟐 : (aa) is symmetrical bidentate ligand
Ex:[Cr(en)cl2 (NH3 )2 ]+

3) Octahedral complexes of type 𝐌(𝐚𝐚)𝟐 𝐛𝟐 :


Ex:[Co(en)2 cl2 ]+

Trans form of [M(AA)2a2]n± does not show optical isomerism due to plane of symmetry.
4) Octahedral complexes of type 𝐌(𝐚𝐚)𝟐 𝐛𝐜:
Ex: [Co(en)2 NH3 Cl]2+

5) Octahedral complexes of type 𝐌(𝐚𝐚)𝟑 :


Ex: [Cr(en)3 ]3+

ACTIVESITE EDUTECH- 9844532971 85


ACTIVE SITE
a
EDUTECH Co-ordination Compounds

6) Octahedral complexes of type 𝐌𝐚𝐛𝐜𝐝𝐞𝐟:


Ex: [Pt(py)(NH3 )NO2 Cl Br I]

Note:
1) Optical isomerism rarely occurs in square planar complexes due to presence of axis of symmetry.
2) Optical isomerism is expected in tetrahedral complexes of the type Mabcd, but no optical isomer has
been isolated till now.

ACTIVESITE EDUTECH- 9844532971 86


ACTIVE SITE
a
EDUTECH Co-ordination Compounds
CLASS EXERCISE
1] The complex ions [Co(NH3)5(NO2)]2+ and [Co(NH3)5(ONO)]2+ are called
a) Ionization isomers b) Linkage isomers
c) Co-ordination isomers d) Geometrical isomers
2] The compounds [Cr(H2O)6]Cl3, [Cr(H2O)5Cl. H2O and [Cr(H2O)4Cl2]Cl.H2O represent
a) Linkage isomers b) Ionisation isomers
c) Hydrate isomers d) Co-ordination isomers
II IV IV II
3] and complexes represent the example of
[Pt(NH3 )4 ] [PtCl6 ] [Pt(NH3 )4 Cl2 ] [PtCl4 ]
a) ligand isomerism b) hydrate isomerism
c) linkage isomerism d) co-ordination isomerism
4] The complexes [Co(NH3)6] [Cr(C2O4)3]and [Cr(NH3)6][Co(C2O4)3]
a) geometrical isomerism b) ionization energy
c) coordination isomerism d) linkage isomerism
5] [Co(NH3)4Cl2]NO2 and [Co(NH3)4Cl.NO2] Cl are
a) optical isomers b) geometrical isomers
c) ionization isomers d) linkage isomers
6] Optical isomerism is shown by octahedral complexes
a) having all monodentate ligands b) having all the three bidentate ligands
c) having two trans bidentate ligands d) having two trans monodentate ligands
7] Which of the following will exhibit optical isomerism?
a) [Cr(en)(H2O)4]3+ b) [Cr(en)3]3+ c) trans-[Cr(en)2Cl2] d) [Cr(NH3)6]3+
8] The total number of possible isomers for the complex compound [Cu(NH3)4][PtCl4]
a)6 b) 5 c) 4 d) 3

HOME EXERCISE
1] Which of the following cannot show linkage isomerism?
a) NO−2 b) NH3 c)CN– d) SCN–
2] [Co(NH3)5Br]SO4and [Co(NH3)5SO4] Br are examplesof which type of isomerism?
a) Linkage b) Optical c)Geometrical d) Ionisation
3] Which of the following does not have optical isomer?
a) [Co(en)(NH3)2Cl2]Cl b)[Co(en)2Cl2]Cl
c) [Co (NH3)3Cl3] d) [Co(en)3]Cl3
4] Change in composition of coordination sphere yields which type of isomer?
a) Geometrical b) Ionisation c) Optical d) None of these
5] Which one of the following has highest number of isomers?
a) [Co(NH3)5CI]2+ b) [Co(en)2Cl2]+ c) [Ru(NH3)4CI–] d) [In(PP3)2H(CO)]2+
6] Which kind of isomerism is exhibited by octahedral [Co(NH3)4Br2Cl]?
a) Geometrical and ionization b) Geometrical only
c) Geometrical and optical d) Optical and ionization
7] Which one of the following is expected to exhibit optical isomerism[en=ethylenediamine]?

ACTIVESITE EDUTECH- 9844532971 87


ACTIVE SITE
a
EDUTECH Co-ordination Compounds
a) Trans–[Co(en)2Cl2] b) Cis – [Pt(NH3)2Cl2]
c) Cis – [Co(en)2Cl2] d) trans – [Pt(NH3)2Cl2]
8] Which of the following compounds shows optical isomerism?
a) [Cr(C2O4)3]3– b) [Cu(NH3)4]2+ c) [Co(CN)6]3– d) [ZnCl4]2–
NCERT Test Book Questions: 9.8, 9.9, 9.10, 9.11, 9.12

CLASS EXERCISE
1) b 2) c 3) d 4) c 5) c 6) b
7)b 8) a
HOME EXERCISE
1) b 2) d 3) c 4) b 5) b 6) a
7)c 8) a

Bonding in Coordination compounds (Werner’s Theory)

1. WERNER’S THEORY
Alfred Werner in 1893 put forward his concept of auxiliary (secondary) valency for advancing a correct
explanation for the characteristics of the coordination compounds. The fundamental postulates of
Werner’s theory may be summarized as:
A) Metals possess two types of valencies, namely primary (principal) or ionizable valency and
secondary (auxiliary) or non-ionizable valency.
• Primary valencies are those which a metal normally exhibits in the formation of its simple salt. Thus
in the formation of PtCl4 , CuSO4 and AgCl the primary valencies of Pt, Cu and Ag are 4, 2 and 1
respectively. Primary valencies are satisfied by negative ions.
• Secondary valencies are those which a metal cation exercises towards a neutral molecule or
negative group in the formation of its complex ions. Thus, secondary valencies may be satisfied by
negative ions, neutral molecules having lone electron pair (e.g. H2O, NH3 etc.) or even sometimes by
some positive groups. In every case, the co-ordination number of the metal must be fulfilled.

ACTIVESITE EDUTECH- 9844532971 88


ACTIVE SITE
a
EDUTECH Co-ordination Compounds
• In modern terminology the primary valency corresponds to oxidation number and the secondary to
co-ordination number. Primary valencies are shown by dotted lines while secondary by thick lines.
B) Every metal has a fixed number of secondary valencies, e.g. cobalt3+ and platinum4+ were
recognized to have six secondary valencies and copper2+ four. The total number of secondary
valencies required by a metal is more commonly known as coordination number (C.N.).
C) The secondary valencies are always directed towards fixed positions in space about the central metal
ion. Thus, the number and arrangements of ligands in space determines the stereochemistry of a
complex. Thus, in case of six secondary valencies the arrangement of secondary valencies was as
directed to the space of a regular octahedral while in case of four secondary valencies arrangement
might be either in a square planar or a tetrahedral manner and thus this postulate predicted the
possibilities of a variety of type of isomerism in coordination compounds. Remember that primary
valencies are non-directional.
D) To distinguish between the two types of valencies, Werner introduced the square brackets[ ] to
enclose atoms making up the coordination complex and which are therefore not ionized. The portion
enclosed in the bracket is known as coordination sphere and the portion present outside the bracket
as ionizing sphere.

Important aspect of structure of 5 different complexes of PtCl4 with ammonia prepared by Werner are as:
Complex Modern formula No. of 𝐂𝐥– ions Total number
precipitated of ions
PtCl4. 6NH3 [Pt(NH3)6]Cl4 4 5
PtCl4. 5NH3 [Pt(NH3)5Cl]Cl3 3 4
PtCl4. 4NH3 [Pt(NH3)4Cl2]Cl2 2 3
PtCl4. 3NH3 [Pt(NH3)3Cl3]Cl 1 2
PtCl4. 2NH3 [Pt(NH3)2Cl4] 0 0

ACTIVESITE EDUTECH- 9844532971 89


ACTIVE SITE
a
EDUTECH Co-ordination Compounds
Determination of the structure of a complex on the basis of Werner’s theory:
Taking the example of complexes formed between CoCl3 and NH3, the ionizable chloride ions are found by
precipitation with AgNO3. These are linked by primary valencies. The remaining Cl atoms and NH3
molecules are present around the central metal atom in such directions that the repulsions between them
are minimum thereby giving a definite shape. These atoms and groups are linked by secondary valencies.
Representing the primary valencies by dotted lines and secondary valencies by solid lines, the structures of
the different complexes formed between CoCl3 and NH3 are given below:
NH3 Cl Cl Nh3
H3N NH3 H3N NH3 H3N NH3 Cl Cl
Cl
Cl Co H3N Co NH3 Co Cl Co
Cl
H3N NH3 Cl Cl H3N NH3 H3N NH3
NH3 NH3 Cl Cl
[Co(NH3)6]Cl3 [CoCl(NH3)5]Cl2 [CoCl2(NH3)4]Cl [CoCl3(NH3)3]
The solid and dotted lines together indicate the Cl atom satisfying both primary and secondary valency.
Some important observations: On the basis of his studies, Werner put forward the following important
observations:
(i) The complex compounds do not retain any of the properties of the central metal ion or of the ligands.
Thus, a solution of [Co (NH3)6] Cl3 does not give any reaction of Co3+ ion or of NH3. However, it gives
the reactions of Cl- ions which have not entered into complex formation. Thus, the addition of AgNO 3
will precipitate all the chloride ions present as counter ions.
(ii) The chloride ions which enter into coordination lose their ionic character. These are no longer chloride
ions as their charges has been utilized in lowering the number of positive charges on the complex ion.
These chloride ions can no longer be precipitated on the addition of silver nitrate solution.
(iii) It is also possible to have complexes in which some of the ligands may be neutral molecules and the
rest may be anions. Such complexes are called mixed complexes or heteroleptic complexes. Thus, Co 3+
ion with coordination number 6 may have six NH3 molecules as ligands or five NH3 molecules and one
Cl- ion as ligands and so on.

Limitations of Werner’s Theory:


Werner was the first to explain the bonding features in complex compounds. However, this theory could
not explain the following queries:
(i) Why the complex forming tendency is limited to a few elements only?
(ii) Why do bonds in the co-ordination complexes are of directional nature?
(iii) Why do certain complexes are of magnetic nature and show geometrical and optical isomerism?
This theory at present has lost its relevance and has been replaced by more exhaustive theories.

ACTIVESITE EDUTECH- 9844532971 90


ACTIVE SITE
a
EDUTECH Co-ordination Compounds
Solved Examples
Example 1. Match the pairs of complexes listed in column-I with the method(s) used for their
differentiation listed in column-II.
Column – I Column–II
(1) [Cr(H2O)6]Cl3 and Cr(H2O)5Cl] Cl2.H2O (p) Only neutral molecule(s) satisfiy(es) all
secondary valences in both complexes.

(2) [Co(NH3)5Br]SO4 and [Co(NH3)5SO4]Br (q) Negative ion(s) satisfiy(es) both primary as
well as secondary valences in both complexes.
(3) [Co(NH3)5Cl]Cl2 and [Co(NH3)6]Cl3 (r) Both complexes give same number of ions in
aqueous solution.
(4) [Cu(H2O)4]SO4.H2O and [Cu(H2O)6](NO3)2 (s) Both complexes give white precipitate with
AgNO3.
Ans. (1) – s ; (2) – q, r ; (3) – s ; (4) – p
Sol. (1) [Cr(H2O)6]Cl3 (aq) [Cr(H2O)6]3+ (aq) + 3Cl– (aq)
[Cr(H2O)5Cl] Cl2.H2O (aq) [Cr(H2O)5Cl]2+ (aq) + 2Cl– (aq)
Ag+ + Cl– —® AgCl ¯ (white)
(2) [Co(NH3)5Br]SO4 (aq) [Co(NH3)5Br]2+ (aq) + SO42– (aq)
1 : 1 electrolyte
[Co(NH3)5SO4]Br (aq) [Co(NH3)5SO4]+ (aq) + Br– (aq)
1 : 1 electrolyte
Br– and SO42– satisfy both primary as well as secondary valences.

(3) [Co(NH3)5Cl]Cl2 (aq) [Co(NH3)5Cl]2+ (aq) + 2Cl– (aq)


[Co(NH3)6]Cl3 (aq) [Co(NH3)6]3+ (aq) + 3Cl– (aq)

(4) [Cu(H2O)4]SO4.H2O (aq) [Cu(H2O)4]2+ (aq) + SO42– (aq)


[Cu(H2O)6](NO3)2 (aq) [Cu(H2O)6]2+ (aq) + 2NO3– (aq)

Example 2. Which of the following has the highest molar conductivity in solution ?
(A) [Pt(NH3)6]Cl4 (B) [PtCl(NH3)5]Cl3 (C) [PtCl2(NH3)4 ]Cl2 (D) [PtCl3(NH3)3]Cl
Sol. (A) [Pt(NH3)6]Cl4 [Pt(NH3)6]4+ (aq) + 4Cl– (aq).
(B) [PtCl(NH3)5]Cl3 [PtCl(NH3)5]3+ (aq) + 3Cl– (aq).
(C) [PtCl2(NH3)4 ]Cl2 [PtCl2(NH3)4 ]2+ (aq) + 2Cl– (aq).
(D) [PtCl3(NH3)3]Cl [PtCl3(NH3)3]+ (aq) + Cl– (aq).
Since [Pt(NH3)6]Cl4 gives maximum number of ions in the solution and as number of ions increases molar
conductivity in solution increases.

ACTIVESITE EDUTECH- 9844532971 91


ACTIVE SITE
a
EDUTECH Co-ordination Compounds
CLASS EXERCISE
1] Primary and secondary valency of Pt in [Pt (en)2Cl2] are
a) 4, 4 b) 4, 6 c) 6, 4 d) 2, 6
2] The number of chlorine atoms acting as ligands in the complex [Co(en)2 (H2O)Cl] Cl2 is
a) 1 b) 2 c) 3 d) None of these
3] Which exhibits highest molar conductivity?
a) [Co(NH3)6]Cl3 b) [Co(NH3)5Cl]Cl2 c) [Co(NH3)4Cl2]Cl d) [Co(NH3)3Cl3]
4] The fraction of chlorine precipitated by AgNO3 solution from [Cr(NH3)5Cl]Cl2 is
a) 1/2 b) 2/3 c) 1/3 d) ¼
5] Which of the following statements is incorrect?
a) In K3[Fe(CN)6], the ligand has satisfied only the secondary valency of ferric ion.
b) In K3[Fe(CN)6], the ligand has satisfied both primary and secondary valency of ferric ion.
c) In K4[Fe(CN)6], the ligand has satisfied only the secondary valency of copper.
d) In [Cu(NH3)4]SO4, the ligand has satisfied only the secondary valency of copper.

HOME EXERCISE
1] According to postulates of Werner theory for coordination compounds
a) Primary valency is ionizable b) Secondary valency is ionizable
c) Primary and secondary valencies are non-ionizable d) Only primary valency is non-ionizable
2] The number of ions per mole of the complex CoCl3.4NH3 in aqueous solution will be
a) 3 b) 8 c) 2 d) 4
3] Given the molecular formula of the hexa coordinated complexes (A) CoCl3.6NH3
(B) CoCl3.5NH3 (C) CoCl3.4NH3. If the number of coordinated NH3 molecules in A, Band C respectively
are 6, 5 and 4, primary valency in (A), (B) and (C) are
a) 0, 1, 2 b) 3, 2, 1 c) 6, 5, 4 d) 3, 3, 3
4] Which one of the following has the highest molar conductivity?
a) diamminedichloroplatinum (II) b) tetraamminedichlorocobalt (III) chloride
c) potassium hexacyanoferrate (II) d) hexaaquachromium (III) bromide
e) pentacarbonyliron (0)
5] In the complex in [Fe(C2O4)3]3–, which statement is correct?
a) Fe3+ is a Lewis acid and C2O42–is a lewis base
b) Primary & secondary valencies are 3 and 6 respectively
c) EAN of Fe is 35
d) All are correct
NCERT Test Book Questions:9.1, 9.28
CLASS EXERCISE
1) d 2) a 3) a 4) b 5) a
HOME EXERCISE
1) a 2) c 3) d 4) c 5)d

ACTIVESITE EDUTECH- 9844532971 92


ACTIVE SITE
a
EDUTECH Co-ordination Compounds

Werner was first to describe the structures and the existence of various types of isomerism in coordination
compounds. However, the nature of bonding between central metal atom and ligands in the
coordination sphere has been explained by the three well known theories. These theories are:
1] Valence Bond Theory (VBT)
2] Crystal Field Theory (CFT)
3] Molecular Orbital Theory (MOT)

VALENCE BOND THEORY


Postulates: It was developed by Pauling. The main postulates of the theory are
1. The central metal loses required number of electrons to form ion. The number of electrons lost is the
valency of the resulting cation. In some cases, the metal atom does not lose e̅ s.
2. The central metal atom or ion in the complex makes available number of empty orbitals for the
formation of coordinate bonds with suitable ligands. The number of empty orbitals made available for
this purpose is equal to C.N. of the central metal or ion.
3. The appropriate number of vacant orbitals of the central metal atom or ion hybridise to give same no.
of equivalent hybrid orbitals.
4. Ligands have atleast one  orbital containing a lone pair of electrons. The empty hybrid orbitals of the
central metal atom or ion overlap with filled orbitals of the ligands containing lone pair of electrons.
5. The d-orbitals involved in the hybridization may belong to the inner (n-1) d subshell or the outer
nd-subshell. The complexes formed in these two ways are called inner orbital complexes (low-spin) and
outer orbital complexes. (high spies) respectively.
6. The non-bonding electrons of the metal occupy the inner orbitals. These are arranged in accordance
with Hund’s rule. But under the influence of some strong ligands, there, may be some re-arrangement
of electrons in the atomic orbital (against the Hund’s rule)
7. If the complex contains unpaired electrons, it is paramagnetic is nature, otherwise, it is diamagnetic

Coordination No. Hybridization Geometry

2 sp Linear
3 sp2 Trigonal Planar
4 sp3 Tetrahedral
4 dsp2 Square Planar
5 dsp3 Trigonal bipyramidal
5 sp3d Square pyramidal
6 sp3d2 (or) d2sp3 Octahedral

ACTIVESITE EDUTECH- 9844532971 93


ACTIVE SITE
a
EDUTECH Co-ordination Compounds

Application of VBT on co-ordination complexes:


a. Complex with Co-ordination Number 4:
Tetra co-ordinate complexes may have either tetrahedral or square planer geometry depending upon
the nature of orbital’s involved in hybridization. If one ns and three np orbital’s are involved in bonding
them geometry will be tetrahedral and hybridization sp 3.If (n - 1) d, ns and two np are involved in
bonding them geometry will be square planar and hybridization dsp 2. Tetra co-ordinate complexes are
common with Ni (II), Cu (II), Pt (II), Pd (II)etc.

i) Tetrahedral Complexes:
1) Ni(CO)4: In Ni(CO)4 Ni has zero oxidation state & exist as Ni (0). Four ligands (CO) are attached to central
metal atom Ni and require four orbitals. The electronic configuration in Ni(CO)4 can be written as.
8 2
28Ni – atom = 3d , 4s
In presence of four CO ligands, which is strong ligand, the electrons pair up against Hund’s rule.
Ni – atom in Ni(CO)4 = 3d10, 4s0

Explanation:
Four sp3 hybrid orbitals are arranged tetrahedral so it is tetrahedral complex. All the electrons are paired
so it is diamagnetic.

ACTIVESITE EDUTECH- 9844532971 94


ACTIVE SITE
a
EDUTECH Co-ordination Compounds
2) [NiCl4]2–: In [NiCl4]2–, Ni is in (II) oxidation state and its electronic arrangement is as follows:

Explanation:
1) Cl- is a weal field ligand, so it cannot pair up the 3d – electrons against Hund’s rule.
2) Geometry is tetrahedral and it will be paramagnetic in nature because these are two unpaired electrons.

(3) [CuCl4]2-: In [CuCl4]2–, Cu is in (II) oxidation state and electronic arrangement can be represented as:

Explanation:
1) Geometry is tetrahedral as four sp3 hybrid orbitals are arranged tetrahedrally.
2) Complex is paramagnetic due to one unpaired electron.

ACTIVESITE EDUTECH- 9844532971 95


ACTIVE SITE
a
EDUTECH Co-ordination Compounds
4) [Zn(NH3)4]2+: In this complex, Zn exists as Zn2+ ion and electronic arrangement is as follows:

Explanation: complex is tetrahedral and diamagnetic in nature because all the electrons are paired.

5) [Zn(OH)4]2- :

b) Square Planar Complexes:


i) [Ni(CN)4]2–: In it, Ni is in ( II) oxidation state and electronic arrangement is as follows:

Explanation:
1] CN- is strong ligand and so it pairs up 3d – electrons against Hund’s rule. The d – orbital thus made
vacant takes part in hybridization.
2] Geometry is square planar, as four dsp2 hybrid orbitals are arranged in this manner.

ACTIVESITE EDUTECH- 9844532971 96


ACTIVE SITE
a
EDUTECH Co-ordination Compounds
3] Complex is diamagnetic because all the electrons are paired.
4] Because the complex makes use of inner d- orbital, so it is known as inner orbital or low spin or hyper
ligated or spin paired complex.

ii) Complexes with Co-ordination Number 6:


Hexa co-ordinated complexes may be of two types, inner orbital complexes and outer orbital complexes.
They have octahedral geometry.
Table: Comparison between inner and outer orbital complexes.
Inner orbital octahedral complexes Outer orbital octahedral complexes
1. Formed by d2sp3 hybridization 1. Formed by sp3 d2 hybridization involving ns,
2
involving (n – 1) dz , (n − 1)dx2 −y2 , ns, three np, (n − 1)dx2 −y2 and (n – 1)dz2 orbitals
npx, npy and npz orbitals.
2. They have comparatively lesser 2. Number of unpaired electrons comparatively
number of unpaired electrons, so also larger and so have high magnetic moment.
known as low spin complexes. Also, known as high spin complexes.
3. Formed by strong ligands. 3. Formed by weak ligands.

Some examples of octahedral complexes are as follows.


a) Inner Orbital Complexes:
1) [Fe(CN)6]4-: Hexacyanoferrate (II) ion: In it, Fe is in (II) oxidation state and electronic configuration is as
follows:

Explanation: It is low spin octahedral complex and is diamagnetic in nature. CN –is strong ligand so d –
electrons get paired up.

2) [Fe (CN6)]3–: Here Fe– atom is in (III) oxidation state.

Explanation: It is low spin octahedral complex and is weakly paramagnetic.

ACTIVESITE EDUTECH- 9844532971 97


ACTIVE SITE
a
EDUTECH Co-ordination Compounds
3) [Cr(NH3)6]3+: Hexaammine chromium (III) ion: Here Cr is in (III) oxidation state.

Explanation: It is low spin octahedral complex and is paramagnetic.

4) [Co(ox)3]3–: tris (oxalate) cobalt (III) ion: Here cobalt is in (III) oxidation state.

Explanation: It is inner orbital octahedral complex, diamagnetic in nature as all the electrons are paired.

b) Outer Orbital Complexes:


1) [CoF6]3–: Hexafluoro cobaltate (III) ion: Cobalt is in (III) oxidation state.

Explanation: F– is weak ligand and can’t pair up the d – electrons so complex is high spin. Because outer
4d– orbitals take part in hybridization so it is outer orbital octahedral complex. Due to four unpaired
electrons, it is highly paramagnetic.

ACTIVESITE EDUTECH- 9844532971 98


ACTIVE SITE
a
EDUTECH Co-ordination Compounds
2) [Fe(H2O)6]3+: Iron is in (III) oxidation state.

Explanation: It is outer octahedral complex and is highly paramagnetic.

Drawbacks of Valence Bond Theory:


Although Valence Bond Theory successfully explains the geometry (or the shapes) and magnetic behavior
of the coordination compounds but it has a number of shortcomings. A few of these are as follows:
i) It cannot explain why some complexes of a metal ion in a particular oxidation state are low spin, i.e.,
inner orbital complexes while some other complexes of the same metal ion in the same oxidation
state are high spin, i.e., outer orbital complexes, For example, [Co(NH3)6]3+ is a low spin, i.e., inner
orbital complex (involving d2 sp3 hybridisation) while [CoF6]3- is a high spin, i.e., outer orbital complex
(involving sp3 d2 hybridisation). As a result, the former is diamagnetic while the latter is
paramagnetic.
ii) The magnetic behavior predicted by Valence Bond Theory is sometimes misleading. For example, the
square planar complex of Ni (II) should be diamagnetic. This is because, for dsp2 hybridisation, the
two unpaired electrons of Ni2+ pair up as follows:
3d 4s 4p
2+
Ni ion
(Ground state)

Square planar diamagnetic


2+
complex of Ni
2
dsp Hybridisation
Thus, we may wrongly assume that all the square planar complexes of Ni (II) should be diamagnetic.
This is actually not so. Some square planar complexes of Ni (II) are known which are paramagnetic. In
their cases, we assume that one of the unpaired 3d electron is excited to 4p orbital so that two
unpaired electrons are present as shown below:
3d 4s 4p
2+
Ni ion
(Excited state)

2
dsp Hybridisation
Valence bond theory cannot explain as to why excitation occurs in one square planar complex but
not in the other. Similarly, in the case of the complex, [Cu(NH3)4]2+, it cannot explain why excitation
of one 3d electron into 4p orbital takes place.

ACTIVESITE EDUTECH- 9844532971 99


ACTIVE SITE
a
EDUTECH Co-ordination Compounds
iii)It could not give any satisfactory explanation for the colour of the complexes, i.e., it fails to explain
the absorption spectra of coordination compounds.
iv) It does not give an exact explanation of thermodynamic or kinetic stabilities of coordination
compounds.
v) It does not distinguish between weak and strong ligands.
vi) In the mathematical treatment, it involves a number of assumptions.
vii) In a number of cases, the experimentally observed values of magnetic moment do not exactly
coincide with the values calculated from valence bond theory.

ACTIVESITE EDUTECH- 9844532971 100


ACTIVE SITE
a
EDUTECH Co-ordination Compounds

ACTIVESITE EDUTECH- 9844532971 101


ACTIVE SITE
a
EDUTECH Co-ordination Compounds

ACTIVESITE EDUTECH- 9844532971 102


ACTIVE SITE
a
EDUTECH Co-ordination Compounds

Solved Examples
Example 1. Sodium nitroprusside, Na2[Fe(CN)5NO] is a diamagnetic substance and a important laboratory
reagent for the testing of sulphide ions. The metal involved in the complexation in this is
present in which of the following hybridization state:
(1) sp3 (2) dsp2 (3) d2sp3 (4) sp3d2

Sol. Fe2+, [Ar] 3d6

[Fe(CN)5NO]2–

Example 2. When the d-orbitals involved in the hybridisation are inner (n–1) d-orbitals ; the complex
formed are referred to as :
(1) high spin-complexes (2) low spin complexes
(3) zero spin complexes (4) positive spin complexes
Sol. When in the formation of complex the inner d-orbital, (n – 1)d is used in hybridization, the
complex is called an inner orbital or low spin or spin paired complex.
When in the formation of complex the outer d-orbital, nd is used in hybridization, the complex
is called a outer orbital or high spin or spin free complex.

Example 3. Explain [Co(NH3)6]3+ is an inner orbital complex whereas [Ni(NH3)6]2+ is an outer orbital
complex.
Sol. In the presence of NH3, the 3d electrons pair up leaving two d-orbitals empty to be involved in
d2sp3 hybridisation forming inner orbital complex in case of [Co(NH3)6]3+.
In Ni(NH3)62+, Ni is in +2 oxidation state and has d8 configuration, the hybridization involved is
sp3d2 forming outer orbital complex.

ACTIVESITE EDUTECH- 9844532971 103


ACTIVE SITE
a
EDUTECH Co-ordination Compounds
CLASS EXERCISE
1] Considering H2O as a weak field ligand, the number of unpaired electrons in [Mn(H2O)6]2+ will be
(At. No. of Mn = 25)
a) two b) three c) four d) five
2] In the complex [MLx]n+, the central metal ion has five unpaired electrons and L is a weak ligand. What
will be the number of unpaired electrons in this complex ion?
a) 0 b) 1 c) 5 d) Cannot be predicted.
3] The complex ion having square planar arrangement with coordination number 4 is
a) [Ni(Cl)4]2- b) [CoCl4]2- c) [Ni(CN)4]2- d) [MnO4]-
4] [Fe(CN)6]3-ion has magnetic moment of 1.73 BM while [Fe(H2O)6]+3 has a magnetic moment of 5.92
BM. Thus, hybridization of Fe in both the complexes are respectively.
a) d2sp3, sp3d2 b) sp3d2, d2sp3 c) d2sp3, d2sp3 d) sp3d2, sp3d2
5] Which statement is incorrect?
a) Ni(CO)4- tetrahedral, paramagnetic b) Ni(CN)42– - square planar, diamagnetic
c) Cu(CN)42– -square planar, paramagnetic d) NiCl42– -tetrahedral, paramagnetic

HOME EXERCISE
1] How many unpaired electrons are present in the central metal ion of [CoCl4]2–?
a) 2 b) 3 c) 4 d) 0
2] Among the following ions, which one has the highest unpaired electrons?
a) [Cr(H2O)6]3+ b) [Zn(H2O)6]2+ c) [Fe(H2O)6]2+ d) [Ni(H2O)6]3+
3] Among [Fe(H2O)6]3+, [Fe(CN)6]3–, [Fe(Cl6)]3– species, the hybridization state of the Fe atom are,
respectively.
a) d2sp3, d2sp3, sp3d2 b) sp3d2, d2sp3, d2sp3
c) sp3d2, d2sp3, sp3d2 d) None of these
4] What is the magnetic moment of K3[FeF6]?
a) 3.87 BM b) 4.89 BM c) 5.91BM d) 6.92 BM
5] The value of ‘spin only’ magnetic moment for one of the following configuration is 2.84 BM the
correct one
a) d4 (in weak ligand field) b) d4 ( in strong ligand field)
c) d2 ( in weak as well as in strong field) d) d5 (in strong ligand field)
NCERT Test Book Questions:9.15, 9.19, 9.20, 9.21, 9.2, 9.24, 9.29

CLASS EXERCISE
1) d 2) c 3) c 4) a 5) a
HOME EXERCISE
1) b 2) c 3) c 4) c 5) b

ACTIVESITE EDUTECH- 9844532971 104


ACTIVE SITE
a
EDUTECH Co-ordination Compounds
CRYSTAL FIELD THEORY
CFT was originally proposed to ionic crystals to explain their optical properties and is therefore called CFT.
Valence Bond Theory represents the ligand metal bond as covalent. VBT is not adequate to explain
properties of complexes such as color & Magnetism.
Postulates: The main points of the theory are:
1) According to CFT, the bonding in complex ions is purely electrostatic. This theory regards the ligand
atoms of ionic ligands (F–, Cl–, CN–) as negative point charges and if the ligand molecules are neutral,
these are regarded as point dipoles, negative end pointing towards the central metal ion.

2) The complex is regarded as a combination of a central metal ion surrounded by ligands. The
arrangement of ligands around the central metal ion is such a way to minimize the repulsions
between them.
3) Interactions between the central metal ion and the ligands are of two types.
a) The attractive forces arise due to the positive metal ion and the negatively charged ligands or the
negative end of the polar neutral molecule.
Ex: In [Fe(CN)6]3–, the interactions are between Fe3+ and negatively charged CN–ion, whereas in
[Cr(NH3)6]3+, the interactions are between Cr3+ and negative end of NH3 molecule.
b) The repulsive forces arise between the d electrons of the metal and lone pair on the ligands.
4) In a free transition metal or ion, there are 5 d-orbitals (dxy, dyz, dzx,dx2 −y2 &dz2). The
5 d-orbitals are divided into two sets depending on the nature of their orientation in space.
a) The three d-orbitals (dxy, dyz, dz2) which orient in the region between the coordinate. Axes are
designated as t2g orbitals. These are three-fold degenerate and are non-axial orbitals.
b) The other two d-orbitals (dx2−y2 &dz2) which orient along the axes are called ‘eg’ orbitals. These
are two-fold degenerate and are axial orbitals.
5) In a free metal atom or ion, all the five d-orbitals are degenerate. When ligands approach the metal
atom/ion, due to repulsions between the d-electrons of the metal and lone pair of ligands, the
energy of the d-orbitals will rise and still they remain degenerate. Due to the difference in the
orientation of d-orbitals, they experience different repulsive interactions from the ligand. Hence the
d-orbitals split into two levels of different energy.

This splitting of the degenerate d-orbitals under the influence of approaching ligands into different energy
sets is called crystal field splitting.

ACTIVESITE EDUTECH- 9844532971 105


ACTIVE SITE
a
EDUTECH Co-ordination Compounds
Crystal field splitting in octahedral complexes:
In octahedral complex, the central metal ion is placed at the center of the octahedron and is surrounded
by six ligands which reside at the six corners of the octahedron. If all the ligands approaching the metal
cation are at an equal distance from each of d-orbitals, the energy of each of the five d-orbitals increases
by the same amount, and still, they are degenerate.

Since the lobes of two eg orbitals lie in the path of approaching ligands, the electrons in these orbitals
experience greater force of repulsion than those in t 2g orbitals whose lobes are directed in space between
the path of ligands i.e., energy of eg orbitals is increased while that of t2g orbitals is decreased.

ACTIVESITE EDUTECH- 9844532971 106


ACTIVE SITE
a
EDUTECH Co-ordination Compounds

The difference of energy between the two sets of degenerate orbitals as a result of crystal field splitting is
called crystal field splitting energy or crystal field stabilizing energy (CFSE) and is represented by0(o
stands for octahedral) or 10Dq

Thus 3 orbitals lie at an energy that is 0.40 or 4Dq below the average d-orbital energy and 2 orbitals lie at
an energy 0.60or 6 Dq above the average energy.

ACTIVESITE EDUTECH- 9844532971 107


ACTIVE SITE
a
EDUTECH Co-ordination Compounds
Crystal field splitting in tetrahedral complexes:
In tetrahedral complexes the t2g orbitals are close to the approaching ligands. As a result of this,
t2gelectrons experience more repulsions than eg electrons. The energy of t2gorbitals increases more than eg
orbitals. The energy gap between the two sets of orbitals is less and is designated as t.
4
t = q0. (This energy is so small that it is not sufficient to force the pairing of electrons; tetrahedral
complexes have high spin configuration)

ACTIVESITE EDUTECH- 9844532971 108


ACTIVE SITE
a
EDUTECH Co-ordination Compounds

ACTIVESITE EDUTECH- 9844532971 109


ACTIVE SITE
a
EDUTECH Co-ordination Compounds

ACTIVESITE EDUTECH- 9844532971 110


ACTIVE SITE
a
EDUTECH Co-ordination Compounds

Spectrochemical series:
For any given metal cation, the magnitude of crystal field splitting energy depends on the nature of the
ligands. The greater the ease with which the ligand can approach the metal ion, the greater will be the
crystal field splitting caused by it. The ligands which affect only a small degree of crystal field splitting are
termed weak field ligands while those which affect a large splitting are called strong field ligands.
When the ligands are arranged in order of the magnitude of crystal field splitting, the arrangement, thus,
obtained is called spectrochemical series.

From the above arrangement it is clear that ligands before H2O such as I–, NO3− , OH–, etc., are weak field
ligands while the ligands after H2O such asNO2− , CN–, CO, etc., are strong field ligands. Stronger field ligands
cause greater crystal splitting i.e., 0, value for octahedral complex is high.

ACTIVESITE EDUTECH- 9844532971 111


ACTIVE SITE
a
EDUTECH Co-ordination Compounds

ACTIVESITE EDUTECH- 9844532971 112


ACTIVE SITE
a
EDUTECH Co-ordination Compounds
Factors Affecting the CFSE:

1. Nature of ligand: The value of  depends upon nature ofligands. Ligands which causes only a small
degree of crystal field splitting are termed as weak field ligands while those ligands which cause large
splitting are called strong field ligands. The common ligands can be arranged in ascending order of
CFSE caused by them. This series remains practically constant for different metals and is known as
spectrochemical series. It is an experimentally determined series. The order is difficult to explain due
to involvement of both σ and π bonding. Some ligands in spectrochemical series are given below:

Generally, the order depends upon the donor atom and is in the following order:
C - donor> N – donor > O – donor > Halogen donor

Weak field ligands lead to a smaller Strong field ligands lead to a larger
splitting energy. splitting energy.

ACTIVESITE EDUTECH- 9844532971 113


ACTIVE SITE
a
EDUTECH Co-ordination Compounds
2. Geometry of the complex: t is approximately 4/9 times to 0. The lower value of tis due to lesser
number of ligands in tetrahedral complex. Also, in tetrahedral complexes the orbitals do not point
towards axes, so there is less interaction.
3. Oxidation state of metal ion: Higher is the charge on central metal atom (or oxidation state), more is
the CFSE.
e.g., 0 for [Fe (H2O)6]2+,0 for [Co (H2O)6]2+ is greater than [Co(H2O)6]2+,0 for [V(H2O)6]2+ is greater
than [Cr (H2O)6 ]3+.
4. Nature of metal ion: The value of CFSE also depends upon the transition series to which metal
belongs and the order is 3d < 4d < 5d. The value ofincreases by 30% to 50% for 3d to 4d series and
form 4d to 5d series. Thus, metals of 4d and 5d series have more tendency to form low spin
complexes. e.g.,
CFSE for given complexes follow the order: [Co (NH3)6]3+< [Rh (NH3)6]3+< [Ir(NH3)6]3+
When two metal ions have same charge but different number of d – electrons, the magnitude of 0
decreases with increase in the number of d- electrons in the central metal atom. e.g.,
 0 for [Co(H2O)6]2+ is greater than 0 for [Ni(H2O)6]2+ becauseCo2+ has 3d7 configuration while Ni2+ has
3d8 configuration.

Applications of CFT
1. Electronic arrangement of metal ion in complexes: Before studying the electronic configurations, we
must also be familiar with another term mean pairing energy (P).
Mean pairing energy (P) is the energy which is required to pair up two electrons against electron –
electron repulsion in same orbital against Hund’s rule.
Let us consider metal ions in octahedral complexes in which d. orbitals split into t 2g and eg. For d1, d2
and d3 ion the arrangement of electrons has only one choice as shown below:

But there are two possibilities for d4 ion.


i) The electrons remain unpaired. Here 4th electron enters in e g orbital. This situation is weal field high
spin situation. It occurs when 0< P.
ii) The 4th electron pair up in tg orbital and eg remain empty. This situation is strong field, low spin
situation. In this situation eg set will remain empty. It occurs when0> P.

Weak field and strong field situations in d4 configuration


Thus, filling of eg orbitals in weak field situation starts from 4th electron while in strong field situation
it starts from 7th electron.

ACTIVESITE EDUTECH- 9844532971 114


ACTIVE SITE
a
EDUTECH Co-ordination Compounds
The electron configurations of d5 – d10 ions are shown in following Table

Thus, it is clear that strong field ligands from low spin complexes while weak field complexes form
high spin complexes.
2. Magnetic character of complexes: Complexes containing unpaired electrons are paramagnetic. On
the other hand, if all the electrons are paired then complex will be diamagnetic and magnetic
moment
(𝐁 ) = √𝐧(𝐧 + 𝟐) B. M., where n is number of unpaired electrons.
Magnetic moment of co-ordination compounds is determined experimentally and these data provide
information to study the nature of co-ordination entities. These measurements are called magnetic
susceptibility measurements. e.g.,
[Fe(H2O)6]2+ is paramagnetic while [Fe(CN)6]4- is diamagnetic. Above observation can be explained on
the basis of electronic configurations of F2+ in [Fe (H2O6)]2+ and [Fe (CN)6]4-.
H2 O is Weak field ligand CN- is strong field ligand. So [Fe(CN)6]4-is outer orbital high spin complex.
The configuration of Fe2+ in both is given below:

ACTIVESITE EDUTECH- 9844532971 115


ACTIVE SITE
a
EDUTECH Co-ordination Compounds

Solved Examples
Example 1. How would you account for the following?
(a) [Fe(CN)6]3– is weakly paramagnetic while [Fe(CN)6]4– is diamagnetic
(b) Ni(CO)4 possesses tetrahedral geometry while [Ni(CN)4]2– is square planar
(c) [Cr(NH3)6]3+ is paramagnetic while [Ni(CN)4]2– is diamagnetic. Explain why?
Sol. (a) [Fe(CN)6]3– involves d2sp3 hybridization.
3d 4s 4p




Fe3+, [Ar] 3d5

[Fe(CN)6]3–

One d-orbital is singly occupied, hence it is weakly paramagnetic in nature.


[Fe(CN)6]4– involves also d2sp3 hybridization but it has Fe2+ ion as central ion.
3d 4s 4p
Fe2+, [Ar] 3d6





[Fe(CN)6]4–
All electrons are paired, hence it is diamagnetic in nature.

(b) In the formation of Ni(CO)4 nickel atom undergoes sp3 hybridization, hence it is tetrahedral
in shape.

Ni(0), [Ar] 3d8

Ni(CO)4

In the formation of [Ni(CN)4]2–, Ni2+ ion undergoes dsp2 hybridization, hence it is square planar
in shape.

Ni2+, [Ar] 3d8





[Ni(CN)4]2–



Rearrangement
dsp2 hybridization

ACTIVESITE EDUTECH- 9844532971 116


ACTIVE SITE
a
EDUTECH Co-ordination Compounds
(c) Chromium in +3 oxidation i.e., [Ar]18 3d3 electron configuration always have three unpaired
electrons with strong as well as with weak field ligands. So,
[Cr(NH3)6]3+

d2sp3 hybrid orbital


[Ni(CN)4]2–

dsp2 hybrid orbitals


It is diamagnetic as all electrons are paired in 3d8 configuration of Ni2+.

3. Colour of complexes: In complexes d – orbitals split in two sets t2g and eg. which have different
energies. The difference in energies of t2g and eg lies in visible region and i.e., why transition metal
complexes may absorb in visible region of spectra.

It makes them coloured and colour being complementary to colour absorbed. This transition involves t 2g
and eg sets of d- orbitals and is called as d-d transition. Thus d-d transition is responsible for colour of
transition metal complexes
Same chemical composition, same # of groups of same types attached to same metal. What made the two
different colors?

ACTIVESITE EDUTECH- 9844532971 117


ACTIVE SITE
a
EDUTECH Co-ordination Compounds

ACTIVESITE EDUTECH- 9844532971 118


ACTIVE SITE
a
EDUTECH Co-ordination Compounds

Limitations of CFT:
1. Treatment of ligands as point charges is objectionable. If this assumption is true then anionic ligands
must be stronger than neutral ligands but OH is weaker ligand than H2O and NH3.
2. It assumes electrostatic interaction only, but in complexes of metals in zero oxidation state this
assumption fails and covalent bonding must be there.
3. CFT considers the d- orbitals of metal ion only and there is no consideration of other orbitals. It also
does not consider the - bonding in complexes.

CLASS EXERCISE
1] In an octahedral crystal field, the eg orbitals are
a) Raised in energy by 0.40 b) Lowered in energy by 0.40
c) Raised in energy by 0.60 d) Lowered in energy by 0.60
2]

The above splitting of d-orbitals takes place in the formation of


a) octahedral complexes b) tetrahedral complexes
c) square-planar complexes d) both tetrahedral and octahedral complexes
3] Magnetic moment is given by
a) μB = √n(n + 2)BM b) μB = √n(n + 1)BM
c) μB = √4n + 1BM d) None

ACTIVESITE EDUTECH- 9844532971 119


ACTIVE SITE
a
EDUTECH Co-ordination Compounds
4] Strong field ligands form A and while weak field ligands form B. A and B are:
a) Both high spin complexes
b) Both low spin complexes
c) Low spin and high spin complexes
d) High spin and low spin complexes
5] The difference in energies of t 2g and eg lies in
a) Infrared region b) U.V region c) Microwave d) Visible region
6] Anionic ligands must be A than neutral ligands (assumption of CFT) but OH is B ligand than H2 O and
NH3 ; A and B are
a) Strong ; weak b) Weak ; strong c) Both strong d) Both weak

HOME EXERCISE
1] Which of the following ligand has lowest 0value?
a) CN– b) CO c) F– d) NH3
2] The correct order for the wavelength of absorption in the visible region is
a) [Ni(NO2)6]4-<[Ni(NH3)6]2+< [Ni(H2O)6]2+
b) [Ni(NH3)6]2+<[Ni(H2O)6]2+<[Ni(NO2)6]4-
c) [Ni(H2O)6]2+<[Ni(NH3)6]2+<[Ni(NO2)6]4-
d) [Ni(NO2)6]4- <[Ni(H2O)6]2+<[Ni(NH3)6)]2+
3] The number of d − orbitals in the free transition metal or ion is/are
a) 4 b) 3 c)5 d) 1
4] The attractive forces are due to the ___ and ___ of polar neutral molecules
a) Positive metal ion, negatively charged ligand
b) Positive metal ion and positively charged ligand
c) Negative metal ion, negatively charged ligand
d) Any one of them
5] Three fold generate and are non − axial orbitals are
a) eg orbitals b) t 2g orbitals c) Both a and b d) a or b
6] Two fold generate and axial orbital which orient along ___ are eg
a) dx2 −y2 and d2z b) d2z c) dxy , dyz d) dxy , dyz , d2z
NCERT Test Book Questions: 9.16, 9.17, 9.18, 9.32

CLASS EXERCISE
1) c 2) a 3) a 4) c 5) d 6) a
HOME EXERCISE
1) c 2) a 3) c 4) a 5) b 6) a

ACTIVESITE EDUTECH- 9844532971 120


ACTIVE SITE
a
EDUTECH Co-ordination Compounds

STABILITY OF COORDINATION COMPOUNDS IN SOLUTION


A complex ion almost does not dissociate into its component parts in the aqueous solution. However, such
a dissociation does take place through it may be small. The extent of dissociation and hence the
thermodynamic stability of the complex ion depends upon the strength of the metal-ligand bond. Stronger
is the metal ligand bond, less is the dissociation in the solution and hence greater is the stability. The
stability of a complex may also be defined as a measure of resistance to the replacement of a ligand by
some other ligand. As the dissociation involves an equilibrium between the dissociated components (ions
or molecules) and the undissociated complex, the stability of a complex in solution can be expressed in
terms of equilibrium constant of the dissociation equilibrium. This constant is called the instability
constant or dissociation constant. For example, for the complex, [Cu (NH3)4]2+ in solution, we have
[Cu(NH3)4]2+⇌ Cu2+ + 4NH3
[Cu2+ ][NH3 ]4
Instability constant,Ki =[Cu(NH 2+
3 )4 ]
If instead of dissociation, we talk of the formation of a complex ion in the solution, the reaction will be
reverse of the above reaction, i.e., we haveCu2+ + 4NH3⇌[ Cu (NH3)4]2+
The equilibrium constant for this reaction is, therefore, called the stability constant. Representing it by ,
[Cu(NH ) ]2+
we have Stability constant,  = [Cu2+ ][NH
3 4
4
3]

Thus, stability constant is the reciprocal of the instability constant ( = 1/Ki).


A general expression for the stability constant may be derived as follows:
The reaction between metal and ligand may be represented as M a+ + nLx -⇌ MLb+ n
Where a +, x – and b + are the charges on the metal, the ligand and the complex ion respectively. In
accordance with the charge balance, (a +) + n (x -) = (b +)
[MLb+ ]
Stability constant, n = [Ma+][Ln x−]n
The numerical value of the stability constant is a measure of stability of the complex in solution. Greater
the magnitude of the stability constant, more stable is the complex.
As most of the complexes are formed from aqueous solution, the reaction involves replacement of H 2O
molecules from the aqua complex by the ligands, i.e., we have the reaction:
[M (H2O)n] + n L ⇌[MLn] + n H2O
[MLn ]
Hence, expression for the stability constant can be written asn= [M(H n
2 O)n ][L]
[H2O] has been ignored as usual because its concentration remains almost constant. The above reaction is
believed to take place in steps with a suitable formation constant for each step, as given below:
[ML (H2 O)n−1 ]
[M (H2O)n] + L⇌ [ML (H2O)n-1] + H2O,K1 = [M(H
2 O)n ][L]
[ML2 (H2 O)n−2 ]
[ML(H2O)n-1] + L⇌ [ML2(H2O)n-2] +H2O, K2 =[ML(H and so on till finally, we have
2 O)n−1 ][L]
[MLn ]
[MLn-1 (H2O)] + L⇌[MLn] + H2O,Kn = [MLn−1 (H2 O)][L]
Adding all these steps, we get the overall reaction and overall stability constant will be
n = K1 × K2× K3 ….. × Kn

ACTIVESITE EDUTECH- 9844532971 121


ACTIVE SITE
a
EDUTECH Co-ordination Compounds
For example, ignoring water molecules, the formation of [Cu (NH3)4]2+ takes place in four steps as follows:
[Cu(NH )]2+
3
Cu2+ + NH3⇌ [Cu(NH3)]2+, K1= [Cu2+][NH
3]
[Cu(NH3 )2 ]2+
[Cu (NH3)]2+ + NH3⇌ [Cu (NH3)2]2+, K2 =[Cu(NH 2+ [NH
and so on where K1, K2 etc. are stepwise stability
3 )] 3]
constants and the overall stability constant will be
[Cu(NH ) ]2+
n = K1 × K2 × K3 × K4 = [Cu2+][NH
3 4
4
3]
As more and more ammine groups are added, the successive stability constants decrease. For example, in
the above case,
Log K1 = 4.0,log K2 = 3.2, log K3 = 2.7,log K4 = 2.0
log4 = 4.0 + 3.2 + 2.7 + 2.0 = 11.9
Similarly, in case of [Cd (NH3)4]2+,log4 = 2.6 + 2.1 + 1.4 + 0.9 = 7.0

Value of logis a measure of the stability of the complex: Thus, as [Cu(NH3)4]2+ has greater log  value
than for [Cd(NH3)4]2+, the former complex is more stable than the latter. That is why in Group II of
qualitative analysis, when H2S gas is passed through the solution containing [Cu(NH3)4]2+ complex, no
precipitate of CuS is formed but [Cd (NH3)4]2+ forms a precipitate of CdS.

Factors affecting the stability of a complex ion: Stability of a complex ion depends upon following factors:

i) Charge on the central metal ion: Greater the charge on the central metal ion, greater is the stability
of the complex.
Thus, complexes with trivalent central metal ion are more stable than those with divalent central
metal ion. It has been observed by Irving and Williams that irrespective of the nature of the ligand,
the stability of the divalent metal ions of the first transition series is in the following order
Mn (II) < Fe (II) < Co (II) < Ni (II) < Cu (II) < Zn (II)
This order is called Irving – Williams order.

ACTIVESITE EDUTECH- 9844532971 122


ACTIVE SITE
a
EDUTECH Co-ordination Compounds
ii) Nature of the metal ion: Metal ions are grouped into two classes:
‘Class a’ acceptors are the metals belonging to groups 1 and 2, earlier members of the transition
series, i.e., groups 3 to 6 and inner transition elements. They form stable complexes when the donor
atoms of the ligands are N, O or F.
‘Class b’ acceptors are the transition metals mostly after group 6 which have relatively filled d-
orbitals (e.g., Rh, Pd, Ag, Au, Hg, etc.) They form stable complexes when the donor atoms of the
ligands are heavier members of N, O and F family.
iii) Basic nature of the ligand: Greater the basic strength of the ligand, greater is the stability of the
complex.
iv) Presence of chelate rings: Formation of chelate rings increases the stability of the complex. The
stabilization due to chelation is called chelate effect. It is found to be maximum for the 5- and 6-
membered rings.

BONDING IN METAL CARBONYLS


The metal-carbon bonds in metal carbonyls have both  and  character. The formation of bonds between
the metal and carbon atom of carbon monoxide is described below:

The first overlap takes place between the filled bonding2p orbital of the carbon monoxide with an empty
metal d-orbital resulting in a -bond between the metal and carbon atom of carbon monoxide. Here,
donation of lone pair of electrons on carbon into a vacant d-orbital of the metal takes place. As CO is a
weak base/weak donor, the-bond formed with the metal atom is weak.
The second overlap takes place between the filled metal d-orbital with an empty antibonding2p* -orbital
of the carbon monoxide resulting in additional  bond between the metal and same carbon monoxide
molecule. Here, donation of electrons from a filled metal d-orbital into a vacant antibonding* -orbital of
CO occurs (back bonding). Here, CO is acting as an acceptor ligand. The effect of  bond formation
strengthens the bond and vice-versa. This is called synergic effect (i.e., working together towards, the
same goal). Thus, as a result of synergic effect, the bond between CO and metal is strengthened.
Further, as explained above, a weak bond is formed by donation of electron pair from carbon to metal
(M C O) and a stronger bond (second bon) is formed by back donation of electrons from filled d-
orbital of metal into empty antibonding * orbital of carbon ( M→C bond). The total bonding is thus M =
C = O. Thus, bond order of C – O bond is reduced from triple bond to double bond. This is supported by the
fact that C – O bond length in C O is 1.128Å and it increases to about 1.15 Å in many carbonyls.

ACTIVESITE EDUTECH- 9844532971 123


ACTIVE SITE
a
EDUTECH Co-ordination Compounds
They are of two types.
(a) Monomeric: Those metal carbonyls which contain only one metal atom per molecule are called
monomeric carbonyls. For examples: [Ni(CO)4] (sp3, tetrahedral) ; [Fe(CO)5 ] (dsp3, trigonal
bipyramidal)
[Cr(CO)6] (d2 sp3, octahedral) ; [V(CO)6] (d2 sp3, octahedral, only carbonyl which is paramagnetic having
one unpaired electron. This is least stable among all the four carbonyls)

(b) Polymeric: Those metal carbonyl which contain two or more than two metal atom per molecule and
they have metal-metal bonds are called polymeric carbonyl. For example: Mn2 (CO)10 , Co2(CO)8, etc.

The metal–carbon bond in metal carbonyls possess both s and p character. The M—C  bond is formed by
the donation of lone pair of electrons on the carbonyl carbon (CO is a weak base) into a vacant orbital of
the metal. The M — C  bond is formed by the donation of a pair of electrons from a filled d orbital of
metal into the vacant antibonding * orbital of carbon monoxide. Thus, carbon monoxide acts as  donor
(OC → M) and a  acceptor (OC  M), with the two interactions creating a synergic effect which
strengthens the bond between CO and the metal as shown in figure.




M C O



Synergic bonding

ACTIVESITE EDUTECH- 9844532971 124


ACTIVE SITE
a
EDUTECH Co-ordination Compounds
Solved Examples
Example 1. What will be the correct order for the wavelength of absorption in the visible region for the
following :
[Ni(NO2)6]4–, [Ni(NH3)6]2+, [Ni(H2O)6]2+?
Sol. (i) The order of the ligand in the spectrochemical series :
H2O < NH3 < NO2–.
Hence the wavelength of the light observed will be in the order :
[Ni(H2O)6]2+ < [Ni(NH3)6]2+ < [Ni(NO2)6]4–.
Thus, wavelengths absorbed (E = hc/) will be in the opposite order.
Example 2. A solution of [Ni(H2O)6]2+ is green but a solution of [Ni(CN)4]2– is colourless. Explain.
Sol. According to crystal field theory the d-d transition of electron in presence of ligands attributes the
colour of the complex. In [Ni(CN)4]2– there is no unpaired electron for d-d transition ; so it is
colourless. In [Ni(H2O)6]2+
there are two unpaired electrons available for d-d transition; so, it is coloured.

Example 3. [Fe(CN)6]4– and [Fe(H2O)6]2+ are of different colours in dilute solutions. Why?
Sol. The colour of the complex in the dilute solution depends on the nature of ligands.
(i) The order of the ligand in the spectrochemical series: H2O < CN–
Hence the wavelength of the light observed will be in the order: [Fe(H2O)6]2+ < [Fe(CN)6]4– .
Thus, wavelengths absorbed (E = hc/) will be in the opposite order.
Therefore, they so different colours in dilute solutions.

Example 4. Calculate the overall complex dissociation equilibrium constant for the
[Cu(NH3)4]2+ ion, given that 4 for this complex is 2.1 × 1013.
1
Sol. The overall dissociation constant is the reciprocal of overall stability constant i.e.
4
= 4.7 × 10–14.

ACTIVESITE EDUTECH- 9844532971 125


ACTIVE SITE
a
EDUTECH Co-ordination Compounds

IMPORTANCE AND APPLICATIONS OF COORDINATIONS COMPOUNDS


The importance of coordination compounds in different fields is briefly described below:
1. In Analytical Chemistry:
a) In qualitative analysis: In salt analysis, the presence of a number of basic radicals is confirmed by
converting them into suitable complexes which have definite colours. For example,
i) The presence of Ni2+ ion is detected by adding dimethylglyoxime in presence of NH4OH to the salt
solution when a brilliant red precipitate is formed due to the formation of a complex of Ni 2+ with
dimethyl glyoxime.

ii)The presence of Co2+ is tested by adding ammonium thiocyanate solution when a blue colour is
obtained due to formation of a complex
CoCl2 + NH4SCN→Co (SCN)2 + 2NH4Cl
Co (SCN)2 + 2NH4SCN→ (NH4)2 [Co (SCN)4]
Ammonium tetrathiocyanatocobaltate (II) (Blue colour)
The presence of Co2+ ion is also tested by potassium ferricyanide test when a reddish brown ppt is
obtained due to the formation of an insoluble complex
𝐖𝐚𝐫𝐦
3CoCl2 + 3K3 [F3 (CN)6]→ Co3 [Fe (CN)6]2 + 6KCl
Pot. Ferricyanide Cobalt hexacyanoferrate (III)
(Reddish brown)

ACTIVESITE EDUTECH- 9844532971 126


ACTIVE SITE
a
EDUTECH Co-ordination Compounds
iii) Fe3+ ions are tested by adding potassium ferrocyanide solution to the salt solution when
ferriferrocyanide or Prussian blue complex is formed
4FeCl3 + 3K4 [Fe (CN)6] → Fe4 [Fe (CN)6]3 + 12KCl
Pot. Ferrocyanide ferri-ferrocyanide
orFeCl3 + K4 [Fe(CN)6] →K Fe [Fe(CN)6] + 3 KCl
Prussian blue
iv) Zn2+ ions are tested by adding potassium ferrocyanide solution to the acidified salt solution when a
bluish white ppt. is obtained due to the formation of a complex
2 ZnCl2 + K4 [Fe (CN)6]→Zn2 [Fe (CN)6] + 4 KCl
Zinc hexacyanoferrate (II)
A mixture of two ions can be separated also by making use of the fact that with the same reagent
and under the same condition, one of the metal ions may form a soluble complex while the other
may form an insoluble complex. For example, with ammoniacal potassium ferrocyanide solution,
cobalt forms an insoluble complex while nickel forms similar complex but soluble in NH 4OH.
b) In Quantitative Analysis (Estimations):
i) Gravimetric Analysis:
The amount of metal present in a given sample can be estimated by converting a known amount of
the sample into an insoluble complex which can be filtered, dried and weighed. For example, the
amount of nickel present in a salt is estimated by precipitating it as nickel dimethyl glyoxime
complex.
ii) Volumetric Analysis (Complexometric titrations):
A number of metal ions react completely with polydentate ligands at an appropriate pH to form
complexes. Hence, the solutions of metal ions can be titrated against the solutions of the
polydentate ligands in the presence of a suitable buffer and the end point can be detected by using a
suitable indicator. The most common polydentate ligand used is ethylene diamine tetra acetic acid
(EDTA).

It is a hexadentate ligand and it coordinates through two N – atoms and four O-atoms of the – COOH
groups. It is usually represented by H4Y. Its disodium salt is commonly used because of its better
solubility in water. Its formula is represented as Na2H2Y. It ionizes as
Na2H2Y → 2 Na+ + H2Y2 –
The reactions of the metal ions like Ca2+, Mg2+, Zn2+ ions etc. (in general, M2+) with EDTA may be
represented as: M2+ + H2Y2 –⇌MY2 – + 2 H+
The indicators most commonly used are organic dyes such as Eriochrome black – T or calcon.
The hardness of water due to presence of Ca2+ and Mg2+ ions is estimated by these complexometric
titrations.

ACTIVESITE EDUTECH- 9844532971 127


ACTIVE SITE
a
EDUTECH Co-ordination Compounds
2) In metallurgy (Extraction of metals): The noble metals like silver and gold are extracted from their
ores through the formation of cyanide complexes, [Ag (CN)2]– and [Au(CN)2 ]– respectively. For
example, silver is extracted from its ore by first dissolving the ore in NaCN solution and then
precipitating out silver by adding the more electropositive metal, zinc.
Ag2S + NaCN⇌Na [Ag (CN)2]+ Na2S
Sodium dicyanoargentate (I)
Na [Ag (CN)2] + Zn→Na2 [Zn (CN)4]+ 2 Ag
Sodium tetracyanozincate (II)
Similarly, the cyanide process used for the extraction of gold is based upon the fact that gold
dissolves in KCN solution in the presence of atmospheric oxygen to form a soluble cyanide complex
4 Au + 8 KCN + 2 H2O + O (air)→4K [Au (CN)2] + 4 KOH
From the complex cyanide solution, gold is precipitated out by adding zinc shavings2 K [Au (CN)2] +
Zn→ K2 [Zn (CN)4] + 2 Au
3) In purification of metals: Some metals are purified by formation of their metal carbonyls followed by
their decomposition, e.g., impure nickel is converted into nickel tetracarbonyl, Ni(CO)4 which on
decomposition gives pure nickel.
4) In Biological Systems: A number of biologically important compounds are the coordination
compounds, i.e., metal complexes. Their use is mainly due to the fact that they are soluble complexes
and thus make the metal available to the biological systems which may not be otherwise easily
available. A few examples are given below:
a) Chlorophyll, the green plant pigment that acts as a photosensitizer in the photosynthesis in plants
is a coordination compound of magnesium.
b) Haemoglobin, a red pigment of blood which acts as an oxygen carrier is a coordination compound
of iron.
c) Vitamin B12 which is chemically cyanocobalamine and is the anti-pernicious anaemia factor is a
coordination compound of cobalt.
Some other biologically important molecules which are coordination compounds include certain
enzymes, e.g., carboxypeptidase -A is a coordination compound with zinc as the central meal atom.
They are biological catalysts.
5) In Industry:
a) As catalysts: Coordination compounds are used as catalysts for many industrial processes. A few
examples are given below:
i) The coordination compound of Rhodium with the formula, [RhCl(Ph3P)], i.e., chloridotris (triphenyl
phosphine) rhodium (I), known as Wilkinson catalyst, is used for selective hydrogenation of alkenes.
The double bond at the end of the chain is hydrogenated but double bonds elsewhere in the chain
are not affected. It is of importance is pharmaceutical industry.
1
ii) Decomposition of water(H2 O →H2 + 2 O2 ) which is thermodynamically not favoured, has been
found to be catalysed by complexes of ruthenium.

ACTIVESITE EDUTECH- 9844532971 128


ACTIVE SITE
a
EDUTECH Co-ordination Compounds
b) In electroplating: Articles can be electroplated with silver or gold much more smoothly and
evenly from solutions of the complexes, [Ag (CN)2]- and [Au (CN)2]-, then from the solutions of
simple metal ions.
c) In photography: The developed film is fixed by washing with hypo solution which dissolves the
undecomposed AgBr to form a complex
AgBr + 2Na2S2O3→Na3 [Ag (S2O3)2]+NaBr
Sod. Dithiosulphate argentite (I)
d) As dyes or pigments: Phthalocyanine blue which is a extremely stable complex of copper (II) is
one of the many complex compounds used as dyes or pigments.
6) In Medical field: Two well-known examples are given below:
i) British anti-Lewisite (BAL) in which the chelating agent is (HS) CH2CH (SH) CH2OH was developed
during war time as an antidote to an arsenic containing poisonous gas (Lewisite). BAL is now used to
treat poisoning by As, Hg, Au, Bi, Sb and even Pb and Cd.
ii) EDTA is quite often used for treatment of lead poisoning.
iii) Metals present is toxic proportions in animals and plants are removed by chelate therapy, e.g.,
excess of copper and iron are removed by using chelating ligands, D – penicillamine and
desferrioxime – B.

iv) Recently, the platinum complex, cis [Pt (NH3)2 Cl2], known as cisplatin has been found to theuseful
in the treatment of cancer (tumours). It has a square planar structure as shown on the right hand
side.

ACTIVESITE EDUTECH- 9844532971 129


ACTIVE SITE
a
EDUTECH Co-ordination Compounds

CLASS EXERCISE
1] Which one of the following is used to separate Cd2+ in presence of Cu2+?
a) Dimethylglyoxime b) EDTA
c) Excess of KCN d) NaCN
2] The instability constant for the two complexes A and B are 0.626 × 10-7& 0.185 × 10-18 respectively.
Which is more stable?
a) A b) B c) Both d) Cannot bepredicted
3] Which of the following exhibits higher stability constant value?
a) [Ni(H2O)6]2+ b) [Ni(NH3)6]2+ c) [Ni(en)3]2+ d) [Ni(EDTA)]2-
4] Stability of complex molecules is increased by
a) high charge density on the central metal b) greater basic strength of the ligand
c) chelation d) All the three
5] A racemic mixture has a net rotation
a) To right of original plane
b) To left of original plane
c) To right or left of original plane
d) Zero
6] The central metal ion in haeme is
a) CO+3 b) Fe+2 c)Mn+2 d) Mg +2

ACTIVESITE EDUTECH- 9844532971 130


ACTIVE SITE
a
EDUTECH Co-ordination Compounds
HOME EXERCISE
1] In which of the following, oxidation state of central metal ion is not +1.
a) Brown ring complex, [Fe(H2O)5NO]SO4 b) Tollen’s reagent, [Ag(NH3)2]OH
c) Sodium nitropruisside, Na2[Fe(CN)5NO] d) Wilkinspn’s catalyst, (PPh3)3RhCl
2] Which of the following statement(s) is/are correct about stability of chelates?
a) as the number of rings in complex increases, stability of complex (chelate) also increases
b) a chelate having five membered ring is more stable if it contains double bonds
c) a chelate having six membered ring is more stable if it does not contain double bonds
d) Chelating ligands are atleast bidentate ligands.
3] Coordination compounds have great importance ion biological systems. In this context which of the
following statements is incorrect?
a) Haemoglobin is the red pigment of blood and contains
b) cyanocobalamine is B12 and contains cobalt
c) Chlorophylls are green pigments in plants and contains calcium
d) Carboxypeptidase-A an enzyme and contains zinc
4] The complex used as an anticancer agent is
a) cis=[PtCl2(NH3)2] b) Na2CO3
c) trans-[Co(NH3)3Cl3] d) cis-K2[PtCl2Br2]
5] The central metal present in chlorophyll
a) Mg b) Co c)Fe d) Ca
6] The metal ion present in vitamin B12 is
a) Co2+ b) Fe2+ c)Co3+ d) Fe3+
7] Oxidation state of Mg in chlorophyll is
a) +6 b) +3 c)+4 d) +2
8] Oxidation state of iron in haemoglobin is:
a) +6 b) 0 c)+2 d) +4
9] Which is not a π − bonded complex
a) Zeisel salt b) Ferrocene
c) Dibenzene chromium d) Tetraethyl lead

NCERT Test Book Questions:9.22, 9.25, 9.26, 9.27, 9.31

CLASS EXERCISE
1) c 2) b 3) d 4) d 5) d 6) b
HOME EXERCISE
1) c 2) c 3) c 4) a 5) a 6) c
7)d 8) c 9) d

ACTIVESITE EDUTECH- 9844532971 131


ACTIVE SITE
a
EDUTECH Co-ordination Compounds
Solved Examples
Example 1. Give evidence that [Co(NH3)4ClBr]Cl and [Co(NH3)4Cl2]Br are ionisation isomers.
Sol. The ionisation isomers dissolve in water to yield different ions and thus react differently to various
reagents :
[Co(NH3)4ClBr]Cl [Co(NH3)4ClBr]+ + Cl– ; Ag+ + Cl– —→ AgCl  (white)

[Co(NH3)4Cl2]Br [Co(NH3)4Cl2]+ + Br– ; Ag+ + Br– —→ AgBr  (yellow)

Example 2. Which is incorrect statement ?


(A) [Co(en)3] [Cr(CN)6] will display coordination isomerism.
(B) [Mn(H2O)5(SCN)] will display linkage isomerism.
(C) [Co(NH3)5(NO2)]SO4 will display ionization isomerism
(D) None of these. Ans. (D)
Sol. (A) Coordination isomerism arises due to exchange of ligands between cationic and anionic
complexes.
(B) SCN– is an ambidentate ligand and thus can attach either through nitrogen or sulphur to central
metal ion causing linkage isomerism.
(C) Exchange of NO2– (in coordination sphere) and SO42– (in ionization sphere) produces ionization
isomerism.
Example 3. Indicate the types of isomerism exhibited by the following complexes.
(i) K[Cr(H2O)2(C2O4)2] (ii) [Co(en)3]Cl3
(iii) [Co(NH3)4(NO2)2](NO3)2 (iv) [Pt(NH3)(H2O)Cl2].
Sol. (i) Both geometrical (cis-, trans-) and optical isomers for cis can exist.
(ii) Two optical isomers can exist.
(iii) There are 10 possible isomers. (Hint : There are geometrical, ionisation and linkage isomers
possible).
(iv) Geometrical (cis-, trans-) isomers can exist.

Example 4. Out of the following two coordination entities which is chiral (optically active)?
(a) cis-[CrCl2(ox)2]3– (b) trans-[CrCl2(ox)2]3–.
Sol. The two entities are represented as

Out of the two, (a) cis-[CrCl2(ox)2]3– is chiral (optically active).

ACTIVESITE EDUTECH- 9844532971 132


ACTIVE SITE
a
EDUTECH Co-ordination Compounds
1. MISCELLANEOUS SOLVED PROBLEMS (MSPS)
1. Give the order of chelating effect of following ligands.
(i) C2O42– (ii) EDTA (iii) NH3
(1) iii > ii > i (2) i > ii > iii (3) ii > i > iii (4) i > iii > ii
Ans. (3)
Sol. As number of donor atoms increases the number of bonds increase. So the chelating effect of
ligands increase as oxalato, dien and EDTA has two, three and six donor atoms respectively.
2. Write the structural formula corresponding to each of the following IUPAC names :
(a) potassium tetracyanidozincate (II)
(b) tetracarbonyl nickel (0)
(c) potassium tris(oxalato)aluminate (III)
Ans. (a) K2[ZnII(CN)4] (b) [Ni0(CO)4] (c) K3[AllII(C2O4)3]
Sol. Refer IUPAC rule.
3. Write IUPAC names of the following :
(a) [V(H2O)6]Cl3 (b) (NH4)3[Co(C2O4)3] (c) [Ni(PPh3)4]
Ans. (a) hexaaquavanadium (III) chloride
(b) ammonium tris(oxalato)cobaltate(III) or ammonium trioxalatocobaltate(III)
(c) tetrakis(triphenylphosphine)nickel(0)
Sol. Refer IUPAC nomenclature rule.
4. A solution containing 0.319 gm of complex CrCl3 . 6H2O was passed through cation exchanger and
the solution given out was neutralised by 28.5 ml of 0.125 M NaOH. The correct formula of the
complex will be [molecular weight of complex = 266.5]
(1) [CrCl(H2O)5]Cl2 . H2O (2) [Cr(H2O)6]Cl3
(3) [CrCl2(H2O)4]Cl. 2H2O (4) All are correct
Ans. (2)
Sol. Cl– = HCl = NaOH
nCl– + nH+ —→ nHCl
Thus 1 mol of complex will form n mol of HCl
1 mole of complex = n mol of HCl = n mole of NaOH
0.319 28.5  0.125
mole of complex = = 0.0012; mole of NaOH used = = 0.0036
266.5 1000
So 0.0012 mole of complex = 0.0036 mole of NaOH = 0.0036 mole of HCl
0.0036
1 mole of complex = = 3 mole of HCl
0.0012
 n=3
So complex is [Cr(H2O)6]Cl3.
5. Calculate the effective atomic number of the metal atom in the following complexes.
(a) [Cr(CO)6] (b) [FeCl4]2– (c) [Co(CN)6]4– (d) [Ni(NH3)6]2+

ACTIVESITE EDUTECH- 9844532971 133


ACTIVE SITE
a
EDUTECH Co-ordination Compounds
[Cr = 24 ; Fe = 26; Co = 27 and Ni = 28 as atomic numbers]
Ans. (a) 36 (b) 32 (c) 37 (d) 38
Sol. (a) [Cr0(CO)6] ; EAN = 24 + 12 = 36
(b) [FeIICl4]2– ; EAN = 26 – 2 + 8 = 32
(c) [CoII(CN)6]4–; EAN = 25 + 12 = 37
(d) [NiII(NH3)6]2+; EAN = 26 + 12 = 38

6. Consider the following complexes :


(i) K2PtCl6 (ii) PtCl4 . 2NH3 (iii) PtCl4 . 3NH3 (iv) PtCl4 . 5NH3
Their electrical conductance in aqueous solution are :
(1) 256, 0, 97, 404 (2) 404, 0, 97, 256 (3) 256, 97, 0, 404 (4) 404, 97, 256, 0
Ans. (1)
Sol. The electrical conductance of the complexes depends upon the number of ions given by them in
the aqueous solution.
(i) K2[PtCl6] 2K+ (aq) + [PtCl6]2– (aq)
(ii) [Pt(NH3)2Cl4] [Pt(NH3)2Cl4] (aq)
(iii) [Pt(NH3)3Cl3]Cl [Pt(NH3)3Cl3]+ (aq) + Cl– (aq)
(iv) [Pt(NH3)5Cl]Cl3 [Pt(NH3)5Cl]3+ + 3Cl–
Complex Number of ions Expected electrical conductance
(i) K2[PtCl6] 3 256
(ii) [Pt(NH3)2Cl4] 0 0
(iii) [Pt(NH3)3Cl3]Cl 2 97
(iv) [Pt(NH3)5Cl]Cl3 4 404
So, the correct option is (A).
7. Explain the following :
(i) All the octahedral complexes of Ni2+ must be outer orbital complexes.
(ii) [CoF6]3– is paramagnetic but [Co(CN)6]3– is diamagnetic.

Sol. (i) Ni2+ configuration


During rearrangement only one 3d-orbital may be made available for pairing of the electrons.
Thus, d2 sp3 hybridization is not possible. So only sp3d2 (outer) hybridization can occur.
(ii) ln [CoF6]3– , Co3+ undergoes sp3d2 hybridization. Four d-orbitals are singly occupied. Hence, it is
paramagnetic.

[CoF6]3–
In [Co(CN)6]3–, Co3+ undergoes d2sp3 hybridization. All electrons are paired and thus it is diamagnetic.

[Co(CN)6]3– `

ACTIVESITE EDUTECH- 9844532971 134


ACTIVE SITE
a
EDUTECH Co-ordination Compounds
8. The increasing order of the crystal field splitting power of some common ligands is :
(1) H2O < OH¯ < NCS¯ < F¯ < NH3 (2) H2O < NCS¯ < OH¯ < NH3< F¯
(3) CN¯ < H2O < OH¯ < NCS¯ < H2O¯ (4) F¯ < OH¯ < H2O < NCS¯ < NH3
Ans. (4)
Sol. Spectro chemical series based on experimental data follows the order:
I– < Br– < SCN– < Cl– < S2– < F– < OH– < C2O42– < H2O < NCS– < edta4– < NH3 < en < NO2– < CN– < CO
9. Which of the following statements is not correct?
(1) The complexes [Fe(H2O)6]2+ and [Fe(CN)6]4– differ in the state of hybridisation of iron.
(2) The complexes [Fe(H2O)6]2+ and [Fe(CN)6]4– differ in the magnetic properties.
(3) Iron ion has the same secondary valency in the complexes [Fe(H2O)6]2+ and [Fe(CN)6]4–.
(4) The complexes [Fe(H2O)6]2+ and [Fe(CN)6]4– differ in primary valences of iron.
Ans. (4)
Sol. The complexes [Fe(H2O)6]2+ and [Fe(CN)6]4– have same primary valences of iron i.e. + 2.

10. [Ni(PPh3)4] and [NiCl4]2– have geometry, hybridization and magnetic moments of the ions
respectively :
(1) tetrahedral, tetrahedral ; sp3, sp3 ; 0, 2.82
(2) tetrahedral, square planar ; sp3, dsp2 ; 0, 2.82
(3) tetrahedral, octahedral ; sp3 , d2sp3 ; 2.82, 0
(4) square planar ,tetrahedral, ; dsp2 ,sp3 ; 2.82 , 2.82
Ans. (1)
Sol. Ni : 3d8 4s2
Ni (0), 3d10

It is diamagnetic, so B.M. = 0
Ni : 3d8 4s2
Ni2+, 3d8
since Cl is a weak field ligand, so it will not cause a paring of electron.

N=2
µ = N(N + 2) = 2(2 + 2) B.M. = 8 B.M. = 2.82 B.M.

ACTIVESITE EDUTECH- 9844532971 135


ACTIVE SITE
a
EDUTECH Co-ordination Compounds
11. It is an experimental fact that :
DMG + Ni()salt + NH4OH —→ Red precipitate
Which of the following is wrong about this red precipitate?
(1) It is a non–ionic complex (2) It has square planar geometry
(3) It has tetrahedral geometry (4) It is a diamagnetic complex
Ans. (3)
Sol. In complex [Ni(DMG)2], the nickel is in +2 oxidation state ; the ion has the electronic configuration
3d8 and dimethylglyoxime is a chelating ligand. So, the hybridisation scheme is as shown in figure.
3d 4s 4p
Ni , [Ar]3d
2+ 8

[Ni(DMG)2] diamagnetic.
dsp2 hybrid orbitals

12. The correct order for the CFSE (numerical value) for the following complexes is
Complex P Q R S
Formula [CoF6] 3– [Co(CN)6] 3– [Co(NH3)6] 3+ [Co(H2O)6]3+
(1) P > Q > R > S (2) Q > R > S > P (3) S > R > P > Q (4) R > Q > P > S
Ans. (2)
Sol. CFSE depends on the relative magnitude of crystal field splitting, o and pairing energy and in turns
o depends upon the field produced by ligand and charge on the metal ion. The order of increasing
crystal field strength is F– < H2O < NH3 < CN–.
Thus the (B) option is correct.
13. Which of the following statements are correct?
(a) [Ni(H2O)6]2+ and [Ni(NH3)6]2+ have same value of CFSE
(b) [Ni(H2O)6]2+ and [Ni(NH3)6]2+ have same value of magnetic moment
(c) [Ni(H2O)6]2+ and [Ni(NH3)6]2+ have same hybridisation of nickel
(1) a and c only (2) b and c only (3) a and b only (4) All of
these
Ans. (2)
Sol. (a) Ammonia is a stronger field ligand than water. So [Ni(NH3)6]2+ will have higher CFSE than
[Ni(H2O)6]2+.
(b) Both complexes [Ni(H2O)6]2+ and [Ni(NH3)6]2+ have sp3d2 hybridisation with two unpaired
electrons. So, they possess same magnetic moment (‘spin only’)

ACTIVESITE EDUTECH- 9844532971 136


ACTIVE SITE
a
EDUTECH Co-ordination Compounds
14. STATEMENT 1 : [CoII(NH3)6]2+ is not readily oxidized to [CoIII(NH3)6]3+ when air is bubbled through it.
STATEMENT 2 : Crystal field stabilization energy of Co(+III) with a d6 configuration is higher than
for Co(+II) with a d7 arrangement.
(1) Statement-1 is True, Statement-2 is True; Statement-2 is a correct explanation for Statement-1.
(2) Statement-1 is True, Statement-2 is True; Statement-2 is NOT a correct explanation for
Statement-1
(3) Statement-1 is True, Statement-2 is False
(4) Statement-1 is False, Statement-2 is True
Ans. (4)
Sol. S1 : It is false statement.
S2 : [CoII (NH3)6]2+ is readily oxidised in [CoIII(NH3)6]3+ in presence of air due to the higher CFSE value
of d6 configuration.

15. Which of the following is true about the complex [PtCl2(NH3)(OH2)] ; [Atomic number of Pt = 78]
(i) It will have two geometrical isomeric forms, cis and trans
(ii) The hybridisation state of Pt() is sp3
(iii) It is a square planar complex (iv) It is a diamagnetic complex
(v) It can show hydrate isomerism (vi) It is a tetrahedral complex
(1) (i), (iii),(iv) (2) (ii),(iv),(v) (3) (ii),(v),(vi) (4) (i),(v),(vi)
Ans. (A)

Sol. Pt2+([Xe]4f14 5d8)

[PtCl2(NH3)(OH2)] diamagnetic
dsp2 hybrid orbitals
Four pairs of electrons from four Cl–
Ma2bc have cis- and trans isomers.

16. In metal carbonyls the metal carbon bond length is found to be less than the expected bond length.
Explain why ?
Sol. It is due to synergic interaction between metal and CO which develops partial double bond
character between metal and CO.



M C O



Synergic bonding
17. (a) Draw all possible constitutional isomers of the compound Ru(NH3)5(NO2)Cl. Label the isomers as
linkage isomers or ionization isomers.

ACTIVESITE EDUTECH- 9844532971 137


ACTIVE SITE
a
EDUTECH Co-ordination Compounds
(b) There are six possible isomers for a square planar palladium(II) complex that contains two Cl –
and two SCN– ligands. Sketch the structures of all six, and label them according to the classification.
Ans. (a) There are three constitutional isomers
(i) [Ru(NH3)5(NO2)]Cl

(ii) [Ru(NH3)5Cl](NO2) or [Ru(NH3)5Cl]ONO

(iii) [Ru(NH3)5 ONO]Cl

(i) & (ii) are ionisation isomers

(i) & (iii) are linkage isomers

(b) (i) (ii) (iii)

(iv) (v) (vi)

18. cis- platin contains :


(1) platinum (2) iron (3) aluminium (4) cobalt
Ans. (1)
Sol. cis- platin is cis[Pt(NH3)2Cl2]. So it contains platinum.

PART - I : SUBJECTIVE QUESTIONS


Section (A) : General introduction of complex salts and definitions to be used
A-1. K2SO4 solution mixed with Cr2(SO4)3 solution in 1 : 1 molar ratio gives the test of Cr3+ ion but CuSO4
solution mixed with aqueous ammonia in 1 : 4 molar ratio does not give the test of Cu 2+ ion. Explain
why?
A-2. What is the coordination number and the oxidation state of the metal in each of the following
complexes?
(a) [AgCl2]– ; (b) [Cr(H2O)5Cl]2+ ; (c) [Co(NCS)4]2–
(d) [Co(NH3)3(NO2)3] ; (e) [Fe(EDTA)]– (f) [Cu(en)2]SO4;
(g) K[Pt(NH3)Cl5]
A-3. Write the name of the following ligands and classify their denticity
(A) CH3NC (B) acac–1 (C) N3– (D) dien (E) edta4–
–1 –
(F) edta 3–
(G) ox 2–
(H) dmg (I) NC (J) NO2–

ACTIVESITE EDUTECH- 9844532971 138


ACTIVE SITE
a
EDUTECH Co-ordination Compounds
(K) O2– (L) O2–
A-4. Predict the different ligating sites by drawing structures in the following ligands.
(A) (NO2)– (B) (SCN)– (C) (C2O2S2)2– (D) (OCN)–

(E) (NOS)
A-5. (a) Determine the denticity of the ligands in complexes [Fe(C2O4)3]3– and [Pt(en)2]2+. What are
the
coordination number and the oxidation number of the central metal ion?
(b) Designate the coordination entities and counter ions in the coordination compounds.
K2[Ni(CN)4] ; [Cr(en)3] Cl3 ; Fe4[Fe(CN)6]3 ; [PtCl2(en)2] (NO3)2.
(c) Identify the Lewis acid and Lewis base components of the following complexes.
(i) [HgBr4]2- (ii) [Ni(H2O)6]2+ (iii) [PdCl2(NH3)2]
– –
(iv) [Al(OH)4] (v) [Ag(CN)2] (vi) [Cr(CO)6]

Section (B) : Nomenclature of coordination compounds


B-1. Name the following compounds
Prepared in 1798 by B.M. Tassaert and considered to be first
(a) [Co(NH3)6]Cl3
complex salt prepared.
A yellow colored complex obtained by heating
(b) [Rh(NH3)5I]I2 0
[Rh(NH3)5(H2O)]I3 above 100 C.
(c) [Fe(CO)5] A highly toxic volatile liquid.
The ion formed when Fe2O3 rust is dissolved in oxalic acid,
(d) [Fe(C2O4)3]3–
H2C2O4.
A deep blue compound obtained when CuSO4 is treated with
(e) [Cu(NH3)4]SO4
excess of NH3.
The compound formed when Cr(OH)3 precipitate is dissolved in
(f) Na[Cr(OH)4]
excess of NaOH.
(g) [Co(gly)3] A complex that contains the anion of amino acid, glycine.
The red complex ion formed in the qualitative analysis test of
(h) [Fe(H2O)5(SCN)]2+
Fe3+ ion.
(i) K2[HgI4] Alkaline solution of this complex is called Nessler’s Reagent.
Deep blue crystalline precipitate obtained in qualitative
(j) Co[Hg(SCN)4]
detection of Hg2+.
Prussian blue, deep blue colored complex obtained in
(k) Fe4[Fe(CN)6]3
detection of Fe2+.
Potassium cobaltinitrite or Fischer salt yellow precipitate
(l) K3[Co(NO2)6]
obtained in detection of Co2+.
(m) [Ni(dmg)2] Rosy red precipitate obtained in detection of Ni2+ ions.
(n) K2[PtCl6] Yellow precipitate obtained in detection of potassium ions.
Sodium nitroprusside used for detection of sulphide
(o) Na2[Fe(CN)5NO+]
ions/sulphur.
(p) [Fe(H2O)5(NO )]SO4 Brown ring complex, obtained in detection of Fe+1 ions.
+

Colourless stable soluble complex obtained in detection of Cu2+


(q) [Cu(CN)4]3–
on adding excess of KCN solution.
Only few compounds of ammonium ions are precipitate this is
(r) (NH4)2[PtCl6]
one of these, a yellow precipitate.

ACTIVESITE EDUTECH- 9844532971 139


ACTIVE SITE
a
EDUTECH Co-ordination Compounds
B-2. Name the following compounds.
(a) [CoBr(en)2(ONO)]+ (b) [Co(NH3)6][Co(ONO)6]
(c) [Co(NH3)5(CO3)]Cl (d) [Pt(NH3)4Cl2][PtCl4]
(e) [Co(en)3]2(SO4)3 ; (f) [(NH3)5Co-NH2-
Co(NH3)4(H2O)]Cl5
(g) [Cr(CO)5(PPh3)] (h) [(CO)5Mn-Mn(CO)5]
(i) Cr(6–C6H6)2 (j) [Co(NH3)4(OH2)2][BF4]3
(k) Ba[Zr(OH)2(ONO)2(ox)] (l) [Co(NH3)6][Co(C2O4)3]
B-3. Write down the formulae of the following compounds
(a) Tetraamminezinc(II) Nitrate The compound formed when zinc nitrate is treated with
an excess of ammonia
(b) Tetracarbonylnickel(0) The first metal carbonyl(prepared in 1888) and an
important compound in the industrial refining of nickel
metal
(c) Potassium A compound that contains a square planar anion
amminetrichloridoplatinate(II)
(d) Dicyanidoaurrate(I) ion An ion important in the extraction of gold from its ores
(e) Sodium hexafluoridoaluminate(III) Called cryolite, used in the electrolytic refining of
aluminium
(f) Diamminesilver(I) ion Ion formed when AgCl is dissolved in excess of
ammonia
B-4. Write down the formulae of the following compounds
(a) diamminetriaquahydroxidochromium (III) nitrate
(b) tetrakis(pyridine)platinum(II) tetraphenylborate(III)
(c) dibromidotetracarbonyliron (II)
(d) ammonium
diamminetetrakis(isothiocyanato)chromate(III).
(e) pentaamminedinitrogenruthenium(II) chloride
(f) barium dihydroxidodinitrito-O-oxalatozirconate(IV)
(g) tetrapyridineplatinum(II) tetrachloridonickelate(II)

Section (C) : Werner's Theory


(Initial bonding theories and EAN rule)
C-1. Werner conducted many experiments to establish the formula of complexes, one of these were
conductivity measurements. On the basis of the experiments performed he obtained the following
values of conductivity for different type of complexes.
Type of complex Electrical Conductivity
0 – 10 (due to
Nonelectrolyte
impurities)
1:1 Electrolyte 90 – 130
1:2 or 2:1
230 – 290
Electrolyte
1:3 or 3:1
390 – 450
Electrolyte
1:4 Electrolyte 500 – 550

ACTIVESITE EDUTECH- 9844532971 140


ACTIVE SITE
a
EDUTECH Co-ordination Compounds
On the basis of above table Match the following two columns.
COLUMN A COLUMN B
Formula of Correct Werner’s
Conductivity
compound Representation
(a) PtCl4.2NH3 6.99 (i) [Cr(NH3)5Cl]Cl2
(b) PtCl4.NH3.KCl 106.8 (ii) [Co(NH3)5Br]Br2
(c) CrCl3.5NH3 260.2 (iii) [Cr(NH3)6]Cl3
(d) PtCl4.2KCl 256.8 (iv) [Pt(NH3)2Cl4]
(e) CrCl3.6NH3 441.7 (v) [Pt(NH3)6]Cl4
(f) PtCl4.6NH3 522.9 (vi) [Pt(NH3)3Cl3]Cl
(g) CoBr3.5NH3 257.6 (vii) K2[PtCl6]
(h) PtCl4.3NH3 96.8 (viii) K[Pt(NH3)Cl5]

C-2. 1 g of complex [Cr(H2O)5Cl]Cl2.H2O was passed through a cation exchanger to produce HCl. The
acid liberated was diluted to 1 litre. What will be the molarity of acid solution [Molecular weight of
complex = 266.5] ?

C-3. Calculate the EAN of central atom in the following complexes


(a) [Cr(CO)6] (b) [Fe(CN)6]4– (c) [Fe(CO)5] (d) [Co(NH3)6]3+
3–
(e) [Ni(CO)4] (f) [Cu(CN)4] (g) [Pd(NH3)6]4+ (h) [PtCl6]2–

C-4. Arrange the following compounds in order of increasing molar conductivity.


(i) K[Co(NH3)2(NO2)4] (ii) [Cr(NH3)3(NO2)3]
(iii) [Cr(NH3)5(NO2)]3[Co(NO2)6]2 (iv) [Cr(NH3)6]Cl3

Section (D) : Valence Bond Theory + Crystal Field Theory (Part-I)


D-1. A metal complex having composition Cr(NH3)4Cl2Br has been isolated in two forms A and B. The
form A reacts with AgNO3 to give a white precipitate readily soluble in dilute aqueous ammonia,
whereas B gives a yellow precipitate soluble in concentrated ammonia.
(i) Write the formulae of A and B.
(ii) State hybridisation of chromium in each.
(iii) Calculate their magnetic moments for each (spin-only value).
(iv) Calculate the EAN for both.
(v) Will they conduct electricity or not.
(vi) Write the formula of the complexes formed when the precipitates dissolve in aqueous ammonia
& the concentrated ammonia respectively.

D-2. Predict the hybridisation and geometry of the following complexes.


(a) [NiBr4]2– (b) [AuCl4]– (c) [Pt(NH3)4]2+

Section (E) : Valence Bond Theory + Crystal Field Theory (Part-II)


E-1. For the complex K2[Cr(NO)(NH3)(CN)4];  = 1.73 BM.
(i) Write IUPAC name.
(ii) What will be structure?
(iii) How many unpaired electrons are present in the central metal ion?
(iv) Is it paramagnetic or diamagnetic?
(v) Calculate the EAN of the complex.
(vi)What will be the hybridisation of the complex?

ACTIVESITE EDUTECH- 9844532971 141


ACTIVE SITE
a
EDUTECH Co-ordination Compounds
E-2. Predict the hybridisation and geometry of the following complexes.
(a) [Fe(CN)6]3– (b) [MnBr4]2– (c) [Fe(H2O)6]2+ (d) [Co(SCN)4]2–

E-3. [Co(NH3)6]3+ & [CoF6]3– both are complexes of Co(III), but [Co(NH3)6]3+ is diamagnetic while
[CoF6]3– is paramagnetic with  = 4.90 B.M. Explain.

E-4. Arrange the following in increasing order as directed.


(a) (i) [CoCl3(NH3)3], (ii) [CoCl(NH3)5]Cl2, (iii) [Co(NH3)6]Cl3, (iv) [CoCl2(NH3)4]Cl - Molar conductance
(b) C, N, O, F (halogen) - tendency of  donation.
(c) Br–, S2–, NO2–, CO, H2O, CN–, NH3, NO3– - strength of ligands.

E-5. For each of the following complexes, draw a crystal field energy-level diagram, assign the electrons
to orbitals, and predict the number of unpaired electrons:
(a) [CrF6]3– (b) [V(H2O)6]3+ (c) [Fe(CN)6]3–
(d) [Cu(en)3]2+ (e) [FeF6]3–

Section (F) : Applications of crystal field theory


(Magnetic moment of complex, Colour of complex, Limitation, Stability of complex)
F-1. Cobalt (II) is stable in aqueous solution but in the presence of complexing reagents(strong field
ligands) it is readily oxidised. Why ?

F-2. The value of 0 for [Ti(H2O)6]3+ is found to be 240 kJ mol–1 then predict the colour of the complex
using the following table. (h = 6 × 10–34 J-sec, NA= 6 × 1023, c = 3 × 108 m/sec)
Absorbed  (nm)
Colour exhibited
light (absorbed)
Blue 435 – 480 Yellow
green-blue 480 – 490 Orange
blue-green 490 – 500 Red
green 500 – 560 purple
yellow-green 560 – 580 violet
Yellow 580 – 595 blue
Red 605 – 700 blue green

F-3. (a) [Ti(H2O)6]3+ absorbs light of wavelength 5000 Å. Name one ligand which would form a
titanium(III) complex absorbing light of lower wavelength than 5000 Å and one ligand which would
form a complex absorbing light of wavelength higher than 5000 Å.
(b) Calculate the magnetic moments (spin only) of the following complexes
(i) [PtCl6]2– (ii) [Cr(CO)6] (iii) [Ir(NH3)6]3+ (iv) [Pd(en)2]2+

Section (G) : Isomerism in Coordination compounds


(Structural Isomerism, Stereoisomerism, Geometrical Isomerism, Optical Isomerism)
G-1. What type of isomers are the following :
(i) [Mn(CO)5SCN] and [Mn(CO)5NCS]
(ii) [Co(en)3] [Cr(CN)6] and [Cr(en)3] [Co(CN)6]
(iii) [Co(NH3)5 NO3]SO4 and [Co(NH3)5SO4]NO3
(iv) [Co(H2O)2Cl2 (py)2]Cl and [Co(H2O)Cl3 (py)2]H2O

ACTIVESITE EDUTECH- 9844532971 142


ACTIVE SITE
a
EDUTECH Co-ordination Compounds

G-2. (a) Draw all possible constitutional isomers of the compound Ru(NH3)5(NO2)Cl. Label the isomers
as linkage isomers or ionization isomers.
(b) There are six possible isomers for a square planar palladium(II) complex that contains two NH3
and two SCN– ligands. Sketch the structures of all six, and label them according to the
classification.

G-3. How many geometrical isomers are possible for each of the following complexes?
(a) [Pt(NH3)2(SCN)2] (b) [CoCl2Br2]2– (tetrahedral)
(c) [Co(NH3)3 (NO2)3] (d) [Pt(en)Cl2]
+
(e) [Cr Br2 (en)2] (f) [Rh(en)3]3+

G-4. Which of the following complexes can exist as enantiomers? Draw their structures
(a) cis-[Co(NH3)4Br2]+ (b) cis-[Cr(H2O)2(en)2]3+ (c) [Cr(gly)3]
3+
(d) [Cr(en)3] (e) cis-[Co(NH3)Cl(en)2]2+ (f) trans-[Co(NH3)2(en)2]2+

Section (H) : Organometallic Compounds


H-1. Draw the structures of the following metal carbonyls
(a) [V(CO)6] (b) [Cr(CO)6] (c) [Mn2(CO)10]
(d) [Fe(CO)5] (e) [Ni(CO)4]




M C O
H-2.



The figure represents the synergic bonding interaction in metal carbonyl complex. On the basis of
this explain the following :
(i) Strength of Metal-ligand bond
(ii) Bond order of CO in carbonyl complex as compared to bond order in carbon monoxide.

PART - II : ONLY ONE OPTION CORRECT TYPE


Section (A) : General introduction of complex salts and definitions to be used
A-1. Ethylene diamine is an example of a .......... ligand :
(A) monodentate (B) bidentate (C) tridentate (D) hexadentate

A-2. The donor sites of (EDTA)4– are ?


(A) O atoms only (B) N atoms only
(C) Two N atoms and four O atoms (D) Three N atoms and three O atoms

A-3. Some salts although containing two different metallic elements give test for one of them in solution.
Such salts are :
(A) complex salt (B) double salt (C) normal salt (D) none

A-4. Ligands are :


(A) Lewis acids (B) Lewis bases (C) neutral (D) none

ACTIVESITE EDUTECH- 9844532971 143


ACTIVE SITE
a
EDUTECH Co-ordination Compounds

A-5. The oxidation state of Mo in its oxido-complex species [Mo2O4(C2H4)2(H2O)2]2– is


(A) +2 (B) +3 (C) +4 (D) +5

A-6. Co-ordination number of platinum in [Pt(NH3)4Cl2]2+ ion is:


(A) 4 (B) 2 (C) 8 (D) 6

A-7. Which of the following is copper() compound ?


(A) [Cu(H2O)4]2+ (B) [Cu(CN)4]3– (C) [Cu(NH3)4]2+ (D) All of
these

A-8. In the complex [CoCl2(en)2]Br, the co-ordination number and oxidation state of cobalt are :
(A) 6 and +3 (B) 3 and +3 (C) 4 and +2 (D) 6 and +1

A-9. What is the charge on the complex [Cr(C2O4)2(H2O)2] formed by Cr() ?


(A) +3 (B) +1 (C) +2 (D) –1

Section (B) : Nomenclature of coordination compounds


B-1. A complex cation is formed by Pt (in some oxidation state) with ligands (in proper number so that
coordination number of Pt becomes six). Which of the following can be its correct IUPAC name ?
(A) Diammineethylenediaminedithiocyanato-S-platinum (II) ion
(B) Diammineethylenediaminedithiocyanato-S-platinate (IV) ion
(C) Diammineethylenediaminedithiocyanato-S-platinum (IV) ion
(D) Diamminebis (ethylenediamine) dithiocyanate-S- platinum (IV) ion

B-2. Which of the following names is impossible ?


(A) Potassium tetrafluoridooxidochromate (VI) (B) Barium tetrafluoridobromate (III)
(C) Dichlorobis(urea)copper (II) (D) All are impossible.

B-3. The formula of the complex tris(ethylenediamine)cobalt(III) sulphate is :


(A) [Co(en)2SO4] (B) [Co(en)3SO4] (C) [Co(en)3]2SO4 (D) [Co(en)3]2(SO4)3

B-4. The correct IUPAC name for the compound [Co(NH3)4CI(ONO)]Cl is :


(A) Tetraamminechloridonitrito–N–cobalt(III) chloride
(B) Chloridonitrito–O–tetraamminecobalt(II) chloride
(C) Dichloridonitrito–O–tetraamminecobalt(III)
(D) Tetraamminechloridonitrito–O–cobalt(III) chloride

B-5. The hypothetical complex triamminediaquachloridocobalt(III) chloride can be represented as :


(A) [CoCl(NH3)3(H2O)2] (B) [Co(NH3)3(H2O)Cl3]
(C) [Co(NH3)3(H2O)2Cl]Cl2 (D) [Co(NH3)3(H2O)3]Cl3 ]

Section (C) : Werner's Theory


(Initial bonding theories and EAN rule)
C-1. EAN of a metal carbonyl M(CO)x is 36. If atomic number of metal M is 26, what is the value of x?
(A) 4 (B) 8 (C) 5 (D) 6

C-2. The EAN of platinum in potassium hexachloridoplatinate(IV) is (Atomic number of Pt = 78) :


(A) 90 (B) 86 (C) 76 (D) 88

ACTIVESITE EDUTECH- 9844532971 144


ACTIVE SITE
a
EDUTECH Co-ordination Compounds
C-3. A compound is made by mixing cobalt (III) nitrite and potassium nitrite solutions in the ratio of 1 : 3.
The aqueous solution of the compound showed 4 particles per molecule whereas molar
conductivity reveals the presence of six electrical charges. The formula of the compound is :
(A) Co(NO2)3.2KNO2 (B) Co(NO2)3.3KNO2 (C) K3[Co(NO2)6] (D) K[Co(NO2)4]

C-4. Which of the following will exhibit maximum ionic conductivity?


(A) K4 [Fe(CN6] (B) [Co(NH3)6] Cl3 (C) [Cu(NH3)4] Cl2 (D) [Ni (CO)4]

C-5. Which of the following shows maximum molar conductance ?


(A) [Co(NH3)6]Cl3 (B) [Co(NH3)3Cl3] (C) [Co(NH3)4 Cl2] Cl (D) [Co(NH3)5Cl]Cl2

C-6. The complex [Cr(H2O)4Br2]Cl gives the test for :


(A) Br– (B) Cl– (C) Cr3+ (D) Br– and Cl– both

C-7. Which of the following complexes will be dehydrated to relatively minimum extent by conc. H2SO4
under identical condition.
(A) [Cr(H2O)5Cl]Cl2.H2O (B) [Cr(H2O)4Cl2]Cl.2H2O
(C) [Cr(H2O)6]Cl3 (D) all of these

C-8. On adding AgNO3 solution to a solution of [Pt(NH3)3Cl3]Cl, the percentage of total chloride ion
precipitated is:
(A) 100 (B) 75 (C) 50 (D) 25

C-9. A complex of platinum, ammonia and chloride produces four ions per molecule in the solution. The
structure consistent with the observation is:
(A) [Pt(NH3)4]Cl4 (B) [Pt(NH3)2Cl4] (C) [Pt(NH3)5Cl]Cl3 (D) [Pt(NH3)4Cl2]Cl2

Section (D) : Valence Bond Theory + Crystal Field Theory (Part-I)


D-1. The complex ion which has no. 'd' electrons in the central metal atom is :
(A) [MnO4]– (B) [Co(NH3)6]3+ (C) [Fe(CN)6]3– (D) [Cr(H2O)6]3+

D-2. For the correct assignment of electronic configuration of a complex, the valence bond theory often
requires the measurement of
(A) molar conductance (B) optical activity
(C) magnetic moment (D) dipole moment
D-3. The magnitude of crystal field stabilisation energy (CFSE of 1) in tetrahedral complexes is
considerably less than that in the octahedral field. Because
(A) There are only four ligands instead of six so the ligand fild is only 2/3 in tetradedral complex
(B) The direction of the orbitals does not coincide with the direction of the ligands. This reduces the
crystal field stabilization energy () by further 2/3
(C) Both points (A) and (B) are correct
(D) Both points (A) and (B) are wrong
D-4. Which of the following factors tends to increase the stability of metal ion complexes ?
(A) Higher ionic radius of the metal ion (B) Higher charge/size ratio of the metal ion
(C) Lower ionisation potential of the metal ion (D) Lower basicity of the ligand
D-5. The crystal field splitting energy for octahedral complex (o) and that for tetrahedral complex (t) are
related as:
4 9
(A) t = 0 (B) t = 0.5 o (C) t = 0.33 o (D) t = 0
9 4

ACTIVESITE EDUTECH- 9844532971 145


ACTIVE SITE
a
EDUTECH Co-ordination Compounds
D-6. All the metal ions contains t2g6 eg0 configurations. Which of the following complex will be
paramagnetic?
(A) [FeCl(CN)4(O2)]4- (B) K4[Fe(CN)6] (C) [Co(NH3)6]Cl3 (D) [Fe(CN)5(O2)]-5

Section (E) : Valence Bond Theory + Crystal Field Theory (Part-II)


E-1. Chromium hexacarbonyl is an octahedral compound involving :
(A) sp3d2 (B) dsp2 (C) d2sp3 (D) dsp3

E-2. Which of the following molecules is not tetrahedral ?


(A) [Pt(en)2]2+ (B) [Ni(CO)4] (C) [Zn(NH3)4]2+ (D) [NiCl4]2–

E-3. The complex [Pt(NH3)4]2+ has ..... structure :


(A) square planar (B) tetrahedral (C) pyramidal (D) pentagonal

E-4. Match Column-I with Column-II and select the correct answer with respect to hybridisation using the
codes given below :
Column - I Column - II
(Complex) (Hybridisation)

(I) [Au F4] (p) dsp2 hybridisation
(II) [Cu(CN)4]3– (q) sp3 hybridisation
(III) [Co(C2O4)3]3– (r) sp3d2 hybridisation
(IV) [Fe(H2O)5NO]2+ (s) d2sp3 hybridisation
Codes :
(I) (II) (III) (IV) (I) (II) (III) (IV)
(A) q p r s (B) p q s r
(C) p q r s (D) q p s r

E-5. The hybridisation and unpaired electrons in [Fe(H2O)6]2+] ion are :


(A) sp3d2 ; 4 (B) d2sp3 ; 3 (C) d2sp3 ; 4 (D) sp3 d2 ; 2

E-6. The number of unpaired electrons in d6, low spin, octahedral complex is :
(A) 4 (B) 2 (C) 1 (D) 0

E-7. Which of the following is a high spin complex ?


(A) [Co(NH3)6]3+ (B) [Fe(CN)6]4– (C) [Ni(CN)4]2– (D) [FeF6]3–

E-8. Which has maximum paramagnetic nature ?


(A) [Cu(H2O)4]2+ (B) [Cu(NH3)4]2+ (C) [Mn(H2O)6]2+ (D) [Fe(CN)6]4–

E-9. The number of unpaired electrons present in complex ion [FeF6]3- is :


(A) 5 (B) 4 (C) 6 (D) 0

E-10. Which of the following complexes has a geometry different from others ?
(A) [Ni Cl4]2– (B) Ni (CO)4 (C) [Ni(CN)4]2– (D) [Zn(NH3)4]2+

ACTIVESITE EDUTECH- 9844532971 146


ACTIVE SITE
a
EDUTECH Co-ordination Compounds
E-11. Select the correct statement.
(A) Complex ion [MoCl6]3– is paramagnetic. (B) Complex ion [Co(en)3]3+ is diamagnetic.
(C) Both (A) and (B) are correct. (D) None of correct.

E-12. Amongst Ni(CO)4, [Ni(CN)4]2– and NiCI42– :


(A) Ni(CO)4 and NiCI42– are diamagnetic and [Ni(CN)4]2– is paramagnetic.
(B) NiCI42– and [Ni(CN)4]2– are diamagnetic and Ni(CO)4 is paramagnetic.
(C) Ni(CO)4 and [Ni(CN)4]2– are diamagnetic and NiCI42– is paramagnetic.
(D) Ni(CO)4 is diamagnetic and NiCI42– and [Ni(CN)4]2– are paramagnetic.

Section (F) : Applications of crystal field theory


(Magnetic moment of complex, Colour of complex, Limitation, Stability of complex)
F-1. The compound which does not show paramagnetism ?
(A) [Cu(NH3)4Cl2 ] (B) [Ag(NH3)2]Cl (C) NO (D) NO2
F-2. Among the following ions, which one has the highest paramagnetism ?
(A) [Cr(H2O)6]3+ (B) [Fe(H2O)6]2+ (C) [Cu(H2O)6]2+ (D) [Zn(H2O)6]2+

Section (G) : Isomerism in Coordination compounds


(Structural Isomerism, Stereoisomerism, Geometrical Isomerism, Optical Isomerism)
G-1. The complexes [Pt(NH3)4] [PtCl6] and [Pt(NH3)4Cl2] [PtCl4] are :
(A) linkage isomers (B) optical isomers
(C) co-ordination isomers (D) ionisation isomers

G-2. [Co(NH3)5NO2]Cl2 and [Co(NH3)5ONO]Cl2 are related to each other as :


(A) geometrical isomers (B) linkage isomers
(C) coordination isomers (D) ionisation isomers

G-3. The number of geometrical isomer of [Co(NH3)3(NO3)3] are :


(A) 0 (B) 2 (C) 3 (D) 4

G-4. Geometrical isomerism is found in coordination compounds having coordination number :


(A) 2 (B) 3 (C) 4 (tetrahedral) (D) 6

G-5. Cis-trans isomerism is found in square planar complexes of molecular formula ('a' and 'b' are
monodentate ligands) :
(A) Ma4 (B) Ma3b (C) Ma2b2 (D) Mab3

G-6. Geometrical isomerism can be shown by :


(A) [Ag(NH3)(CN)] (B) Na2[Cd(NO2)4] (C) [PtCl4I2] (D) [Pt(NH3)3Cl][Au(CN)4]

Section (H) : Organometallic Compounds


H-1. Which one is not an organometallic compound ?
(A) RMgX (B) (C2H5)4Pb (C) (CH3)4Sn (D) C2H5ONa

H-2. Formula of ferrocene is:


(A) [Fe(CN)6]4– (B) [Fe(CN)6]3+ (C) [Fe(CO)5] (D) [Fe(C5H5)2]

ACTIVESITE EDUTECH- 9844532971 147


ACTIVE SITE
a
EDUTECH Co-ordination Compounds
PART - III : MATCH THE COLUMN
1. Match the column :
Column-I Column-II
d2sp3 hybridisation of central
(A) [Fe(en)3]3+ (p)
metal
sp3d2 hybridisation of central
(B) [Co(ox)3]3– (q)
metal
(C) [Cr(CN)6]3– (r) paramagnetic
(D) [NiCl6]4– (s) diamagnetic
(t) metal ion has +3 oxidation state

2. There are some coordination compounds given in column-I which may exist in different isomeric
forms as given in column-II. Select the correct option(s) for the coordination compounds and their
respective isomeric forms.
Column-I Column-II
(A) [Co(en)2NH3Cl]SO4 (p) Enantiomer
Geometrical
(B) [Co(NH3)4(NO2)2](NO3)2 (q)
isomer
(C) [Co(en)(pn)(NO2)2]Cl (r) Ionization isomer
(D) [Co(gly)3] (s) Linkage isomer

 Marked questions are recommended for Revision.


PART - I : ONLY ONE OPTION CORRECT TYPE
1. A complex anion is formed by Osmium (in some oxidation state) with ligands (in proper number so
that coordination number of osmium becomes six). Which of the following can be its correct IUPAC
name?
(A) pentachloridonitridoosmium(VI) (B) pentachloridonitridoosmate(VI)
(C) azidopentachloridoosmate(VI) (D) None of these

2. The EAN of metal atoms in [Fe(CO)2(NO+)2] and Co2(CO)8 respectively are :


(A) 34, 35 (B) 34, 36 (C) 36, 36 (D) 36, 35

3. Which of the following is inner orbital complex as well as diamagnetic in nature ?


(A) [Ir(H2O)6 ]3+ (B) [Ni(NH3)6]2+ (C) [Cr(NH3)6]3+ (D)
[Co(NH3)6]2+

4. Which of the following statement is correct ?


(A) The oxidation state of iron in sodium nitro prusside Na2[Fe(CN)5(NO))] is +3
(B) [Ag(NH3)2]+ is linear in shape
(C) In [Fe(H2O)6]3+ , Fe is d2sp3 hybridized
(D) In Ni(CO)4, the oxidation state of Ni is 1

ACTIVESITE EDUTECH- 9844532971 148


ACTIVE SITE
a
EDUTECH Co-ordination Compounds
5. The complex K4[Zn(CN)4(O2)2] is oxidised into K2[Zn(CN)4(O2)2], then which of the following is
correct?
(A) Zn(II) is oxidised into Zn(IV) (B) Paramagnetic moment decreases
(C) O–O bond length increases (D) Paramagnetic moment increases

6. All the following complexes show decrease in their weights when placed in a magnetic balance then
the group of complexes having tetrahedral geometry is :
I Ni (CO)4 II K [AgF4] III Na2 [Zn (CN)4]
IV K2 [PtCl4] V [RhCl (PPh3)3]
(A) II, III, V (B) I, II, III (C) I, III, IV (D) none of these

7. The complex [Fe(H2O)5NO]2+ is formed in the brown ring test for nitrates when freshly prepared
FeSO4 solution is added to aqueous solution of NO3– ions followed by addition of conc. H2SO4.
Select correct statement about this complex.
(A) Hybridisation of iron is sp3d2.
(B) Iron has +1 oxidation state.
(C) It has magnetic moment of 3.87 B.M. confirming three unpaired electrons in Fe.
(D) All the above are correct statements.

8. Which of the following statements is not correct?


(A) TiCl4 is a colourless compound. (B) [Cr(NH3)6)]Cl3 is a coloured compound.
(C) K3[VF6] is a colourless compound. (D) [Cu(NCCH3)4][BF4] is a colourless
compound.

9. Among TiF62 − , CoF63 − , Cu2Cl2 and NiCl24 − the colourless species are:
(A) CoF63 − and NiCl24 − (B) TiF62 − and CoF63 − (C) NiCl24 − and Cu2Cl2 (D) TiF62 − and Cu2Cl2

10. The number of geometrical isomers for octahedral [Co(NH3)2Cl4]–, square planar AuCl2Br2– are :
(A) 4, 2 (B) 2, 2 (C) 3, 2 (D) 2, 3

11. Which of the following statements is not true about the complex ion [Pt(en)2Cl2]2+ ?
(A) It has two geometrical isomers – cis and trans.
(B) Both the cis and trans isomers display optical activity.
(C) Only the cis isomer displays optical activity.
(D) Only the cis isomer has non–superimposable mirror image.

12. Both geometrical and optical isomerism are shown by :


(A) [Co(en)2Cl2]+ (B) [Co(NH3)5Cl]2+ (C) [Co(NH3)4Cl2]+ (D) [Cr(OX)3]3–

13. Among the following, metal carbonyls, the C–O bond is strongest :
(A) [Mn(CO)6]+ (B) [Cr(CO)6] (C) [V(CO)6]– (D) [Ti(CO)6]2–

PART - II : SINGLE AND DOUBLE VALUE INTEGER TYPE


1. Sum of denticity of following ligands are
Glycinate ion, Oxalate ion, o-phenathroline, 2,2-bipyridyl, diethylenetriamine, ethylenediamine

2. How many total sodium ions are present in one formula unit of sodium ethane-1,2-
diaminetetraacetatochromate (II) and sodium hexanitrito cobaltate (III) ?

ACTIVESITE EDUTECH- 9844532971 149


ACTIVE SITE
a
EDUTECH Co-ordination Compounds
3. A blue colour complex is obtained in the analysis of Fe+3 having formula Fe4[Fe(CN)6]3
Let a = oxidation number of Iron in the coordination sphere
b = no. of secondary valencies of central iron ion.
c = Effective atomic number of Iron in the coordination sphere.
Then find the value of (c + a – 2b)

4. Coordination number of Cr in CrCl3.5H2O as six. The possible volumes of 1 M Ag NO3 needed to
precipitate the chlorine inoutersphere in 200 ml of 0.1 M solution of the complex is/are.
Write your answer as V1 + V2 + V3 + .........

5. Ni+2 form a complex ion in water having the formula [Ni(H2O)6]+2. How many of the following
statements are true for the complex ion ?
(i) The complex is octahedral in shape. (ii) The complex is diamagnetic in
nature.
(iii) Ni+2 has incompletely filled 3d subshell. (iv) Secondary valency of Ni+2 is 6.
(v) All the bonds (metal-ligand) are perpendicular to each other.
(vi) All the 3d orbitals of Ni+2 are degenerate
(vii) Total spin of the complex is 1. (viii) The hybridisation of Ni+2 is d2sp3
+2
(ix) The complex is more stable than [Ni(en)3] (x) Effective atomic number of Ni+2 is 36.

6. How many of the following is correctly matched complex ?


Oxidation no. on central Electronic
Complex
metal configuration
(a) K3[Co(C2O4)3] +3 t 62g
(b) (NH4)2 [CoF4] +2 t 52g eg2
(c) Cis -
+3 t 32g e0g
[Cr(en)2Cl2]Cl
(d) [Mn(H2O)6]SO4 +2 t 32g eg2

7. Total number of paramagnetic complexes which are inner orbital complexes :
(i) [Cr(NH3)6]Cl3 (ii) [Co(NH3)6](NO3)2 (iii) [Ni(NH3)6]SO4
(iv) K2[PtCl6], (v) [V(H2O)6]SO4 (vi) [Mn(NH3)6]SO4
(vii) [Fe(H2O)5 (NO)]SO4 (viii) K3[CuCl4] (ix) Na4[Fe(CN)5(NOS)]

8. The number of coordination isomers possible for [Fe(NH3)6]3+ [Cr(C2O4)3]3– is _______

9. Find the sum of number of geometrical isomers for following complexes.


(a) [CoCl2Br2]2– (b) [Rh(en)3]3+ (c) [Cr(en)2 Br2]+
(d) [Pt en Cl2] (e) [Co(NH3)3(NO2)3]

10. What is the sum of bond order of Fe–C bond and C–O bond in Fe(CO)5 ?

11. How many isomeric forms are possible for the octahedral complex, [Rh(en)2(NO2)(SCN)]+ ?

ACTIVESITE EDUTECH- 9844532971 150


ACTIVE SITE
a
EDUTECH Co-ordination Compounds
PART - III : ONE OR MORE THAN ONE OPTIONS CORRECT TYPE
1. Which of the following statement(s) are incorrect ?
(A) Those additional compounds which lose their identity in solution are called double salts.
(B) In K3[Fe(CN)6] Fe2+ and CN– ion can give quantitative identification test.
(C) [KAl(SO4)2.12H2O] is a coordination compound.
(D) All acids are lewis acids and  donors.

2. The effective atomic number of Co(CO)4 is 35 and hence is less stable. It attains stability by
(A) oxidation of Co (B) reduction of Co (C) dimerization (D) none

3. Select the correct statements ;


(A) Potassium ferrocyanide and potassium ferricyanide can be differentiated by measuring the solid
state magnetic moment.
(B) The complex [Co(NH3)5Br]SO4 and [Co(NH3)5SO4]Br can be differentiated by adding aqueous
solution of barium chloride
(C) The complex [Co(NH3)5Cl]Br and [Co(NH3)5Br]Cl can be differentiated by adding aqueous
solution of silver nitrate.
(D) the complex [Co(NH3)6]Cl3 and [Co(NH3)5Cl]Cl2 can be differentiated by measuring molar
conductance

4. Consider the following statements :


S1 : [MnCl6]3–, [FeF6]3– and [CoF6]3– are paramagnetic having four, five and four unpaired electrons
respectively.
S2 : Low value of formation constant of a complex indicates its high thermodynamic stability.
S3 : The crystal field splitting o, depends upon the field produced by the ligand and charge on the
metal ion.
and arrange in the order of true/ false.
(A) T T T (B) T F T (C) F T F (D) T F F

5. Which of the following is/are correctly matched ?


(A) [Ni(CO)4] - dsp2 and diamagnetic.
(B) [Ni(en)3](NO2)2 - sp3d2 and two unpaired electrons.
(C) [V(NH3)6]Cl3 - sp3d2 and two unpaired electrons.
(D) [Mn(NO+)3(CO)] - sp3 and diamagnetic.

6. Which of the following statement(s) is/are correct with respect to the crystal field theory ?
(A) It considers only the metal ion d–orbitals and gives no consideration at all to other metal
orbitals.
(B) It cannot account for the  bonding in complexes.
(C) The ligands are point charges which are either ions or neutral molecules
(D) The magnetic properties can be explained in terms of splitting of d- orbitals in different crystal
field.

7. 'Spin only' magnetic moment of Ni in [Ni(dmg)2] is same as that found in :


(A) Ni in [NiCl2 (PPh3)2] (B) Mn in [MnO4]–
2–
(C) Co in [CoBr4] (D) Pt in [Pt(H2O)2Br2]

8. Which complex of the following pairs has the larger value of 0 ?

ACTIVESITE EDUTECH- 9844532971 151


ACTIVE SITE
a
EDUTECH Co-ordination Compounds
(i) [Co(CN)6] 3 – and [Co(NH3)6]3+ (ii) [Co(H2O)6]3+ and [Rh(H2O)6]3+
(iii) [Co(H2O)6]3+ and [Co(H2O)6]2+ (iv) [Co(NH3)6]3+ and [CoF6] 3–
Select the correct one
(A) [Co(CN)6]3 – > [Co(H2O)6]3+ (B) [Co(H2O)6]2+ < [Co(H2O)6]3+
(C) [Co(H2O)6]3+ > [Rh(H2O)6 ]3+ (D) [Co(NH3)6]3+ < [CoF6] 3 –

9. Which of the following isomerisms is/are shown by the complex [CoCl2(OH2)2(NH3)2]Br ?


(A) Ionization (B) Linkage (C) Geometrical (D) optical
10. Three arrangements are shown for the complex [Co(en)(NH3)2Cl2]+. Pick up the wrong statement.

(A) I and II are geometrical isomers (B) II and III are optical isomers
(C) I and III are optical isomers (D) II and III are geometrical isomers

11. Consider the following complexies [V(CO)6]–, [Cr(CO)6] and [Mn(CO)6]+. Then incorrect statement
(s) about metal carbonyls is /are.
(A) ‘C–O’ bond is strongest in the cation and weakest in the anion
(B) ‘C–O’ bond order is less in the cation than in anion.
(C) ‘C–O’ bond longer in the cation than in anion or neutral carbonyl.
(D) ‘M–C’ bond order is higher in the cation than in anionic or neutral carbonyl.
12. Following Sidwick's rule of EAN, Co(CO)x will be :
(A) Co2(CO)4 (B) Co2(CO)3 (C) Co2(CO)8 (D) Co2(CO)10
PART - IV : COMPREHENSION
Read the following passage carefully and answer the questions.
Comprehension # 1
In coordination chemistry there are a variety of methods applied to find out the structure of
complexes. One method involves treating the complex with known reagents and from the nature of
reaction, the formula of the complex can be predicted. An isomer of the complex
Co(en)2(H2O)Cl2Br, on reaction with concentrated H2SO4 (dehydrating agent) it suffers loss in
weight and on reaction with AgNO3 solution it gives a white precipitate which is soluble in NH3 (aq).
1. The correct formula of the complex is :
(A) [CoClBr(en)2]H2O (B) [CoCl(en)2(H2O)]BrCl
(C) [Co Br(en)2(H2O)]Cl2 (D) [CoBrCl(en)2]Cl.H2O
2. If all the ligands in the coordination sphere of the above complex be replaced by F–, then the
magnetic moment of the complex ion (due to spin only) wll be :
(A) 2.8 BM (B) 5.9 BM (C) 4.9 BM (D) 1.73 BM
3. Similarly if all the ligands in the coordination sphere be replaced by NO2¯, then the magnetic
moment of the complex ion (due to spin only) will be :
(A) 1.73 BM (B) 0.0 BM (C) 4.9 BM (D) 5.9 BM
4. If one mole of original complex is treated with excess Pb(NO 3)2 solution, then the number of moles
of white precipitate (of PbCl2) formed will be :
(A) 0.5 (B) 1.0 (C) 0.0 (D) 3.0

ACTIVESITE EDUTECH- 9844532971 152


ACTIVE SITE
a
EDUTECH Co-ordination Compounds
5. The number of geometrical isomers of the formula of the above original complex are (including the
complex):
(A) 2 (B) 3 (C) 4 (D) 1

Comprehension # 2
Co2+ (aq.) + SCN– (aq.) ⎯⎯
→ Complex (X).
NH OH
Ni2+ (aq.) + Dimethylglyoxime ⎯⎯⎯⎯ 4
→ Complex (Y).
The corrdination number of cobalt and nickel in complexes X and Y are four.

6. The IUPAC names of the complexes (X) and (Y) are respectively :
(A) tetrathiocyanato-S-cobalt(II) and bis(dimethylglyoximate) nickel(II).
(B) tetrathiocyanato-S-cobaltate (II) and bis(dimethylglyoximato)nickel (II).
(C) tetrathiocyanato-S-cobaltate (II) and bis(dimethylglyoximato)nickelate(II).
(D) tetrathiocyanato-S-cobaltate(III) and bis(dimethylglyoximato)nickel(II).

7. The geometry of complexes (X) and (Y) are respectively :


(A) tetrahedral and square planar.
(B) both tetrahedral.
(C) square planar and tetrahedral
(D) both square planar.

8. Select the correct statement for the complexes (X) and (Y).
(A) (X) is paramagnetic with two unpaired electrons.
(B) (Y) is diamagnetic and shows intermolecular H-bonding.
(C) (X) is paramagnetic with three unpaired electrons and (Y) is diamagnetic.
(D) (X) and (Y) both are diamagnetic.

Comprehension # 3
Q.9, Q.10 and Q.11 by appropriately matching the information given in the three columns of the
following table.
Let us consider following columns
Column 1 Column 2 Column 3
 (in B.M>) Hybridisation state No. of geometrical isomers
(I)  = 2.83 B.M. (i) sp3 (P) 2
(II)  = 5.93 B.M. (ii) sp3d2 (Q) 3
(III)  = 3.88 B.M. (iii) d2sp3 (R) 4
(IV)  = 0 B.M. (iv) dsp2 (S) 5
[Note: Atomic Number of Cr = 24, V = 23, Pt = 78)]
9. About (CrCl3(NH3)3) which of following combination is correct ?
(A) (III), (iii), P (B) (II), (iv), Q (C) (IV), (i), R (D) (I), (ii), S

10. Correct combination for [VCl2(NO2)2(NH3)2]–.


(A) (II), (i), P (B) (I), (iii), S (C) (III), (ii), R (D) (IV), (iv), Q

11. Correct combination for [PtCl2(NH3)2] is :


(A) (II), (iii), Q (B) (I), (iv), S (C) (IV), (iv), P (D) (III), (ii), R

ACTIVESITE EDUTECH- 9844532971 153


ACTIVE SITE
a
EDUTECH Co-ordination Compounds

* Marked Questions may have more than one correct option.


PART - I : JEE (ADVANCED) / IIT-JEE PROBLEMS (PREVIOUS
YEARS)
1. A green complex, K2[Cr(NO)(NH3)(CN)4] is paramagnetic and has eff = 1.73 BM. Write the IUPAC
name of the complex and draw the structure of anion and find out the hybridisation of metal ion.
[JEE 2003, 4/60]

2. The species having tetrahedral shape is : [JEE 2004, 3/84]


(A) [PdCl4]2– (B) [Ni(CN)4]2– (C) [Pd(CN)4] 2–
(D) [NiCl4]2–

3. The spin magnetic moment of cobalt in the compound, Hg [Co(SCN)4] is :[JEE 2004, 3/84]
(A) 3 (B) 8 (C) 15 (D) 24

4. When dimethyl glyoxime is added to the aqueous solution of nickel(II) chloride in presence of dilute
ammonia solution, a bright red coloured precipitate is obtained. [JEE 2004, 4/60]
(a) Draw the structure of bright red substance.
(b) Write the oxidation state of nickel in the substance and hybridisation.
(c) State whether the substance is paramagnetic or diamagnetic.

5. Which kind of isomerism is exhibited by octahedral [Co(NH3)4Br2]Cl ? [JEE 2005, 3/84]


(A) Geometrical and ionization (B) Geometrical and optical
(C) Optical and ionization (D) Geometrical only

6. The bond length in CO is 1.128 Å. What will be the bond length of CO in Fe(CO) 5 ?[JEE 2006,
5/184]
(A) 1.158 Å (B) 1.128 Å (C) 1.178 Å (D) 1.118 Å

Comprehension # (Q.7 to Q.9)


NiCl2 ⎯⎯⎯
KCN
→ complex A
HCN

NiCl2 ⎯⎯

KCl
complex B
excess

A & B complexes have the co-ordination number 4.

7. The IUPAC name of complexes ‘A’ & ‘B’ are respectively : [JEE 2006, 5/184]
(A) Potassium tetracyanonickelate() and Potassium tetrachloronickelate()
(B) Potassium tetracyanonickel() and Potassium tetrachloronickel()
(C) Potassium cyanonickelate() and Potassium chloronickelate()
(D) Potassium cyanonickel() and Potassium chloronickel()

8. The hybridisation of both complexes are : [JEE 2006, 5/184]


(A) dsp2 (B) sp2 & dsp2 2
(C) dsp & sp 3
(D) both sp3

9. What are the magnetic nature of ‘A’ & ‘B’ ? [JEE 2006, 5/184]
(A) Both diamagnetic.
(B) ‘A’ is diamagnetic & ‘B’ is paramagnetic with one unpaired electrons.

ACTIVESITE EDUTECH- 9844532971 154


ACTIVE SITE
a
EDUTECH Co-ordination Compounds
(C) ‘A’ is diamagnetic & ‘B’ is paramagnetic with two unpaired electrons.
(D) Both are paramagnetic.

10. Among the following metal carbonyls, the C–O bond order is lowest in : [JEE 2007, 3/162]
+
(A) [Mn(CO)6] (B) [V(CO)6]¯ (C) [Cr(CO)6)] (D) [Fe(CO)5]

11. Match the complexes in Column-I with their properties listed in Column-II. [JEE 2007, 6/162]
Column-I Column-II
(A) [Co(NH3)4(H2O)2]Cl2 (p) Geometrical isomers
(B) [Pt(NH3)2Cl2] (q) Paramagnetic
(C) [Co(H2O)5Cl]Cl (r) Diamagnetic
Metal ion with +2 oxidation
(D) [Ni(H2O)6]Cl2 (s)
state

12. The IUPAC name of [Ni(NH3)4] [NiCl4] is : [JEE 2008, 3/163]


(A) Tetrachloronickel(II) tetraamminenickel (II) (B) Tetraamminenickel(II) tetrachloronickel
(II)
(C) Tetraamminenickel(II) tetrachloronickelate (II) (D) Tetraamminenickel(II) tetrachloronickelate
(0)

13. Both [Ni(CO)4] and [Ni(CN)4]2– are diamagnetic. The hybridisation of nickel in these complexes,
respectively, are : [JEE 2008, 3/163]
3 3 3 2 2 3
(A) sp , sp (B) sp , dsp (C) dsp , sp (D) dsp2, sp2

14. Statement-1 : The geometrical isomers of the complex [M(NH3)4Cl2] are optically inactive, and
Statement-2 : Both geometrical isomers of the complex [M(NH3)4Cl2] possess axis of symmetry.
[JEE 2008, 3/163]
(A) Statement-1 is true, statement-2 is true; statement-2 is a correct explanation for statement-1.
(B) Statement-1 is true, statement-2 is true; statement-2 is NOT a correct explanation for statement-
1.
(C) Statement-1 is true, Statement-2 is false.
(D) Statement-1 is false, Statement-2 is true.

15. Statement-1 : [Fe(H2O)5NO]SO4 is paramagnetic, and


Statement-2 : The Fe in [Fe(H2O)5NO]SO4 has three unpaired electrons. [JEE 2008, 3/163]
(A) Statement-1 is true, statement-2 is true; statement-2 is a correct explanation for statement-1.
(B) Statement-1 is true, statement-2 is true; statement-2 is NOT a correct explanation for statement-
1.
(C) Statement-1 is true, Statement-2 is false.
(D) Statement-1 is false, Statement-2 is true.

16.* The compound(s) that exhibit(s) geometrical isomerism is(are) : [JEE 2009, 4/160]
(A) [Pt(en)Cl2] (B) [Pt(en)2]Cl2 (C) [Pt(en)2Cl2]Cl2 (D) [Pt(NH3)2Cl2]

17. The spin only magnetic moment value (in Bohr magneton units) of Cr(CO)6 is : [JEE 2009, 3/160]
(A) 0 (B) 2.84 (C) 4.90 (D) 5.92

18. The correct structure of ethylenediaminetetraacetic acid (EDTA) is : [JEE 2010, 3/163]

ACTIVESITE EDUTECH- 9844532971 155


ACTIVE SITE
a
EDUTECH Co-ordination Compounds

(A)

(B)

(C)

(D)

19. The ionization isomer of [Cr(H2O)4Cl (NO2)]Cl is : [JEE 2010, 3/163]


(A) [Cr(H2O)4(O2N)]Cl2 (B) [Cr(H2O)4Cl2](NO2)
(C) [Cr(H2O)4Cl(ONO)]Cl (D) [Cr(H2O)4Cl2(NO2)].H2O

20. The complex showing a spin-only magnetic moment of 2.82 B.M. is : [JEE 2010, 5/163]
(A) Ni(CO)4 (B) [NiCl4] 2– (C) Ni(PPh3)4 (D) [Ni(CN)4] 2–

21. Total number of geometrical isomers for the complex [RhCl(CO)(PPh3)(NH3)] is : [JEE 2010, 3/163]

22. Geometrical shapes of the complexes formed by the reaction of Ni2+ with Cl–, CN– and H2O,
respectively, are [JEE 2011, 3/160]
(A) octahedral, tetrahedral and square planar (B) tetrahedral, square planar and octahedral
(C) square planar, tetrahedral and octahedral (D) octahedral, square planar and octahedral

23. Among the following complexes (K–P),


K3[Fe(CN)6] (K), [Co(NH3)6]Cl3 (L), Na3[Co(oxalate)3] (M), [Ni(H2O)6]Cl2 (N), K2[Pt(CN)4] (O) and
[Zn(H2O)6](NO3)2 (P)
the diamagnetic complexes are : [JEE 2011, 3/160]
(A) K, L, M, N (B) K, M, O, P (C) L, M, O, P (D) L, M, N, O

24. The volume (in mL) of 0.1 M AgNO3 required for complete precipitation of chloride ions present in
30 mL of 0.01 M solution of [Cr(H2O)5Cl]Cl2, as silver chloride is close to. [JEE 2011, 4/160]

25. As per IUPAC nomenclature, the name of the complex [Co(H2O)4(NH3)2]Cl3 is : [JEE 2012, 3/143]
(A) Tetraaquadiaminecobalt (III) chloride (B) Tetraaquadiamminecobalt (III) chloride
(C) Diaminetetraaquacobalt (III) chloride (D) Diamminetetraaquacobalt (III) chloride

26. NiCl2 {P(C2H5)2(C6H5)}2 exhibits temperature dependent magnetic behaviour (paramagnetic/


diamagnetic). The coordination geometries of Ni2+ in the paramagnetic and diamagnetic states are
respectively [JEE 2012, 3/143]
(A) tetrahedral and tetrahedral (B) square planar and square planar
(C) tetrahedral and square planar (D) square planar and tetrahedral

27. Consider the following complex ions, P, Q and R. [JEE(Advanced) 2013, 2/120]

ACTIVESITE EDUTECH- 9844532971 156


ACTIVE SITE
a
EDUTECH Co-ordination Compounds
P = [FeF6]3–, Q = [V(H2O)6]2+ and R = [Fe(H2O)6]2+.
The correct order of the complex ions, according to their spin-only magnetic moment values (in
B.M.) is
(A) R < Q < P (B) Q < R < P (C) R < P < Q (D) Q < P < R

28.* The pair(s) of coordination complexes/ions exhibiting the same kind of isomerism is(are) :
[JEE(Advanced) 2013,
4/120]
(A) [Cr(NH3)5Cl]Cl2 and [Cr(NH3)4Cl2]Cl (B) [Co(NH3)4Cl2]+ and [Pt(NH3)2(H2O)Cl]+
(C) [CoBr2Cl2]2- and [PtBr2Cl2]2- (D) [Pt(NH3)3(NO3)]Cl and [Pt(NH3)3Cl]Br

29. EDTA4– is ethylenediaminetetraacetate ion. The total number of N–Co–O bond angles in
[Co(EDTA)]1– complex ion is : [JEE(Advanced)
2013, 4/120]

30. A list of species having the formula XZ4 is given below.


XeF4, SF4, SiF4. BF4–, BrF4–, [Cu(NH3)4]2+ , [FeCl4]2–, [CoCl4]2– and [PtCl4]2–.
Defining shape on the basis of the location of X and Z atoms, the total number of species having a
square planar shape is [JEE(Advanced) 2014,
3/120]

31. Match each coordination compound in List-I with an appropriate pair of characteristics from List-II
and select the correct answer using the code given below the lists. [JEE(Advanced) 2014,
3/120]
{en = H2NCH2CH2NH2 ; atomic numbers : Ti = 22; Cr = 24; Cp = 27; Pt = 78}
List-I List-II
Paramagnetic and exhibits ionisation
P. [Cr(NH3)4Cl2)Cl] 1.
isomerism
Diamagentic and exhibits cis-trans
Q. [Ti(H2O)5Cl](NO3)2 2.
isomerism
Paramagentic and exhibits cis-trans
R. [Pt(en)(NH3)Cl]NO3 3.
isomerism
Diamagentic and exhibits ionisation
S. [Co(NH3)4(NO3)2]NO3 4.
isomerism
Code :
P Q R S P Q R S
(A) 4 2 3 1 (B) 3 1 4 2
(C) 2 1 3 4 (D) 1 3 4 2

32. For the octahedral complexes of Fe3+ in SCN– (thiocyanato-S) and in CN– ligand environments, the
difference between the spin-only magnetic moments in Bohr magnetons (when approximated to the
nearest integer) is : [Atomic number of Fe = 26] [JEE(Advanced) 2015,
4/168]

33. In the complex acetylbromidodicarbonylbis(triethylphosphine)iron(II), the number of Fe–C bond(s) is


[JEE(Advanced) 2015, 4/168]

34. Among the complex ions, [Co(NH2–CH2–CH2–NH2)2Cl2]+, [CrCl2(C2O4)2]3–, [Fe(H2O)4(OH)2]+,


[Fe(NH3)2(CN)4]–, [Co(NH2–CH2–CH2–NH2)2 (NH3)Cl]2+ and [Co(NH3)4(H2O)Cl ]2+, the number of

ACTIVESITE EDUTECH- 9844532971 157


ACTIVE SITE
a
EDUTECH Co-ordination Compounds
complex ion(s) that show(s) cis-trans isomerism is : [JEE(Advanced) 2015,
4/168]

35. Among [Ni(CO)4], [NiCl4]2–, [Co(NH3)4Cl2]Cl, Na3[CoF6], Na2O2 and CsO2, the total number of
paramagnetic compounds is : [JEE(Advanced) 2016, 3/124]
(A) 2 (B) 3 (C) 4 (D) 5

36. The number of geometric isomers possible for the complex [CoL2Cl2]– (L = H2NCH2CH2O–) is
[JEE(Advanced) 2016, 3/124]

37. The geometries of the ammonia complexes of Ni2+, Pt2+ and Zn2+, respectively, are
[JEE(Advanced) 2016, 3/124]
(A) octahedral, square planar and tetrahedral (B) square planar, octahedral and tetrahedral
(C) tetrahedral, square planar and octahedral (D) octahedral, tetrahedral and square planar

38.* Addition of excess aqueous ammonia to a pink coloured aqueous solution of MCl2.6H2O (X) and
NH4Cl gives an octahedral complex Y in the presence of air. In aqueous solution, complex Y
behaves as 1 : 3 electrolyte. The reaction of X with excess HCl at room temperature results in the
formation of a blue coloured complex Z. The calculated spin only magnetic moment of X and Z is
3.87 B.M., whereas it is zero for complex Y.
[JEE(Advanced) 2017, 4/122]
Among the following options, which statement(s) is (are) correct?
(A) The hybridization of the central metal ion in Y is d2sp3
(B) Addition of silver nitrate to Y gives only two equivalents of silver chloride
(C) When X and Z are in equilibrium at 0ºC, the colour of the solution is pink
(D) Z is a tetrahedral complex

39.* The correct statement(s) regarding the binary transition metal carbonyl compounds is (are)
(Atomic numbers: Fe = 26, Ni = 28) [JEE(Advanced) 2018, 4/128]
(A) Total number of valence shell electrons at metal centre in Fe(CO)5 or Ni(CO)4 is 16
(B) These are predominantly low spin in nature
(C) Metal–carbon bond strengthens when the oxidation state of the metal is lowered
(D) The carbonyl C−O bond weakens when the oxidation state of the metal is increased

40. Among the species given below, the total number of diamagnetic species is ___.
[JEE(Advanced) 2018, 3/120]

H atom, NO2 monomer, O2 (superoxide), dimeric sulphur in vapour phase,
Mn3O4, (NH4)2[FeCl4], (NH4)2[NiCl4], K2MnO4, K2CrO4

41. The ammonia prepared by treating ammonium sulphate with calcium hydroxide is completely used
by NiCl2.6H2O to form a stable coordination compound. Assume that both the reactions are 100%
complete. If 1584 g of ammonium sulphate and 952 g of NiCl2.6H2O are used in the preparation, the
combined weight (in grams) of gypsum and the nickel-ammonia coordination compound thus
produced is ____. (Atomic weights in g mol–1: H = 1, N = 14, O = 16, S = 32, Cl = 35.5, Ca = 40, Ni
= 59) [JEE(Advanced) 2018, 3/120]

42.* The correct option(s) regarding the complex [Co(en)(NH3)3(H2O)]3+


(en = H2NCH2CH2NH2) is (are) [JEE(Advanced) 2018, 4/120]
(A) It has two geometrical isomers
(B) It will have three geometrical isomers if bidentate 'en' is replaced by two cyanide ligands

ACTIVESITE EDUTECH- 9844532971 158


ACTIVE SITE
a
EDUTECH Co-ordination Compounds
(C) It is paramagnetic
(D) It absorbs light at longer wavelength as compared to [Co(en)NH3)4]3+

43. Match each set of hybrid orbitals from LIST-I with complex(es) given in LIST-II.
[JEE(Advanced) 2018, 3/120]
LIST-I LIST-II
(P) dsp2 (1) [FeF6]4–
(Q) sp3 (2) [Ti(H2O)3Cl3]
(R) sp3d2 (3) [Cr(NH3)6]3+
(S) d2sp3 (4) [FeCl4]2–
(5) Ni(CO)4
(6) [Ni(CN)4]2–
The correct option is :
(A) P → 5 ; Q → 4, 6 ; R → 2, 3 ; S → 1 (B) P → 5, 6 ; Q → 4 ; R → 3 ; S → 1, 2
(C) P → 6 ; Q → 4, 5 ; R → 1 ; S → 2, 3 (D) P → 4, 6 ; Q → 5, 6 ; R → 1, 2 ; S → 3

PART - II : JEE (MAIN) / AIEEE PROBLEMS (PREVIOUS YEARS)

JEE(MAIN) OFFLINE PROBLEMS

1. One mole of Co(NH3)5Cl3 gives 3 moles of ions on dissolution in water. One mole of this reacts with
two moles of AgNO3 to give two moles of AgCl. The complex is : [AIEEE 2003,
3/225]
(1) [Co(NH3)4Cl2]Cl.NH3 (2) [Co(NH3)4Cl]Cl2.NH3
(3) [Co(NH3)5Cl]Cl2 (4) [Co(NH3)3Cl3].2NH3

2. Ammonia forms the complex [Cu(NH3)4]2+ with copper ions in alkaline solution but not in acid
solution. The reason for it is : [AIEEE 2003,
3/225]
(1) in alkaline solution Cu(OH)2 is precipitated which is soluble in excess of alkali.
(2) copper hydroxide is amphoteric.
(3) in acidic solution hydration protects Cu2+ ions.
(4) in acidic solution protons coordinates with ammonia molecule forming NH4+ ions and NH3
molecules are not available.

3. In the coordination compound K4[Ni(CN)4], the oxidation state of nickel is : [AIEEE 2003,
3/225]
(1) – 1 (2) 0 (3) + 1 (4) + 2

4. The co-ordination number of a central metal atom in a complex is determined by : [AIEEE 2004,
3/225]
(1) the number of only anionic ligands bonded to metal ion
(2) the number of ligands around a metal ion bonded by pi bonds
(3) the number of ligands around a metal ion bonded by sigma and pi bonds
(4) the number of ligands around a metal ion bonded by sigma bonds

5. Which one is an outer orbital complex ? [AIEEE 2004,


3/225]

ACTIVESITE EDUTECH- 9844532971 159


ACTIVE SITE
a
EDUTECH Co-ordination Compounds
(1) [Ni(NH3)6]2+ (2) [Mn(CN)6]4– (3) [Co(NH3)6]3+ (4) [Fe(CN)6]4–

6. Co-ordination compounds have great importance in biological systems. In this context, which
statement is incorrect ? [AIEEE
2004, 3/225]
(1) Carboxypeptidase–A is an enzyme and contains zinc.
(2) Haemoglobin is the red pigment of blood and contains iron.
(3) Cyanocobalmin is B12 and contains cobalt.
(4) Chlorophylls are green pigments in plants and contain calcium.

7. Which one has largest number of isomers ? [AIEEE 2004, 3/225]


(1) [Co(en)2Cl2]+ (2) [Co(NH3)5Cl]2+ (3) [Ir(PhR3)2H(CO)]2+ (4) [Ru(NH3)4Cl2]+

8. The correct order of magnetic moments (only spin value in BM) among is : [AIEEE 2004,
3/225]
(1) Fe(CN)64– > [CoCl4]2– > [MnCl4]2– (2) [MnCl4]2– > [Fe(CN)6]4– > [CoCl4]2–
(3) [Fe(CN)6]4– > [MnCl4]2– > [CoCl4]2– (4) [MnCl4]2– > [CoCl4]2– > [Fe(CN)6]4–

9. The oxidation state of Cr in [Cr(NH3)4Cl2]+ is : [AIEEE 2005,


1½/225]
(1) 0 (2) +1 (3) +2 (4) +3

10. The IUPAC name of K3Fe(CN)6 is : [AIEEE 2005, 3/225]


(1) Potassium hexacyanoferrate(II) (2) Potassium hexacyanoferrate(III)
(3) Potassium hexacyanoiron(II) (4) Tripotassium hexacyanoiron(II)

11. Which of the following will show optical isomerism ? [AIEEE 2005, 3/225]
(1) [Cu(NH3)4]2+ (2) [ZnCl4]2– (3) [Cr(C2O4)3]3– (4) [Co(CN)6]3–

12. Which one of the following complexes would exhibit the lowest value of paramagnetic behaviour ?
[AIEEE 2005, 3/225]
3– 3– 3–
(1) [Co(CN)6] (2) [Fe(CN)6] (3) [Mn(CN)6] (4)
3–
[Cr(CN)6]

13. The value of 'spin only' magnetic moment for one of the following configurations is 2.84 BM. The
correct one is: (Assuming octahedral complex) [AIEEE 2005,
4½/225]
(1) d4 (in strong field ligand) (2) d4 (in weak field ligand)
3
(3) d (in weak as well as strong field ligand) (4) d5 (in strong field ligand)

14. Nickel (Z = 28) combines with a uninegative monodentate ligand X– to form a paramagnetic
complex [NiX4]2–. The number of unpaired electron(s) in the nickel and geometry of this complex ion
are, respectively : [AIEEE 2006,
3/165]
(1) one, tetrahedral (2) two, tetrahedral (3) one, square planar (4) two, square planar

15. The IUPAC name for the complex [Co(NH3)5(NO2)]Cl2 is : [AIEEE 2006, 3/165]
(1) Nitrito-N-pentaamminecobalt(III) chloride (2) Nitrito-N-pentaamminecobalt(II) chloride
(3) Pentaamminenitrito-N-cobalt(II) chloride (4) Pentaamminenitrito-N-cobalt(III) chloride

ACTIVESITE EDUTECH- 9844532971 160


ACTIVE SITE
a
EDUTECH Co-ordination Compounds
16. In Fe(CO)5, the Fe – C bond possesses : [AIEEE 2006, 3/165]
(1) -character only (2) both  and  characters
(3) ionic character only (4) -character only

17. How many EDTA (ethylenediaminetetraacetic acid) molecules are required to make an octahedral
complex with a Ca2+ ion ? [AIEEE 2006, 3/165]
(1) Six (2) Three (3) One (4) Two

18. The 'spin only' magnetic moment (in units of Bohr magneton, B) of Ni2+ in aqueous solution would
be (atomic number Ni = 28) [AIEEE 2006,
3/165]
(1) 2.84 (2) 4.80 (3) 0 (4) 1.73

19. Which one of the following has a square planar geometry? [AIEEE 2007, 2/120]
(1) [NiCl4]2– (2) [PtCl4]2– (3) [CoCl4]2– (4) [FeCl4]2–
(At. no. Co = 27, Ni = 28, Fe = 26, Pt = 78)

20. The coordination number and the oxidation state of the element 'E' in the complex [E(en) 2(C2O4)]
NO2 (when 'en' is ethylene diamine) are, respectively, [AIEEE
2008, 3/105]
(1) 4 and 2 (2) 4 and 3 (3) 6 and 3 (4) 6 and 2

21. In which of the following octahedral complexes of Co (at no. 27), will the magnitude of 0 be the
highest? [AIEEE 2008, 3/105]
(1) [Co(C2O4)3]3– (2) [Co(H2O)6]3+ (3) [Co(NH3)6]3+ (4) [Co(CN)6]3–

22. Which of the following has an optical isomer ? [AIEEE 2009,


4/144]
(1) [Co (en) (NH3)2]2+ (2) [Co(H2O)4 (en)]3+ (3) [Co (en)2 (NH3)2]3+ (4) [Co (NH3) 3 Cl]+

23. Which of the following pairs represents linkage isomers ? [AIEEE 2009, 4/144]
(1) [Pd(PPh3)2(NCS)2] and [Pd(PPh3)2(SCN)2]
(2) [Co(NH3)5NO3]SO4 and [Co(NH3)5(SO4)]NO3
(3) [PtCl2(NH3)4Br2 and [Pt Br2(NH3)4]Cl2
(4) [Cu(NH3)4][Pt Cl4] and [Pt(NH3)4[CuCl4]

24. A solution containing 2.675 g of CoCl3.6 NH3 (molar mass = 267.5 g mol–1) is passed through a
cation exchanger. The chloride ions obtained in solution were treated with excess of AgNO3 to give
4.78 g of AgCl (molar mass = 143.5 g mol–1). The formula of the complex is (At. mass of Ag = 108
u)
[AIEEE 2010, 8/144]
(1) [Co(NH3)6 ] Cl3 (2) [CoCl2 (NH3)4] Cl (3) [CoCl3(NH3)3] (4) [CoCl(NH3)5] Cl2

25. Which one of the following has an optical isomer ? [AIEEE 2010, 4/144]
(1) [Zn(en)(NH3)2]2+ (2) [Co(en)3]3+ (3) [Co(H2O)4(en)]3+ (4) [Zn(en)2]2+
(en = ethylenediamine)

26. Which of the following facts about the complex [Cr(NH3)6]Cl3 is wrong ? [AIEEE 2011,
4/144]

ACTIVESITE EDUTECH- 9844532971 161


ACTIVE SITE
a
EDUTECH Co-ordination Compounds
(1) The complex involves d2sp3 hybridisation and is octahedral in shape.
(2) The complex is paramagnetic.
(3) The complex is an outer orbital complex.
(4) The complex gives white precipitate with silver nitrate solution.

27. The magnetic moment (spin only) of [NiCl4]2– is : [AIEEE 2011,


4/144]
(1) 1.82 BM (2) 5.46 BM (3) 2.82 BM (4) 1.41 BM

28. Which among the following will be named as dibromidobis (ethylene diamine) chromium (III)
bromide?
[AIEEE 2012, 4/144]
(1) [Cr (en)3]Br3 (2) [Cr(en)2Br2]Br (3) [Cr(en)Br4]– (4) [Cr(en)Br2]Br

29. Which of the following complex species is not expected to exhibit optical isomerism ?
[JEE(Main) 2013, 4/120]
3+ +
(1) [Co(en)3] (2) [Co(en)2 Cl2] (3) [Co(NH3)3 Cl3] (4) [Co(en) (NH3)2 Cl2]+

30. The octahedral complex of a metal ion M3+ with four monodentate ligands L1, L2, L3 and L4 absorb
wavelengths in the region of red, green, yellow and blue, respectively. The increasing order of
ligand strength of the four ligands is : [JEE(Main) 2014,
4/120]
(1) L4 < L3 < L2 < L1 (2) L1 < L3 < L2 < L4 (3) L3 < L2 < L4 < L1 (4) L1 < L2 < L4 < L3

31. The number of geometric isomers that can exist for square planar [Pt(Cl)(py)(NH3)(NH2OH)]+ is
(py = pyridine ) : [JEE(Main) 2015, 4/120]
(1) 2 (2) 3 (3) 4 (4) 6

32. The pair having the same magnetic moment is : [At. No.: Cr = 24, Mn = 25, Fe = 26, Co = 27]
[JEE(Main) 2016, 4/120]
2+ 2+
(1) [Cr(H2O)6] and [Fe(H2O)6] (2) [Mn(H2O)6] and [Cr(H2O)6]2+
2+

2– 2+
(3) [CoCl4] and [Fe(H2O)6] (4) [Cr(H2O)6]2+ and [CoCl4]2–

33. Which one of the following complexes shows optical isomerism ? [JEE(Main) 2016,
4/120]
(1) cis[Co(en)2Cl2]Cl (2) trans[Co(en)2Cl2]Cl (3) [Co(NH3)4Cl2]Cl (4) [Co(NH3)3Cl3]
(en = ethylenediamine)

34. On treatment of 100 mL of 0.1 M solution of CoCl3.6H2O with excess AgNO3; 1.2 × 1022 ions are
precipitated. The complex is : [JEE(Main) 2017, 4/120]
(1) [Co(H2O)3Cl3].3H2O (2) [Co(H2O)6]Cl3
(3) [Co(H2O)5Cl]Cl2.H2O (4) [Co(H2O)4Cl2]Cl.2H2O

35. Consider the following reaction and statements :


[Co(NH3)4Br2]+ + Br– → [Co(NH3)3Br3] + NH3
(I) Two isomers are produced if the reactant complex ion is a cis-isomer.
(II) Two isomers are produced if the reactant complex ion is a tran-isomer.
(III) Only one isomer is produced if the reactant complex ion is a trans-isomer.
(IV) Only one isomer is produced if the reactant complex ion is a cis-isomer.
The correct statements are : [JEE(Main) 2018, 4/120]

ACTIVESITE EDUTECH- 9844532971 162


ACTIVE SITE
a
EDUTECH Co-ordination Compounds
(1) (III) and (IV) (2) (II) and (IV) (3) (I) and (II) (4) (I) and (III)

36. The oxidation states of Cr in [Cr(H2O)6]Cl3, [Cr(C6H6)2], and K2[Cr(CN)2(O)2(O2)(NH3)] respectively


are :
[JEE(Main) 2018, 4/120]
(1) +3, 0, and +6 (2) +3, 0, and +4 (3) +3, +4, and +6 (4) +3, +2, and +4

JEE(MAIN) ONLINE PROBLEMS

1. An octahedral complex of Co3+ is diamagnetic. The hybridisation involved in the formation of the
comlex is: [JEE(Main) 2014 Online (09-04-14),
4/120]
(1) sp3d2 (2) dsp2 (3) d2sp3 (4) sp3d

2. The correct statement about of the magnetic prperties of [Fe(CN)6]3– and [FeF6]3– is : (Z = 26)
[JEE(Main) 2014 Online (09-04-14), 4/120]
(1) both are paramagnetic
(2) both are diamagnetic
(3) [Fe(CN)6]3– is diamagnetic, [FeF6]3– is paramagnetic.
(4) [Fe(CN)6]3– is paramagnetic, [FeF6]3– is diamagnetic.

3. Which of the following name formula combinations is not correct ?


[JEE(Main) 2014 Online (11-04-14), 4/120]
Formula Name
(1) K2[Pt(CN)4] Potasium tetracyanoplatinate (II)
(2) [Mn(CN)5]2– Pentacyanomagnate (II) ion
Potassium diammine tetrachlorochromate
(3) K[Cr(NH3)2Cl4]
(III)
Tetraammine aquaiodo cobalt (III)
(4) [Co(NH3)4(H2O)l]SO4
sulphate

4. Consider the coordination compound, [Co(NH3)6]Cl3. In the formation of the complex, the species
which acts as the Lewis acid is : [JEE(Main) 2014 Online (11-
04-14), 4/120]
(1) [Co(NH3)6]3+ (2) Cl– (3) Co3+ (4) NH3

5. Among the following species the one which causes the highest CFSE, 0 as a ligand is :
[JEE(Main) 2014 Online (12-04-14), 4/120]

(1) CN (2) NH3 (3) F– (4) CO

6. Which one of the following complexes will most likely absorb visible light ?
(At nos. Sc = 21, Ti = 22, V = 23, Zn = 30) [JEE(Main) 2014 Online (12-04-14), 4/120]
3+ 4+
(1) [Sc(H2O)6] (2) [Ti(NH3)6] (3) [V(NH3)6]3+ (4) [Zn(NH3)6]2+

7. An octahedral complex with molecular composition M.5NH3.Cl.SO4 has two isomers, A and B. The
solution of A gives a white precipitate with AgNO3 solution and the solution of B gives white
precipitate with BaCl2 solution. The type of isomerism exhibited by the complex is:
[JEE(Main) 2014 Online (19-04-14), 4/120]
(1) Linkage isomerism (2) Ionisation isomerism

ACTIVESITE EDUTECH- 9844532971 163


ACTIVE SITE
a
EDUTECH Co-ordination Compounds
(3) Coordinate isomerism (4) Geometrical isomerism

8. Nickel (Z = 28) combines with a uninegative monodenate ligand to form a diamagnetic complex
[NiL4]2–. The hybridisation involved and the number of unpaired electrons present in the complex
are respectively: [JEE(Main) 2014 Online (19-04-14),
4/120]
(1) sp3, two (2) dsp2, zero (3) dsp2, one (4) sp3, zero

9. The correct statement on the isomerism associated with the following complex ions,
[JEE(Main) 2015 Online (10-04-15), 4/120]
(a) [Ni(H2O)5NH3]2+, (b) [Ni(H2O)4(NH3)2]2+ and (c) [Ni(H2O)3(NH3)3]2+ is :
(1) (a) and (b) show only geometrical isomerism.
(2) (b) and (c) show geometrical and optical isomerism
(3) (b) and (c) show only geometrical isomerism
(4) (a) and (b) show geometrical and optical isomerism

10. Which molecule/ion among the following cannot act as a ligand in complex compounds?
[JEE(Main) 2015 Online (10-04-15), 4/120]

(1) CH4 (2) CN (3) Br– (4) CO

11. Which of the following complex ions has electrons that are symmetrically filled in both t 2g and eg
orbitals? [JEE(Main) 2015 Online (11-04-15), 4/120]
(1) [FeF6]3– (2) [Mn(CN)6]4– (3) [CoF6]3– (4) [Co(NH3)6]2+

12. Identify the correct trend given below: (Atomic No.: Ti = 22, Cr = 24 and Mo = 42)
[JEE(Main) 2016 Online (09-04-16), 4/120]
(1) o of [Cr(H2O)6] < [Mo(H2O)6] and o of [Ti(H2O)6]3+ < [Ti(H2O)6]2+
2+ 2+

(2) o of [Cr(H2O)6]2+ > [Mo(H2O)6]2+ and o of [Ti(H2O)6]3+ > [Ti(H2O)6]2+


(3) o of [Cr(H2O)6]2+ > [Mo(H2O)6]2+ and o of [Ti(H2O)6]3+ < [Ti(H2O)6]2+
(4) o of [Cr(H2O)6]2+ < [Mo(H2O)6]2+ and o of [Ti(H2O)6]3+ > [Ti(H2O)6]2+

13. Which one of the following complexes will consume more equivalents of aqueous solution of
Ag(NO3) ?
[JEE(Main) 2016 Online (09-04-16), 4/120]
(1) Na3[CrCl6] (2) [Cr(H2O)5Cl]Cl2 (3) [Cr(H2O)6]Cl3 (4) Na2[CrCl5(H2O)]

14. Which of the following is an example of homoleptic complex ?


[JEE(Main) 2016 Online (09-04-16), 4/120]
(1) [Co(NH3)4Cl2] (2) [Co(NH3)6]Cl3 (3) [Co(NH3)5Cl]Cl2 (4) [Pt(NH3)2Cl2]

15. sp3d2 hybridization is not displayed by : [JEE(Main) 2017 Online (08-04-17),


4/120]
(1) PF5 (2) SF6 (3) [CrF6]3– (4) BrF5

16. [Co2(CO)8] displays : [JEE(Main) 2017 Online (09-04-17), 4/120]


(1) one Co–Co bond, four terminal CO and four bridging CO
(2) one Co–Co bond, six terminal CO and two bridging CO
(3) no Co–Co bond, four terminal CO and four bridging CO
(4) no Co–Co bond, six terminal CO and two bridging CO

ACTIVESITE EDUTECH- 9844532971 164


ACTIVE SITE
a
EDUTECH Co-ordination Compounds

17. The correct combination is : [JEE(Main) 2018 Online (15-04-18), 4/120]


(1) [NiCl4] – square-planar; [Ni(CN)4] – paramagnetic
2– 2–

(2) [Ni(CN)4]2– – tetrahedral; [Ni(CO)4] – paramagnetic


(3) [NiCl4]2– – paramagnetic; [Ni(CO)4] – tetrahedral
(4) [NiCl4]2– – diamagnetic; [Ni(CO)4] – square-planar

18. The correct order of spin-only magnetic moments among the following is :
(Atomic number : Mn = 25, Co = 27, Ni = 28, Zn = 30) [JEE(Main) 2018 Online (15-04-18),
4/120]
(1) [ZnCl4]2– > [NiCl4]2– > [CoCl4]2– > [MnCl4]2– (2) [CoCl4]2– > [MnCl4]2– > [NiCl4]2– > [ZnCl4]2–
(3) [NiCl4]2– > [CoCl4]2– > [MnCl4]2– > [ZnCl4]2– (4) [MnCl4]2– > [CoCl4]2– > [NiCl4]2– > [ZnCl4]2–

19. The total number of possible isomers for square-planar [Pt(Cl)(NO2)(NO3) (SCN)]2– is :
[JEE(Main) 2018 Online (15-04-18), 4/120]
(1) 8 (2) 12 (3) 16 (4) 24

20. In a complexometric titration of metal ion with ligand


M (Metal ion) + L (Ligand) → C (Complex) end point is estimated spectrophotometrically (through
light absorption). If 'M' and 'C' do not absorb light and only 'L' absorbs, then the titration plot
between absorbed light (A) versus volume of ligand 'L' (V) would look like :
[JEE(Main) 2018 Online (16-04-18), 4/120]

A A A A
(1) (2) (3) (4)

V (Ligand) V (Ligand) V (Ligand) V (Ligand)

21. In Wilkinson's catalyst, the hybridization of central metal ion and its shape are respectively :
[JEE(Main) 2018 Online (16-04-18), 4/120]
(1) sp3d, trigonal bipyramidal (2) d2sp3, octahedral
(3) dsp2, square planar (4) sp3, tetrahedral

22. Which of the following complexes will show geometrical isomerism ?


[JEE(Main) 2018 Online (16-04-18), 4/120]
(1) Potassium tris(oxalato)chromate(III)
(2) Pentaaquachlorochromium(III)chloride
(3) Aquachlorobis(ethylenediamine)cobalt(II) chloride
(4) Potassium amminetrichloroplatinate(II)

23. Two complexes [Cr(H2O)6]Cl3 (A) and [Cr(NH3)6]Cl3 (B) are violet and yellow coloured respectively.
The incorrect statement regarding them is : [JEE(Main) 2019 Online (09-01-19),
4/120]
(1) 0 value for (A) is less than that of (B).
(2) both absorb energies corresponding to their complementary colors.
(3) 0 values of (A) and (B) are calculated from the energies of violet and yellow light, respectively.
(4) both are paramagnetic with three unpaired electrons.

ACTIVESITE EDUTECH- 9844532971 165


ACTIVE SITE
a
EDUTECH Co-ordination Compounds
24. Homoleptic octahedral complexes of a metal ion 'M3+' with three monodentate ligands L1, L2 and L3
absorb wavelengths in the region of green, blue and red respectively. The increasing order of the
ligand strength is : [JEE(Main) 2019 Online (09-01-19),
4/120]
(1) L1 < L2 < L3 (2) L3 < L2 < L1 (3) L2 < L1 < L3 (4) L3 < L1
< L2

25. The complex that has highest crystal field splitting energy (), is :
[JEE(Main) 2019 Online (09-01-19), 4/120]
(1) K2[COCl4] (2) [CO(NH3)5(H2O)]Cl3 (3) [CO(NH3)5Cl]Cl2 (4) K3[CO(CN)6]

26. Wilkinson catalyst is : [JEE(Main) 2019 Online (10-01-19), 4/120]


(1) [(Et3P)3rCl] (Et = C2H5) (2) [(Et3P)3RhCl]
(3) [(Ph3P)3RhCl] (4) [(Ph3P)3rCl]

27. The total number of isomers for a square planar complex [M(F) (Cl) (SCN) (NO2)] is :
[JEE(Main) 2019 Online (10-01-19), 4/120]
(1) 16 (2) 4 (3) 12 (4) 8

28. The difference in the number of unpaired electrons of a metal ion in its high-spin and low-spin
octahedral complexes is two. The metal ion is: [JEE(Main) 2019 Online (10-01-19),
4/120]
(1) Co2+ (2) Fe2+ (3) Mn2+ (4) Ni2+

29. A reaction of cobalt(III) chloride and ethylenediamine in a 1 : 2 mole ratio generates two isomeric
products A (violet coloured) and B(green coloured). A can show optical activity, but, B is optically
inactive. What type of isomers does A and B represent? [JEE(Main) 2019 Online (10-01-19),
4/120]
(1) Ionisation isomers (2) Linkage isomer
(3) Coordination isomers (4) Geometrical isomers

30. Match the metals (column I) with the coordination compound(s)/ emzyme (s) (column II):
[JEE(Main) 2019 Online (11-01-19), 4/120]
(column I) (column II)
Metal Coordination compound(s)/enzyme(s)
(A) Co (i) Wilkinson catalyst
(B) Zn (ii) ChlorophyII
(C) Rh (iii) Vitamin B12
(D) Mg (iv) Carbonic anhydrase
(1) (A)-(i);(B)-(ii);(C)-(iii);(D)-(iv) (2) (A)-(iv);(B)-(iii);(C)-(i);(D)-(ii)
(3) (A)-(iii);(B)-(iv);(C)-(i);(D)-(ii) (4) (A)-(ii);(B)-(i);(C)-(iv);(D)-(iii)

31. The coordination number of Th in K4[Th(C2O4)4(OH2)2] is: [JEE(Main) 2019 Online (11-01-19),
4/120]
(C O
2
2–
4 = Oxalato )
(1) 14 (2) 10 (3) 6 (4) 8

32. The number of bridging CO ligand(s) and Co-Co bond (s) in Co2(CO)8, respetively are :
[JEE(Main) 2019 Online (11-01-19), 4/120]

ACTIVESITE EDUTECH- 9844532971 166


ACTIVE SITE
a
EDUTECH Co-ordination Compounds
(1) 4 and 0 (2) 0 and 2 (3) 2 and 1 (4) 2 and 0

33. The metal d-orbitals that are directly facing the ligands in K3[Co(CN)6] are :
[JEE(Main) 2019 Online (12-01-19), 4/120]
(1) dxz, dyz and dz 2 (2) dxy and d x 2 − y 2 (3) dxy, dxz and dyz (4) d x 2 − y 2 and dz 2

34. Mn2(CO)10 is an organometallic compound due to the presence of :


[JEE(Main) 2019 Online (12-01-19), 4/120]
(1) Mn – C bond (2) C – O bond (3) Mn – O bond (4) Mn – Mn bond

35. The magnetic moment of an octahedral homoleptic Mn(II) complex is 5.9 BM. The suitable ligand
for this complex is: [JEE(Main) 2019 Online (12-01-19),
4/120]
(1) Ethylenediamine (2) CN– (3) NCS– (4) CO

ACTIVESITE EDUTECH- 9844532971 167


ACTIVE SITE
a
EDUTECH Co-ordination Compounds

EXERCISE - 1
PART - I
A-1. K2SO4.Cr2(SO4)3 2K (aq) + 2Cr (aq) + 4SO42–
+ 3+

So chrome alum is a double salt. It when dissolved in water gives its constituent ions. Hence it
gives the test of K+, Cr3+ and SO42– ions.
CuSO4.4NH3 [Cu(NH3)4] SO4 [Cu(NH3)4]2+ (aq) + SO42– (aq)
As copper (II) is present in coordination sphere it will not give the test of Cu2+ ion.

A-2. The coordination number of the central atom/ion is determined by the number of sigma bonds
between the ligands and the central atom/ions i.e. the number of ligand donor atoms to which the
metal is directly attached.
The oxidation number of the central atom is defined as the charge it would carry if all the ligands
are removed along with the electron pairs that are shared with the central atom.
Coordination Oxidation
Complex
Number State

(a) [AgCl2] 2 1
(b) [Cr(H2O)5Cl]2+ 6 3
2–
(c) [Co(NCS)4] 4 2
(d) [Co(NH3)3(NO2)3] 6 3

(e) [Fe(EDTA)] 6 3
(f) [Cu(en)2]SO4 4 2
(g) K[Pt(NH3)Cl5] 6 4

A-3. (A) methyl isocyanide, monodentate. (B) acetylacetonato, bidentate


(C) azido, monodentate (D) diethylenetriamine, tridentate
(E) ethylenediamine tetraacetato, hexadentate (F) ethylenediamine triacentato, pentadentate
(G) oxalato, bidentate (H) dimethylglyoximato
(I) isocyanido, monodentate (J) nitrito, monodentate
(K) oxido, monodentate (L) superoxido, monodentate

A-4. (A) nitrito-N M  O—N=O nitrito-O


(B) M  SCN thiocyanato or thiocyanato-S, M  NCS isothiocyanato or thiocyanato-N

(C) or dithioxalate

(D) M  OCN cyanato-O or cyanato-N, M  NCO isothiocyanato or thiocyanato-N


(E) M  NOS thionitrito-N or , M  SON thionitrito-S

A-5.

ACTIVESITE EDUTECH- 9844532971 168


ACTIVE SITE
a
EDUTECH Co-ordination Compounds
O 3–

C O
O C
O O
C O
(a) Fe
C O O
O
O C
C O

O
[Fe(C2O4)3]3– [Pt(en)2]2+
The ligands, oxalate and ethylenediamine are bidentate as each ligand has two donor
atoms. So in 1st case the number of chelate rings (five membered) are three where as in
2nd case the number of chelate rings (five membered) are two.
The coordination number and oxidation state of iron are six and +3 respectively and the
coordination number and oxidation state of platinum are four and +2 respectively.
(b) K2[Ni(CN)4] 2K+(aq) + [Ni(CN)4]2–(aq)
[Cr(en)3]Cl3 [Cr(en)3]3+(aq) + 3Cl–(aq)
Fe4[Fe(CN)6]3 4Fe3+(aq) + 3[Fe(CN)6]4–(aq)
[PtCl2(en)2] (NO3)2 [PtCl2(en)2]2+ (aq) + 2(NO3)–(aq)
So, [Ni(CN)4] , [Cr(en)3]3+, 3[Fe(CN)6]4– and [PtCl2(en)2]2+ are coordination entities and K+,
2–

Cl–, Fe3+ and NO3– are counter ions.


(c) Coordination compounds are acid-base adduct. Cations are electron deficient, therefore, are
called Lewis acids where as ligands are electrons donors, therefore, are called as Lewis
base.
LEWIS
LEWIS BASE
ACID
(i) [HgBr4] 2-
Hg2+ 4Br–
(ii) [Ni(H2O)6]2+ Ni2+ 6H2O
(iii) [PdCl2(NH3)2] Pd2+ 2Cl– & 2NH3
(iv) [Al(OH)4]– Al3+ 4OH–
(v) [Ag(CN)2]- Ag+ 2CN–
(vi) [Cr(CO)6] Cr0 6CO

B-1.
(a) [Co(NH3)6]Cl3 Hexaamminecobalt(III) chloride
(b) [Rh(NH3)5I]I2 Pentaammineiodidorhodium(III) iodide
(c) [Fe(CO)5] Pentacarbonyliron(0)
Trioxalatoferrate(III) ion OR
(d) [Fe(C2O4)3]3–
Tris(oxalato)ferrate(III) ion
(e) [Cu(NH3)4]SO4 Tetraamminecopper(II) sulphate
(f) Na[Cr(OH)4] Sodium tetrahydroxidochromate(III)
(g) [Co(gly)3] Triglycinatocobalt(III) OR Tris(glycinato)cobalt(III)
(h) [Fe(H2O)5(SCN)]2+ Pentaaquathiocyanato–S–iron(III) ion
(i) K2[HgI4] Potassium tetraiodidomercurate(II)
(j) Co[Hg(SCN)4] Cobalt(II) tetrathiocyanato–S–mercurate(II)
(k) Fe4[Fe(CN)6]3 Iron(III) hexacyanidoferrate(II)

ACTIVESITE EDUTECH- 9844532971 169


ACTIVE SITE
a
EDUTECH Co-ordination Compounds
(l) K3[Co(NO2)6] Potassium hexanitrito–N–cobaltate(III)
(m) [Ni(dmg)2] Bis(dimethylglyoximato)nickel(II)
(n) K2[PtCl6] Potassium hexachloridoplatinate(IV)
(o) Na2[Fe(CN)5NO+] Sodium pentacyanidonitrosoniumferrate(II)
(p) [Fe(H2O)5(NO+)]SO4 Pentaaquanitrosoniumiron(I) sulphate
(q) [Cu(CN)4]3– Tetracyanidocuperate(I) ion
(r) (NH4)2[PtCl6] Ammonium hexachloridoplatinate(IV)

B-2.
(a) [CoBr(en)2(ONO)]+1 Bromidobis(ethylenediamine)nitrito-O-cobalt(III)
(b) [Co(NH3)6][Co(ONO)6] Hexaamminecobalt(III) hexanitrito-O-cobaltate(III)
(c) [Co(NH3)5(CO3)]Cl Pentaamminecarbonatocobalt(III) chloride
Tetraamminedichloridoplatinum(IV)
(d) [Pt(NH3)4Cl2][PtCl4]
tetrachloridoplatinate(II)
Tris(ethylenediamine)cobalt(III) sulphate or
(e) [Co(en)3]2(SO4)3
Tris(ethane-1, 2-diamine)cobalt(III) sulphate.
[(NH3)5Co-NH2- Pentaamminecobalt(III)--
(f)
Co(NH3)4(H2O)]Cl5 amidotetraammineaquacobalt (III) chloride
(g) [Cr(CO)5(PPh3)] Pentacarbonyltriphenylphosphinechromium(0)
(h) [(CO)5Mn-Mn(CO)5] Decacarbonyldimanganese(0)
(i) Cr(-C6H6)2 Bis(6-benzene)chromium(0)
(j) [Co(NH3)4(OH2)2][BF4]3 Tetraamminediaquacobalt(III) tetrafluoridoborate(III)
(l) Ba[Zr(OH)2(ONO)2(ox)] Barium dihydroxidodinitrito-O-oxalatozirconate(IV)
(l) [Co(NH3)6][Co(C2O4)3] Hexaamminecobalt(III) trioxalatocobaltate(III)

B-3.
(a) Tetraamminezinc(II) Nitrate [Zn(NH3)4](NO3)2
(b) Tetracarbonylnickel(0) [Ni(CO)4]
Potassium
(c) K[Pt(NH3)Cl3]
amminetrichloridoplatinate(II)
(d) Dicyanidoaurrate(I) ion [Au(CN)2]–
(e) Sodium hexafluoridoaluminate(III) Na3[AlF6]
(f) Diamminesilver(I) ion [Ag(NH3)2]+

B-4.
(a) Diamminetriaquahydroxidochromium(III) nitrate [Cr(NH3)2(H2O)3(OH)](NO3)2
(b) Tetrakis(pyridine)platinum(II) tetraphenylborate(III) [Pt(Py)4][B(ph)4]2
(c) Dibromidotetracarbonyliron(II) [Fe(Br)2(CO)4]
Ammonium
(d) (NH4)[Cr(NH3)2(NCS)4]
diamminetetrakis(isothiocyanato)chromate(III)
(e) Pentaamminedinitrogenruthenium(II) chloride [Ru(NH3)5N2]Cl2
(f) Barium dihydroxidodinitrito-O-oxalatozirconate(IV) Ba[Zr(OH)2(ONO)2(ox)]
(g) Tetrapyridineplatinum(II) tetrachloridoplatinate(II) [Pt(py)4][PtCl4]
C-1. (a) – iv, (b) – viii, (c) – i, (d) – vii,
(e) – iii, (f) – v, (g) – ii, (h) – vi

ACTIVESITE EDUTECH- 9844532971 170


ACTIVE SITE
a
EDUTECH Co-ordination Compounds
C-2. 0.0075.

C-3. (a) 36 (b) 36 (c) 36 (d) 36


(e) 36 (f) 36 (g) 54 (h) 86

C-4. ii < i < iv < iii.

D-1. (i) [Cr(NH3)4Cl Br]Cl [Cr(NH3)4Cl Br]+ + Cl–; Ag+ + Cl– ⎯⎯ → AgCl  (white) ; soluble in dilute
NH3.
[Cr(NH3)4Cl2]Br [Cr(NH3)4Cl2]+ + Br–; Ag+ + Br– ⎯⎯ → AgBr  (yellow) ; soluble in conc. NH3.
So, A = [Cr(NH3)4Cl Br]Cl and B = [Cr(NH3)4Cl2]Br.
(ii) In both complexes chromium is in +3 oxidation state. Chromium with 3d3 configuration has 3
unpaired electrons with weak field as well as strong field ligand. So, the hybridisation scheme is as
follow :

(iii)  = n (n + 2) = 15
(iv) EAN = 24 – 3 + 12 = 33
(v) Yes, both have two ions per formula unit.
(vi) AgCl + 2NH3 [Ag(NH3)2]Cl ; AgBr + 2NH3 [Ag(NH3)2]Br

D-2.
Complex Hybridization Geometry
(a) [NiBr4]2– sp3 Tetrahedral
square
(b) [AuCl4]– dsp2
planar
square
(c) [Pt(NH3)4]2+ dsp2
planar

E-1. (i) potasium amminetetracyanidonitrosoniumchromate(I)


(ii) Octahedral
(iii) One unpaired electron (iv) It is paramagnetic with one unpaired electron
(v) EAN = 24 – 1 + 2 × 6 = 35 (vi) d2sp3

E-2.
Complex Hybridization Geometry
(a) [Fe(CN)6]3– d2sp3 octahedral
(b) [MnBr4]2– sp3 Tetrahedral
(c) [Fe(H2O)6]2+ sp3d2 Octahedral
(d) [Co(SCN)4]2– sp3 Tetrahedral

E-3. Since ammonia is a strong field ligand so can pair up the electrons of Co(III), so will form an inner
d-orbital complex having zero magnetic moment while fluoride being a weak field ligand can not pair

ACTIVESITE EDUTECH- 9844532971 171


ACTIVE SITE
a
EDUTECH Co-ordination Compounds
up electrons and forms outer d-complex with higher magnetic moment equal to four unpaired
electrons.

E-4. (a) i < iv < ii < iii (b) X < O < N < C (c) Br– < S2– < NO3– < H2O < NH3 < NO2– < CN–
< CO

E-5.

F– is weak
So number of unpaired
(a) field ligand.
electrons = 3
Cr3+, 3d3

eg

bari centre
H2O is weak d-orbitals So number of unpaired
in presence
(b) field ligand. of ligand field electrons = 2
V3+, 3d2 t2g

d-orbitals
in absence
of ligand field
eg

bari centre
CN– is strong d-orbitals So number of unpaired
in presence
(c) field ligand. of ligand field electron = 1.
Fe3+, 3d5 t2g

d-orbitals
in absence
of ligand field

ACTIVESITE EDUTECH- 9844532971 172


ACTIVE SITE
a
EDUTECH Co-ordination Compounds
eg

bari centre
en is strong d-orbitals So number of unpaired
in presence
(d) field ligand. of ligand field
electron = 1.
Cu2+, 3d9
t2g

d-orbitals
in absence
of ligand field
eg

bari centre
F– is weak d-orbitals So number of unpaired
in presence
(e) field ligand. of ligand field
electrons = 5.
Fe3+, 3d5
t2g

d-orbitals
in absence
of ligand field

F-1. As 3d6 configuration has higher CFSE as compared to 3d7 so it gets oxidised in presence of
complexing reagent to easily have d2sp3 hybridisation.

F-2. Yellow colour

F-3. (a) CN–, F–


(b) (i) 0 (ii) 0 (iii) 0 (iv) 0

G-1. (i) Linkage (ii) Coordination (iii) Ionisation (iv) Hydrate

G-2. (a) There are three constitutional isomers


(i) [Ru(NH3)5(NO2)]Cl
(ii) [Ru(NH3)5Cl](NO2) or [Ru(NH3)5Cl]ONO
(iii) [Ru(NH3)5 ONO]Cl
(i) & (ii) are ionisation isomers
(i) & (iii) are linkage isomers

(b) (i) (ii) (iii)

(iv) (v) (vi)

G-3. (a) Two (b) None (c) Two (d) None (e) Two (f) None

G-4. (a) No ; (b) Yes ; (c) Yes ; (d) Yes ; (e) Yes ; (f) No.

ACTIVESITE EDUTECH- 9844532971 173


ACTIVE SITE
a
EDUTECH Co-ordination Compounds
CO CO

CO CO
OC OC

H-1. (a) V (b) Cr

CO CO
OC OC

CO CO
CO
OC
OC CO CO

OC
(c) OC Mn Mn CO (d) Fe CO

OC
CO CO
CO CO
CO
CO

(e) Ni

OC CO
CO

H-2. (i) Increases (ii) Decreases

PART - II
A-1. (B) A-2. (C) A-3. (A) A-4. (B) A-5. (B)

A-6. (D) A-7. (B) A-8. (A) A-9. (D) B-1. (C)

B-2. (A) B-3. (D) B-4. (D) B-5. (C) C-1. (C)

C-2. (B) C-3. (C) C-4. (A) C-5. (A) C-6. (B)

C-7. (C) C-8. (D) C-9. (C) D-1. (A) D-2. (C)

D-3. (C) D-4. (B) D-5. (A) D-6. (A) E-1. (C)

E-2. (A) E-3. (A) E-4. (B) E-5. (A) E-6. (D)

E-7. (D) E-8. (C) E-9. (A) E-10. (C) E-11. (C)

E-12. (C) F-1. (B) F-2. (B) G-1. (C) G-2. (B)

G-3. (B) G-4. (D) G-5. (C) G-6. (C) H-1. (D)

ACTIVESITE EDUTECH- 9844532971 174


ACTIVE SITE
a
EDUTECH Co-ordination Compounds
H-2. (D)

PART - III
1. (A - p,r,t); (B - p,s,t); (C - p,r,t); (D - q,r) 2. (A - p,q,r); (B - q,r,s) ; (C - p,q,r,s) ; (D -
p,q)
EXERCISE - 2
PART - I
1. (B) 2. (C) 3. (A) 4. (B) 5. (D)
6. (D) 7. (D) 8. (C) 9. (D) 10. (B)
11. (B) 12. (A) 13. (A)

PART - II

1. 13 2. 05 3. 26 4. (40 + 20) ml = 60 ml
5. 4 (i, iii, iv, vii) 6. 3 (a, c, d) 7. 2 (i & v) 8. 4
9. 4 (0 + 0 + 2 + 0 + 2 = 4) 10. 4 11. 12

PART - III
1. (BCD) 2. (BC) 3. (ABCD) 4. (B) 5.
(BD)
6. (ABCD) 7. (BD) 8. (AB) 9. (ACD) 10.
(BCD)
11. (BCD) 12. (C)

PART - IV
1. (D) 2. (C) 3. (B) 4. (A) 5. (A)

6. (B) 7. (A) 8. (C) 9. (A) 10. (B)

11. (C)

EXERCISE - 3
PART - I
1. IUPAC name is :
Potassium amminetatracyanidonitrosoniumchromate(I)
or Potassium amminetatracyanidonitrocyliumchromate(I).
Let n is the number of unpaired electron in the chromium ion.
Since  = n(n + 2) or 1.73 = n(n + 2) B.M. or 1.73 × 1.73 = n2 + 2n.
Hence n = 1.
As the CN– and NH3 are strong fields ligands, they compel for pairing of electrons. So,

ACTIVESITE EDUTECH- 9844532971 175


ACTIVE SITE
a
EDUTECH Co-ordination Compounds

[Cr(NO)(CN)4(NH3)]2– =
Hence, the oxidation state of chromium is +1 (having 3d5 configuration). So according to charge on
the complex NO should be NO+ and the structure of this complex is octahedral with d2sp3
hybridisation as given below

2. (D) 3. (C)

Ni2+ + 2dmg ⎯⎯⎯→ [Ni(dmg)2]  (bright red).


NH OH
4. 4

It acquires stability through chelation and intra molecular H-bonding.


In [Ni(dmg)2] the nickel is in +2 oxidaiton state and to have square planar geometry because of
chelation the pairing of electrons takes place. So

[Ni(dmg)2]

As all electrons are paired, so complex is diamagnetic. Nickel with coordination number four will
have the structure as given below.

CH3 – C = N N = C – CH3
+2
Ni
H3C – C = N N = C – CH3

rosy red ppt


5. (A) 6. (A) 7. (A) 8. (C) 9. (C)
10. (B) 11. (A - p,q,s); (B - p,r,s); (C - q,s); (D - q,s) 12. (C) 13. (B)
14. (B) 15. (A) 16.* (CD) 17. (A) 18. (C)
19. (B) 20. (B) 21. 3 22. (B) 23. (C)
24. 6 25. (D) 26. (C) 27. (B) 28.* (BD)
29. 8 30. 4 31. (B) 32. 4 33. 3
34. 6 35. (B) 36. 5 37. (A) 38. (ACD)
39.* (BC) 40. 1 41. 2992 42.* (ABD) 43. (C)

PART - II
JEE(MAIN) OFFLINE PROBLEMS
1. (3) 2. (4) 3. (2) 4. (4) 5. (1)

ACTIVESITE EDUTECH- 9844532971 176


ACTIVE SITE
a
EDUTECH Co-ordination Compounds
6. (4) 7. (1) 8. (4) 9. (4) 10. (2)
11. (3) 12. (1) 13. (1) 14. (2) 15. (4)
16. (2) 17. (3) 18. (1) 19. (2) 20. (3)
21. (4) 22. (3) 23. (1) 24. (1) 25. (2)
26. (3) 27. (3) 28. (2) 29. (3) 30. (2)
31. (2) 32. (1) 33. (1) 34. (4) 35. (4)
36. (1)
JEE(MAIN) ONLINE PROBLEMS
1. (3) 2. (1) 3. (2) 4. (3) 5. (4)
6. (3) 7. (2) 8. (2) 9. (3) 10. (1)
11. (1) 12. (4) 13. (3) 14. (2) 15. (1)
16. (2) 17. (3) 18. (4) 19. (2) 20. (1)
21. (3) 22. (3) 23. (3) 24. (4) 25. (4)
26. (3) 27. (3) 28. (1) 29. (4) 30. (3)
31. (2) 32. (3) 33. (4) 34. (1) 35. (3)

ACTIVESITE EDUTECH- 9844532971 177


08.
Coordination Compounds
4. IUPAC name of the complex,
1. Isomerism of Coordination K3[Al(C2O4)3] is:
Compound (a) Potassium alumino oxalate
(b) Potassium trioxalatoaluminate (III)
(c) Potassium aluminium (III) oxalate
1. During the qualitative analysis of salt with
(d) Potassium trioxalato aluminate (VI)
cation y2+, addition of a reagent (X) to alkaline
solution of the salt gives a bright red CG PET-22.05.2022
precipitate. The reagent (X) and the cation (y2+) Ans. (b) : The given compound is K3[Al(C2O4)3]. The
present respectively are: IUPAC name of the compound is potassium trioxalato
(a) Dimethylglyoxime and Ni2+ aluminate (III).
(b) Dimethylglyoxime and Co2+ 5. Among the following the compounds, that is
(c) Nessler's reagent and Hg2+ both paramagnetic and coloured:
(d) Nessler's reagent and Ni2+ (a) K2Cr2O7 (b) (NH4)2 [TiCl6]
JEE Main-24.06.2022, Shift-I (c) VOSO4 (d) K3[Cu(CN)4]
CG PET-22.05.2022
Ans. (a) : Ni 2 + dmg 
AmmonicalSolution
→[Ni(dmg) 2 ] ↓
Ans. (c) : The given compound is K3[Al(C2O4)3]. The
2. One mole of Co(NH3)5 Cl3, gives 3 moles of ions compound which has the unpaired electron, shows the
on dissolution in water. One mole of the same paramagnetic behaviour.
complex reacts with two moles of AgNO3 (a) K2Cr2O7
solution to yield two moles of AgCl. The Let, x be the oxidation state of Cr.
structure of the complex is: 2(+1) + 2x + 7 (−2) = 0
(a) [Co(NH3)5Cl]Cl2 2 + 2x − 14 = 0
(b) [Co(NH3)3Cl3] .2NH3 x = +6
(c) [Co(NH3)4Cl2]Cl.NH3 Cr6+ = [Ar]3d0 4s0
(d) [Co(NH3)4Cl]Cl2.NH3 The compound is diamagnetic but it shows the colour
CG PET-22.05.2022 due to ligand to metal charge transfer.
Ans. (a) : The dissolution of complex is given as – (b) (NH4)2[TiCl6]
Let, x be the oxidation state of Ti.
2(+1) + x + 6 (−1) = 0
x = +4
2Cl− + 2Ag+ → 2AgCl Ti4+ = [Ar]3d0
3. In following compounds, which one contains There is no electron present in the outer shell that’s why
least oxidation state of iron: it does not show the paramagnetic character as well as
(a) K3[Fe(OH)6] (b) K2 [FeO4] colour.
(c) FeSO4(NH4)2SO4.6H2O (d) K3[Fe(CN)6] (c) VOSO4
CG PET-22.05.2022 Let, x be the oxidation state of V.
x + 1(−2) + 1 (−2) = 0
Ans. (c) : Let, x be the oxidation state of Fe of all
compound x = +4
(a) K3[Fe(OH)6] V4+ = [Ar] 3d1
It has the one unpaired electron thus it shows the
3(+1) + x + 6 (−1) = 0 paramagnetic character and electron transition take
x=+3 place from lower energy state to higher energy state
(b) K2[FeO4] thus shows the colour.
2(+1) + x + 4(−2) = 0 (d) K3[Cu(CN)4]
x = +6 Let, x be the oxidation state of Cu.
(c) FeSO4(NH4)2SO4.6H2O ∴ 3(+1) + x + 4(−1) = 0
x + 1(−2) + 2 (+1) + 1(−2) = 0 x = +1
x = +2 CN is a strong field ligand so pairing occur in inner
(d) K3[Fe(CN)6] orbital and forms the square planar complex.
3(+1)+x+6(−1) = 0 Cu+ = [Ar]3d10
x = +3 It is diamagnetic in nature because the compound has
Hence, FeSO4(NH4)2SO4.6H2O has the least oxidation not any unpair electron and there is no transition take
state. place thus it does not show the colour.

Objective Chemistry Volume-II 518 YCT


6. BeO reacts with HF in presence of ammonia to
give [A] which on thermal decomposition
produces [B] and ammonium fluoride.
Oxidation state of Be in [A] is_____.
JEE Main-27.06.2022, Shift-II Unpaired electron (n) = 4
Ans. (2) : BeO reacts with HF in presence of ammonia
to give the BeF2. ∴ µs = 4(4 + 2)
µs = 4.89 BM
BeO + 2HF  NH3
→ (NH 4 ) 2 BeF4
[A] (4) [Mn(CN)6]3−

(NH 4 ) 2 BeF4  → NH 4 F2 + BeF2 Let, x be the oxidation state of Mn.
[B] x + 6(−1) = −3
Here, [A] is (NH4)2BeF4. x = +3
Let, x be the oxidation state of Be. Mn3+ = [Ar] 3d4 (Pairing occur because CN is SFL)
∴ 2(+1) + x + 4 (−1) = 0
x = +2
7. Arrange the following coordination compounds
in the increasing order of magnetic moments.
(Atomic numbers: Mn = 25; Fe = 26) Unpaired electron (n) = 2
(1) [FeF6]3– (2) [Fe(CN)6]3–
3–
(3) [MnCl6] (high spin) (4) [Mn(CN)6] 3– ∴ µs = 2(2 + 2)
Choose the correct answer from the options µs = 2.82 BM
given below: Hence, the increasing order of magnetic moment is
(a) 1 < 2 < 4 < 3 (b) 2 < 4 < 3 < 1 2 < 4 < 3 < 1.
(c) 1 < 3 < 4 < 2 (d) 2 < 4 < 1 < 3
8. Identify the incorrect statement related for
JEE Main-27.06.2022, Shift-II PCl5 from the following.
Ans. (b) : (1) [FeF6]3− (a) In this molecule, orbitals of phosphorous are
Let, x be the oxidation state of Fe assumed to undergo sp3d hybridization.
x + 6 (−1) = −3 (b) The geometry of PCl5 is trigonal bipyramidal.
x = +3 (c) PCl5 has two axial bonds stronger than three
F is a weak field ligand so pairing of electron will not equatorial bonds.
occur (d) The three equatorial bonds of PCl5 lie in a
Fe3+ = [Ar]3d5 plane
JEE Main-27.06.2022, Shift-II
Ans. (c) : (a) PCl5
Hybridisation = Bond pair + Lone pair
=5+0
unpaired electron (n) = 5 = 5 (sp3d)
∴ µs = n(n + 2) The hybridisation of PCl5 is sp3d hence the geometry of
the molecule is trigonal bipyramidal.
µs = 5(5 + 2) Hence, both (a) and (b) statement are true.
µs = 5.9BM
(2) [Fe(CN)6]−3
Let, x be the oxidation state of Fe.
x + 6 (−1) = −3
x = +3
Fe3+ = [Ar] 3d5 (Pairing occur because CN is SFL.)
Here, PCl5 has two axial bond and three equatorial
bond. The two axial bond is less stronger than
equatorial bond because axial bond have three times
bond pair- bond pair repulsion rather than the repulsion
between equatorial bond. Hence, statement is false.
Unpaired electron(n) = 1 (d) The equatorial bond lie in the plane. Hence, the
µs = 1(1 + 2) statement is true.
µs = 1.732 BM 9. Spin only magnetic moment of [MnBr6]4- is
(3) [MnCl6]−3 _____ B.M.[ round off to the closest integer]
Let, x be the oxidation state of Mn. JEE Main-29.06.2022, Shift-II
x + 6 (−1) = −3 Ans. (6) : The compound is [MnBr6]4−. The oxidation
x = +3 state of Mn is +2.
Mn3+ = [Ar] 3d4 (High spin) ∴ Mn2+ = [Ar]3d5

Objective Chemistry Volume-II 519 YCT


Now, from relation between ∆t and ∆o –
4
∆t = ∆o
9
Or
4
∆o = ∆t
9
Number of unpaired electron (n) = 5 4
∆ o = × 0.034034 × 10−17 J
∴ Spin only magnetic moment (µs) = n(n + 2) BM 9
µs = 5(5 + 2) ∆ o = 0.07657 × 10−17 J
µs = 5.91BM ∆ o ≈ 766 × 10 −21 J
µs ≈ 6BM 12. White precipitate of AgCl dissolves in aqueous
ammonia solution due to formation of:
10. For the reaction given below: (a) [Ag(NH3)4]Cl2 (b) [Ag(Cl)2(NH3)2]
CoCl 3 ⋅ xNH 3 + AgNO 3 (aq)  → (c) [Ag(NH3)2]Cl (d) [Ag(NH3)Cl]Cl
If two equivalents of AgCl precipitate out, then JEE Main-25.06.2022, Shift-I
the value of x will be__. Ans. (c) : When white precipitate of AgCl dissolves in
JEE Main-29.06.2022, Shift-II aqueous solution of ammonia then [Ag(NH3)2]Cl is
Ans. (5) : Following reaction take place which is given formed.
below- 2NH3(aq) + AgCl → (Ag(NH3)2)Cl
CoCl3.xNH3 + AgNO3 → AgCl (↓) Soluble
2 mol 14. An acidified manganate solution undergoes
[Co(NH3)5Cl]Cl2 + AgNO3 → AgCl(↓) disproportionation reaction. The spin-only
magnetic moment value of the product having
2 mol
manganese in higher oxidation state is ––––
Here, the value of x will be 5. B.M. (Nearest integer)
11. Reaction of [Co(H2O)6]2+ with excess ammonia JEE Main-29.06.2022, Shift-I
and in the presence of oxygen results into a' Ans. (0) : The disproportionation reaction of acidified
diamagnetic product. Number of electrons maganate solution is-
present in t2g-orbitals of the product is ––––––. +7 +4
JEE Main-26.06.2022, Shift-II 3MnO 24− + 4H +  → 2 MnO 4− + MnO 2 + 2H 2 O
Ans. (6) : Reaction of [Co(H2O)6]2+ with excess
Here, MnO −4 has the highest value of oxidation state.
ammonia gives the following product-
[Co(H2O)6]2+ + NH3(excess) → [Co(NH3)6]3+ Hence, it’s spin only magnetic moment is given as–
3+ +7
[Co(NH3)6] is a octahedral complex. The oxidation Mn =[Ar]3d 0
state of Co is +3.
Co3+ = [Ar]3d6 (low spin complex) Unpair electron(n) =0
∴ µs = n(n + 2) BM
µs = 0(0 + 2) BM
µs = 0
Here, the number of electron in t2g orbital is 6. 15. Acidified potassium permanganate solution
12. If [Cu(H2SO)4]2+ absorbs a light of wavelength oxidises oxalic acid. The spin-only magnetic
600 nm for d-d transition, then the value of moment of the manganese product formed
octahedral crystal field splitting energy for from the above reaction is ______ B.M.
[Cu(H2O)6]2+ will be______×10–21 J. [Nearest (Nearest Integer)
Integer] JEE Main-27.06.2022, Shift-I
–34 8 –1
[Given: h= 6.63 ×10 Js and c= 3.08 ×10 ms ] Ans. (6) : The acidified reaction of potassium
permanganate solution with oxalic acid is –
JEE Main-25.06.2022, Shift-I 2KMnO4 + 5H2C2O4 + 3H2SO4→2MnSO4 + K2SO4 +
Ans. (766) : Given : λ = 600 nm 10CO2 + 8H2O
MnSO4
∆o = ?
Let, x be the oxidation state of Mn.
hc
∴ ∆t = E = x + 1(−2) = 0
λ x = +2
6.63 ×10−34 (Js) × 3.08 × 108 (ms −1 ) Mn2+ = [Ar]3d5
∆t =
600 × 10−9 m
∆ t = 0.034034 ×10 −17

Objective Chemistry Volume-II 520 YCT


Unpaired electron (n) = 5
∴ µs = n(n + 2)
µs = 5(5 + 2)
µs = 5.91≈ 6 BM Unpaired electron (n) = 0 (diamagnetic)
Here, the number of paramagnetic complex is 2 i.e.
16. Which of the following will have maximum [Fe(CN)6]−3 and [Ti(CN)6]−3.
stabilization due to crystal field?
18. Transition metal complex with highest value of
(a) [Ti(H2O)6]3+ (b) [Co(H2O)6]2+
3– crystal field splitting (∆o ) will be
(c) [Co(CN)6] (d) [Cu(NH3)4]2+
JEE Main-27.06.2022, Shift-I (a) [Cr(H2O)6]3+ (b) [Mo(H2O)6]3+
3+
(c) [Fe(H2O)6] (d) [Os(H2O)6]3+
Ans. (c) : According to the spectrochemical series, the
ligands which has more ligand field, have the maximum JEE Main-24.06.2022, Shift-II
stabilization energy. Here, CN has the greater ligand Ans. (d) : The value of crystal field splitting energy
field thus [Co(CN)6]−3 has the maximum crystal field depends upon the size of metal. As the size of metal
stabilization energy. increases, as well as the value of CFSE increases. Here,
17. [Fe(CN)6]4- Os has the maximum size thus [Os(H2O)6]3+ has the
maximum CFSE.
[Fe(CN)6]3-
[Ti(CN)6]3- 19. The correct IUPAC name of cis-platin is
[Ni(CN)4]2- (a) Diammine dichloride platinum (O)
[Co(CN)6]3- (b) Dichloride diammine platinum (IV)
(c) Diammine dichloride platinum (II)
Among the given Complexes, number of
paramagnetic complexes is ______. (d) Diammine dichloride platinum (IV)
JEE Main-28.06.2022, Shift-II Karnataka CET-17.06.2022, Shift-II
Ans. (c) : The cis-platin is the anti-cancer agent. The
Ans. (2) : [Fe(CN)6]4− chemical formula of cis-platin is [Pt(NH3)2Cl2]. The
x + 6(−1) = −4 IUPAC name of the compound will be diammine
x=+2 dichloride platinum (II).
Fe2+ = [Ar]3d6 (low spin complex) 20. The complex hexamine platinum (IV) chloride
will give _____ number of ions on ionization.
(a) 3 (b) 2
(c) 5 (d) 4
Karnataka CET-17.06.2022, Shift-II
Unpaired electron (n) = 0 (diamagnetic) Ans. (c) : The chemical formula of hexamine platinum
[Fe(CN)6]3− (IV) chloride is [Pt(NH3)6]Cl4.
x + 6(−1) = −3
x = +3
Fe3+= [Ar]3d5 (low spin complex)
21.
A transition metal exists in its highest oxidation
state. It is expected to behave as
(a) An oxidizing agent
(b) A reducing agent
Unpaired electron (n) = 1 (paramagnetic) (c) A chelating agent
[Ti(CN)6]−3 (d) A central metal in a co-ordination compound
x + 6(−1) = −3 Karnataka CET-17.06.2022, Shift-II
x = +3 Ans. (a) : As we know that transition metal shows the
Ti = [Ar]3d24s2 highest oxidation state due to which it has more
Ti3+ = [Ar]3d1 tendency to accept the electron. Thus, it behaves as an
Unpaired electron (n) = 1 (Paramagnetic) oxidizing agent.
−2 22. Amongst FeCl33H2O, K3[Fe(CN)6] and
[Ni(CN)4]
x = +2 [Co(NH3)6]Cl3, the spin-only magnetic moment
value of the inner-orbital complex that absorbs
Ni2+ = [Ar]3d8 light at shortest wavelength is–––B.M. [nearest
CN is a strong field ligand and it forms the square integer]
planar complex. Here, the number of unpaired electron
is zero thus it is diamagnetic in nature. JEE Main-25.06.2022, Shift-II
[Co(CN)6] 3– Ans. (2) : The given compounds are –
x + 6(–1) = –3 (1) FeCl3.3H2O
x = +3 Fe3+

Objective Chemistry Volume-II 521 YCT


E.C. = [Ar]3d5(WFL) 24.[Co(NH3)5SO4]Br and [Co(NH3)Br] SO4 are
Unpaired e− = 5 (a) linkage isomers
(b) coordination isomers
µs = 5(5 + 2)
(c) ionisation isomers
µs = 5.91 BM (d) optical isomers
JCECE - 2018
(2) K3[Fe(CN)6] AMU-2013
Fe3+ CG PET -2005, 2004
E.C. = [Ar]3d5(SFL)
− Ans. (c) : [Co(NH3)5SO4] Br and [Co(NH3)5Br]SO4 are
Unpaired e = 1 ionisation isomers. These compounds give different
µs = 1(1 + 2) ions in the solution although they have same
µs = 1.73 BM composition. It arises when the counter ion in a
complex salt is itself a potential ligand and can displace
µs ≈ 2 BM a ligand which can then become the counter ion.
[Co(NH3)6]Cl3
3+ 25. The complexes [Co(NH3)6][Cr(CN)6] and
Co [Cr(NH3)6][Co(CN)6] are the examples of
6
E.C. = [Ar]3d (SFL) which type of isomerism?
Unpaired e− = 0 (a) Ionisation isomerism
Here, K3[Fe(CN)6] is the inner orbital complex which (b) Coordination isomerism
has the 2 magnetic moment. (c) Geometrical isomerism
23. The metal ion (in gaseous state) with lowest (d) Linkage isomerism
spin- only magnetic moment value is UPTU/UPSEE-2016
(a) V2+ (b) Ni2+ JIPMER-2014
(c) Cr2+ (d) Fe2+ AIPMT -2011
JEE Main-25.06.2022, Shift-II Ans. (b) : When both positive and negative ions are
Ans. (b) : complex, isomerism is possible due to the interchange
2+ of ligands between the two complex ion. This type of
(a) V
3 2 isomerism is called co-ordination isomerism and is
E.C. = [Ar]3d 4s illustrated below-
2+ 3
E.C. of V = [Ar]3d [Co(NH3)6][Cr(CN)6] and [Cr(NH3)6][Co(CN)6]
unpaired electron (n) = 3
26. The geometrical and optical isomers of complex
µs = n(n + 2)  Pt ( NH 3 ) ( Br )( Cl )( Py )  are respectively
µs = 3(3 + 2) (a) 2,2 (b) 0,3
µs = 3.87 BM (c) 2,1 (d) 3,0
(b) Ni2+ GUJCET-2022
2+ 8
E.C. of Ni = [Ar]3d CG PET- 2013
AIPMT -2011
Ans. (d): [Pt(NH3) (Br) (Cl) (Py)] is a square planer
Unpaired electron (n) = 2 complex. Square planar complexes of this type exist in
three isomeric forms. Which is illustrated below-
∴ µs = 2(2 + 2)
µs = 2.82 BM
(c) Cr2+
E.C. of Cr2+ = [Ar]3d4

Unpaired electron (n) = 4 Square planar complex do not exhibit optical isomers as
µs = 4(4 + 2) they possess a plane of symmetry.
27. Which complex possess facial isomer?
µs = 2.82 BM
(d) Fe2+ (a) K  Fe ( NH 3 )2 ( CN )4 
E.C. of Fe2+ = [Ar]3d6 (b) Co ( NH 3 )3 ( NO 2 )3 
(c) Co ( NH 3 ) 4 CO3  Cl
Unpaired electron (n) = 4
µs = 4(4 + 2) (d)  Ni ( H 2 O ) 4 ( NH 3 ) 2  SO 4
µs = 4.89 BM JEE Main-2020, 8 Jan Shift-I
Here, Ni2+ has the lowest magnetic moment. GUJCET-2018
Objective Chemistry Volume-II 522 YCT
Ans. (b): When the three ligrands (with same donor 31. Which one of the following is expected to
atoms) are on the same trianglular face of the exhibit optical isomerism?
octahedron, the compound is called facial or fac-isomer. (en = ethylenediamine)
The given compound is [Co(NH3)3 (NO2)3]. (a) cis – [Pt (NH3)2Cl2]
(b) trans – [Co (en)2Cl2]
(c) trans – [Pt (NH3)2Cl2]
(d) cis – [Co (en)2Cl2]
JIPMER-2007, AIPMT -2005
Ans. (d) : The optical isomerism shows by such type of
compound which are non superimposible mirror image
to each other.
Hence, cis- [Co(en)2Cl2] shows the optical isomerism.

28. One of the following complexes shows


geometrical isomerism. The complex is
(a) PtCl4 (b) Pt(NH3)2Cl2
(c) Pt(NH3)3Cl (d) Ni(NH3)3Cl
J & K CET-2015, 2013
Ans. (b) : Geometrical isomerism is also known by cis -
trans isomerism. The tetrahedral complex do not show
geometrical isomer because all the ligands make the 32. Number of+ stereoisomers exhibited by
same angle with adjacent ligand. Thus, Ni(NH3)3Cl do [Co(en)2Cl2] is
not show the geometrical isomerism. (a) 4 (b) 2
[Pt(NH3)2Cl2] is a square planar complex. It forms the (c) 5 (d) 3
cis and trans isomerism. Karnataka-CET-2020, NEET-2015
Ans. (d) : This is the [M(AA)2B2] type of complex. The
complex [Co(en)2Cl2]+ contains the bidentate ligands.
The cis -form of this complex are optically active
whereas trans form of this compound is optically
inactive.
29. Square planar complexes of the type MABXL
(where A, B, X and L are unidentates) show
(a) two cis and one trans isomer
(b) two trans and one cis isomer
(c) two cis and two trans isomer
(d) one cis and one trans isomer
Karnataka-CET-2017, J & K CET-2011
Ans. (a) : The square planar complex of the type is
MABXL. It forms the two C is and one trans isomer
which is illustrated below.

30. [Co(NH3)5ONO]Cl2and[Co(NH3)5NO2]Cl2 show


(a) geometrical isomerism
(b) linkage isomerism
(c) coordination isomerism
(d) ionization isomerism
Kerala CEE -03.07.2022 33. The existence of two different coloured
complexes of [Co(NH3)4Cl2]+ is due to :
J & K CET-1998, 1997
(a) ionisation isomerism
Ans. (b) : Existence of ambidentate ligands (e.g.- SCN–, (b) co-ordination isomerism
NO2) gives rise to the possibility of linkage isomerism,
in which the same ligand may link through different (c) linkage isomerism
atoms. Hence, [Co(NH3)5ONO]Cl2 and (d) geometrical isomerism
[Co(NH3)5NO2]Cl2 shows the linkage isomerism. Manipal-2019, AIPMT -2010
Objective Chemistry Volume-II 523 YCT
Ans. (d) : The compound is [Co(NH3)4Cl2]+. It is a (c) [Cr(H2O)6]Cl3 – solvate isomerism.
octahedral complex type of [MA4B2]. This complex When water molecules are intercharged between co-
exhibit the cis and trans isomerism which is illustrated ordination sphere and ionisation sphere, the resulting
below- isomers are called hydrate isomers.
(d) cis–[CrCl2(ox)2]2 – Optical isomerism.
The cis form of [M(AA)2B2] type complex shows the
optical activity because of unavailability of plane of
symmetry.

34. The number of stereoisomers possible for


[Co(ox)2 (Br)(NH3)]2+ is ………[ox=oxalate].
JEE Main-2021, 26 March Shift-II
Ans. (3): [Co(ox)2Br NH3]2+ ion is a octahedral
complex type of [M(AA)2BC]. This complex ion exists
in cis and trans isomers. The cis-form is optically active
because plane of symmetry is absent and trans form is
optically inactive.

36.
Indicate the complex/complex ion which did
not show any geometrical isomerism.
(a) [CoCl2(en)2] (b) [Co(CN)5(NC)]3-
(c) [Co(NH3)3(NO2)3] (d) [Co(NH3)4Cl2]+
JEE Main-2021, 26 July Shift-I
35. Match List-I and List-II. Ans. (b): The complex ion with monodentate ligands A
and B, no isomers are possible for [MA5B] or [MAB5]
List-I List-II
type of complex.
a. [Co(NH3)6[Cr(CN)6] 1. Linkage
isomerism 37. The number of geometrical isomers found in
b. [Co(NH3)3(NO2)3] 2. Solvate the metal complexes [PtCl2(NH3)2], [Ni(CO)4],
isomerism [Ru(H2O)3Cl3] and[CoCl2(NH3)4]+ respectively
c. [Cr(H2O)6]Cl3 3. Co-ordination are
isomerism (a) 1,1,1,1 (b) 2,1,2,2
d. cis-[CrCl2(ox)2]3- 4. Optical (c) 2,0,2,2 (d) 2,1,2,1
isomerism JEE Main-2021, 27 July Shift-I
Chose the correct answer from the options Ans. (c): The metal complexes are [PtCl2(NH3)2],
given below. [(Ni(CO)4], [Ru(H2O)3Cl3] and [CoCl2(NH3)4]+ type of
a b c d [MA2B2], [MA4], [MA3B3] and [MA2B4] respectively.
(a) 3 1 2 4
[PtCl2](NH3)2] is a square planar complex which form
(b) 4 2 3 1
the cis and trans isomerism i.e. two isomer.
(c) 2 1 3 4
(d) 1 2 3 4
JEE Main-2021, 18 March Shift-I
Ans. (a):
(a) [Co(NH3)6] [Cr(CN)6] – Co-ordination isomerism
salts that contain complex cations and anions may
exhibit isomerism through the interchange of ligands
between cation and anion.
(b) [Co(NH3)3(NO2)3]– Linkage isomerism.
The complex contains the NO2 ligand which has two
donor site due to which it can show the linkage Ni(CO)4 is a tetrahedral complex which is not show any
isomerism. geometrical isomerism.

Objective Chemistry Volume-II 524 YCT


Ru(H2O)3Cl3 is the octahedral complex which form the
facial and meridional isomerism.
(d)
[CoCl2(NH3)4]+

JEE Main-2021, 18 March Shift-I


Ans. (d) : The structure of trans - [NiBr2(pph3)2] and
meridional [Co(NH3)3(NO2)3] are given below -

trans meridional
40. The number of optical isomers possible for
38. The number of geometrical isomers possible in [Cr(C2O4)3]3- is…
triamminetrinitrocobalt (III) is X and in JEE Main-2021, 27 Aug Shift-II
trioxalatochromate (III) is Y. Then, the value
Ans. (2): This is a octahedral complex type of
of X + Y is ……….
[M(AA)3]. The compound shows the optical activity
JEE Main-2021, 27 July Shift-I which is illustrated below–
Ans. (2): The name of the compound is triammine
trinitrocobalt (III). The molecular formula of compound
will be [Co(NH3)3(NO2)3]. This is a octahedral
compound type of [MA3B3]. It from the cis and trans
isomer i.e. X = 2

Hence, there is two optical isomerism is possible.


41. Which type of Isomerism in isomers [Co(NH3)5
(SO4)]Br and [Co(NH3)5Br]SO4?
(a) Linkage (b) Ionization
(c) Coordination (d) Geometrical
GUJCET-2021
The name of the compound is trioxalatochromate (III). Ans. (b): Ionization Isomerism:- The isomers which
The chemical formula will be [Cr(ox)3]–3. It is type of form different ions in solution, although, they have
[M(AA)3], complexes of this type do not exhibit same composition are called ionization isomers.
geometrical isomerism i.e. Y = 0 [Co(NH3)5 (SO4)]Br and [Co(NH3)5Br]SO4 are
∴X + Y = 2 + 0 = 2 ionization isomer. This type of isomerism occurs due to
39. The correct structures of trans- [NiBr2(PPh3)2] exchange of groups between the complex ion and ion
and meridonial-[Co(NH3)3(NO2)3], respectively outside it. Such compounds behave differently when
are dissolved in water.
[Co(NH3)5Br]SO4 → Red-violet
[Co(NH3)5SO4]Br → Red
(a) 42. Which of the following does not show
geometrical isomerism?
(a) [Co(NH3)4 Cl2]+ (b) [CoCl2(en)2]
(c) [Pt(NH3)2Cl2] (d) [Cr(NH3)5Cl]2+
(b) Assam CEE-2021
Ans. (d) : For monodentate ligands A and B, no isomers
are possible for [MA5B] or [MAB5].
Hence, [Cr(NH3)5Cl]2+ has not any isomers.
(c) 43. The number of stereoisomers possible for
[Co(ox)2(Br)(NH3)]2– is _________.
JEE Main 26.02.2021,Shift-II
Objective Chemistry Volume-II 525 YCT
Ans. (3) : Ans. (c) :
Co-ordination complex Isomerism
(a) [Co(NH3)6][Cr(CN)6] – Co-ordination isomerism
(b) [Co(NH3)3(NO2)3] – Linkage isomerism
(c) [Cr(H2O)6]Cl3 – Solvate isomerism
(d) cis-[CrCl2(ox)2]3– – Optical isomerism
46. Consider the complex ions, trans-[Co(en)2Cl2]+
(A)and cis-[Co(en)2Cl2]+ (B). The correct
statement regarding them is
(a) Both (A) and (B) cannot be optically active
(b) (A) can be optically active, but (B) cannot be
optically active
(c) Both (A) and (B) can be optically active
(d) (A) cannot be optically active, but (B) can be
optically active
JEE Main-2020, 5 Sep Shift-II
Ans. (d) : The complexs are-
Total stereoisomer = 2 + 1 = 3 trans-[Co(en)2Cl2]+ (A) and cis-[Co(en)2Cl2]
44. The type of isomerism shown by the complex
+
CoCl ( en )  is
 2 2
(a) Geometrical isomerism
(b) Coordination isomerism
(c) Linkage isomerism
(d) Ionization isomerism
TS EAMCET 09.08.2021, Shift-I
Ans. (a) : The type of isomerism shown by the complex The cis-form formed the optically active compound
+ because it has not plane of symmetry.
CoCl2 ( en ) 2  is geometrical isomerism. This complex
47. The complex that can show optical activity is
show cis-trans isomerism. In the cis form, two Cl (a) trans-[Cr(Cl2)(ox)2]3-
ligands are adjacent to each other and two en ligands are
(b) trans-[Fe(NH3)2(CN)4]-
adjacent to each other. In the trans form, two Cl ligands
are opposite to each other and two en ligands are (c) cis-[Fe(NH3)2(CN)4]-
opposite to each other. (d) cis-[CrCl2(ox)2]3-(ox=oxalate)
JEE Main-2020, 3 Sep Shift-I
Ans. (d) : Due to absence of plane of symmetry on cis -
[CrCl2(OX)2]3– , it shows the optically activity.

Geometrical isomerism in [CoCl2(en)2]+


45. Match List-I with List-II:
List-I List-II
(a) [Co(NH3)6][Cr(CN)6] (i) Linkage isomerism 48. Among(A)-(D), the complexes that can display
geometrical isomerism are
(b) [Co(NH3)3 (NO2)3] (ii) Solvate isomerism
(A) [Pt(NH3)3Cl]+ (B) [Pt(NH3)Cl5]-
(c) [Cr(H2O)6]Cl3 (iii) Co-ordination
(C) [Pt(NH3)2Cl(NO2)] (D) [Pt(NH3)4ClBr]2+
isomerism
(a) (D) and (A) (b) (C) and (D)
(d) cis–[CrCl2(ox)2]3– (iv) Optical isomerism
(c) (A) and (B) (d) (B) and (C)
Choose the correct answer from the options
JEE Main-2020, 8 Jan Shift-II
given below:
(a) (a)-(i), (b)-(ii), (c)-(iii), (d)-(iv) Ans. (b):
(b) (a)-(ii),(b)-(i),(c)-(iii),(d)-(iv) (A) [MA3B] type of square planar complex not show
(c) (a)-(iii),(b)-(i),(c)-(ii), (d)-(iv) the geometrical isomerism.
(d) (a)-(iv),(b)-(ii),(c)-(iii),(d)-(i) (B) [MAB5] is a octahedral complex and it is not shows
JEE Main 17.03.2021, Shift-II the geometrical isomerism.

Objective Chemistry Volume-II 526 YCT


(C) [MA2BC] type of complex also exist in cis and Ans. (c): Such type of isomerism is not possible in
trans-form. complexes having co-ordination number four
(tetrahedral complex). This is due to the fact that in
these arrangements all the position are adjacent to each
other. Hence, [Ni(NH3)2Cl2] do not show the isomerism.
52. Which of the following is not an organometallic
compound?
(a) Zeise’s salt (b) Grignard reagent
(c) Cis-platin (d) None of these
COMEDK-2020
(D) [Pt (NH3)4ClBr]2+ ion is an important example of Ans. (c) : Zeise's salt and Grignard reagent has at least
octahedral complex of [MA4BC] type. This ioc has one carbon and metal atom which shows the property of
cis and trans isomer whose structures are given organometallic compound. Cis-platin is the anti cancer
below– drug, its chemical formula not contains any carbon atom
due to which this is not organometallic compound.
53. CuSO4 reacts with KCN solution and forms
(a) K3[Cu(CN)4] (b) [Cu(CN)]
(c) [Cu(CN)2] (d) K4[Cu(CN)6]
COMEDK-2020
Ans. (a) : Copper sulphate is dissolved in potassium
cyanide solution. Copper sulphate reacts with potassium
cyanide to form potassium sulphate and cupric cyanide-
CuSO 4 + 2KCN → K 2SO 4 + Cu ( CN )2
Copper sulphate Potassium Potassium
cyanide Cupric
Sulphate cyanide

Cupric Cyanide is unstable. It decomposes to form


cuprous cyanide and cyanogens gas.
49. The number of isomers possible for
[Pt(en)(NO2)2] is 2Cu ( CN )2 → 2CuCN + ( CN )2
(a) 3 (b) 2 In this reaction, cupric cyanide is reduced to cuprous
(c) 4 (d) 1 cyanide. One mole of cuprous cyanide reacts with three
JEE Main-2020, 4 Sep Shift-I moles of potassium cyanide to form the complex
Ans. (a): [Pt (en) (NO2)2] is a square planar complex compound.
type of [M(AA) B2]. CuCN + 3KCN  → K 3 [Cu ( CN ) 4 ]
54. Which of the following complex will exhibit
optical isomerism?
(a) [Co(NH3)4]3+ (b) [Fe(NH3)2(CN)4]–
2+
(c) cis[PtCl2(en)2] (d) trans-[PtCl2(en)2]2+
Assam CEE-2020
Ans. (c) : Cis- [Co(en)2Cl2]+ is optically active. Trans-
[Co(en)2Cl2]+ is symmetric and hence it is optically
inactive.

There are three isomers formed.


50. The total number of coordination sites in
ethylenediaminetetraacetate [EDTA4-] is…..
JEE Main-2020, 5 Sep Shift-I
Ans. (6): Ethylenediaminetetraacetic acetate ion 55. Which isomerism is possible in hexa ammine
[EDTA]4– is a hexadentate ligand which has six donor cobalt (III) hexa cyanide chromate (III)
site and formed the six co-ordinate bond. complex?
51. The one that is not expected to show isomerism (a) Linkage isomerism
is (b) Co-ordination isomerism
(a) [Ni(NH3)4(H2O)2]2+ (b) [Ni(en)3]2+ (c) Ionisation isomerism
(c) [Ni(NH3)2Cl2] (d) [Pt(NH3)2Cl2] (d) Solvate isomerism
JEE Main-2020, 2 Sep Shift-II GUJCET-2020
Objective Chemistry Volume-II 527 YCT
Ans. (b): The name of the compound is hexa ammine Ans. (a): The square planar complex has four ligands.
cobalt-III hexa cyanide chromate-III. The chemical Here, NO2 and SCN have two donor site atom. The total
number of isomers for a square planar complex is 12.
formula of the compound will be [Co(NH3)6] [Cr
(CN)6]. This complex can interchange of ligands
between cation and anion and shows the co-ordination
isomerism.
56. Which of the following complex ion shows
geometrical isomerism ?
+ +
(a) Cr ( H 2O )4 Cℓ 2  (b)  Pt ( NH 3 )3 Cℓ 
3+ 3−
(c) Co ( NH 3 )6  (d) Co ( CN )5 ( NC ) 
AP EAMCET (Engg.) 18.09.2020, Shift-I
Ans. (a) : [Cr(H2O)4Cl2]+ shows geometrical
isomerism. It is complex compounds of [MA4B2] type.

+ 3+
 Pt ( NH 3 )3 Cℓ  , Co ( NH 3 )6  and
3−
Co ( CN )5 ( NC )  do not show stereoisomerism.
57. The species that can have a trans-isomer is
(en=ethane -1,2-diamine, ox=oxalate) 59. A reaction of cobalt (III) chloride and ethylene
diamine in a 1:2 mole ratio generates two
isomeric products A (violet coloured) and B
(green coloured). A can show optical activity,
but B is optically inactive. What type of
isomers does A and B represent?
(a) Ionisation isomers
(a) [Pt(en)Cl2] (b) [Cr(en)2(ox)]+ (b) Coordination isomers
(c) [Pt(en)2Cl2]2+ (d) [Zn(en)Cl2] (c) Geometrical isomers
(d) Linkage isomers
JEE Main-2019, 10 April Shift-I JEE Main-2019, 10 Jan Shift-II
2+
Ans. (c) : [Pt(en)2Cl2] is a octahedral complex type of Ans. (c): According to the question–
[M(AA)2B2]. It shows the both cis and trans isomerism. CoCl3 + 2en  → Co ( en )2 Cl 2  Cl

[Pt(en)Cl2], [Cr(en)2(ox)]+ and [Zn(en)Cl2] exhibit the


only cis isomerism.
58. The total number of isomers for a square
planar complex
[M(F)(Cl)(SCN)(NO2)] is
(a) 12 (b) 16
(c) 4 (d) 8 The structure of complex shows the cis and trans
JEE Main-2019, 10 Jan Shift-II isomerism i.e. geometrical isomerism.
Objective Chemistry Volume-II 528 YCT
60. Which of the following complex is optically Ans. (c) : MX3Y3 has a trigonal prismatic geometry.
inactive?
(a) [RhCl(CO)(PPh3)(NH3)]
(b) [Fe(C2O4)3]3–
(c) [Fe(en)2Cl2]
(d) [Pd(en)2Cl2]
AIIMS 26 May 2019 Morning
Ans. (a) : The isomer which is not capable of rotating
the plane of polarized lilght is called optically inactive.
[Rh(Cl)(CO)(PPh3)(NH3)] is optically inactive
compound because they posses a plane of symmetry.
61. Which of the following complexes posses
meridional isomer?
(a) Co ( NH 3 )3 Cl3  (b) Co ( NH 3 )4 Cl 2 
(c) Co ( NH 3 )2 Cl 4 
(d) Co ( NH 3 )5 Cl 
GUJCET-2019
Ans. (a): Octahedral complexes of the type [MA3B3]
like [Co(NH3)3Cl3] also exist in two geometrical
isomerism i.e. facial and meridional. When the three
ligands are on the same equatorial plane of the
octahedron i.e. around the meridian of the octahedron,
the compound is called meridional or mer isomer.
64. Ethylene diamine is an example of :
(a) monodentate ligand (b) bidentate ligand
(c) tridentate ligand (d) polydentate ligand
Manipal-2018
Ans. (b) : Ethylene diamine is the example of bidentate
ligand in which two donor site are present for donation
of electron. It is denoted by 'en'. It is a neutral ligand.

65. The type of isomerism present in


nitropentamine chromium (III) chloride is :
62. The complexes : [Pd(C6H5)2 (SCN)2]] and (a) optical (b) coordination
[Pd(C6H5)2 (NCS)2] are the example of which (c) linkage (d) polymerisation
type of isomerism? Manipal-2018
(a) Ionization isomerism Ans. (c) : The chemical formula of the compound will
(b) Linkage isomerism be [Cr(NO2)(NH3)5]Cl. In this complex, NO2 ligand is
present which is a major reason for exhibit the linkage
(c) Coordination isomerism isomerism because NO2 has two donor site of donation.
(d) Hydrate isomerism
66. Which of the following complexes is optically
Jharkhand – 2019 active?
Ans. (b) : The given complexes contain thiocyanato (a) [Co(NH3)5Cl]+ (b) [Co(NH3)Cl3]3–
(NCS) as a ligand which is an ambidenate ligand and (c) Cis-[Co(en)2Cl2] (d) Trans-[Co(en)2Cl2]
can coordinate through two donor atoms N and S. AMU-2018
Ans. (c) : The cis-form of [Co(en)2Cl2] complex which
is unsymmetrical and can form two optical isomers. The
Hence, in [Pd(C6H5)2 (SCN)2] and [Pd(C6H5)2 (NCS)2] trans- [Co(en)2Cl2] is optically inactive due to presence
linkage isomerism is possible. of a plane of symmetry.
63. For a complex, MX3Y3 possessing trigonal
prismatic geometry, the number of possible
isomer is
(a) 2 (b) 4
(c) 3 (d) 6
UPTU/UPSEE-2018
Objective Chemistry Volume-II 529 YCT
67. The type of isomerism shown by the complex
[CoCl2(en)2] is
(a) geometrical isomerism
(b) coordination isomerism
(c) ionization isomerism
(d) linkage isomerism.
NEET 2018
Ans. (a) : [CoCl2(en)2] is a octahedral complex type of
[M(AA)2B2]. The complex shows the cis and trans 71. Select correct match:
isomerism (geometrical isomerism) which is illustrated (a) Co ( ox )( H 2O )3 ( NH 3 )  Br: Optical
below-
isomerism
(b) Cr ( SCN )( H 2O )3 ( en )  ( C 2 O 4 ) : Ionization
isomerism
(c)  ZnBr ( CN )( SCN )( NH 3 )  : Geometrical
isomerism
(d) CoBrCl ( H 2O )4   Ag ( CN ) 2  : Co-ordination
isomerism
68. How many geometrical isomers are possible AIIMS-27 May, 2018 (E)
with complexes of the type [M(ab)3]? Ans. (d) : When both positive and negative ions are
(a) 2 (b) 4 complex, isomerism is possible due to the interchange
(c) 3 (d) 5 of ligands between the two complex ions. This type of
isomerism is called co-ordination isomerism. In [Co
J & K CET-2018
BrCl (H2O)4] [Ag(CN)2], [CoBrCl(H2O)4]2+ is a cation
Ans. (a) : The type of the complex is [M(ab)3]. It is a and [Ag(CN)2]2- is an anion ligands interchange their
octahedral complex which form the facial or meridional position to form [CoBrCl(H2O)4] [Ag(CN)2].
isomerism (geometrical isomerism) 72. The correct statement for the isomerism
associated with the following complex ions is:
2+
(A)  Ni ( H 2 O )5 NH 3  ,
2+
(B)  Ni ( H 2 O ) 4 ( NH 3 )2  and
2+
(C)  Ni ( H 2 O )3 ( NH 3 )3  is :
(a) (A) and (B) show only geometrical
isomerism.
(b) (A) and (B) show geometrical and optical
Hence, it contains the two geometrical isomer. isomerism.
(c) (B) and (C) show geometrical and optical
69. [Fe (NO2)3Cl3] and [Fe (O–NO)3Cl3] shows isomerism.
(a) linkage isomerism (d) (B) and (C) show only geometrical
(b) geometrical isomerism isomerism.
(c) optical isomerism AIIMS-27 May, 2018 (M)
(d) hydrate isomerism Ans. (d) : (A) For monodentate ligands A and B, no
Karnataka CET-2018 isomers are possible for [MA5B] type of complex.
2+
Ans. (a) : [Fe(NO2)3Cl3] and [Fe(ONO)3)Cl3] shows the Hence, [Ni(H2O)5NH3] does not show the geometrical
linkage isomerism because it contains the NO2 ligands. as well as optical isomerism.
(B) [MA4B2] i.e. [Ni(H2O)4(NH3)2]2+ type of octahedral
70. Which of the following complex compounds
complexes exist as cis and trans isomer (geometrical
shows optical isomerism? isomerism).
(a) cis–[Co(en)2Cl2]NO2 (b) [Co(H2O)6]2+
(c) [Co(CN)6]4– (d) K3[Fe(ox)3]
CG PET-2018
Ans. (a):The given species are the example of
octahedral complex. [M(AA)2ab] type complex shows
the optical isomerism (only cis form). The compound
is- cis–[Co(en)2Cl2] NO2
Objective Chemistry Volume-II 530 YCT
(c) [MA3B3] i.e. [Ni(H2O)3(NH3)3]2+ type of octahedral 76. Which of the following complex compounds
complexes are known to give cis and trans isomers gives 5 isomers?
(Geometrical isomerism) (M = metal; a, b, c, d, e, f = ligands)
• When the three ligands are in the same triangular face (a) Ma4b2 (b) Ma2b2c2
of the octahedron, the compound is called facial or (c) Mabcdef (d) Ma3b3
face-isomer. When the three ligands are on the same CG PET-2016
equatorial plane of the octahedron i.e. around the Ans. (b) : [Ma2b2c2] type complex is an important
meridian of the octahedron, the compound is called example of octahedral complex. This complex can exist
meridional or mer isomer. theoretically in five geometrical isomers that have been
shown below-

73. The complexes [Co(NH3)6][Cr(CN)6]and


[Cr(NH3)6][Co(CN)6] are the examples of which
type of isomerism?
(a) Linkage isomerism
(b) Ionization isomerism
(c) Coordination isomerism
(d) Geometrical isomerism.
COMEDK-2017 77. Which one of the following does not exhibit
Ans. (c) : The complexes Co ( NH 3 )6  Cr ( CN )6  and geometrical isomerism?
(a) Octahedral complex with formula [MX2L4]
Cr ( NH 3 )6  CO ( CN )6  are the examples of co- (b) Square planar complex with formula [MX2L2]
ordination isomerism. Both complex interchange the (c) Tetrahedral complex with formula [MABXL]
ligand between the metal in the cation and the metal in (d) Octahedral complex with formula [MX2(L –
the anion. L)2]
74. Which of the following coordination TS EAMCET-2016
compounds would exhibit optical isomerism? Ans. (c) : Tetrahedral complex does not show
(a) pentamminenitrocobalt(III) iodide geometrical isomerism because relative positions of the
(b) diamminedichloroplatinum(II) unidentate ligands attached to the central metal atom are
(c) trans-dicyanobis (ethylenediamine) chromium same with respect to each other.
(III) chloride 78. [Pt(NH3)2Cl2] complex shows ____ isomerism.
(d) tris-(ethylendiamine) cobalt (III) bromide (a) linkage (b) hydrate or solvate
AIIMS-2017 (c) co-ordination (d) cis-trans
SRMJEEE-2016
Ans. (d) : Tris (ethylenediamine) cobalt (II) complex
ion exhibit optical isomerism as illustrated below. Ans. (d) : Complex [Pt(NH3)2Cl2] shows cis trans
isomerism.

79.
Which of the following complexes does not
show geometrical isomerism?
75. Correct formula for Wilkinson's catalyst is (a) [Pt (NH3)2Cl2] (b) [Co (NH3)4Cl2]
(a) [(Ph3P)3RhCl] (b) [(Ph3P)2RhCl2] (c) [CoCl2(en)2] (d) [Ni(CO)4]
(c) [(Ph3P)3RhCl2] (d) [(Ph3P)2RuCl2] AMU-2016
AMU-2017 Ans. (d) : [Ni(CO) ] is a tetrahedral complex and it is
4
Ans. (a) : Wilkinson catalyst is the co-ordinate clear that geometrical isomerism cannot be shown by
compound of the rhodium with triphenyl phosphine and tetrahedral complexes because all the four ligands in
chloride. The chemical formula of the compound is this geometry have adjacent position (i.e cis position) to
[(PPh3)3RhCl]. The hybridization possessed by the one another and all the four bond angles are the same.
catalyst is dsp2 and the shape is a square planer. (109.5o).
Objective Chemistry Volume-II 531 YCT
80. The number of geometric isomers that can exist 84. Aluminium reacts with NaOH and forms
for square planar compound 'X'. If the coordination number of
[Pt(Cl) (Py) (NH3) (NH2OH)]+ is (Py = pyridine) aluminium in 'X' is 6, the correct formula of X
(a) 2 (b) 3 is
(c) 4 (d) 6 (a) [Al(H2O)4(OH)2]+ (b) [Al(H2O)3(OH)3]
JEE Main-2015 (c) [Al(H2O)2(OH)4]– (d) [Al(H2O)6](OH)3
Ans. (b): The given compound is [Pt(Cl) (py) (NH3) VITEEE- 2009
(NH2OH)]+. The geometry of the compound is square
planar. Square planar complexes of this type exist in Ans. (c) : 2Al + 2NaOH + 2H 2O  → 2NaAlO3 + 3H 2
sodium meta
three isomeric forms. aluminate

Sodium meta aluminate is soluble in water and changes


into the complex [Al(H2O)2(OH)4]–, in which
coordination number of Al is 6.
85. [Co(NH3)5SO4] Br and [Co(NH3)5Br]SO4 are a
pair......of isomers.
81. The compounds [Co(SO4)(NH3)5]Br and (a) ionisation (b) ligand
[Co(SO4)(NH3)5]Cl represent (c) Coordination (d) Hydrate
(a) linkage isomerism AP EAMCET-2008
(b) ionisation isomerism Ans. (a) : In ionisation isomer, the molecular formula
(c) coordination isomerism of both should be same but they will give different ion
(d) no isomerism in solution.
JCECE - 2015 +

Ans. (d) : The given complexes are not isomers as their  (


Co NH 3 )5 SO 4  Br ⇌ Co ( NH 3 )5 SO 4  + Br −
composition is not same. (Isomers have same 2+
Co ( NH 3 )5 Br  SO 4 ⇌  Co ( NH 3 )5 Br  + SO 4 2−
composition but different arrangement and properties.)
82. [Cr(NH3)6][Cr(SCN)6] and [Cr(NH3)2(SCN)4] Both are the pair of ionization isomers.
[Cr(NH3)4 (SCN)2] are the examples of what 86. Which one is an organo-metallic compound in
type of isomerism? the following
(a) lonisation isomerism (a) C2 H 5ONa (b) C2 H 5 – S – S – C2 H 5
(b) Linkage isomerism
(c) Coordination isomerism (c) Al2 (CH3 )6 (d) Al(C6 H 5S)3
(d) Solvate isomerism MPPET-2008
MHT CET-2015 Ans. (c) : Organometallic-compound that contain
Ans. (c) : The co-ordination isomerism take place when carbon metal bonds are called organometallic
two cation or anion exchange own ligands. compound.
Hence, [Cr(NH3)6] [Cr(SCN)6] and [Cr(NH3)2(SCN)4] Al2 (CH3)6 is an organometallic compound.
[Cr(NH3)4(SCN)2] are the example of co-ordination
isomerism. 87. In a particular isomer of [Co(NH3)4Cl2]0, the
Cl–Co–Cl angle is 90°, the isomer is known as
83. Which one of the following coordination entity
is chiral? (a) optical isomer (b) cis-isomer
(a) trans-[CrCl2(ox)2] 3–
(b) cis-[Co(NH3)4Cl2] + (c) position isomer (d) linkage isomer
3– + Karnataka NEET-2013
(c) cis-[CrCl2(ox)2] (d) trans-[Co(NH3)4Cl2]
COMEDK-2015 Ans. (b) :The given compound Co ( NH ) Cl  is the
 3 4 2
Ans. (c) : The compound which have not symmetry
element then compound shows the chirality and optical type of [ MA 4 B2 ] . [ MA 4 B2 ] type of octahedral complex
isomer in nature. exist as cis isomer ( the two monodentate ligands in 1,2-
+
trans – CrCl2 ( ox ) 2  ,Cis  Co ( NH 3 )4 Cl 2  , and
3–
positions) and trans-isomers (the two legends in 1,6
+
position).
trans – Co ( NH 3 ) 4 Cl2  have the symmetry and it
does not show the optical activity
cis –  CrCl2 ( ox ) 2  shows the chirality i.e.
3–

Objective Chemistry Volume-II 532 YCT


88. The complex [PtCl2(en)2]2+ ion shows 91. Which of the following complex does not show
(a) structural isomerism geometrical isomerism?
+
(b) optical isomerism only (a) Co ( NH 3 )4 Cl2  ⇒ [ Ma 4 b 2 ] ( V )
(c) geometrical and optical isomerism

(d) geometrical isomerism only (b)  Fe ( NH 3 ) 2 ( CN ) 4  ⇒ [ Ma 2 b 4 ]
COMEDK-2014 3−
Ans. (c) : Type [M(AA)2X2]– Such complexes show (c) Cr ( ox )3  [ M(aa)3 ]
both geometrical and optical isomerism. The trans (d) Co ( NH 3 )3 ( NO 2 )3  [ Ma 3 b3 ] ( V )
isomers will be optically inactive due to the presence of
symmetry, only cis-form is unsymmetrical and can form GUJCET-2007
two optical isomers. Ans. (c) : [Ma4b2], [Ma2b4] and [Ma3b3] are type of
octahedral complexes exist as cis-isomer and trans-
isomers which are exhibit geometrical isomerism.
[M(aa)3] exhibit optical isomerism as illustrated below–
(Where, A stands for bidentate ligand).

89. How many isomers does [Co(en)2Cl2]+ have?


(a) 1 (b) 3
92. [Fe(NO2)3Cl3] and [Fe(ONO)3Cl3] shows
(c) 2 (d) 4
(a) linkage isomerism
VITEEE- 2006 (b) geometrical isomerism
Ans. (b) : (c) optical isomerism
(d) none of the above
AIIMS-2008
Ans. (a) : In NO 2− ion, either oxygen or nitrogen can
act as donor giving two different isomers. This type of
isomerism occurs when more than a single atom in a co-
ordainated group may act as a donor. Two isomers of
iron having ligands chloride and NO 2− showing linkage
isomerism are [Fe(NO2)3Cl3] and [Fe (ONO)3Cl3]. They
are linkage isomers to each other.
93. The type of isomerism in coordination
compounds
[Cu (NH3)4] [PtCl4] and [Pt (NH3)4] [CuCl4] is
(a) coordination isomerism
(b) geometrical isomerism
(c) ionization isomerism
(d) linkage isomerism
AMU-2007
Ans. (a) : In coordination compounds, [Cu(NH3)4]
90. The structure of paramagnetic nickel complex, [PtCl4] and [Pt(NH3)4][CuCl4] exist in coordination
[NiCl4]2– is isomerism. This type of isomerism is shown by
(a) tetrahedral compounds in which both cation as well as anion are
(b) square planar complexes ( coordination entities) and ligands may
(c) trigonal bipyramidal intercharge their position between the two complex
(d) distorted octahedral ions.
AMU-2013 94. [(C6H5)2Pd(NCS)2] and[(C6H5)2Pd(SCN)2]
Ans. (a) : The oxidation state of Ni in [NiCl4]2– is – are
x+4(–1)= –2 (a) linkage isomeric
x = +2 (b) coordinated isomeric
Ni2+ = [Ar]3d8 , unpaired e– =2 (Paramagnetic) (c) ionic isomeric
The hybridization of the complex is sp3. (d) geometrical isomeric
Hence, the geometry will be tetrahedral. CG PET-2008
Objective Chemistry Volume-II 533 YCT
Ans. (a) : The given compound is- Ans. (d): Optical isomerism arises when the structure of
[(C6H5)2Pd(NCS)2] and [(C6H5)2Pd(SCN)2] one form is the mirror image of the other and both are
The complex contain the SCN ligand which is a not super imposable. Such type of compounds are
ambidentate ligand and it can form M–SCN and M– optical isomers. The condition for optical isomerism is
NCS. It shows the linkage isomerism because more than atleast one bidenatate ligand present in complex. Here,
a single atom in a co–ordinated group may act as a [Cr(NH3)4SO4]+ do not contain any bidentate ligand due
donor.
to which they do not show the optical isomerism.
95. Formula of hexaaqua manganese (II)
phosphate is 99. Which of the following compounds shows
optical isomerism?
(a)  Mn ( H 2 O )6  PO 4 (b)  Mn ( H 2O )6  PO 4
3 (a) [Co(CN)6]3– (b) [Cr(C2O4)3]3–
(c)  Mn ( H 2O )6  ( PO4 )2 (d)  Mn ( H 2 O )6  ( PO 4 )3 (c) [ZnCl4] 2–
(d) [Cu(NH3)4]2+
3

CG PET-2012 AIEEE-2005
Ans. (c) : The name of the complex is hexa aqua Ans. (b): Optical isomerism arises when the structure of
manganese (II) phosphate. The metal contains the +2 one form is the mirror image of the other and both are
oxidation state. not super imposable. Such compounds are called optical
(a) [Mn(H2O)6]PO4 (b) [Mn(H2O)6]3 PO4 isomerism [M(AA)3] type of complex exhibit the
x +6(0)+(–3)=0 3x+3×6×(0)+(–3)=0 optical isomerism. Thus, [Cr(C2O4)3]3+ shows the
x = +3 x = +1 optical isomerism.
(c) [Mn(H2O)6]3(PO4)2 (d) [Mn(H2O)6]6(PO4)3
3x+3×6×(0)+2(–3)=0 6x+6×6(0)+(–3) ×3 =0
3x = 6 6X–9 = 0
9 3
x = +2 x = =
6 2
Hence, the correct option will be (c).
96.  Pt ( NH 3 ) Cl 2  Br2 and  Pt ( NH 3 ) Br2  Cl 2
 4   4 
are related to each other as
(a) optical isomers (b) coordinate isomers
(c) ionisation isomers (d) linkage isomers 100. Which one of the following complex ions has
CG PET-2007 geometrical isomers?
Ans. (c): [Pt (NH3)4Cl2] and [Pt (NH3)4 Br2] Cl2. (a) [Co(en)3]3+ (b) [Ni (NH3)5]+
3+
The complex can exchange groups between the (c) [Co(NH3)2 (en)2] (d) [Cr(NH3)4 (en)]3+
complex ion and ion outside it. This type of isomerism AIEEE-2011
is called as ionization isomerism. Such compounds Ans. (c): Geometrical isomerism is exhibited by
behave differently when dissolved in water.
complexes in which same two ligands occupy positions
97. Which of the following does not show optical adjacent to each other or opposite to each other.
isomerisation?
3−
[Co(NH3)2(en)2]3+ is the type of [M(AA)2B2]. This ion
(a)  Pt ( NH 3 ) 2 C12  (b) Co ( ox )3  shows geometrical isomerism.
3+ 3+
(c) Co ( en )3  (d) Cr ( diph )3 
CG PET-2006
Ans. (a): Square planer complex do not exhibit optical
activity as they possess a plane of symmetry. Here,
[Pt(NH3)2Cl2] is a square planar complex.
98. Which of the following complex does not show
optical isomerism?
3−
(a) Cr ( C2O 4 )3 
2+
(b) Cis  Pt ( Br )2 ( en ) 2  101. Which one of the following has an optical
+
isomer?
(c) CrCl 2 ( NH 3 ) 2 en  (en = ethylenediamine)
+ (a) [Zn(en)(NH3)2]2+ (b) [Co(en)3]3+
(d) Cr ( NH 3 ) 4 SO 4  3+
(c) [Co(H2O)4(en)] (d) [Zn(en)2]2+
GUJCET-2014 AIEEE-2010
Objective Chemistry Volume-II 534 YCT
Ans. (b): [M(AA)3] is a type octahedral complex in Ans. (d) : [Pt(NH3)2Cl2] is the square planar complex of
which non-super impossible 3+ mirror image are MA2B2 type. It forms the cis and trans isomerism which
formed and hence. [Ce(en)3]3+ has the optical is illustrated below-
isomerism.

• [MA5B] type complex do not show the geometrical


isomerism.
107. The total number of possible isomers for the
complex compound [CuII(NH3)4][PtIICl4] are
(a) 3 (b) 6
102. Which of the following pairs represents linkage (c) 5 (d) 4
isomers? J & K CET-2005
(a) [Cu(NH3)4][PtCl4] and [Pt(NH3)4][CuCl4]
Ans. (d) : When the cationic and anionic complexes
(b) [Pd(PPh3)2(NCS)2] and [Pd(PPh3)2(SCN)2] contain the same metal atom in tetrahedral complexes,
(c) [Co(NH3)5]NO3SO4 and [Co(NH3)5SO4]NO3 there will be 2 isomers and if the metal atoms are
(d) [PtCl2(NH3)4]Br2 and [PtBr2 (NH3)2 different in the two ionic parts, there will be 4 isomers.
(PPh3)2(SCN)2]
108. Which of the following does not have optical
AIEEE-2009
isomer?
Ans. (b): The complex compound which contains the (a) [Co(NH3)3Cl3] (b) [Co(en)3]Cl2
bidentate ligands, shows the linkage isomerism. Hence,
[Pd(PPh3)2(NCS)2] and [Pd(PPh3)2(SCN)2] has the (c) [Co(en)2Cl2]Cl (d) [Co(en) (NH3)2Cl2]Cl
bidentate ligand i.e. NCS. JCECE - 2007
103. Which of the following complex species is not Ans. (a) : [Co(NH3)3Cl3] do not show optical isomerism
expected to exhibit optical isomerism? whereas it shows the geometrical isomerism (facial and
(a) [Co(en)3]3+ (b) [Co(en)2Cl2]+ meridional isomerism). Rest of all complexes shows the
(c) [Co(NH3)3Cl3] (d) [Co(en)(NH3)Cl2]+ optical isomerism.
JEE Main-2013 109. [Co(NH3)4Cl2]NO2 and [Co(NH3)4ClNO2]Cl
Ans. (c): Octahedral complexes of [MA3B3] i.e. exhibit which type is isomerism?
[Co(NH3)3Cl3] type exist in cis and trans isomers. Both (a) Geometrical (b) Optical
these isomers are optically inactive and hence do not (c) Linkage (d) Ionisation
show optical isomerism. JCECE - 2012
104. Which is the metal present in the coordination Ans. (d) : Both give different ions when subjected to
compound chlorophyll? ionisation.
(a) Ca (b) Mg [Co(NH3)4Cl2]NO2 ⇌ [Co(NH3)4Cl2]+ + NO−2
(c) K (d) Na
J & K CET-2014 [Co(NH3)4ClNO2]Cl ⇌ [Co(NH3)4ClNO2]+ + Cl–
Ans. (b) : Molecular formula of chlorophyll is They exhibit ionisation isomerism.
C55H72O5N4Mg. From molecular formula magnesium 110. The correct IUPAC name of [Pt (NH3)2Cl2] is
(Mg) is the only mineral element present in chlorophyll. (a) diammine dicholorido platinum (II)
105. The complexes [PtCl2(NH3)4]Br2 and (b) diammine dicholorido platinum (IV)
[PtBr2(NH3)4]Cl2 are examples for which type (c) diammine dicholorido platinum (0)
of isomerism (d) dicholorido diammine platinum (IV)
(a) geometrical (b) optical JCECE - 2013
(c) ionization (d) linkage
Ans. (a) : The correct IUPAC name of the compound is
J & K CET-2008
diammine dichlorido platinum (II).
Ans. (c) : The isomers which involve exchange of
ligands between co-ordination sphere and ionization 111. The IUPAC name of the complex ion formed
sphere are called ionization isomerism. Here, when gold dissolves in aqua-regia is
[PtCl2(NH3)4]Br2 and [PtBr2(NH3)4]Cl2 are example of (a) tetrachloridoaurate (iii)
ionization isomerism. (b) tetrachloridoaurate (i)
106. Which one amongst the following, exhibit (c) tetrachloridoaurate (ii)
geometrical isomerism (d) dichloridoaurate (iii)
(a) [CoIII(NH3)5Br]SO4 (b) CoIII [EDTA]–1 Karnataka CET-2014
(c) [CrIII(SCN)6]3– (d) [PtII(NH3)2Cl2] Ans. (a) : The correct IUPAC name of the compound is
J&K CET-2005 tetrachloridoaurate (iii).

Objective Chemistry Volume-II 535 YCT


112. Which one of the following complexes is not (c) linkage isomerism, ionization isomerism and
expected to exhibit isomerism? geometrical isomerism
(a) [Ni(NH3)4(H2O)2]2+ (b) [Pt(NH3)2Cl2] (d) ionization isomerism, geometrical isomerism
(c) [Ni(NH3)2Cl2] (d) [Ni(en)3]2+ and optical isomerism.
AIPMT -2010 AIPMT -2006
Ans. (c) : This type of isomerism is not possible in Ans. (c) : [Co(NH3)4(NO2)2] is a octahedral complex
complex having co-ordination number 4 (tetrahedral type of [MA4B2]. NO2 has the two donor site due to
complex). This is due to the fact that in these which they exhibit the linkage isomerism. If exchange
arrangements all the positions are adjacent to each of groups between the complex ion and ion outside it,
other. then the complex exhibit the ionization isomerism. As
113. Which of the following does not show optical we know that [MA4B2] type complex shows the geo
isomerism? metrical isomerism which is illustrated below-
(a) [Co(NH3)3Cl3] 0
(b) [Co(en)Cl2(NH3)2]+
(c) [Co(en)3]3+
(d) [Co(en)2Cl2]+ (en = ethylenediamne)
AIPMT -2009
Ans. (a) : [MA3B3] type of complex do not show the
optical isomerism but it is only exhibit the geometrical
isomerism. Thus, [Co(NH3)3Cl3] do not show the
optical isomerism.
• [Co(en)Cl2(NH3)2]+, [Co(en)3]3+ and [Co(en)2Cl2]+ are 117. Which one of the following octahedral
exhibit the optical activity because they do not contain complexes will not show geometrical
the plane of symmetry. isomerism? (A and B are monodentate ligands)
114. Which of the following does not show optical (a) [Ma4B2] (b) [MA5B]
isomerism? (c) [MA2B4] (d) [MA3B4]
(a) [Cr(NH3)6][Co(CN)6] UPTU/UPSEE-2007
(b) [Co(en)2Cl2]Cl Ans. (b) : [MA5B] type of complex do not exhibit the
(c) [Pt(NH3)4][PtCl6] geometrical isomerism.
(d) [Co(NH3)4Cl2]NO2
(en = NH2CH2CH2NH2) 2. Nomenclature
AIPMT -2007
Ans. (b) : The compound [CO(en)2Cl2]Cl does not 118. The IUPAC name of the following complex is–
show optical isomerism. [Cr(NH3)3(H2O)2Cl]Cl2
115. The anion of acetylacetone (acac) forms (a) Triamminediaqua chlorido chromium (III)
Co(acac)3 chelate with Co3+. The rings of the chloride
chelate are (b) Diaquatriammine chlorido chromium (III)
(a) five membered (b) four membered chloride
(c) six membered (d) three membered. (c) Chlorido diaquatriammine chromium (III)
Karnataka NEET 2013 chlorido
Ans. (c) : (d) Triammine diaqua trichlorido chromium (III)
AP EAMCET-08.07.2022, Shift-II
Ans. (a) : [(CrCNH3)3 (H2O) Cl]Cl2
IUPAC – Triamminediaqua chlorido chromium (III)
chloride.
119. The IUPAC name of the coordination
compound [Co(H2O)2(NH3)4] Cl3 is
(a) tetraamminediaquacobalt (III) chloride
(b) cobalt (III) tetraamminediaqua chloride
(c) diaquatetraamminecobalt (III) chloride
(d) tetraamminediaquacobalt (II) chloride.
J & K CET-2015, 1999, 1997
The acetylacetone makes the six membered ring with Ans. (a) : The IUPAC name of the coordination
3+
Co . compound [Co(H2O)2(NH3)4] Cl3 is- tetraamminediaqua
116. [Co(NH3)4(NO2)2]Cl exhibits cobalt (III) chloride
(a) linkage isomerism, geometrical isomerism 120. The IUPAC name for the complex
and optical isomerism [Co(NO2)(NH3)5]Cl2 is:
(b) linkage isomerism, ionization isomerism and (a) Pentaammine nitrito - N- cobalt (II) chloride
optical isomerism (b) Pentaammine nitrito - N- cobalt (III) chloride
Objective Chemistry Volume-II 536 YCT
(c) Nitrito-N-pentaamminecobalt (III) chloride Ans. (c) : When dimethylglyoxime solution is added to
(d) Nitrito-N-pentaamminecobalt(II) chloride an aqueous solution of nickel chloride in the presence of
AIIMS 26 May 2019 (Morning) ammonium hydroxide (NH4OH), a rosy red precipitate
AIEEE-2006 is formed.
Ans. (b) : [CO(NO2) (NH3)5]Cl2 is a octahedral
compound. The IUPAC name of the compound is
pentaammine nitrito-N-cobalt (III) chloride.
121. Mn2(CO)10 is an organometallic compound due
to the presence of
(a) Mn– C bond (b) Mn– O bond
(c) C–O bond (d) Mn – Mn
JEE Main-2021, 26 Feb Shift-II
JEE Main-2019, 12 Jan Shift-I
Ans. (a) : For a complex to be organometallic
compound, there should be atleast one metal-carbon
bond i.e. Mn – C bond.
122. Which is correct formula for pentammine
carbonatocobalt(III) chloride coordination
compound? (Rosy red precipitate)
(a) [Co(NH3)5(CO3)]Cl (b) [Co(NH3)5(CO2)]Cl Dimethyl glyoxime is negative bidentate ligand.
(c) [Co(NH3)5(CO3)]Cl2 (d) [Co(NH2)5(CO3)]Cl 126. An amine on reaction with benzenesulphonyl
GUJCET-2021 chloride produces a compound insoluble in
Ans. (a): The formula of pentamminecarbonato alkaline solution. This amine can be prepared
cobalt(III) chloride is - [Co(NH3)5(CO3)]Cl. by ammonolysis of ethyl chloride. The correct
123. Which product is obtained during reaction of structure of amine is:
MnO 4– with I– in faintly alkaline condition? (a) CH 3 CH 2 CH 2 NHCH 3
(a) I2 (b) IO3– H
(b) CH 3 CH 2 CH 2 N– CH 2 CH3
(c) IO– (d) IO 4–
GUJCET-2021 (c)
Ans. (b) : Reaction of MnO 4– with I– in faintly alkaline
condition IO3– is formed. (d) CH 3 CH 2 NH 2
JEE Main 26.02.2021, Shift-I
MnO 4– + I – → IO3– + Mn 2 +
Ans. (b) : According to the question the amine should
Hence the product form is IO3– . be 2°–amine in which one of the alkyl group should be
124. Which of the following complex ions does not ethyl because it can be formed by ammonolysis of ethyl
exist? chloride
(a) [B(H2O)6]3+ (b) [Al(H2O)6]3+ O
(c) [Ga (H2O)6]3+ (d) [In(H2O)6]3+ ||
TS EAMCET 04.08.2021, Shift-I R − NH 2 
C6H5SO2Cl
→ R − NH − S − C6 H 5
3+ (1°-amine) ||
Ans. (a) : [B(H2O)6] complex is not exist because the O
size of boron is very small and when 6 ligand are Solublein alkalines

attached the boron then steric hindrence are generate


between ligand.
125. Given below are two statements :
Statements I :
The identification of Ni2+ is carried out by
dimethyl glyoxime in the presence of NH4OH.
According to the question the amine should be 2°-
Statements II : amine, in which one of the alkyl group should be ethyl,
The dimethyl glyoxime is a bidentate neutral because it can be formed by ammonolysis of ethyl
ligand. chloride.
In the light of the above statements, choose the
Therefore, amine is CH3–CH2–CH2–NH–CH2–CH3
correct answer from the options given below :
(a) Both Statement I and Statement II are true. 127. Which of the following element do not form
(b) Both Statement I and Statement II are false. complex with EDTA?
(c) Statement I is true but Statement II is false. (a) Ca (b) Mg
(d) Statement I is false but Statement II is true. (c) Be (d) Sr
JEE Main 25.02.2021, Shift-II BITSAT-2021
Objective Chemistry Volume-II 537 YCT
Ans. (c) : Be do not form stable complex with EDTA Ans. (a) : The given compound is Co ( NH 3 )5 ( CO3 ) 
because it belongs to the second period. Ca2+ and Mg2+
form the stable complex with EDTA. AS we Know that Cl. The IUPAC name of compound will be
EDTA is the hexadentate ligand. pentaamminecarbonatocobalt (III) chloride.
128. [Fe(en)2(H2O)2]2+ + en → complex (X). The 132. Which of the following gives the maximum
correct statement about the complex (X) is– number of ions in aqueous solution?
(a) it is a low spin complex (a) Ni (CO)4 (b) CoCl3 . 5 H2O
(b) it is diamagnetic (c) PtCl4 . 6 NH3 (d) [Cr (NH3)3 (NO2)3]
(c) it shows geometrical isomerism AP EAPCET 25.08.2021, Shift-II
(d) (a) and (b) both Ans. (c) : The maximum number of ions in aqueous
solution is PtCl4.6NH3.
BITSAT-2021
133. The different types of bonds present in CuSO4 .
Ans. (d) : 5 H2O are ––––––––
[Fe(en)2 (H2O)2]2+ + en → [Fe(en)3]2+ 2H2O (a) Only Ionic and Covalent
'X' (b) Only Ionic, Covalent and Coordinate covalent
Fe formed the complex with ethylene diamine in which (c) Only Covalent, Coordinate covalent and
'en' act as strong field ligand. The pairing of electron Hydrogen bonding
occurred in inmer orbital of the metal. There is no (d) Ionic, Covalent, Coordinate covalent,
unpaired electron present in Fe2+. Hence, it is a inner Hydrogen bonding only
orbital complex with diamagnetic character. AP EAPCET 25.08.2021, Shift-II
• It shows only optical isomerism. Ans. (d) : CuSO4 . 5H2O is an inorganic compound blue
129. The secondary valency and the number of colour. The name of CuSO4 . 5H2O is copper sulfate
hydrogen bonded water molecules in CuSO4. Pentahydrate.
5H2O, respectively, are
(a) 6 and 4 (b) 4 and 1
(c) 6 and 5 (d) 5 and 1
JEE Main-2021, 18 March Shift-II
Ans. (b) : CnSO4 SH2O is the complex compound. The
structure of the compound is given below-

Metal connected with the four water molecule hence


secondary valence is 4 and no. of hydrogen bonded CuSO4 . 5H2O shows – Ionic, covalent, coordinate and
water molecule is 1. hydrogen bonding.
130. In the structure of the dichromate ion, there is 134. Which among the following doesn’t form a
a white precipitae with AgNO3 solution ?
(a) [Co(NH3)6]Cl3 (b) [Co(NH3)5 Cl]Cl2
(a) linear symmetrical Cr–O–Cr bond
(c) [Co(NH3)4 Cl2]Cl2 (d) [Co(NH3)3 Cl3]
(b) non-linear symmetrical Cr–O–Cr bond
AP EAPCET 24.08.2021, Shift-I
(c) linear unsymmetrical Cr–O–Cr bond
(d) non-linear unsymmetrical Cr–O–Cr bond Ans. (d) : The [Co(NH3)3Cl3] complex contain elements
of symmetry and is optically inactive. It does not give
JEE Main-2021, 31 Aug Shift-I white precipitate is present in the co-ordination sphere.
Ans. (b) : The chemical formula of dichromate ion 135. Complex X of composition Cr(H2O)6 Cln has a
[Cr2O7]2–. The structure of the ion is given below. spin only magnetic moment of 3.83BM. It
reacts with AgNO3 and shows geometrical
isomerism. The IUPAC nomenclature of X is
(a) tetraaquadichlorido chromium (IV) chloride
dihydrate
The dichromate ion has the symmetrical structure with (b) hexaaqua chromium (III) chloride
non-linear (c) dichloridotetraaqua - chromium (IV) chloride
131. The IUPAC name of [Co(NH3)5(CO3)] Cl is dihydrate
(a) Pentaamminecarbonatocobalt (III) chloride (d) tetraaquadichlorido - chromium (III) chloride
(b) Carbonatopentamminecobalt (III) chloride dihydrate
(c) Pentaamminecarbonatocobaltate (III) JEE Main-2020, 9 Jan Shift-II
chloride Ans. (d) : Given,
(d) Pentaamminecobalt (III) carbonatechloride spin only magnetic moment of Cr(H2O)6Cln=3.83 BM
Kerala CEE-29.08.2021 ∴ Number of unpaired electron (n)–
Objective Chemistry Volume-II 538 YCT
Ans. (b) : The IUPAC name of the complex is
µs = n ( n + 2 )
diamminechlorido (methanamine) platinum (II) chloride
3.83 = n ( n + 2 ) 139. Give the IUPAC name of [Pt(NH3)4] [PtCl4] is
putting the value of n =3 then we get the LHS equal to (a) tetraammine platinum (0) tetrachlorido
platinum (IV)
RHS. Hence, the value of n will be +3. That means
metal has +3 oxidation state. (b) tetraammine palatinate (II) tetrachlorido
platinum (II)
∴ The complex is Cr(H2O)6Cln→Cr(H2O)6Cl3 (c) tetraammine palatinate (0) tetrachlorido
Thus, the formula of the complex will be platinum (IV)
[Cr(CH2O)4Cl2]Cl.2H2O The IUPAC name of the (d) tetraamine platinum (II) tetrachlorido
compound is tetraaquadichloro chromium (III) chloride palatinate (II)
dihydrate.
Karnataka CET-2020
136. Complexes (ML5) of metals Ni and Fe have Ans. (d) : x + 0 + x + (–4) = 0
ideal square pyramidal and trigonal
bipyramidal geometries, respectively. The sum 2x = +4
of the 90°, 120° and 180° L-M-L angles in the x = +2
two complexes is ....... Oxidation state of platinum is +2,
JEE Main-2020, 8 Jan Shift-II IUPAC name of [Pt(NH3)4][Pt Cl4] is
tetraamminplatinum (II) tetrachlorido palatinate (II).
Ans. (20o) : The geometry of the complex are square
pyramidal and trigonal pipyramidal. The structure is 140. The correct formula of dichlorobis
given as- (triphenylphosphine) nickel (II) is
(a) [NiCl2(PPh3)2]Cl (b) [NiCl2(PPh3)]
(c) [NiCl2(PPh2)3] (d) [NiCl(PPh3)2]
(e) [NiCl2(PPh3)2]
Kerala CEE-2020
Ans. (e) : The formula of di-chloro bis-(triphenyl
phosphine) nickel (II) is –
[NiCl2 (PPh3)2]
Oxidation state of Ni = x – 2 + 0 × 2 = 0
(∵ PPh3 is a neutral ligand)
The sum of the 90o, 120o and 180o L–M–L angles in the
two complex is 20o. ⇒x=+2
137. The isomer (s) of [Co(NH3)4Cl2] that has/have a 141. Indicate the correct IUPAC name of the
Cl-Co-Cl angle of 900,is/are coordination compound shown in the figure.
(a) cis and trans
(b) meridional and trans
(c) cis only
(d) trans only
JEE Main-2020, 9 Jan Shift-II
Ans. (c) : [CO(NH3)4Cl2] is the octahedral complex (a) Cis-dichlorotetraminochromium(III) chloride
type of [MA4B2]. It forms the cis and trans isomerism (b) Trans-dichlorotetraminnochromium(III)
i.e. geometaical isomerism chloride
(c) Trans-tetraaminedichlorochromium(III)
chloride
(d) Cis-tetraaminedichlorochromium(III) chloride
WB JEE-2020
Ans. (d): IUPAC naming of ligands occurs in
alphabetical sequence
Ammine> chloro
Only cis form contains the Cl–CO–Cl 90o If two same ligand present at 90o with each other than if
form ‘cis’ and if they are at 180o then if form ‘trans’.
138. The IUPAC name of the complex In the given compound two chloride are at 90o so it
[Pt(NH3)2Cl(NH2CH3)]Cl is show cis isomer. Hence IUPAC name of this complex is
(a) diammine (methanamine) chlorido platinum Cis– tetraaminedichlorochromium (III) chloride
(II) chloride
142. Which among the following complexes can
(b) diamminechlorido (methanamine) platinum named "dichlorido-bis-(ethane-1, 2-diamine) -
(II) chloride platinum (IV) nitrate" ?
(c) diamminechlorido (aminomethane) platinum
(II) chloride (a) [ PtCℓ 2 (en)2 ] ( NO3 )2 (b) [ PtCℓ 2 (en)2 ] ( NO3 )4
(d) bisammine (methanamine) chlorido platinum (c) PtCℓ 2 (en)2 ( NO3 )  (d) PtCℓ 2 (en)2 ( NO2 ) 
(II) chloride
JEE Main-2020, 7 Jan Shift-I AP EAMCET (Engg.) 17.09.2020, Shift-II

Objective Chemistry Volume-II 539 YCT


Ans. (a) : The formula of coordination compound Ans. (d): Dissociation of following compounds–
dichlorido-bis-(ethane-1, 2-diamine)-platinum (IV) (a) [CO(NH3)6]Cl3 → [CO(NH3)6]3+ + 3Cl–
nitrate is– NO. of ions =4
[ PtCℓ 2 (en)2 ] ( NO3 )2 (b) [CO(NH3)5 SO4]Cl → [CO(NH3)SO4]+ + Cl–
No. of ions = 2
143. In photography, hypo is used as
(a) complexing agent (b) oxidising agent (c) [CO(NH3)5 Cl]SO4 → [CO(NH3)Cl]2+ + SO 24+
(c) reducing agent (d) all of these. No. of ions = 2
COMEDK-2020 (d) [CO(NH3)5 Cl]Cl2 → [CO(NH3)5Cl]2+ + 2Cl–
No. of ions =3
Ans. (a) : Hypo is used in photography because it is a
strong complexing agent. sodiumthiosulfate used as a Hence, option (d)
fixed in photography. The negative plate or film Pentaaminechlorido cobalt (III) chloride on dissociation
obtained at the end of photographic development with a gives 3 ions in solution.
reducing agent (i.e. potassium ferrous oxate, 148. The following ligand is
phrogallol) contains AgBr which has not been light-
activated and reduced to Ag metal. This is treated with
sodiumthiosulfate solution so that AgBr is removed as a
soluble complex compound.
144. How many ions obtain after dissociation of this
complex [Co(NH3)6] Cl3?
(a) 3 (b) 2
(c) 5 (d) 4
JIPMER-2019
Ans. (d) : In coordination complexes, only those ions (a) Hexadentate (b) Tetradentate
will dissociate which are present outside the (c) Bidentate (d) Tridentate
coordination sphere. JEE Main-2019, 8 April Shift-I
Dissociation
[Co(NH 3 )6 ]Cl3 → [Co(NH 3 )6 ]+ + 3Cl− Ans. (b) : The ligand contains the four donor site due to
which this is a tetradentate ligand
Hence, total 4 ions obtains after dissociation.
149. The incorrect statement is
145. The formula of pentaaquanitratochromium (a) the gemstone, ruby, has Cr3+ ions occupying
(III) nitrate is, the octahedral sites of beryl
(a) [Cr(H2O)6(NO)3] (b) the color of [CoCl(NH3)5]2+ is violet as it
(b) [Cr(H2O)6(NO2)2] absorbs the yellow light
(c) [Cr(H2O)5NO3] (NO3)2 (c) the spin only magnetic moments of
(d) [Cr(H2O)5NO2]NO3 Fe(H2O)6]2+ and [Cr(H2O)]2+ are nearly
Karnataka CET-2019 similar
Ans. (c) : The formula of pentaaquanitrato chromium (d) the spin only magnetic moment of [Ni(NH3)4
(III) nitrate is [Cr(H2O)5(NO3)](NO3)2 (H2O)2]2+ is 2.83 BM
146. IUPAC name of the complex Ba[CuCl4] is JEE Main-2019, 10 April Shift-II
(a) barium tetrachlorocuprate (II) Ans. (d) : The gemstone, ruley has Cr3+ accupying
(b) tetrachlorobariumcuprate (II) octohedral sites of Al3+ (aluminium oxide).
(c) barium tetrachlorocuprate (III) (b) According to complementary colour theory, if
complex absorbs yellow light then it shows violet.
(d) tetrachlorobarium copper (II)
(c) [Fe(H2O)6]2+ ⇒ Fe2+ ⇒ d6(H.S)
MHT CET-03.05.2019, SHIFT-I
Ans. (a) : If coordination sphere contains negative d 6⇒
charge then suffix 'ate' or Latin name of metal is used. Unpaired electron = 4
II [Cr(H2O)6]2+ ⇒ Cr2+ ⇒ d4(H.S)
IUPAC name of Ba[Cu Cl 4 ] is barium tetrachloro
cuprate (II). d 4⇒
147. The conductivity measurement of a Unpaired electron = 4
coordination compound of cobalt (III) shows Hence, both complex having nearly similar spin only
that it dissociates into 3 ions in solution. The magnetic moment.
compound is (d) [Ni(NH3)4(H2O)2]2+ ⇒ Ni2+ ⇒ d8
(a) hexaamminecobalt (III) chloride d 8⇒
(b) Pentaamminesulphatocobalt (III) chloride No. of unpaired electron = 2
(c) Pentaamminechloridocobalt (III) sulphate
(d) Pentaamminechloridocobalt (III) chloride Magnetic moment = n(n + 2) = 2(2 + 2) = 8
WB JEE-2019 = 2.83 B.M

Objective Chemistry Volume-II 540 YCT


150. IUPAC name for the complex 155. The cationic complex is
[Co(H2NCH2CH2 NH2)3]2(SO4)3is (a) cuprammonium ion (b) ferrocyanide ion
(a) Tetrakis (ethane-1-2-diammine) cobalt (II) (c) argentocyanide ion (d) mercuric iodide ion
sulphate COMEDK-2018
(b) tetrakis (ethane-1,2-diammine) cobalt (III)
Ans. (a) : The complexes are given as follow:-
sulphate
(c) tris(ethane-1,2-diammine)cobalt (II) sulphate.
(d) tris (ethane-1,2- diammine) cobalt (III)
sulphate.
J & K CET-2019
Ans. (d) : IUPAC naming of [Co(H2NCH2CH2NH2)3]2
(SO4)3 Cobalt is present in +3 oxidation state.
[Co(H2CH2CH2NH2)3]2 (SO4)3 ⇒ tris (ethane- 1, 2-
diamine) cobalt (III) sulphate
151. Which of the following species is not expected
to be a ligand?
(a) NO (b) NH +4
(c) NH2CH2CH2NH2 (d) CO
HP CET-2018
Ans. (b) : All the ligands are arranged in the
electrochemical series of co-ordination compound NO, (c) [ Ag(CN) ]–
2
NH2CH2CH2NH2 and CO exist in the electriochemical
series but NH +4 is not present in the series. (d) HgI2
156. When an aqueous solution of copper (II)
NO → Nitrosyl
sulphate is saturated with ammonia, the blue
NH2CH2CH2NH2 → Ethylenediamine
compound crystallises on evaporation. The
CO → Carbonyl formula of this blue compound is:
152. The IUPAC name of [Co (NH3)4Cl(NO2)]Cl is
(a) Cu ( NH 3 ) 4  SO 4 ⋅ H 2O ( square planar )
(a) tetraamminechloridonitrito-N-cobalt(III)
chloride (b) Cu ( NH 3 ) 4  SO 4 ( Tetrahedral )
(b) tetraamminechloridonitrocobalt(II) chloride
(c) tetraamminechloridonitrocobalt(I) chloride (c) Cu ( NH 3 )6  SO 4 ( Octahedral )
(d) tetraamminechloridonitrocobalt (III) chloride
Karnataka CET-2018 (d) Cu ( SO 4 )( NH 3 )5  ( Octahedral )
III AIIMS-27 May, 2018 (E)
Ans. (a) : The IUPAC name of [Co(NH 3 ) 4 Cl(NO 2 )]Cl
Ans. (a) : When an aqueous solution of copper (II)
is tetraammine chloride nitrito-N- cobalt (III) chloride. sulphate is is saturated with ammonia, the blue colour of
153. Ferric ion forms a prussian blue precipitate [Cu (NH3)4]2+ ion is formed which is square planar in
due to the formation of nature.
(a) K4[Fe(CN)6] (b) K3[Fe(CN)6] CuSO 4 ⋅ 5H 2 O + 4NH3  → [Cu(NH 3 ) 4 ]SO4 ⋅ H 2 O + 4H 2 O
(c) Fe(CNS)3 (d) Fe4[Fe(CN)6]3 Square planar

WB JEE-2018 157. Which one of the following ions has an


Ans. (d) : Ferric ion forms a Prussian blue precipitate of octahedral structure?
ferric ferrocyanide when reacts with potassium (a) [Ag(CN)2]– (b) [Cu(NH3)4]2+
ferrocyanide. (c) [HgI4] 2–
(d) [Fe(CN)6]4–
Fe3+ + Ku[Fe(CN)6] → Fe4[Fe(CN)6]3
COMEDK-2017
154. Silver chloride dissolves in excess of
ammonium hydroxide solution. The cation Ans. (d) :
present in the resulting solution is Compound Hybridisat Structure
(a) [Ag(NH3)6)]+ (b) [Ag(NH3)4)]+ ion
(c) Ag+ (d) [Ag(NH3)2]+ (a)  Ag ( CN ) 2 
– sp [ NC – Ag – CN ]

Linear
WB JEE-2018
Ans. (d): Silver chloride dissolves in excess of (b) Cu ( NH 3 ) 4 
2+ dsp2
ammonium hydroxide solution and forms diammin
silver (I) ion.
AgCl + NH 4OH → [Ag(NH 3 ) 2 ]+
(excess)

Objective Chemistry Volume-II 541 YCT


sp3
(c)
[ HgI4 ] Ans. (b) : When chloride ion present outside the
2–

coordination then it reacts with silvers nitrate and form


AgCl.
[Co(NH3)6]Cl3 + AgNO3 → 3AgCl
[Co(NH3)5Cl]Cl2 + AgNO3 → 2AgCl
(d) 4− 2
d sp 3 [Co(NH3)4Cl2]Cl + AgNO3 → AgCl
 Fe ( CN )6  161. [CuCl4]2– exists while [CuI4]2– does not exist,
because
(a) I– is stronger reductant than Cl–
(b) I– is weaker reductant than Cl–
(c) I– is stronger oxidant than Cl–
158. Primary valency of Co in [Co(en)2Cl2]Cl is
(a) + 1 (b) + 2 (d) None of the above
(c) + 3 (d) + 6 JIPMER-2016
2–
AMU-2017 Ans. (a) : [CuI 4 ] is not exist because I– is stronger
– –
Ans. (c) : Werner's' stated that primary valency is reductant than 2+
Cl , I act as reducing agent which
satisfied by oxidation state. The compound is –– reduces Cu into Cu+ and oxidizes itself into I2.
2–
[CO(ex)2Cl2]Cl Therefore [CuI4] complex does not exist.
Let, x be the oxidation state of Co 162. As per IUPAC norms, the name of the complex
∴ X + 2 (0) + 2 (–1) + (–1) = 0 [Co(en)2 (ONO) Cl] Cl is
or x=+3 (a) chlorido bis (ethane-1, 2-diamine) nitro-O-
Hence, the primary valency of the compound is + 3. cobalt (III) chloride
(b) chloro bis (ethylenediamine) nitro-O-cobalt
159. On treatment of 100 mL of 0.1 M solution of
(III) chloride
CoCl3.6H2O with excess of AgNO3; 1.2×1022
ions are precipitated. The complex is (c) chloride di (ethylenediamine) nitrocobalt (III)
(a) [Co(H2O)4Cl2]Cl.2H2O chloride
(b) [Co(H2O)3Cl3]3H2O (d) chloro ethylenediaminenitro-O-cobalt (III)
(c) [Co(H2O)6]Cl3 chloride
(d) [Co(H2O)5Cl]Cl2.H2O Karnataka CET-2016
III
JEE Main-2017 Ans. (a) : The IUPAC name of [Co(en)
2 (ONO)Cl]Cl is
Ans. (d) : Given chlorido bis (ethan -1, 2- diamine) nitro-O- cobalt (III)
Volume of solution = 100mL chloride.
concentration of solution = 0.1 M
22 163. Which of the following complexes would give
ions of AgCl = 1.2×10
white precipitate with excess of AgNO3 sol.
COCl3. 6H2O+AgNO3 →AgCl↓+[COClx6H2O]+(ppt) (a) [Co(NH3)2Cl2]NO3 (b) [Co(NH3)5SO4]Cl
∴ Number of moles of CoCl3.6H2O=Molarity ×Volume (c) [Co(NH3)4Cl2] (d) [Co(NH3)5NO3]NO2
100
= 0.1× AMU-2016
1000 Ans. (b) : [Co(NH3)5SO4]Cl is a complex compound
= 0.01 which give white precipitate with excess of AgNO3
Total ions of AgCl solution.
number of mole of AgCl =
Avogadro 's number [CO(NH3)5SO4]Cl (aq) + AgNO3(aq) → AgCl(s) ↓
ppt
1.2 ×1022
n AgCl = 164. The hypothetical complex chloro-
8.023 × 1023 diaquatriamminecobalt (III) Chloride can be
nAgCl = 0.02 mol
represented as
∵ [Co(H2O)zClx]Cly
(a) [CoCl(NH3)3 (H2O)2] Cl2
∴ y×0.01 = 0.02
(b) [Co(NH3)3 (H2O)2Cl3]
y=2
(c) [Co(NH3)3 (H2O)2 Cl]
Hence, the complex is [Co(H2O)5Cl]Cl2H2O
(d) [Co(NH3)3 (H2O)3] Cl3
160. The correct order of the stoichiometries of
BITSAT-2016
AgCl formed when AgNO3 in excess in treated
with the complexes: CoCl3.6NH3, CoCl3.5NH3, Ans. (a) : the chemical formula of chloro-diaqva
CoCl3.4NH3 respectively is triammine cobalt (III) chloride is [COCl(NH3)3(H2O)2]
(a) 3AgCl, 1AgCl, 2AgCl Cl2. It is a octahedral complex.
(b) 3AgCl, 2AgCl 1AgCl 165. Out of the following statements with respect to
(c) 2AgCl, 3AgCl, 2AgCl the postulates of Werner’s theory,
(d) 1AgCl, 3AgCl, 2AgCl 1. The central metal atom of coordination
NEET-2017 compounds shows two types of valencies
Objective Chemistry Volume-II 542 YCT
2. The primary valencies are normally non Ans. (a) : If more than one ligand is present then
ionisable and are satisfied by neutral IUPAC naming occurs in alphabetical order.
molecules or negative ions. III
3. The secondary valencies are ionsable and IUPAC name of [Co(NH 3 )3 (NO 2 )3 ] is triammine
are satisfied by negative ions only trinitro -N-cobalt (III).
4. The ions/groups bound by the secondary
linkages to the metal have characteristic 170. The name of complex ion, [Fe(CN)6]3– is
spatial arrangements corresponding to (a) hexacyanitoferrate (III) ion
different coordination numbers. (b) tricyanoferrate (III) ion
The incorrect statements are (c) hexacyanidoferrate (III) ion
(a) (1) and (2) (b) (3) and (4) (d) hexacyanoiron (III) ion.
(c) (2) and (4) (d) (2) and (3) AIPMT-2015
J & K CET-2016 Ans. (c) : The IUPAC naming of coordination sphere
Ans. (d) : According to Werner's theory- containing negative charge is different from other. If
The central metal atom of coordination compounds coordination sphere is negatively charged then suffix
shows two types of valencies 'ate' or Latin name of metal is used,
(i) Primary valencies- are generally ionisable and +3 −
always satisfied by negative ions. Primary valencies IUPAC name of [Fe(CN)6 ]3 is hexacyanido ferrate
corresponds to oxidation state of the metal ion. (III) iron.
(ii) Secondary valencies- are normally non-ionisable
and always satisfied by negative ions or neutral 171. Cobalt (III) chloride forms several octahedral
molecules. Secondary valencies corresponds to complexes with ammonia. Which of the
coordination number of the metal ion. following will not give test for chloride ions
with silver nitrate at 25°C?
166. Which of the following is not an anionic
complex? (a) CoCl3.5NH3 (b) CoCl3.6NH3
(a) [Cu(NH3)4]Cl2 (b) K4[Fe(CN)6] (c) CoCl3.3NH3 (d) CoCl3.4NH3
(c) K3[Fe(CN)6] (d) [NiCl4]2– NEET-2015, (Cancelled)
SRMJEEE-2016 Ans. (c) : When no chloride ion present outside the
Ans. (a) : Given compound are– coordination sphere then this complex will not given
(a) [Cu(NH3)4]Cl2 → [Cu(NH3)4]+2 + 2Cl– test with silver nitrate.
(Cationic complex) [Co(NH3)5Cl]Cl2 + AgNO3 → 2AgCl
(b) K4[Fe(CN)6] → 4K+ + [Fe(CN)6]–4 [Co(NH3)6]Cl3 + AgNO3 → 3AgCl
(Anionic complex) [Co(NH3)4Cl2]Cl + AgNO3 → AgCl
(c) K3[Fe(CN)6] → 3K+ + [Fe(CN)6]–3 [Co(NH3)6]Cl3 + AgNO3 → No AgCl is formed
(Anionic complex) Hence, CoCl3 3NH3 will not give test for chloride ion
(d) [NiCl4]–2 (Anionic complex) with silver nitrate at 25ºC
∴ [Cu(NH3)4]+2 is only cationic complex. 172. When conc. HCl is added to the solution of Cr
167. Select the correct IUPAC name of [Co (NO3)2, then it slowly turns green due to
(NH3)5(CO3)]Cl. formation of the following:
(a) Penta ammonia carbonate cobalt (III) chloride (a) CrCl3 (b) Cr2O3
(b) Pentammine carbonate cobalt chloride (c) CrO4 (d) Chlorocomplex
(c) Pentammine carbonato cobalt (III) chloride
MPPET - 2012
(d) Cobalt (III) pentammine carbonate chloride
AP EAMCET – 2016 Ans. (b) : Cr2O3 is dark green free crystal.
Ans. (c) : The IUPAC name of [Co(NH3)5(CO3)]Cl is 173. The number of ions given by the complex
pentammine carbonato cobalt (III) chloride. compound [Co (NH3)4 Cl2] Cl is:
168. The common name of K[PtCl3(η2C2H4) is (a) 2 (b) 3
(a) potassium salt (b) Zeise’s salt (c) 4 (d) 5
(c) complex salt (d) None of these MPPET - 2012
BITSAT-2015 Ans. (a) :
Ans. (b) : The common name of K[P + Cl3(n2 – C2H4)] [Co(NH3)4Cl2]Cl → [Co(NH3)4Cl2]+ + Cl–
is Zeise's salt. The IUPAC of this compound is Hence, the complex compound is [Co (NH3)4 Cl2] Cl
potassium trichloro (ethylene) palatinate (II). has two ions.
169. The correct IUPAC name of [Co(NH3)3(NO2)3] 174. The IUPAC name of Ni(CO)4 is :
is (a) Tetracarbonylnickelate(0)
(a) Triammine trinitro-N cobalt (III)
(b) Triammine trinitro-N cobalt (II) (b) Tetracarbonylnickelate(II)
(c) Triammine cobalt (III) nitrite (c) Tetracarbonylnickel(0)
(d) Triammine trinitro-N cobaltate (III) (d) Tetracarbonylnickel(II)
MHT CET-2015 MPPET-2013
Objective Chemistry Volume-II 543 YCT

Ans. (c) : Nickel carbonyl (IUPAC name 179. When SCN is added to an aqueous solution
tetracarbonylnickel) is the organonickel compound with containing Fe(NO3)3, the complex ion produced
the formula Ni(CO)4. is
This colourless liquid is the principal carbonyl of 2+
nickel. (a)  Fe ( OH 2 ) 2 ( SCN − ) 
175. The effective atomic number of cobalt in the 2+
complex [Co(NH3)6]3+ is (b)  Fe ( OH 2 )5 ( SCN − ) 
(a) 36 (b) 24 2+
(c) 33 (d) 30 (c)  Fe ( OH 2 )8 ( SCN − ) 
VITEEE- 2011 6+
3+ (d)  Fe ( OH 2 ) ( SCN − ) 
Ans. (a) : The compound is Co ( NH3 )6  .
VITEEE- 2008
Effective Atomic Number (EAN) = atomic number of
metal – oxidation state + 2(ligand contribution) Ans. (b) : Fe (III) ion from ferric nitrate will react with

EAN = 27 – 3 + 6 × 2 = 36 thiocyante, SCN ion to form a blood red complex i.e.
FeSCN2+. But in presence of water it forms a complex
176. The IUPAC name for the complex containing five water molecules i.e [Fe(H O) (SCN–)]2+.
2 5
[Co(NO2)(NH3)5]Cl2 is 2+
(a) nitrito-N-pentaammine cobalt (III) chloride 180. The IUPAC name of [Ni(PPh )
3 2 Cl 2 ] is
(b) nitrito-N- pentaammine cobalt (II) chloride (a) bis dichloro (triphenylphosphine) nickel (II)
(c) pentaamminenitrito-N-cobalt (II) chloride (b) dichloro bis (triphenylphosphine) nickel (II)
(d) pentaamminenitrito-N-cobalt (III) chloride (c) dichloro triphenylphosphine nickel (II)
VITEEE- 2011 (d) triphenyl phosphine nickel (II) dichloride
Ans. (d) : The given complex is [Co(NO2)(NH3)5]Cl2. VITEEE- 2007
2+
Hence, the IUPAC name of the given compound is Ans. (b) : The IUPAC name of [Ni(PPh3)2Cl2] is
pentaamminenitrito-N-cobalt (III) chloride. dichloro bis (triphenylphosphine) nickel (II).
177. The correct formula of the complex 181. Name the complex Ni(PF3)4
tetraammine aqua chlorocobalt (III) chloride is (a) tetrakis (phosphorus (III) fluoride) nickel (0)
(a) [Cl(H2O)(NH3)4 Co]Cl (b) tetra (phosphorus (III) fluoride) nickel
(b) [CoCl(H2O)(NH3)4]Cl (c) Nickel tetrakis phosphorus (III) fluoride
(c) [Co (NH3)4 (H2O)Cl]Cl (d) (phosphorus (III) tetrakis fluoride) nickel (0)
(d) [CoCl(H2O)(NH3)4]Cl2 VITEEE- 2006
VITEEE- 2010
Ans. (a) : Ni(PF3)4 – tetrakis phosphours (III) fluoride
Ans. (d) : The correct formula of the given complex is nickel (0).
tetraammine aqua chlorocobalt (III) chloride is
[CoCl(H2O)(NH3)4]Cl2, because in it the oxidation 182. A coordination compound in its molecular
number of Co is+3. While in rest other options, formula contains five ammonia molecules, one
oxidation number of Co is + 2 nitro group, two chloride ions and one cobalt
[CoCl(H2O)(NH3)4]Cl2 ion. One mole of the complex salt produces
⇒ x + (–1) +0 + (0 × 4) + (–1) 2 = 0 three moles of ions in its aqueous solution.
⇒x–3=0 With excess of AgNO3 solution, one mole of it
gives two moles of AgCl ppt. Formula of the
⇒ x = +3
complex is
178. The IUPAC name of the given compound (a) [Co(NH3)5NO2]Cl2
Co ( NH 3 )5 Cl  Cl 2 is (b) [Co(NH3)5NO2Cl2]
(a) pentaamino cobalt chloride chlorate (c) [Co(NH3)3Cl2NO2].(NH3)2
(b) cobalt pentaamine chloro chloride (d) [Co(NH3)4NO2Cl].NH3.Cl
(c) pentaamine chloro cobalt (III) chloride SRMJEEE-2013
(d) pentaamino cobalt (III) chlorate Ans. (a) :
VITEEE- 2008
Ans. (c) : For writing IUPAC name of a co-ordination Ionisation
[Co (NH 3 )5 NO 2 ]Cl2 ↽ ⇀
compound we write the name of cation first of the
2+
1mole
complex here Co ( NH 3 )5 Cl  .
[Co (NH 3 )5 NO 2 ]2+
The names of ligands will come first in alphabetical 1mole
order, followed by metal ion with its oxidation state
written in bracket or parentheis in roman number i.e. Co
(III) here. IUPAC name for cationic complex
pentaammine chloro cobalt (III).
This will follow the name of anion with a gap. i.e.
pentaammine chloro cobalt (III) chloride.

Objective Chemistry Volume-II 544 YCT


183. Which of the following is soluble in ammonium 187. The IUPAC name of [Co (NH3)6 ] [Cr(C2O4)3] is
hydroxide solution? (a) Hexa-amine cobalt (III) tris (oxalato)
(a) AgCl (b) PbCl2 chromium
(c) AlCl3 (d) MgCl2 (b) Hexa-amine cobalt (III) tris (oxalato)
chromate (III)
SRMJEEE – 2013
(c) Hexa-amine cobalt tris (oxalato) chromium
Ans. (a) : In ammonium hydroxide solution AgCl is (III)
soluble. (d) Hexa-amine cobalt (III) chromium (III)
AgCl + 2NH 4 OH  → [Ag(NH 3 ) 2 ]Cl + 2H 2 O oxalate
Ammonium soluble MPPET-2008
hydroxide Ans. (b) : The IUPAC name of [Co (NH3)6] [Cr(C2O4)3]
184. Identify the coordination compound, that has Hexa-amine cobalt (III) tris (oxalato) chromate (III)
the lowest electrical conductance in the 188. The IUPAC name of K4 [Ni(CN)4] is
aqueous solution. (a) Tetrapotassium tetracyno nickelate (II)
(a) [Cr(NH3)5Cl]Cl2 (b) [Cr(NH3)4Cl2]Cl (b) Potassium tetracyno nickel (II)
(c) [Cr(NH3)3Cl3] (d) [Cr(NH3)6]Cl3 (c) Potassium tetracyno nickelate (II)
SRMJEEE-2013 (d) Potassium tetracyno nickelate (II)
Ans. (c) : Electrical conductance is directly proportional MPPET-2008
to the number of ions produced in the solution. Ans. (c) : The compound is K4[Ni(CN)4]. The IUPAC
(i) [Cr(NH3)5Cl]Cl2 = 3 ions name of compound is potassium tetracyno nickelate (II).
(ii) [Cr(NH3)4Cl2]Cl = 2 ions 189. Excess of concentrated sulphuric acid on
heating with copper produces
(iii) [Cr(NH3)3Cl3] = no dissociation.
(a) CuSO 4 + H 2
(iv) [Cr(NH3)6]Cl3 = 4 ions
Therefore, [Cr(NH3)3Cl3] has lowest electrical (b) CuSO 4 + H 2 O + SO 2
conductance. (c) CuO + H 2SO3
185. One mole of COCl 3 .YNH 3 complex compound (d) CuO + H 2S
on complete ionisation in water produces three SCRA-2009
moles of ions. If one chloride satisfies both Ans. (b) : Copper reacts with hot and concentrated
primary and secondary valencies of cobalt ion, H2SO4 to form CuSO4, SO2 and H2O. The reaction is
the value of Y is given below-
(a) 3 (b) 4 Cu + H 2SO 4 → CuSO 4 + SO 2 + H 2 O
(c) 5 (d) 6
190. The formula of the product formed, when
AP- EAMCET(Medical) -2010 sodium thiosulphate solution is added to silver
Ans. (c): It is given that one mole of a complex bromide is
compound COCl3 ⋅ YNH 3 gives three moles of ions on (a) Ag2S2O3 (b) Ag2S
dissolution in water. As one mole of complex give three (c) Na 3 [Ag(S2 O 3 ) 2 ] (d) Ag3[Na(S2O3)2]

moles of ions, two Cl ions must present outside the AP EAMCET (Engg.) -2007
sphere. Ans. (c) : When sodium thiosulphate solution is added
2+ to silver bromide then Na3[Ag(S2O3)2] is formed.
CO ( NH3 )5 C1 Cl2 
Ionisation
→ CO ( NH3 )5 Cl  + 2Cl – Na2S2O3 + AgBr → Na3[Ag(S2O3)2] + NaBr
Total ions = 3 191. A coordinate complex contains Co3+,Cl– and
Therefore, on dissolution in water, three ions are NH3. When dissolved in water, one mole of this
produced and they are 2Cl– and [CO (NH3)5 Cl]2+ ions. complex gave a total of 3 moles of ions. The
The value of Y will be 5. complex is
186. A coordinate complex contains Co , Cl and 3+ – (a) Co ( NH3 )6  Cl3 (b) Co ( NH3 )5 Cl  Cl2
NH3. When dissolved in water, one mole of this (c) Co ( NH3 )4 Cl2  Cl (d) Co ( NH 3 )3 Cl3 
complex gave a total of 3 moles on ions. The
complex is AP EAMCET (Engg.) 2012
(a) [Co(NH3)6]Cl3 (b) [Co(NH3)5Cl]Cl2 Ans. (b) : When the complex dissolved in water, it
(c) [Co(NH3)4Cl2]Cl (d) [Co(NH3)3Cl3] gives three ions which shows that two Cl– ions are
present outside the coordination sphere.
AP EAMCET (Engg.) - 2012
Ans. (b) : The complex contains the CO3+, Cl– and NH3 ∴ The formula of the complex is Co ( NH 3 )5 Cl  Cl 2 .
 
Here, [CO (NH3)5Cl] Cl2 dissolved in water then one
mole of this complex gave a total of 3 mole of this
complex gave a total of 3 moles on ions.
[CO (NH3)5Cl]2+ + 2Cl– → 3

Objective Chemistry Volume-II 545 YCT


192. CO ligand is 197. Molar conductance of a complex of cobalt is
(a) carbonyl (b) carbonium zero. Then its structure would be
(c) carboso (d) carbanosium (a) [CO(NH3)]3Cl3 (b) [Co(NH3)Cl]Cl3
COMEDK-2011 (c) [Co(NH3)3Cl2]Cl (d) [Co(NH3)3Cl3]
Ans. (a) :The name of the 'CO' ligand is carbonyl which COMEDK-2014
is the π − acceptor ligand. The position of this ligand in Ans. (d) : As we know if metal has zero molar
spetrochemical series is in right side. conductance value then this is not ionises i.e.
193. An example of complex anion is Co ( NH 3 )3 Cl3 
(a) hexaammineplatinum ion
(b) cuprammonium ion 198. Which of the following acts as best
(c) ferrocyanide ion semipermeable membrane?
(d) hexaamminechrmium ion. (a) Parchment paper (b) Cu 2  Fe ( CN )6 
COMEDK-2011
(c) Plant cell wall (d) Cellophane
Ans. (c) : The example of complex anion is
GUJCET-2007
ferrocyanide-ion. The chemical formula of the complex
Ans. (b) : The membrane that allows only the solvent
is  Fe ( CN )6 
4–
molecules to pass through it and not the solute molecule
is called semipermeable membrane. Copper
ferrocyanide serves best as a semipermeable membrane.
In this membrane, pressure upto 150 atmosphere can be
applied.
199. The name of the ring structure complex
compound formed between metal ion and
194. IUPAC name of [CrCl2(NH3)4]NO3 is polydentate ligand is..........
(a) tetraamminedichlorochromium (I) nitrate (a) Polynuclear complex (b) Chelate complex
(b) dichlorotetraamminechromium (III) nitrate (c) Simple complex (d) None of these
(c) tetraamminedichlorochromium (IV) nitrate GUJCET-2008
(d) tetraamminedichlorochrmium (III)nitrate. Ans. (b) : The ring structure complex compound
COMEDK-2012 formed between metal ion and polydentate ligand is
Ans. (d) : The IUPAC name of CrCl 2 ( NH 3 )4  is chelate complex. The complex produced by this process
is called a chelate, and the polydentate ligand is referred
tetraamminedichlorochromium (III) nitrate. to as a chelating agent.
195. The correct IUPAC name for [CrF2(en)2] Cl is 200. The hypothetical complex chloro-
(a) chloro difluorido–ethylene–diaminechromium diaquatriamminecobalt (III) chloride can be
(III) chloride
represented as
(b) difluoridobis–(ethylene diamine)–chromium
(a) [CoCl(NH3)3(H2O)2]Cl2
(III) chloride
(b) [Co(NH3)3(H2O)Cl3]
(c) difluorobis–(ethylene diamine) chromium
(III) chloride (c) [Co(NH3)3(H2O)Cl]
(d) chloro difluoridobis–(ethylene diamine) (d) [Co(NH3)3(H2O)3]Cl3
–chromium (III) PMET-2007
Karnataka NEET-2013 Ans. (a) : The IUPAC name of the compound is chloro-
Ans. (b) : The correct IUPAC name for diaquatriammine cobalt (III) chloride. The chemical
CrF2 ( en )2  Cl is difluridobis (ethylene diamine) formula of the complex is.
[Co(NH3)3(H2O)2]Cl2
chromium (III) chloride.
201. When AgNO3 is added to a solution of
196. The complex [Ag(NH3)2][Ag(CN)2] has the
CoCl3.5NH3, the precipitate of AgCl shows two
IUPAC name
ionizable chloride ions. This means:
(a) diamminesilver (I) dicyanosilver (I)
(a) Two chlorine atome satisfy primary valency
(b) diamminesilver (I) dicyanoargentate (I)
and one secondary valency.
(c) dicyanosilver (I) diammineargentate (I)
(b) One chlorine atom satisfies primary as well as
(d) diamminesilver (I) dicyanoargentate (I).
secondary valency.
COMEDK-2014
(c) Three chlorine atoms satisfy primary valency.
Ans. (b,d): The given complex is
(d) Three chlorine atome satisfy secondary
 Ag ( NH 3 )2   Ag ( CN )2  The IUPAC name of the valency.
compound is diamminesilver(I) dicyanoargentate (II) AIIMS-2012
Objective Chemistry Volume-II 546 YCT
Ans. (a) : As we know that when AgNO3 is added to a [CuSO4 + 2KCN → Cu(CN)2 + K2SO4] × 2
solution of COCl3 ⋅5NH3 then the following reaction Cu(CN)2 → Cu2(CN)2 + (CN)2
occurred –– Cu2CN2 + 6KCN → 2K3Cu(CN)4
[CO(NH3)5Cl]Cl2 + 2AgNO3 →[Co(NH3)5Cl](NO3)2 ––––––––––––––––––––––––––––––––––––––
+ 2CuSO4 + 10KCN → 2K3Cu(CN)4 +2K2SO4 + (CN)2
2AgCl 207. The IUPAC name of Hg[Co(NCS)4] is
Hence, two chlorine atom satisfy the primary valency (a) mercury cobalt(II)tetrasulphocyanide
and one, secondary valency. (b) mercury tetrathiocyanatocobalt(II)
202. The IUPAC name of the complex (c) mercury tetrathiocyanato–N–cobaltate(II)
Hg[Co(CNS)4] is (d) tetrathiocyanatocobalt(II) mercurate.
(a) mercury tetrathiocyanatocobaltate (II) AMU – 2008
(b) Mercury cobalttetrasulphocyano (II) Ans. (c) : The IUPAC name of Hg[Co(NCS)4] is
(c) mercury tetrasulphocyanidecobalt (II) mercury tetrathiocyanato-N-cobaltate(II).
(d) tetrasulphocyantocobalt mercurate (II) 208. Which of the following compounds is generally
AIIMS-2010 used for hydrogenation of alkene?
Ans. (a) : The complex is Hg[CO(CNS)4]. The IUPAC (a) Ni(CO)4 (b) [(C6H5)3P]3RhCl
name of the compound will be mercury (c) (CH3)3Al (d) (C5H5)2Fe
tetrathiocyanatocobaltate (II). AMU-2005
203. Which of the following is the major source of Ans. (b) : [Rh(PPh3)3 Cl] is used as hydrogenation of
magnesium and is also a double salt? alkene. The name of the compound is Wilkinson's
(a) MgCO3 (b) Mg2P2O7 catalyst.
209. NO3− is detected by ring test. Ring formed has
(c) (d) KCl.MgCl2.6H2O
formula
AMU–2010 (a) FeSO 4 ⋅ HNO 2 (b) FeSO 4 ⋅ NO 2
Ans. (d) : Carnallite is the example of double salt. the 2+
chemical formula of carnallite is KCl. Mg Cl2.6H2O. It (c)  Fe ( H 2 O )5 ⋅ NO  (d) Fe ( OH )2 ⋅ NO
is a major source of magnesium. CG PET- 2012
204. Which of the following is Vaska’s compound ? Ans. (c) : A common nitrate test known as the brown
(a) [Ni (PPh3)2Cl2] ring test can be performed by adding iron (II) sulfate to
(b) [Rh(CO)2Cl]2 a solution of a nitrate, Then slowly adding concentrated
(c) trans-IrCl(CO)(PPh3)2 H2SO4 such that the acid forms a layer below the
(d) IrCl(CO)2(PPh3)2 aqueous solution.
AMU – 2010 KNO3+ H2SO4 → KHSO4 + HNO3
Ans. (c) : Vaska's complex is the trivial name for the 6FeSO4 + 3H2SO4+2HNO3→3Fe2(SO4)3+4H2O+
chemical compound trans-chlorocarbonyl-bid- 2NO
(triphenylphosphine) iridium (I), which has the formula FeSO4 + NO + 5H2O → [Fe(H2O)5NO]SO4
[Ir(Cl)(CO)(PPh3)2]. or [Fe(H2O)5NO)2+ SO 24 −
205. The basic structural unit in silicates is (Brown ring)
(a) SiO2 (b) [Si2O7]2–
4− 210. The complex Co ( ONO )( NH 3 )5  SO 4 is named
(c) SiO 4 tetrahedron (d) [Si2O5]2–.
AMU – 2009 as
(a) nitropentamminecobalt (III) sulphate
Ans. (c) : The basic structural unit in silicates is SiO 44− , (b) nitropentamminecobalt (II) sulphate
each silicon atom is bonded to four oxide ions (c) nitropentamminecobalt (III) sulphate
tetrahedrally. (d) pentamminenitritocobalt (III) sulphate
206. The compound formed when an excess of KCN CG PET- 2010
is added to the aqueous solution of CuSO4 is Ans. (d) : The IUPAC name of the compound is
(a) [Cu(CN)2] (b) K2[Cu(CN)4] pentamminenitritocobalt (III) sulphate.
(c) K [Cu(CN)2] (d) K3[Cu(CN)4]
211. In Fe(CO)5, the Fe–C bond possesses
AMU – 2008
(a) π-character only
Ans. (d) : KCN reacts with copper sulphate to form (b) both σ and π-characters
cupric cyanide which decomposes to give cuprous
cynaide and cyanogen gas. Cuprous cynaide dissolves (c) ionic character
in excess of KCN forming a soluble complex, (d) σ-character only
K3[Cu(CN)4]. AIEEE-2006

Objective Chemistry Volume-II 547 YCT


Ans. (b) : In metal carbonyl compound the back 216. IUPAC name of the complex K3[Al(C2O4)3] is
bonding occurred between metal and ligand. Firstly, (a) potassium tris (oxalato) aluminate(III)
ligand donate your lone pair electron in empty orbital of (b) potassium trioxalatoaluminate(III)
metal and formed ionic bond. After that, the metal (c) potassium tris(oxalato) aluminium (III)
formed the π – bond with ligand with the help of back (d) potassium trioxalatoaluminium (III)
bonding. Hence, both σ and π character is present in Fe J & K CET-2010
– C bond.
Ans. (b) : IUPAC name of K3[Al(C2O4)3] here
212. The IUPAC name of the coordination coordination sphere is negatively charged then suffix
compound K3[Fe(CN)6] is 'ate' or latin name of metal is used.
(a) tripotassium hexacyanoiron (II) Here, Aluminium is present in +3 oxidation state,
(b) potassium hexacyanoiron (II) K3[Al(C2O4)3]⇒ Potassium trioxalatoaluminate (III).
(c) potassium hexacyanoferrate (III) 217. The metal-carbon bond in metal carbonyls
(d) potassium hexacyanoferrate (II) possesses
AIEEE-2005 (a) only s character (b) only p character
Ans. (c) : The IUPAC name of the co–ordination (c) both s and p character (d) only d character
compound K3[Fe(CN)6] is potassium hexachanoferrate J & K CET-2010
(III)
Ans. (c) : Metal-carbon bond in metal carbonyls
213. Which among the following will be named as possesses both s and p character. This is due to
dibromidobis – (ethylenediamine) chromium formation of back bonding between metal to carbon
(III) bromide? atom of carbonyl group.
(a) [Cr(en)3]Br3 (b) [Cr(en)2Br2]Br
(c) [Cr(en)Br4]– (d) [Cr(en) Br2]Br
AIEEE-2012
Ans. (b) : The name of the compound is dibromidobis -
(ethylenediamine) chromium (III) bromide. The
chemical formula of the complex will be [Cr(en)2Br2]
Br.
214. A solution containing 2.675 g of CoCl3.6NH3
(molar mass = 267.5 g mol–1) is passed through
a cation exchanger. The chloride ions obtained
in solution were treated with excess of AgNO3
to give 4.78 g of AgCl (molar mass = 143.5 g
mol–1). The formula of the complex is (Atomic
218. The non-existent metal carbonyl among the
mass of Ag = 108 u)
following is
(a) [Co(NH3)6]Cl3 (b) [CoCl2(NH3)4]Cl
(a) Cr(CO)6 (b) Mn(CO)5
(c) [CoCl3(NH3)3] (d) [CoCl(NH3)5]Cl2
(c) Ni(CO)4 (d) Fe(CO)5
AIEEE-2010
J & K CET-2009
Ans. (a) : Given:- Amount of CoCl3. 6NH3 = 2.675 g
Ans. (b) : When metal carbonyls follows EAN rule then
Amount of AgCl = 4.78g complexes are exist but if it violates the EAN rule then
2.675 complexes are not exist.
Number of moles of CoCl3.6NH3= = 0.01
267.5 EAN- Atomic number of metal + 2 × Coordination No.
AgNO3 ( aq ) + Cl − ( aq ) 
→ AgCl ↓ If value of EAN is equal to electronic E–. configuration
of noble gases then complex while exist.
( white ppt ) (a) Cr(CO)6 → Stable
4.78 EAN = 24 + 2 × 6 = 36
Number of moles of AgCl = = 0.03 (b) Mn(CO)5 → Unstable or not exist
143.5
∵ 0.01 mole CoCl3.6NH3 gives = 0.03 mol AgCl EAN = 25 + 2 × 5 = 35
∵ 1 mole CoCl3.6NH3 ionizes to gives = 3 mol Cl– (c) Ni(CO)4 → Stable
Hence, the formula of compound is [Co(NH3)6]Cl3 EAN = 28 + 4 × 2 = 36
215. In which year, IUPAC draft recommends that (d) Fe(CO)5 → Stable
anionic ligands will end with-ido so that chloro EAN = 26 + 5 × 2 = 36
would become chlorido? 219. The co-ordination compound of which one of
(a) 1994 (b) 1984 the following compositions will produce two
(c) 2000 (d) 2004 equivalents of AgCl on reaction will aqueous
J & K CET-2014 silver nitrate solution?
Ans. (d) : In year 2004, IUPAC draft recommends that (a) CoCl3.3NH3 (b) CoCl3.6NH3
.
anionic will end with ido so that chloro would become (c) CoCl3 4NH3 (d) CoCl3.5NH3
chloride. J & K CET-2009
Objective Chemistry Volume-II 548 YCT
Ans. (d) : When chloride ion present outside the 224. Which does not give a precipitate with AgNo3
coordination sphere then it reacts with silvers nitrate soulution ?
solution and gives precipitate of AgCl. (a) [Co(NH3)6]Cl3 (b) [Co(NH3)5Cl]Cl2
(i) [Co(NH3)3Cl3] + AgNO3 → No AgCl formed (c) [Co(NH3)4Cl2]Cl (d) [Co(NH3)3Cl3]
(ii) [Co(NH3)6]Cl3 + AgNO3 → 3AgCl ↓ JIPMER-2008
(iii) [Co(NH3)4Cl2]Cl + AgNO3 → AgCl ↓ Ans. (d) : When chloride ion present outside the
(iv) [Co(NH3)5Cl]Cl2 + AgNO3 → 2AgCl ↓ coordination sphere then it will reacts with silvers
Hence, correct option is (d). nitrate salt and forms precipitate of AgCl.
220. Prussian blue is obtained by mixing together (i) [Co(NH3)6]Cl3 + AgNO3 → 3AgCl ↓
aqueous solution of Fe3+ salt with (ii) [Co(NH3)5Cl]Cl2 + AgNO3 → 2AgCl ↓
(a) Ferricyanide (b) Ferrocyanide (iii) [Co(NH3)4Cl2]Cl + AgNO3 → AgCl ↓
(c) Hydrogen cyanide (d) Sodium cyanide (iv) [Co(NH3)3Cl3] + AgNO3 → No ppt of AgCl
J & K CET-2007 formed.
Ans. (b) : When Fe3+ salt reacts with potassium Hence, correct option is (d).
ferrocyanide it gives Prussian blue colour solution of 225. The IUPAC name of the complex
ferric ferrocyanide. [Co(NH3)4 Cl2] Cl is
II III II (a) dichloro tetrammine cobalt (III) chloride
Fe3+ + K 4 [Fe(CN)6 ] → Fe 4 [Fe(CN)6 ]3 (b) tetraammine dichloro cobalt (III) chloride
Ferric ferrocyanide (c) tetraammine dichloro cobalt (II) chloride
(Prussian Blue)
(d) tetraammine dichloro cobalt (IV) chloride
221. The number of ions formed when Karnataka CET-2011
hexaamminecopper (II) sulphate is dissolved in
water: Ans. (b) : IUPAC name of [Co(NH3)4Cl2]Cl is
tetraammine dichloro cobalt (III) chloride.
(a) 1 (b) 2
(c) 4 (d) 6 226. IUPAC name of Na3[Co(NO2)6] is
J & K CET-2008 (a) sodium hexanitrito cobaltate (II)
(b) sodium hexanitro cobaltate (III)
Ans. (b) : Only those ions will dissociates which are
present outside the coordination sphere. (c) sodium hexanitrito cobaltate (III)
(d) sodium cobaltinitrite
In case of [Co(NH3)6]SO4 Only SO 2-4 is dissociated form
Karnataka CET-2007
coordination sphere.
Ans. (b) : The IUPAC name of Na3[Co(NO2)6] is
[Co(NH3)6]SO4 ↽ ⇀ [Co(NH3)6]2+ + SO 2-4 sodium hexanitro cobaltate (III).
2 ions formed when hexamine cobalt (II) Sulphate is 227. The IUPAC name of the complex
dissolved in water. [Co(NH3)2 (H2O)4]Cl3 is
222. The number of moles of ions given on complete (a) diaminetetraaquacobalt (III) trichloride
ionisation of one mole of [Co(NH3)3Cl3] is (b) diaminetetraaquacobalt (III) chloride
(a) 4 (b) 3 (c) diaminetetraaquacobalt (II) chloride
(c) 2 (d) 1 (d) tetraaquaiaminecobalt (III) trichloride
J & K CET-2006 (e) tetraaquaiaminecobalt (II) chloride
Ans. (d) : In [Co(NH3)3Cl3] complex only one mole of Kerala CEE-2014
ions given on complete ionisation of because in III
coordination complexes only those ions will ionised Ans. (b) : The IUPAC name of [Co(NH 3 ) 2 (H 2 O) 4 ]Cl3
which are present outside the coordination sphere. is diamminetetraaqua cobalt (III) chloride
Hence, only one ion i.e. [Co(NH3)3Cl3] is formed. 228. Copper sulphate dissolves in excess of KCN to
223. A chelating agent has two or more than two give
donor atoms to bind a single metal ion. Which (a) Cu(CN)2 (b) CuCN
of the following is not a chelating agent? (c) [Cu(CN)4]3– (d) [Cu(CN)4]2–
(a) Thiosulphato (b) Glycinato AIPMT -2006
(c) Oxalato (d) Ethane-1, 2-diamine Ans. (c) : Copper sulphate dissolves in excess of KCN
JIPMER-2013 to give [Cu(CN)4]3–
Ans. (a) : A chelating agent has two or more than two CuSO4 + KCN → Cu(CN)2
donor atoms to bind a single metal ions and formed Cu(CN)2 → Cu(CN) + (CN)2
chelating complex. unstable

 COO  CuCN + 3KCN→ K3[Cu(CN)4] or [Cu(CN)4]3–
For example- EDTA4–, Oxalato  |  , Glycinato 229. The correct IUPAC name for [CrF (en) ]Cl is
 COO −  2 2
  (a) chlorodifluoridoethylenediaminechromium

(H2N–CH2–COO ), Ethane- 1, 2 diamine (H2N–CH2– (III) chloride
CH2–NH2) (b) difluoridobis-(ethylene diamine) chromium
But thiosulphato (S2 O32− ) is an ambidentate ligand.c (III) chloride
Objective Chemistry Volume-II 549 YCT
(c) difluorobis-(ethylene diamine) chromium (III)Ans. (a) : Copper sulphate reacts with potassium
chloride ferrocyanide and forms reddish brown precipitate of
(d) chlorodifluoridobis (ethylene diamine) cuppic ferrocyanide.
chromium (III).
2CuSO4+K4[Fe(CN)6]→ Cu2 [Fe(CN)6]
Karnataka NEET 2013
(ppt)
Ans. (b) : IUPAC name of [CrF2(en)2]Cl is
difluoridobis- (ethylene diamine) chromium (III) 235. According to IUPAC system, what is the
chloride. correct name of the compound
230. An excess of AgNO3 is added to 100 mL of a [Cr(NH3)3(H2O)3]Cl3?
0.01 M solution of dichlorotetraaquachromium (a) Triamminetriaquachromium (III) chloride
(III) chloride. The number of moles of AgCl (b) Triamminetriaquachromium chloride (III)
precipitated would be (c) Tetraammoniumtriaquachromium (III)
(a) 0.003 (b) 0.01 chloride
(c) 0.001 (d) 0.002 (d) None of the above.
NEET-2013
J & K CET-2014
Ans. (c) : The complex [Cr(NH3)4Cl2]Cl when reacts
with AgNO3 will form AgCl. Ans. (a) : Correct name of the compound
[Cr(NH3)4Cl2]Cl + AgNO3 → 1AgCl [Cr(NH3)3(H2O)3] Cl3 is Triamminetriaqua chromium
0.01M 0.01M (III) chloride.
100mL 236. The correct IUPAC name of [Pt(NH3)3 Br(NO2)
No. of moles of AgCl C1 ] C1 is
=Molarity × Volume (1000 mL or 1 L) (a) Triammine chlorohromonitroplatinum (IV)
= 0.01 × 0.1 = 0.001 mole chloride
231. When copper sulphate reacts with excess of (b) Triamminebromochloronitrophlatinate (IV)
KCN, it gives chloride
(a) [Cu(CN)4]3– (b) [Cu(CN)4]2– (c) Triamminebromochloronitroplatinum (IV)
4+
(c) [Cu(CN)6] (d) CuCN chloride
UP CPMT-2014 (d) Triamminebromochloronitroplatinum (II)
Ans. (a) : chloride

CuSO 4 + 2KCN Cu(CN)2 + K 2SO 4 CG PET- 2011


Cu(CN) 2 CuCN + (CN)2 Ans. (c) : [Pt(NH3)3Br(NO2)Cl]Cl
Triammine bromochlronitro platinum chloride (IV)
232. The IUPAC name of K2 [PtCl6] is: Here oxidation state of platinum is 4 since it is neutral
(a) hexachloroplatinate potassium so no ion is added.
(b) Potassium hexachloroplatinate (IV) 237. IUPAC name of [Pt (NH3)2C12] is
(c) Potassium hexachloroplatinate (a) diamine dichloro platinum (II)
(d) Potassium hexachloroplatinum (IV)
(b) amine, chloro platinum (III)
UPTU/UPSEE-2006
(c) chloro diamine platinum (II)
Ans. (b): When coordination sphere is negatively
charged the suffix ‘ate’ or Latin name of metal is used (d) None of the above
for IUPAC naming. CG PET -2006
IUPAC name of K2[PtCl6] is potassium Ans. (a) : [Pt(NH ) Cl ]
3 2 2
hexachloroptatinate (IV).
IUPAC name of coordination compound is
233. Mohr salt is made up of which combination of
salt? Diamine dichlro platinum (II)
(a) Ammonium sulphate and potash Where II is oxidation state.
(b) Ammonium sulphate and ferrous sulphate
(c) Ammonium Sulphate and copper sulphate 3. Magnetic Nature and
(d) Amonium sulphate and magnesium sulphate.
UPTU/UPSEE-2005 Coordination Number
Ans. (b) : Mohr salt is an example of double salt
formed by combination ammonium slphate and ferrous 238. Sum of oxidation state (magnitude) and
sulphate salt. Mohr salt ⇒ FeSO4 (NH4)2 SO4.6H2O coordination number of cobalt in
234. When copper sulphate solution is added to Na Co ( bpy ) Cl 4  is ___.
potassium ferrocyanide, the formula of product
obtained is
Given bpy =
(a) Cu2Fe(CN)6 (b) Cu(CN)3
(c) CuFe(CN)6 (d) Cu(CN)2
UPTU/UPSEE-2010 JEE Main 29.07.2022, Shift-II

Objective Chemistry Volume-II 550 YCT


Ans. (9) : Coordination no. = 6 242. The pair in which both the species have same
Oxidation state =3 magnetic moment (spin only) is
Sum of oxidation state (magnitude) and coordination (a) [ COCl4 ]2–, [ Fe(H2O6 ]2–
number of cobalt in Na Co ( bpy ) Cl4  is nine (9). (b) [ Mn(H2O)6 ]2+, [ Cr(H2O)6 ]2+
(c) [ Cr(H2O6) ]2+, [ Fe(H2O6)Cl4 ]2–
239. Consider the following metal complexes: (d) [ Cr (H2O6)]2+, [COCl4]2–
[Co(NH3)]3+ TS EAMCET-20.07.2022, Shift-I
[CoCl(NH3)5]2+
[Co(CN)6]3− Ans. (c) : µ = n(n + 2); n = no. of unpaired electron
[Co(NH3)5(H2O)]3+ (a) Cr(2 = 24) ; 3d54s1 : Cr2+ ; 3d44s0
The spin-only magnetic moment value of the
complex that absorbs light with shortest
wavelength is B.M. (Nearest integer) Co(2 = 27) ; 3d74s2 : CO+2 + 3d74s0
JEE Main 25.07.2022, Shift-I
Ans. (0) : [Co(CN)6]3− absorbs light with shortest (b) Cr(2 = 24) ; 3d54s2 : Cr2+ ; 3d44s0 n = 4
wavelength because CN−1 is strong field ligand so, more Fe (2 = 26) 3d64s2 ; Fe2+. 3d64s0
spiliting takes place and t2g and eg orbital have more
energy difference. It has no unpaired electrons and will 5 2 2+ 5 0
be in low spin configuration. Hence, it’s magnetic (c) Mn (2 = 25) ; 3d 4s ; Mn ; 3d 4s
moment value is 0. Cr ; n = 4
240. Among Co3+, Ti2+, V2+ and Cr2+ions, one of used ,
as a reagent cannot liberate H2 from dilute (d) n = 4, n = 3
mineral acid solution, its spin-only magnetic 243. The homoleptic complex in the following is
moment in gaseous state is B.M. (Nearest ⊕ ⊕
integer) (a) Co ( NH 3 )4 Br2  (b) Co( C2O4 ) ( NH3 )4 
JEE Main 25.07.2022, Shift-I 3⊕ ⊕
Ans. (5) : Among given ions. Co3+ has more value of (c) Co ( NH 3 )6  (d) Co ( CN )4 Cl2 
standard reduction potential. so, it cannot liberate H2
from dilute acid solution. AP EAMCET-05.07.2022, Shift-I
3+
Magnetic moment of Co is− Ans. (c) : Homoleptic complexes are compound in
no of unpaired electrons = 4 which all the ligands bound to the metal center are
identical. Homoleptic complexes have only one type of
Magnetic moment = n(n + 2) ligands.
= 4(4 + 2) So, option (c) is correct answer.
= 24 244. Regarding magnetic properties of the
= 4.92 ≃ 5 B.M complexes Ni(CO)4 [I], NiCl -24 [ II ]
241. The difference between spin only magnetic Which of the following are correct?
moment values of [Co(H2O)6]Cl2 and (a) I= Diamagnetic II= Paramagnetic
[Cr(H2O)6]Cl3 is ____ (b) I= Paramagnetic II= Paramagnetic
JEE Main 26.07.2022, Shift-I (c) I= Diamagnetic II= Diamagnetic
Ans. (0) : [Co(H2O)6]2+ (d) I= Paramagnetic II= Diamagnetic
AP EAPCET-11.07.2022, Shift-II
Ans. (a) : [Ni(CO)4] (1)
CO is a strong field ligand. Therefore, it causes the
µ = n(n + 2) pairing of unpaired 3d electrons. It cause 4s electrons to
Where n = number of unpaired electrons. shift to the 3d orbital since, no unpaired electrons are
Number of unpaired e– = 3 present in the case of [Ni(CO)4] is diamagnetic.
µ = 3(3 + 2) [(NiCl42-)]
Ni in +2 oxidation state with the configuration 3d8 4s0
µ = 15 BM Cl– ion being weak ligand it cannot pair up the electrons
in 3d orbital. Hence, it is Para magnetic.
245. The correct order of magnetic moments (spin
only values in B.M.) anong is
µ = n (n + 2) (a) [Fe(CN)6]4– > [MnCl4]2– > [CoCl4]2–

No. of unpaired e = 3 (b) [MnCl4]2– > [Fe(CN)6]4– > [CoCl4]2–
µ = 3(3 + 2) (c) [MnCl4]2– > [CoCl4]2– > [Fe(CN)6]4–
(d) [Fe(CN)6]4–> [CoCl4]2– > [MnCl4]2–
µ = 15 BM (Atomic nos. : Mn=25, Fe=26, Co=27)
Difference in Spin only magnetic moment = 0 BITSAT-2014, AIEEE 2004
Objective Chemistry Volume-II 551 YCT
Ans. (c) : [Fe(CN)6]4– 248. Which of the following ion has the maximum
Fe2+ : d6 theoretical magnetic moment?
(a) Fe3+ (b) Cr3+
CN– is strong field ligand and cause pairing of d 3+
(c) Ti (d) Co3+
electron.
Number of unpaired electron = 0 GUJCET-2016, 2015
[MnCl4]2–, Cl– is weak field ligand and causes no (a)
pairing of d– electron. Ans. (a) :
Mn2+ : d5 Number of unpaired electron = 5
No. of unpaired electron = 5 Magnetic moment = 5(5 + 2) = 35 = 5.9B.M.
[CoCl4]2–, Cl– is weak field ligand and does not do
pairing of d-electron. (b)
Co2+ : d7 Number of unpaired electron = 3
Number of unpaired electron = 3 Magnetic moment = 3(3 + 2) = 15 = 3.8B.M.
Magnetic moment ∝ number of unpaired electron
Order:- [MnCl4]2– > [CoCl4]2– > [Fe(CN)6]4– (c)
246. The pair in which both species have same Number of unpaired electron = 1
magnetic moment (spin only value) is:
Magnetic moment = 1(1 + 2) = 3 = 1.73B.M.
(a) [Cr(H2O)6]2+, [CoCl4]2–
(b) [Cr(H2O)6]2+, [Fe(H2O)6]2+ (d)
(c) [Mn(H2O)6]2+, [Cr(H2O)6]2+
(d) [CoCl4]2–, [Fe (H2O)6]2+ Number of unpaired electron = 4
JEE Main 2016 Magnetic moment = 4(4 + 2) = 24 = 4.9B.M.
BITSAT-2013 3+
Hence, Fe has the maximum theoretical magnetic
Ans. (b) : H2O act as weak field ligand and pairing not moment.
occurs. 249. Which of the following substance possess
[Cr(H2O)6]2+ ⇒ Cr2+ antiferromagnetic property?
Cr2+ : d4 (a) Fe3O4 (b) CrO2
Number of unpaired electron = 4 (c) H2O (d) MnO
[Fe(H2O)6]2+ ⇒ Fe2+:d6 GUJCET-2016, 2015
No. of unpaired electron = 4 Ans. (d) : Anti-ferromagnetic substance having anti-
Both [Fe(H2O)6]2+ and [Cr(H2O)6]2+ having same parallel or opposite direction of dipole moment of
number of unpaired electrons and that’s why there spin domains due to this net magnetic moment of substance
only magnetic moment is also same. is zero, MnO is an example of anti-ferromagnetic
substance.
247. Which one of the following cyano complexes
would exhibit the lowest value of paramagnetic 250. The coordination number and the oxidation
behavior? state of the element 'E' in the complex
(a) [CoCN)6]3– (b) [Fe(CN)6]3– [E(en)2(C2O4)]NO2 (Where (en) is ethylene
3– diamine) are, respectively
(c) [Mn(CN)6] (d) [Cr(CN)6]3–
(a) 6 and 2 (b) 4 and 2
(At. NOs. Cr=24, Mn=25, Fe=26, Co= 27)
(c) 4 and 3 (d) 6 and 3
BITSAT-2018
AIEEE 2005 Karnataka CET-2017
AIEEE 2008
Ans. (a) : act as strong field ligand and cause
pairing of metal d-electron. Ans. (d) : For complex [E(en)2(C2O4)]NO2 coordination
number = No. of ligands attached with central metal.
[Co(CN)6]3– ⇒ Co3+ : d6
C. No. = 6 Let oxidation state of metal = x
Number of unpaired electron = 0 x + 2 × 0 + 1 × (–2) = + 1
[Fe(CN)6]3– ⇒ Fe3+ : d5 x=+3
Number of unpaired electron = 1 251. The d electron configurations of Cr2+, Mn2+,
[Mn(CN)6]3– ⇒ Mn3+ : d4 Fe2+ and Ni2+ are 3d4, 3d5, 3d6 and 3d8
Number of unpaired electron = 2 respectively. Which one of the following aqua
complexes will exhibit the minimum
[Cr(CN)6]3– ⇒ Cr3+ : d3 paramagnetic behaviour?
Number of unpaired electron = 3 (a) [Fe(H2O)6]2+ (b) [Ni(H2O)6]2+
Hence, [Co(CN)6]3– exhibit the lowest value of (c) [Cr(H2O)6] 2+
(d) [Mn(H2O)6]2+
paramagnetic behaviour. AIPMT -2011, 2007
Objective Chemistry Volume-II 552 YCT
Ans. (b) : 2+
2+
(d) Cr ( H 2 O )6 
(a)  Fe ( H 2 O )6 
Cr 2+ = [ Ar ] 3d 4
Fe 2+ = [ Ar ] 3d 6
Unpaired electrons (n) = 4
253. A magnetic moment at 1.73 BM will be shown
Unpaired electrons (n) = 4
2+
by one among of the following
(b)  Ni ( H 2O )6  (a) TiCl4 (b) [CoCl6]4–
2+
(c) [Cu(NH3)4] (d) [Ni(CN)4]2–
Ni 2+ = [ Ar ] 3d8
NEET 2013
UPTU/UPSEE-2013
Ans. (c) : (a) TiCl4
Unpaired electrons (n) = 2
2+
x + 4(−1) = 0
(c) Cr ( H 2 O )6  x = +4
Ti4+ = [Ar] 3d0
Cr 2+ = [ Ar ] 3d 4
It has not unpaired electron so it will not show the
magnetic moment.
Unpaired electrons (n) = 4 (b) [CoCl6]4−
(d)  Mn ( H 2O )6 
2+
x + 6(−1) = −4
x = +2
Mn 2+ = [ Ar ] 3d5 Co2+ = [Ar] 3d7

Unpaired electrons (n) = 5


Since, [Ni(H2O)6]2+ has minimum number of unpaired Unpaired electron (n) = 3
2+
electrons. Therefore,  Ni ( H 2O )6  has the minimum ∴ µs = n ( n + 2 )
paramagnetic behaviour. µs = 3 ( 3 + 2 )
252. The d-electron configurations of Cr2+, Mn2+,
Fe2+ and Co2+ are d4,d5, d6 and d7 respectively. µs = 15 = 3.87 BM
Which one of the following will exhibit (c) [Cu(NH3)4]2+
minimum paramagnetic behaviour? x + 4(0) = +2
(a) [Mn(H2O)6]2+ (b) [Fe(H2O)6]2+ x = +2
2+
(c) [Co(H2O)6] (d) [Cr(H2O)6]2+ NH3 act as strong field ligand with Cu2+. Thus pairing
(At. nos. Cr = 24, Mn = 25, Fe = 26, Co = 27) of electron takes place.
Karnataka NEET 2013 Cu2+ = [Ar] 3d9
AIPMT -2011
Ans. (c) : The complex ion which has no unpaired
electrons, are diamagnetic in nature.
Unpaired electron (n) = 1
(a) [Mn(H2O)6]2+
2+ ∴ µs = 1(1 + 2 )
 Mn ( H 2O )6 
µs = 1.732 BM
Mn 2+ = [ Ar ] 3d5
(d) [Ni(CN)4]2−
x + 4(−1) = −2
Unpaired electrons (n) = 5 x = +2
2+ pairing occurred because CN is strong field ligand.
(b)  Fe ( H 2 O )6 
Ni2+ = [Ar] 3d8
Fe 2+ = [ Ar ] 3d 6

Unpaired electrons (n) = 4


(c) [Co(H2O)6]2+ Unpaired electron (n) = 0
Co2+ = [Ar]3d7 ∴ µs = 0
Ground state– Only [Cu(NH3)4]2+ shows the 1.73 BM magnetic
Unpaired electrons (n) = 3 moment.

Objective Chemistry Volume-II 553 YCT


254. Which ion among the following is diamagnetic? Ans. (a) : (A) [Fe(CN)6]3−
[NiCl4]2–, [Ni(CN)4]2–, [CuCl4]2–, [CoF6]3– x + 6(−1) = −3
(a) [NiCl4]2– (b) [Ni(CN)4]2– x = +3
(c) [CuCl4]2– (d) [CoF6]3–
∵ pairing occurred because CN is a strong field ligand.
Tripura JEE-2021
Ans. (b) : The compounds which have no unpaired Fe3+ = [Ar] 3d5
electron are diamagnetic in nature.
(a) [Ni(Cl)4]2−
x +4(−1) = −2
x = +2
Ni2+ = [Ar] 3d8 Unpaired electron (n) = 1
µs = n ( n + 2 )
Unpaired electron (n) = 2, paramagnetic µs = 3
(b) [Ni(CN)4]2− µs = 1.732 BM
x +4(−1) = −2 (B) [Fe(H2O)6]3+
x = +2 x + 6(0) = +3
CN is a strong field ligand and it paired the electron in x= +3
inner orbital. Fe3+ = [Ar] 3d5
Ni2+ = [Ar] 3d8

Unpaired electron (n) = 5


µs = 5 ( 5 + 2 )
Unpaired electron (n) = 0, Diamagnetic µs = 5.92 BM
(c) [CuCl4]2− (C) [Fe(CN)6]4–
x +4(−1) = −2 x + 6(−1) = −4
x = +2 x = +2
Cu2+ = [Ar] 3d9 pairing of electron will take place because CN is strong
field ligand.
Fe2+ = [Ar] 3d6
Unpaired electron (n) = 1, paramagnetic
(d) [CoF6]3−
x +6(−1) = −3
x = +3
Co3+ = [Ar] 3d6 Unpaired electron (n) = 0
µs = 0 ( 0 + 2 )
Unpaired electron (n) = 4, paramagnetic µs = 0 BM
Thus, [Ni(CN)4]2− is diamagnetic because it has zero (D) [Fe(H2O)6]2+
unpaired electron. x + 6(0) = +2
255. Match List-I with List-II x= +2
List I List II Fe2+ = [Ar] 3d6
(A) [Fe(CN)6]3– (i) 5.92 BM
(B) [Fe(H2O)6]3+ (ii) 0 BM
(C) [Fe(CN)6]4– (iii) 4.90 Unpaired electron (n) = 4
(D) [Fe(H2O)6]2+ (iv) 1.73 BM µs = 4 ( 4 + 2 )
Choose the correct answer from the options µs = 4.9 BM
given below:
(a) (A)-(iv), (B)-(i),(C)-(ii), (D)-(iii) Hence, the correct match is –
(b) (A)-(iv), (B)-(ii),(C)-(i), (D)-(iii) A. [Fe(CN)6]3− − 1.732 BM
(c) (A)-(ii), (B)-(iv),(C)-(iii), (D)-(i) B. [Fe(H2O)6]3+ − 5.92 BM
(d) (A)-(i), (B)-(iii),(C)-(iv), (D)-(ii) C. [Fe(CN)6]4− − 0 BM
NEET 2021 D. [Fe(H2O)6]2+ − 4.9 BM

Objective Chemistry Volume-II 554 YCT


256. Which of the following does not represent
property stated against it ?
(a) Co2+ < Fe2+ < Mn2+ – Ionic size
(b) Ti < V < Mn – Number of oxidation states Hybridisation ⇒ d2 Sp2
(c) Cr2+ < Mn2+ < Fe2+ – Paramagnetic behavior [Fe(CN)6]3– ⇒ Fe3+ : d5
(d) Sc > Cr > Fe – Density
Kerala CEE-29.08.2021
Ans. (c & d) :
(A) In the 3d-series of transition metal ions, the size of Hybridisation ⇒ d2 sp3
Mn2+ decreases left to right. Hence, order is correct i.e.
Co2+ < Fe2+ < Mn2+. [Co(C2O4)3]3– ⇒ Co3+ : d6
(B) As we know that Mn shows the highest oxidation
state i.e. 7 and the stable oxidation state of
Vanadium and Titanium are +5, +4 respectively.
Hence, the given order is correct i.e. Ti < V < Mn- Hybridisation ⇒ d2 sp3
number of oxidation state. Cl– and F– act as weak field ligand so, pairing of
(C) The given ions which have more number of electrons not possible [MnCl6]3– ⇒ Fe3+: d4
unpaired electron having the more paramagnetic
behaviour. d4
Ions No. of unpaired electron No. of unpaired electron = 4
Cr2+ 4
Mn2+ 5 [FeF6]3– ⇒ Fe3+: d5
Fe2+ 4 No. of unpaired electron = 5
Hence, the given order is wrong. Hence, both statement I and II are true.
(D) Density is directly proportional to the mass of 259. Spin only magnetic moment in BM of [Fe(CO)4
metal. By this reason, the given order is wrong. (C2O4)]+ is
Hence, the correct order of density is Fe > Cr > Sc. (a) 5.92 (b) 0
257. Which one of the following species responds to (c) 1 (d) 1.73
an external magnetic field JEE Main 2021, 31 Aug Shift-II
(a) [Fe(H2O)6]3+ (b) [Ni(CN)4]2– Ans. (d) : Co and C 2 O 24 − is a strong field ligand, ∆o > P
3–
(c) [Co(CN)6] (d) [Ni(Co)4] So pairing of electrons occurs. [Fe (CO) (C2O4)]+ ⇒ Fe+
JEE Main 2021, 25 July Shift-I : 3d6 4s1
Ans. (A) : If complex is paramagmetive in nature then
it respond to external magnetic field in[Fe(H2O)6]3+,
H2O act as weak field ligand ∆0 < P then pairing of
electrons is not possible. No. of unpaired electron = 1
Magnetic moment = 1(1 + 2) = 3 = 1.73 B.M.
260. The spin only magnetic moment of a divalent
No. of unpaired electron = 5 ion in aqueous solution (atomic number = 29) is
Hence, [Fe(H2O)6]3+ complex respond to external .................BM.
magnetic field. JEE Main 2021, 25 Feb Shift-II
258. Given below are two statements. Ans. (1.73) : Let metal be M.
Statement I [Mn(CN)6]3-, [Fe(CN6]3- and M = 29
[Co(C2O4)3]3- are d2 sp3 hybridised. Electronic configuration: 1s2 2s2 2p6 3s2 3p6 3d10 4s1
Statement II [MnCl6]3- and [FeF6]3- are
paramagnetic and have 4 and 5 unpaired M2+⇒ 3d9
electrons, respectively. No. of unpaired electron = 1
In the light of the above statements, choose the Magnetic moment = 1(1 + 2) = 1.73B.M.
correct answer from the options given below
(a) Statement I is true but statement II is false 261. The calculated magnetic moments (spin only
(b) Both Statement I and statement II are false value) for species
(c) Statement I is false but statement II is true [FeCl4]2–, [Co(C2O4)3]3– and MnO42–
respectively are
(d) Both Statement I and statement II are true
(a) 5.82, 0 and 0 BM
JEE Main 2021, 27 July Shift-II (b) 4.90, 0 and 1.73 BM
Ans. (D) : CN– and C 2 O 24 − act as strong field ligand (c) 5.92, 4.90 and 0 BM
and do paring of electrons. (d) 4.80, 0 and 2.83 BM
[Mn(CN)6)]3– ⇒ Mn3+ : d4 JEE Main 2021, 24 Feb Shift-II
Objective Chemistry Volume-II 555 YCT
Ans. (b) : In [FeCl4]2– , ∆t < P then pairing of electrons Ans. (a) : Let metal B.M.
not possible M = 25
Fe2+ : d6 Electronic configuration of M ⇒ 1s2, 2s2 2p6, 3s2 3p6
3d5 4s2 M2+: 3d5
No, of unpaired electron = 4
d5
Magnetic moment = 4(4 + 2) = 24
No. of unpaired electron =5
= 4.90 B.M.
Magnetic moment = 5(5 + 2) = 35 = 5.92 B.M.
[Co(C2O4)3] , ∆o > P then pairing of d–electrons occurs
3–

264. Arrange the following metal complex


CO3+ : d6 /compounds in the increasing order of spin only
No of unpaired electron = 0 magnetic moment. Presume all the three, high
Magnetic moment = 0 B.M. spin system.
(Atomic numbers Ce=58, Gd=64 and Eu = 63.)
MnO 4 ⇒ Mn +6 ,d1
2−
A. (NH4)2[Ce(NO3)6]
d1 B. Gd(NO3)3 and
No. of unpaired electron = 1 C. Eu(NO3)3 and
(a) (B) < (A) < (C) (b) (C) < (A) < (B)
Magnetic moment = 3 = 1.73 B.M. (c) (A) < (B) < (C) (d) (A) < (C) < (B)
JEE Main 2021, 16 March Shift-II
262. In which of the following order the given Ans. (d) : High spin complex, ∆o, < P
complex ions are arranged correctly with (a) (NH4)2 [Ce(NO ) ] ⇒ Ce+4 : fo
respect to their decreasing spin only magnetic Magnetic moment 3=6 0 B.M.
moment?
(b). [Gd(NO3)3] ⇒ Gd3+ f7
i. [FeF6]3– ii. [Co(NH3)6]3+
iii. [NiCl4]2– iv. [Cu(NH3)4]2+ f7
(a) (i) > (iii) > (iv) > (ii) No. of unpaired electron = 7
(b) (ii) > (iii) > (i) > (iv) Magnetic moment = 7.9 B.M.
(c) (iii) > (iv) > (ii) > (i) (c) [Eu(NO3)3]⇒ Eu3+ : f6
(d) (ii) > (i) > (iii) > (iv) No. of unpaired electron = 6
JEE Main 2021, 25 Feb Shift-II Magnetic moment = 6.9 B.M.
Ans. (a) : (i) [FeF6]3–, ∆o < P then pairing of electrons Increasing order of spin only magnetic moment :–
not possible (A) < (C) < (B)
3+ 5 265. The total number of unpaired electrons present
Fe : d
in the complex K3[Cr(oxalate)3] is ..........
No. of unpaired electron = 5 JEE Main 2021, 18 March Shift-I
Magnate moment = 5(5 + 2) = 5.9 B.M. Ans. (3) : K3 [Cr (ox)3] or [Cr (ox)3]3–
(ii) [Co(NH3)6]3+, for Co3+ ∆o > P
Co3+ ⇒ d6
No. of unpaired electron = 3
266. The spin only magnetic moment value for the
Magnetic moment = 0 B.M. complex [Co(CN)6]4– is ..... BM.
2– –
(iii) [NiCl4] , Cl act as weak field ligand so no pairing [Atomic number of Co = 27]
of electrons is possible. JEE Main 2021, 20 July Shift-I
4–
2+
Ni ; d 8 Ans. (2) : [Co(CN)6] ,
CN– act as strong field ligand, ∆0 > P then pairing is
Magnetic moment = 2(2 + 2) = 8 = 2.84B.M. possible.
(iv) [Cu(NH3)4]2+
In Cu2+ complexes always 1 unpaired electron is
present.
Magnetic moment = 1(1 + 2) = 1.73 B.M.
Decreasing order of spin only magnetic moment:– No. of unpaired electron = 1
(i) > (iii) > (iv) > (ii) Spin only magnetic moment = 1.73 B.M.
263. What is the spin-only magnetic moment value 267. If atomic number of element is 26, then
(BM) of a divalent metal ion with atomic magnetic moment is –––––– BM of its divalent
number 25, in it's aqueous solution? aqueous ion?
(a) 5.92 (b) 5.0 (a) 1.73 (b) 3.87
(c) zero (d) 5.26 (c) 2.83 (d) 4.90
JEE Main 2021, 17 March Shift-II GUJCET-2021
Objective Chemistry Volume-II 556 YCT
Ans. (d) : Given, atomic number of element = 26 Ans. (a) : (i) [FeF6]3–
and its divalent aqueous ion = Fe2+ Oxidation no. of Fe in [FeF6]3– = x+6×(–1)= –3
= x – 6 = –3
Fe2+ ⇒ [Ar]3d6 = [unpaired e–(n)=4]
= x = +3
Magnetic moment (µs) = n ( n + 2 )

µs = 4 ( 4 + 2 )
µs = 4.90 BM (ii) [Co(NH3)6]3+
The oxidation no. of Co is,
268. According to Valence Bond Theory, the x + 6 × 0 = +3
number of unpaired electrons present in
[MnCl6]3–, [Fe(CN)6]3–, and [CO(C2O4)3]3–, x = +3
respectively, are Co3+ : [Ar]3d6
(a) 0; 5; 0 (b) 4; 3; 2
(c) 4; 1; 0 (d) 5; 4; 3
TS EAMCET 05.08.2021, Shift-I
Ans. (c) : (i) [MnCl6]3–
Mn+3 = 3d4 (iii) [NiCl4]2–
Cl is weak field ligand that possible pairing. This Oxidation no. of Ni is,
complex has 4 water e–. ⇒ x+4×(–1) = –2
(ii) [Fe(CN)6]3– ⇒ x–4 = –2
x = +2
Fe+3 = 3d5
CN is strong field ligand, possible pairing. This
complex has 1 unpaired e–.
(iii) [Co(C2O4)3]3–
Co+3 = 3d6 4so
C2O4 behave as a strong field ligand. (iv) [Cu(NH3)4]2+
This complex has 0 unpaired e–. The oxidation no. of Cu is,
Hence, the no. of unpaired electron are 4, 1, 0 x + 4 × 0 = +2
respectively.
x = +2
269. The magnetic moment of the high spin complex
is 5.92 BM. What is the electronic
configuration?
(a) t 2g3 eg1 (b) t 2g4 eg 2
(c) t 2g3 eg 2 (d) t 2g5 eg 0
TS EAMCET 05.08.2021, Shift-I
Ans. (c): The spin only magnetic moment is directly proportional
to the number of unpaired electron. So, order will be (i)
> (iii) > (iv) > (ii)
271. Identify the correct sequence representing the
order of decreasing magnetic moments of the
given cations.
(a) Mn2+ > Fe2+>Co2+>Ni2+
t2g has 3 e–, and eg has 2 e–. Then, the magnetic (b) Ni2+ > Co2+>Fe2+>Mn2+
moment is the high spin complex is 5.92BM. (c) Fe2+ > Co2+>Ni2+>Mn2+
270. In which of the following order the given (d) Co2+ > Ni2+>Mn2+>Fe2+
complex ions are arranged correctly with AP EAPCET-6 Sep. 2021, Shift-II
respect to their decreasing spin only magnetic
Ans. (a): Magnetic moment depends upon the no. of
moment ?
unpaired electron present in the ion.
(i) [FeF6]3– (ii) [Co(NH3)6]3+ (iii) [NiCl4]2– (iv)
[Cu(NH3)4]2+
(a) (i) > (iii) > (iv) > (ii)
(b) (ii) > (iii) > (i) > (iv)
(c) (iii) > (iv) > (ii) > (i)
(d) (ii) > (i) > (iii) > (iv) 2+ 2+ 2+ 2+
JEE Main 25.02.2021, Shift-II So, order will be – Mn > Fe >Co >Ni
Objective Chemistry Volume-II 557 YCT
272. In the ground state of atomic Fe(Z=26), the 275. Which of the following complexes formed by
spin-only magnetic moment is _______×10–1 Nickel is tetrahedral and paramagnetic?
BM. (Round off to the Nearest Integer). (a) [Ni(CN)4]2– (b) [Ni(CO)4]
2–
[Given: 3 = 1.73, 2 = 1.41 ] (c) [Ni(C l) 4 ] (d) [Ni(NH3)6]2+
JEE Main 17.03.2021, Shift-II AP EAPCET 20.08.2021 Shift-I
Ans. (49 × 10–1 BM) : Ans. (c) :
Fe = 1s2 2s2 2p6 3s2 3p6 3d6 4s2 2−
(a)  Ni ( CN )4 
Ni +2 = 3d8
2 unpaired electrons are present.
In 3d6, So, Nature = Paramagenetic and Shape = Square planar
Number of unpaired electron (n) = 4 (b)  Ni ( CO )4 
∴ Spin only magnetic moment ( µs ) = n(n + 2) Ni = 3d8 4s 2
µs = 4(4 + 2) Shape = tetrahedral
As CO is strong ligand, it causes pairing of electrons,
µs = 24 therefore nature is diamagnetic.
µs = 4.89 BM 2−
(c)  Ni ( Cl4 )4 
µs = 48.9 × 10 BM
–1
Ni 2− = 3d8
or µs ≈ 49 × 10 BM
–1
Shape = tetrahedral

273. Arrange the following metal As Cl is weak ligand, no pairing occurs therefore,
complex/compounds in the increasing order of nature is paramagnetic.
spin only magnetic moment. Presume all the (d)  Ni ( NH )  2+
three, high spin system.  3 6

(Atomic number Ce = 58, Gd = 64 and Eu = Shape = octahedral and nature = paramagnetic.


63.) 276. What is co-ordination of the metal in [Co (en)2
(A) (NH4)2[Ce(NO3)6] (B) Gd(NO3)3 and Cl2]?
(C) Eu(NO3)3 answer is: (a) 3 (b) 4
(a) (A) < (B) < (C) (b) (A) < (C) < (B) (c) 5 (d) 6
(c) (B) < (A) < (C) (d) (C) < (A) < (B) AP EAPCET 19-08-2021 Shift-I
JEE Main 16.03.2021, Shift-II Ans. (d) : Given:- [Co(en)2Cl2]
Ans. (b) : (A) (NH4)2[Ce(NO3)6] Oxidation state = +4 In this compound ethylenediamine (en) ligand act as a
∴ 58Ce = 4f25d06s0 bidentet ligand so two donor site present in the ligand
∴ Ce4+ = [Xe]4f05d06s0 Hence total donor site in compound is 6 so the co-
There is no any unpaired electron so µS = 0 ordination number will be 6.
(B) Gd(NO3)3 oxidation state = +3 277. Which among the following is most
∴ 64Gd3+ = [Xe]4f75d06s0 paramagnetic ?
There are seven unpaired electron, So (a) [Cr(H2O)6]3+ (b) [Fe(H2O)6]2+
2+
(c) [Cu(H2O)6] (d) [Zn(H2O)2]2+
µS = 7 ( 7 + 2 ) = 63 B.M.
AP EAPCET 24.08.2021, Shift-I
(C) Eu(NO3)3 oxidation state = +3 Ans. (b) :
∴ 63Eu3+ = [Xe]4f65d06s0 (Six unpaired electron
Compound Electronic Unpaired
present)
Configuration Electron
µS = 6 ( 6 + 2 ) = 48 BM (n)
3+ 5 1
Hence, the order of spin only magnetic moment is [Cr(H 2 O) 6 ] [Ar], 3d 4s 3
A<C<B. = [Ar] 3d3
2+
274. The geometries of [Ni (CO)4], [PtCl4]2- and [Co [Fe(H2O)6] [Ar]3d6 4s2 4
3+
(NH3)6] respectively are : = [Ar] 3d6
(a) Tetrahedral. Tetrahedral and Octahedral [Cu(H2O)6]2+ [Ar] 3d10 4s1 1
(b) Tetrahedral. Square planar and Square = [Ar] 3d3
pyramidal [Zn(H2O)6]2+ [Ar] 3d10 4s2 0
(c) Square planar. Square planar and Octahedral = [Ar] 3d10
(d) Tetrahedral, Square planar and Octahedral 278. In the following set of compounds, the pair
AP EAPCET 24.08.2021 Shift-II with same magnetic moment is
Ans. (d): [Ni (CO)4] is tetrahedral (a) FeCl2 and MnCl2 (b) CoCl2 and MnCl2
[PtCl4]2- is square planar (c) VOCl 2 and CuCl 2 (d) ZnCl2 and MnCl2
[Co(NH)6]3+ is Octahedral TS EAMCET 10.08.2021, Shift-II
Objective Chemistry Volume-II 558 YCT
Ans. (c) : 282. The species that has a spin-only magnetic
moment of 5.9 BM, is (Td = tetrahedral)
Magneticmomeut(µ) = n ( n + 2 ) (n = unpaired electron) (a) [Ni(CN)4]2– (Square Planar)
+4 (b) [NiCl4]2– (Td)
VOCl2 = 23V = 1s 2 2s 2 2p 6 3s 2 3p 6 3d1
(c) [Ni(CO)4] (Td)
(d) [MnBr4]2– (Td)
n = 1e– JEE Main 2020, 6 Sep Shift-I
Ans. (d) : If spin-only magnetic moment of complex is
3d 5.9 B.M then complex contains 5 unpaired electrons in
its orbital.
µ s = 1 (1 + 2 ) = 3 = 1.73BM In [MnBr4]2– complex, Br– act as week field ligand (∆t <
P), then pairing of electrons is not possible,
+2 2 2 6 2 6 9
CuCl 2 = Cu = 1s 2s 2p 3s 3p 3d Mn2+: d5
n = 1e–
No. of unpaired electron = 5
µ = 1 (1 + 2 ) = 3 =1.73 BM Magnetic moment = 5(5 + 2) = 35 = 5.9 B.M.
283. The pair in which both the species have the
279. The magnetic moment of Fe2+ is ____ B.M. same magnetic moment (spin only) is
(a) 3.87 (b) 0 (a) [Mn(H2O)6]2+ and [Cr(H2O)]2+
(c) 4.9 (d) 1.73 (b) [Cr(H2O)6]2+ and [Fe(H2O)6]2+
AP EAPCET 19-08-2021, Shift-II (c) [CO(OH)4]2– and [Fe(NH3)6]2+
Ans. (c) : Electronic configuration of Fe2+ = (d) [Cr(H2O)6]2+ and [CoCl4]2–
JEE Main 2020, 4 sep Shift-I
= 18 [ Ar ] ,3d 6 , 4sº
Ans. (b) : When compounds contains equal number of
= unpaired electron then its spin only magnetic moment is
no. of unpaired electron (n) = 4 also same.
H2O act as weak field ligand ∆o < P then pairing of
∴ Formula of magnetic moment = n ( n + 2) electrons not possible [Cr(H2O)6]2+ ⇒ Cr2+: d4
= 4(4 + 2) d4
No. of unpaired electron = 4
= 24 Magnetic moment = 24 = 4.9 B.M.
= 4.9
d6
280. The coordination number of Fe and Co in the No. of unpaired electron = 4
complex ions, [Fe(C2O4)3]3– and [Co(SCN)4]2–
are respectively Magnetic moment = 24 = 4.9 B.M.
(a) 3 and 4 (b) 6 and 8 Hence, [Cr(H2O)6]2+ and [Fe(H2O)6]2+ having same spin
only magnetic moment.
(c) 4 and 6 (d) 6 and 4
284. The one that can exhibit highest paramagnetic
Karnataka CET-2020 behaviour among the following is
Ans. (d) : In [Fe(C2O4)3]3−, C2O42− is a bidentate ligand gly = glycinato;
then coordination number of complex is 6. bpy = 2, 2'-bipyridine
In [Co(SCN)4]2−, SCN− is monodentate ligand then (a) [Pd(gly)2]
coordination number of complex is 4. (b) [Fe(en)(bpy)(NH3)2]2+
281. Identify the set of paramagnetic ions among the (c) [Co(OX)2 (OH2)]– (∆0 > P)
following. (d) [Ti(NH3)6]3+
(a) V2+, Co2+, Ti4+ (b) Ni2+, Cu2+, Zn2+ JEE Main 2020, 4 Sep Shift-II
3+ 2+
(c) Ti , Cu , Mn 3+
(d) Sc3+, Ti3+, V3+ Ans. (c) : [Co(OX)2 (OH2)]– (∆0 > P)
Karnataka CET-2020 Co= 3d74s2
Ans. (c) : Paramagnetic ions having unpaired electrons.
Ti3+ : d2
No. of unpaired electron = 1 It has highest number of unpaired e–, so it is most
paramagnetic.
No. of unpaired electron = 1 285. Coordination number of caesium in caesium
chloride is
(a) 8 (b) 6
No. of unpaired electron = 4 (c) 4 (d) 12
Hence, Ti2+, Cu2+ and Mn3+ act paramagnetic ions. COMEDK-2020
Objective Chemistry Volume-II 559 YCT
Ans. (a) : Caesium chloride crystal, caesium ion Number of unpaired electron = 0
occupies the center and chloride ions, occupy each Magnetic moment = 0 B.M.
corner of the cube. The coordination number of caesium (iii) Na 3 [Fe(C 2 O 4 )3 ( ∆0 > P)
chloride is 8.
286. The d-electron configurations of Cr2+, Mn2+, Then pairing of electron occurs.
Fe2+ and Co2+ are d4, d5, d6 and d7 respectively.
Which one of the following will exhibit the
lowest paramagnetic behavior? Number of unpaired electron = 1
(Atomic no. Cr=24, Mn=25, Fe=26, Co=27). Magnetic moment = 1(1 + 2) = 3 = 1.73B.M.
(a) [Co(H2O)6]2+ (b) [Cr(H2O)6]2+
2+ (iv) (Et4N)2 [COCl4] or [CoCl4]2–
(c) [Mn(H2O)6] (d) [Fe(H2O)6]2+
BITSAT-2020 Cl– act as weak field ligend (∆ < P), then no pairing of
electrons are possible,
Ans. (a) : Paramagnetic behavior depends on the
number of unpaired present in complex. CO2+; d7
H2O act as weak field ligand in all the complexes and No. of unpaired electron = 3
does not cause pairing of metal d-atoms.
Magnetic moment = 3(3 + 2) = 15 = 3.8 B.M.
(a) [Co(H 2 O)6 ]2+ ⇒ Co 2+ : d 7
Order of magnetic moment is:–
(I)> (IV)> (III)> (II)
Number of unpaired electron = 3 288. Which among the following statements is
(b) [Cr(H 2 O)6 ]2+ ⇒ Cr 2+ :d 4 incorrect for manganate and permanganate
ions?
(a) Both ions are tetrahedral structure.
Number of unpaired electron = 4 (b) In both the ions, pπ - dπ bonding is observed
(c) [Mn(H 2 O)6 ]2+ ⇒ Mn 2+ :d 5 between oxygen and manganese.
(c) Both ions are paramagnetic.
(d) Permanganate is stronger oxidising agent than
manganate.
Number of unpaired electrons = 5
AP EAMCET (Engg.) 17.09.2020, Shift-II
(d) [Fe(H 2 O)6 ]2+ ⇒ Fe 2+ :d 6
Ans. (c) : Manganate ion

Number of unpaired electron = 4


Hence, [Co(H 2 O)6 ]2+ will exhibit the lowest
paramagnetic behavior because it contains lowest
unpaired electrons. • Tetrahedral
287. The correct order of the spin only magnetic • dπ (Mn) — pπ (O) is present.
moments of the following complexes is • Paramagnetic, due to presence of one unpaired d-
(I) [Cr(H2O)6]Br2
electron of Mn+6(3d') of MnO 24− .
(II) Na4[Fe(CN)6]
(III) Na3[Fe(C2O4)3](∆0 > P) • Oxidising agent.
(IV) (Et4N)2 [CoCl4] Permanganate ion
(a) (II) ≈ (I) > (IV) > (III)
(b) (I) > (IV) > (III) > (II)
(c) (III) = (I) > (IV) > (II)
(d) (III) = (I) > (II) > (IV)
JEE Main 2020, 9 Jan Shift-II
• Tetrahedral
Ans. (b) : (i) [Cr(H 2 O)6 ]Br2 ⇒ Cr 2+ • dπ(Mn)—pπ(O) is present.
• Diamagnetic, due to the absence of unpaired d-
Number of unpaired electron = 4 electron in Mn+7 (3dº) of MnO −4 .

Magnetic moment = 4(4 + 2) = 24 = 4.9 B.M. • Stronger oxidising agent.


289. Which complex among the following has the
(ii) Na 4 [Fe(CN)6 ] highest value of spin only magnetic moment?
CN− act as strong field ligend and do pairing of metal Fe ( CN )6  ,
3-
Fe ( CN )6  ,
4-
 Ni ( CN )4  ,
2-

d- electrons.
 Ni ( Cl )4 
2-

Objective Chemistry Volume-II 560 YCT


3− 4−
(a)  Fe ( CN )6  (b)  Fe ( CN )6  (b) [Fe[H2O)6]Cl3
x +6(0) +3(−1) = 0 ⇒ x = +3
2− 2−
(c)  Ni ( CN )4  (d)  Ni ( Cℓ ) 4  Fe3+ = [Ar] 3d5
AP EAMCET (Engg.) 21.09.2020, Shift-I
3−
Ans. (d) : (a)  Fe ( CN )6  has d2sp3 (inner d - Unpaired electron (n) = 5
complex) hybridisation with one electron unpaired. (c) K3[FeF6]
4− 3(+1) +x +6(−1) = 0 ⇒ x = +3
(b) The Fe2+ ion in  Fe ( CN )6  has outer electronic Fe3+ = [Ar] 3d5
configuratin of 3d6. The complex is a low spin Unpaired electron (n) = 5
complex. It contains 0 unpaired electrons with (d) K4[MnF6]
magnetic moment of 0 BM. Therefore, under the 4(+1) +x +6(−1) = 0 ⇒ x = +2
possible electronic arrangement of Fe (II) ion is Mn2+ = [Ar] 3d5
t 62g , e0g .
2−
(c)  Ni ( CN )4  is a square planar geometry formed
Unpaired electron (n) = 5
2−
be dsp2 hybridisation.  Ni ( CN )4  is 291. Which of the following ions shows maximum
diamagnetic, so Ni2+ ion has 3d8, outer paramagnetic property?
configuration with two unpaired electrons. (a) [Cr(H2O)6]3+ (b) [Cu(H2O)6]2+
2+
For the formation of the square planar sructure by (c) [Fe(H2O)6] (d) [Zn(H2O)6]2+
structure by dsp2 - hybridisation, two unpaired d- Tripura JEE-2019
3+
electrons are paired up due to energy made Ans. (c) : (a) [Cr(H2O)6]
available by the approach of ligands, making one x + 6(0) = +3
of the 3d-orbitals empty. x = +3
Cr3+ = [Ar] 3d3

Unpaired electron (n) = 3


(b) [Cu(H2O)6]2+
x +6(0) = +2
x = +2
Cu2+ = [Ar] 3d9

Unpaired electron (n) = 1


(c) [Fe(H2O)6]2+
x +6(0) = +2
x = +2
Fe2+ = [Ar] 3d6
∴ Cl– is a weak ligand so, there is no pairing of
electrons.
Number of unpaired = 2 Unpaired electron (n) = 4
290. Compounds with spin-only magnetic moment (d) [Zn(H2O)6]2+
equivalent to five unpaired electrons are x +6(0) = +2
(a) K4[Mn(CN)6] (b) [Fe(H2O)6]Cl3 x = +2
(c) K3[FeF6] (d) K4[MnF6] Zn2+ = [Ar] 3d10
WB JEE-2019
Ans. (b,c,d) :
(a) K4[Mn(CN)6] Unpaired electron (n) = 0
4(1) +x +6(−1) = 0 ⇒ x = +2 Thus, [Fe(H2O)6]2+ shows maximum paramagnetic
Mn2+ = [Ar] 3d5 (pairing occurred because CN property because it has maximum number of unpaired
is SFL) electrons.
292. The coordination number of Th in
K4[Th(C2O4)4(H2O)2] is(C2O42–) = Oxalato
(a) 14 (b) 10
(c) 8 (d) 6
Unpaired electron (n) = 1 JEE Main 2019, 11 Jan Shift-II

Objective Chemistry Volume-II 561 YCT


Ans. (b) : Coordination number is equal to number of Ans. (b) : 4n [V(CN)6]4– and [Fe(CN)6]4–, CN– act as
ligands attached with metal. strong field ligand, (∆o > P) so pairing of electrons
In K4 [Th (C2O4)4 (H2O)2] occurs.
C2O4 is a bidentate ligand. One C2O4 has two donor
sites. V 2+ : d 3
Coordination number = 2 × 4 + 2 = 10 No. of unpaired electron = 3
293. The coordination numbers of Co and Al in Magnetic moment = 3(3 + 2) = 15 = 3.8 B.M
[CoCl(en)2]Cl and K3[Al(C2O4)3], respectively, 2+ 6
are (en = ethane–1, 2-diamine) Fe : d (Low spin)
(a) 5 and 3 (b) 3 and 3
(c) 6 and 6 (d) 5 and 6
JEE Main 2019, 12 April Shift-II
Ans. (d) : Coordination number of [CoCl(en)2]Cl is 5
because ethylene diammine act as bidentate ligand and No. of unpaired electron = 0
it has two d donor site. Magnetic moment = 0 B.M.
Then, C. No. = 2 × 2 + 1 = 5 In [Ru(NH3)6]3+, NH3 act as strong field ligand with
Coordination number of K3[Al(C2O4)3] is 6 because Ru2+, ∆o > P so pairing of d-electrons occurs Ru3+ : d5
oxalate (C2O4) is bidentate ligand and having two donor (low spin)
site.
C. No. = 3 × 2 = 6
294. Specify the coordination number of the metal
in [CoBr2(en)2]+
(a) 3 (b) 5 No. of unpaired electron = 1
(c) 4 (d) 6 Magnetic moment = 1(1 + 2) = 3 = 1.73B.M.
J & K CET-2019 [Cr(NH3)6] , ∆o < P so pairing is not possible Cr2+:d4
2+
Ans. (d) : Coordination number is the number of
ligands attached with central metal. In [CoBr2(en)2]+
complex, ethylene diamine act as bidentate ligand has No. of unpaired electron = 4
two donor site and Br – is monodentate ligand.
Magnetic moment = 4(4 + 2) = 24 = 4.9 B.M.
C. No. = 2 + 2 × 2 = 6
295. The magnetic moment of an octahedral Order of magnetic moment:–
homoleptic Mn (II) complex is 5.9 BM. The Cr2+ > V2+ > Ru3+ > Fe2+
suitable ligand for this complex is 297. Assertion: Potassium ferricyanide, [Fe(CN)6]3– is
(a) CN– (b) ethylenediamine diamagnetic, while potassium ferrocyanide,
(c) NCS– (d) CO [Fe(CN)6]4– is paramagnetic.
JEE Main 2019, 12 Jan Shift-II Reason: Oxidation state of Fe in potassium
Ans. (c) : Magnetic moment of Mn (II) complex is 5.9 ferricyanide is +2, but in potassium ferrocyanide
B.M. So that Mn contains 5 unpaired electron. Mn can
its oxidation state is +3.
form complex with weak field ligand and due to this
∆0 < P (a) If both Assertion and Reason are correct and
Magnitude of ∆0 is lowest for NCS– ligand. Reason is the correct explanation of
Assertion.
(b) If both Assertion and Reason are correct, but
Reason is not the correct explanation of
Assertion.
(c) If Assertion is correct but Reason is incorrect.
Number of unpaired electron = 5 (d) If both the Assertion and Reason are
Magnetic moment = 5(5 + 2) = 35 = 5.9 B.M. incorrect.
AIIMS 25 May 2019 (Evening)
296. The correct order of the spin only magnetic
moment of metal ions in the following low spin 3 Ans. (d): In case of potassium ferricyanide, [Fe(CN)6]–
complexes ,
4– 4– 3+
[V(CN)6] , [Fe(CN)6] , [Ru(NH3)6] , and Fe is present in +3 oxidation state
[Cr(NH3)6]2+, is
(a) Cr2+ > Ru3+ > Fe2+ > V2+
(b) V2+ > Cr2+ > Ru3+ > Fe2+
(c) V2+ > Ru3+ > Cr2+ > Fe2+
(d) Cr2+ > V2+ > Ru3+ > Fe2+ Fe3+ having 1 unpaired electron that’s why it act as
JEE Main 2019, 8 April Shift-I paramagnetic.
Objective Chemistry Volume-II 562 YCT
In potassium ferrocyanide, [Fe(CN)6]4–, Fe is present in Ans. (a) : (i) [Fe (H2O)6]2+ – Octahedral
+2 oxidation state.

(ii) K3[Cr(CN)6] – Octahedral

Fe2+ having no unpaired electron that’s why it act as


(iii) K3[Fe(CN)6] – Octahedral
diamagnetic.
Hence, both the assertion and reason are incorrect.
298. The hybridization and magnetic behavior of
(iv) K2[Ni(CN)4] – Square planar
complex [Ni (CO)4] is:
2 2
(a) dsp , paramagnetic (b) dsp , diamagnetic
(c) sp3, paramagnetic (d) sp3, diamagnetic
AIIMS 25 May 2019 (Morning) Thus (i), (ii) and (iii) are paramagnetic species but (i)
has more paramagnetic.
Ans. (d) : In [Ni(CO)4], Ni is present in zero oxidation
state and CO act as strong field ligand that’s why 301. Among the following species, the one which
pairing of electron occurs. become paramagnetic on heating at 850 K is
(a) Fe3O4 (b) MnO
(c) ZnFe2O4 (d) Both (a) and (c)
JIPMER-2018
Ans. (d) : Ferrimagnetic substance on heating at 850 K
loose its ferrimagnetic behaviour and behaves like
paramagnetic substance.
Examples of ferrimagnetic substance are Fe3O4,
ZnFe2O4, MgFe2O4 etc.
Complex having no unpaired electron, so it act as
diamagnetic in nature. 302. The coordination number of central metal
atom in a complex is determined by :
(a) The number of ligands around a metal ion
Ni⇒3d10 bonded by sigma bonds
(b) the number of ligands around a metal ion
bonded by π bonds
Hence, hybridization of [Ni(CO)4] complex is sp3. (c) the number of ligand around a metal ion
299. The coordination number, oxidation state and bonded by sigma and π bonds both
the number of d electrons of the metal in the (d) None of the above
complex [Co(NH3)5 (CO3)]Cl are respectively Manipal-2018
(a) 6, 3, 6 (b) 7, 2, 7 Ans. (a) : The co-ordination number of central metal
(c) 7, 1, 6 (d) 6, 2, 7 atom in a complex is determined by the number of
Assam CEE-2019 ligands around a metal ion bonded by sigma bonds.
Ans. (a) : In [Co (NH3)5 (CO3)]Cl five ammonia
ligands and one carbonate ligand is attached to the
centred cobalt atom. Hence, the coordination number is
six
Let X be the oxidation number of cobalt. X + (–2) + (–
1) =0 or X=3. Hence, the oxidation number is 3.
303. Coordination number of Zn in ZnS is :
The outer electronic configuration of cobalt (atomic
7 2
number 27) is 3d 4s . The outer electronic (a) 4 (b) 6
+3 6 o
configuration of Co will be 3d 4s . Thus it contains (c) 2 (d) None of these
six d electrons. Manipal-2018
[Co(NH3)5CO3]ClO4 is inner orbited or low spin Ans. (a) : ZnS occurs in two common polytypes,
complexes. all the electron are paired due to d2sp3 zincblende (also called sphalerite) and wurtzite. In each
hybridization of Co+3 ion. Hence, the number of from, the co-ordination geometry at Zn and S is
unpaired electron is zero. tetrahedral. Thus, each atom connected with four.
Hence, the co-ordination number is 4.
300. Choose the complex which is paramagnetic
304. The geometry and magnetic behaviour of the
(i) Fe ( H 2O )6  (ii) K 3 Cr ( CN )6 
2+
complex [Ni(CO)4] are
(a) square planar geometry and diamagnetic
(iii) K 3 Fe ( CN )6  (iv) K 2  Ni ( CN )4  (b) tetrahedral geometry and diamagnetic
(a) (i), (ii) and (iii) (b) (i), (iii) and (iv) (c) square planar geometry and paramagnetic
(c) (ii), (iii) and (iv) (d) (i), (ii) and (iv) (d) tetrahedral geometry and paramagnetic.
VITEEE -2019 NEET 2018
Objective Chemistry Volume-II 563 YCT
Ans. (b) : The complex is [Ni(CO)4]. Hence, it act as paramagnetic
[Ni(CO)4] (c) [Ni(CO)4 ] ⇒ Ni : 3d 8 4s 2
x + 4(0) = 0 CO act as strong field ligand and do pairing of d-
x=0 electron.
CO is a strong field ligand, pairing will take place.
Ni = [Ar] 3d8 4s2

Number of unpaired electron = 0


(d) [CO(NH 3 )6 ]Cl3 or [CO(NH 3 )6 ]3+
CO3+ : d6

Number of unpaired electron = 0

307. What will be the geometry and magnetic


Four sp3 hybrid orbital shows the diamagnetic character moment of the complex [NiCl4]2–?
because no unpaired e− is present. (a) Tetrahedral and 3.87 B.M.
305. Which shows the maximum magnetic moment? (b) Tetrahedral and 2.82 B.M.
(a) V3+ (b) Cr3+ (c) Square planar and 2.82 B.M.
3+
(c) Fe (d) Co3+ (d) Square planar and 4.89 B.M.
HP CET-2018 J & K CET-2018

Ans. (c) : Ans. (b) : Cl act as weak field ligand , and pairing is
not possible.
Number of unpaired electron = 2 [NiCl4]2– ⇒ Ni2+ ; d8
Magnetic moment = 2(2 + 2) = 8 = 2.84 B.M.

Number of unpaired electron = 3


Cl– occupies sp3 orbitals.
Magnetic moment = 3(3 + 2) = 15 = 3.8B.M. Hybridization = sp3
Geometry = tetrahedral
No. of unpaired electron = 2
Number of unpaired electron = 5 Magnetic moment = 2(2 + 2) = 8
Magnetic moment = 5(5 + 2) = 35 = 5.9 B.M. = 2.82 B.M.
308. Which ion has the least value of theoretical
magnetic moment?
Number of unpaired electron = 4 (a) Cr3+ (b) Co3+
3+
Magnetic moment = 4(4 + 2) = 24 = 4.9 B.M. (c) Ti (d) V3+
3+
GUJCET-2018
Hence, Fe maximum magnetic moment.
(a)
Ans. (c) :
306. Which of the following is paramagnetic? Number of unpaired electron = 4
(a) K4 [Fe(CN)6] (b) K3[Fe(CN)6]
Magnetic moment = 4(4 + 2) = 24
(c) Ni(CO)4 (d) [Co(NH3)6]Cl3
HP CET-2018 = 4.9 B.M.
2+ 3+ 6
Ans. (b) : (a) K 4 [Fe(CN)6 ] ⇒ Fe (b) Co : d

CN act as strong field ligand and do pairing of d- Number of unpaired electron = 4
electron. Magnetic moment = 4(4 + 2) = 24
= 4.9 B.M.
Number of unpaired electron = 0
(c)
K 4 [Fe(CN)6 ] act as diamagnetic substance.
Number of unpaired electron = 1
(b) K 3 [Fe(CN)6 ] ⇒ Fe3+
Magnetic moment = 1(1 + 2) = 3
=1.73 B.M.
Number of unpaired electron = 1
Objective Chemistry Volume-II 564 YCT
(a) If both Assertion and Reason are correct and
(d) the Reason is the correct explanation of
Number of unpaired electron = 2 Assertion.
(b) If both Assertion and Reason are correct, but
Magnetic moment = 2(2 + 2) = 8
Reason is not the correct explanation of
= 2.84 B.M. Assertion.
Hence, Ti3+ has least value theoretical magnetic (c) If Assertion is correct but Reason is incorrect.
moment. (d) If both the Assertion and Reason are
309. A gas metal in bivalent state has approximately incorrect.
23e–, what is spin magnetic moment in AIIMS-26 May, 2018(M)
elemental state? Ans. (d) : In Zeise’s salt, [PtCl3(C2H4)]– coordination
(a) 2.87 (b) 5.5 number of Pt is four.
(c) 5.9 (d) 4.9
AIIMS-26 May, 2018 (E)
Ans. (c): When metal is present in +2 oxidation state
then it contains 23e–
M2+ : 23e–
M : 25e– Ethene act as monodentate ligand.
2 2 6 2 6 2 5 Hence, both Assertion and Reason are incorrect.
Electronic configuration: 1s 2s 2p 3s 3p 4s 3d
5
d No. of unpaired electron = 5 312. Which of the following is a pair of diamagnetic
complex ?
Magnetic moment (µ) = n(n + 2) = 5(5 + 2) 3− 4−

= 5.9 B.M. (a) Co ( ox )3  ,  Fe ( CN )6 


3−
(b) Co ( ox )3  , [ FeF6 ]
3−
310. Which of the following complexes shows
paramagnetic?
3−
(c)  Fe ( ox )3  , [ FeF6 ]
3−
(a) [Zn(NH3)3]2+ (b) [PdCl2(PPh3)2]
3+
(c) [NiCl2(PPh3)2] (d) [Co(en)3] 3−
(d)  Fe ( CN )6  , [ CoF6 ]
3−
CG PET -2018
Ans. (c) : If complex compounds contains unpaired AIIMS-26 May, 2018(M)
electron then it act as paramagnetic.
Ans. (a) : Diamagnatic substance having no unpaired
(a) [Zn(NH 3 )3 ]2+ ⇒ Zn 2+ :d10 electrons.
(a) [Co(ox)3]3−, ⇒ Co3+ = d6
In [Co(ox)3]3−, Co3+ act as strong field ligand with
Number of unpaired electron = 10 oxalate.
(b) In[PdCl 2 (PPh 3 )2 ] complex, Pd is from 4th period
and it act as low spin complex and pairing of metal d Co3+ having no unpaired electron and act as
electrons occurs. diamagnetic.
(b) [Fe(CN)6]4−, ⇒ Fe2+ = d6
Number of unpaired electron = 0
(c) In [NiCl2 (PPh 3 ) 2 ] complex, Ni belongs to 3rd period
Fe2+ with CN− act as strong field ligand so no unpaired

and Cl , PPh 3 belongs to weak field ligand so pairing electron is present. Hence it is also act as diamagnetic
of electron are not possible, complex.
2+
Ni : d 8 313. Which of the following is not expected to show
paramagnetism?
(a) [Ni(H2O)]2+ (b) Ni(CO)4
2+
Number of unpaired electron = 2 (c) [Ni(NH )
3 4 ] (d) [Co(NH3)6]2+
Hence, [NiCl2 (PPH 3 ) 2 ] complex shows paramagnetic JIPMER-2017
behavior. Ans. (b) : (a) H2O act as weak field ligand. So pairing is
3+ not possible.
(d) In [Co(en)3 ] Complex, ethylene diamine act as
strong field ligand with Co3+ complex and pairing of
metal d- electrons occurs. This complex having unpaired electron it behave like
paramagnetic.
Co3+: d6 (b) In [Ni(Co)4], Co act as strong field ligand ∆ o > p .
Number of unpaired electron = 0
311. Assertion : In Zeise's salt, coordination no. of Pt
is five.
Reason: Ethene is bidentate ligand.
Objective Chemistry Volume-II 565 YCT
No. unpaired electron so [Ni(Co)4] act as diamagnetic.
(c) [Ni(NH3)4]2+, NH3 act as weak field ligand so
pairing of electron is not possible.

[Ni(NH3)4]2+ have 2 unpaired electron so behave as


paramagnetic.
No. of unpaired electron = 1
(d) [Co(NH3)6]2+, NH3 act as weak field ligand ∆ o < p
Spin only Magnetic moment = 1(1 + 2) = 1.73 B.M.
317. A common observation seen in Fe3O4 is that it
[Co(NH3)6]2+ has 3 unpaired electron so it behave as is ferrimagnetic at room temperature but at
paramagnetic. 850 K it becomes.
314. The colour and magnetic nature of manganate (a) diamagnetic (b) paramagnetic
ion (MnO42–) is (c) ferromagnetic (d) non-magnetic
(a) green, paramagnetic J & K CET-2017
(b) purple, diamagnetic
Ans. (b) : Fe3O4 is ferrimagnetic at room temperature
(c) green, diamagnetic
but at 850 K it becomes paramagnetic.
(d) purple, paramagnetic
Ferrimagnetic substances having anti parallel and
MHT CET-2017 parallel direction of magnetic moment of domain which
Ans. (a) : [ MnO 4 ]
2−
is the manganate ion. The cancel each other of but on heating ferrimagnetic
property of substance loose and it become paramagnetic
oxidation state of Mn is +6. Due to the electronic
Fe ( CN )6  is
3-
transition, they exhibit the green colour. 318. Assertion : weakly
Mn +6 = [ Ar ] 3d 5
Fe ( CN )6  is
4-
paramagnetic while
diamagnetic.
Reason: Fe ( CN )6  has +3 oxidation state
3-
It contain the five unpair electron due to which they
shows the paramagnetic character.
while Fe ( CN )6  has +2 oxidation state.
4-

315. The paramagnetic species is


(a) SiO2 (b) TiO2 (a) If both Assertion and Reason are correct and
(c) BaO2 (d) KO2 the Reason is the correct explanation of
UPTU/UPSEE-2017 Assertion.
Ans. (d) : The species which has unpaired electron, is (b) If both Assertion and Reason are correct, but
paramagnetic in nature. Reason is not the correct explanation of
KO2: Assertion.
Let, x be the oxidation state of K. (c) If Assertion is correct but Reason is incorrect.
(d) If both the Assertion and Reason are
x +2(−2) = 0
incorrect.
x = +4
4+ AIIMS-2017
K = 1s2, 2s2, 2p6, 3s2, 3p3
Ans. (b) : [Fe(CN)6]3–, Fe has +3 oxidation state and
CN act as strong field ligand which causes pairing of
electrons.
Unpaired electron (n) = 3, Paramagnetic
316. What will be the spin only magnetic moment of
high spin and low spin d5 electronic system in
an octahedral complex?
Number of unpaired electron = 1
(a) µ high spin = 1.73 MB and µ low spin = 5.92 BM
That's why is act as paramagnetic in nature.
(b) µ high spin = 5.92 BM and µ low spin = 1.73 BM [Fe(CN)6]2–, Fe has +2 oxidation state and pairing
(c) µ = 4.9 BM and µ = 2.83 BM occurs because of CN.
high spin low spin

(d) µ high spin = 1.73 BM and µ low spin = 1.73 BM


J & K CET-2017 Number unpaired electron = 0
5
Ans. (b) : High spin d octahedral complex That’s why it act as diamagnetic in nature
Hence, both assertion and Reason are correct but reason
is not the correct explanation of assertion.
No. of unpaired electron = 5 319. When an aqueous solution of copper (II)
sulphate is saturated with ammonia, the blue
Spin only magnetic moment = 5(5 + 2) = 5.92 B.M. compound crystallizes on evaporation. The
Low spin d5 octahedral complex formula of this compound is:

Objective Chemistry Volume-II 566 YCT


(a) [Cu(NH3)4] SO4. H2O (square planar) No. of unpaired electron = 5
(b) [Cu(NH3)4] SO4 (Tetrahedral) Magnetic moment = 5 ( 5 + 2 ) = 35 = 5.92 B.M.
(c) [Cu(NH3)6] SO4 (Octahedral)
(d) [Cu(SO4) (NH3)5] (Octahedral) (z) [Cr(H2O)6]3+ ⇒
BITSAT-2017 No. of unpaired electron = 3
Ans. (a) : CuSO4 + 4NH3 + H2O  → [Cu(NH3)4]SO4 Magnetic moment = 3 ( 3 + 2 ) = 15 = 3.8 B.M.
H 2O
Aqueous solution of copper sulphate reacts with Order of Magnetic moment :
ammonia and give blue color compound of tetrammine x<y<z
copper (II) sulphate. 322. Consider the following two complex ions :
Copper is present in +2 oxidation and hybridization of [CoF6]3- and [Co (C2O4)3]3-. Which of the
complex is dsp2. Shape of [Cu(NH3)4]SO4 is square following statements (s) is/are false?
planar. (I) Both are octahedral.
320. Identify, from the following, the diamagnetic, (II) [Co(C2O4)3]3- is diamagnetic while [CoF6]3-
tetrahedral complex is paramagnetic.
(a) [Ni(Cl)4]2– (b) [Co(C2O4)3]3– (III) Both are outer orbital complexes.
2–
(c) [Ni(CN)4] (d) [Ni(CO)4] (IV). In both the complexes, the central metal is
AP EAMCET-2017 in the same oxidation state.
2– +2 8
Ans. (d) : (i) [NiCl4] = Ni (3d ) (a) Both (II) and (III) (b) (II), (III) and (IV)
(c) Only (III) (d) Both (III) and (IV)
(e) (I), (II) and (IV)
Kerala CEE-2016
∴ Cl is a weak field legend. It docent pair. Ans. (c) The complex are [ CoF6 ]
3−
and Co ( C 2O 4 )3  .
3−

Hence [NiCl4]2– is tetrahedral and two unpaired


electron. (I) Both complex are octahedral in which first
complex contains the monodentate ligand whereas
(ii) [Co(C2O4)3]3– , (C 2 O 4 )32– is a bidentate ligand thus other one contains the bidentate ligands. Hence,
give octahedral geometry. statement is true.
(iii) [Ni(CN4)]2– = Ni+2(3d8) 3−

∴ CN– is a strong field ligand (II) Co ( C 2O 4 )3 


x + 3 ( −2 ) = −3
x = +3
Due to strong nature of bidentate ligand pairing
occurred in inner orbital.

[Ni(CN4)]2– show square planar geometry.


(iv) [Ni(CO4)]
Ni has zero oxidation state. 3−
[Ni(CO4)] Co ( F )6 
x + 6 ( −1) = −3
Thus [Ni(CO)4] show no unpaired electron and x = +3
tetrahedral geometry i.e diamagnetic. ∵ F is a weak field ligand
321. Magnetic moment of [Ti(H2O)6]4+, ∴ Co3+ = [ Ar ] 3d 6
2+ 3+
[Mn(H2O)6] and [Cr(H2O)6] can be
represented as X, Y and Z.
They are in order of
(a) X < Z < Y (b) Y < Z < X (n = 4, Paramagnetic)
(c) X < Y < Z (d) Z < Y < X Statement is true.
JIPMER-2016 (III) Co ( C O )  3− is a inner orbital complex with
 2 4 3
Ans. (a) : H2O act as weak field ligand and pairing of 2 3
electron is not possible. d sp hybridisation due to strong bidentate ligand
3−
(x) [Ti(H2O)6] ⇒ Ti : d
4+ 4+ 0
whereas Co ( F )6  is outer orbital complex.
No. of unpaired electron = 0
Hence statement is not true.
Magnetic moment = 0 B.M.
(IV) It is dear from above discussion, both metal are
(y) [Mn(H2O)6]2+ ⇒ same oxidation state. Hence, Statement is true.

Objective Chemistry Volume-II 567 YCT


323. Permanganate ions are Ans. (c) : Coordination number (CN) of a metal ion in a
(a) tetrahedral and paramagnetic complex can be defined as the number of ligand donar
(b) tetrahedral and diamagnetic atoms to which the metal is directly bonded.
(c) octahedral and paramagnetic Hence, the coordination number of the given compound
(d) octahedral and diamagnetic will be 6.
AMU-2016 327. Which of the following metal ions has a
Ans. (b) : Permanganate ion is MnO −4 . calculated magnetic moment value of 24 BM?
(a) Mn2+ (b) Fe2+
Mn+7 = 3d° 4s° 3+
(c) Fe (d) Co2+
TS EAMCET-2016
Ans. (b) : ∵ Magnetic moment (µs ) = n(n + 2) BM

Hybridisation = d3s
Shape = tetrahedral
Number of unpaired electron = 0 So, µs = 4(4 + 2)
Diamagnetic in nature. µs = 24 BM
324. The spin only magnetic moment (µs) of a • Mn2+ = [Ar]3d5
complex [MnBr4]2– is 5.9 BM. The geometry of
the complex will be
(a) tetrahedral (b) square planar ∴ µs = 5(5 + 2) = 35 BM
(c) square pyramidal (d) tetragonal
AMU-2016 • Fe = [Ar]3d5
3+

Ans. (a) : In [MnBr4]2–, Mn is present in +2 oxidation


state.
∴ µs = 5(5 + 2) = 35 BM
Mn2+ : d5
• Co = [Ar]3d7
2+
Number of unpaired electron = 5
Spin only magnetic moment = 5(5 + 2) = 5.9 B.M.

Br act as weak field ligand so, pairing of d-electrons ∴ µs = 3(3 + 2) = 15 BM
not possible. Hence, [MnBr4]2– forms outer-orbital Hence, correct option is (b).
complex with sp3 hybridisation and geometry is
328. In which silicate, the central metal has
tetrahedral
coordination number 6?
[MnBr4] 2– (a) Zn2[SiO4] (b) Be2[SiO4]
(c) Mg2[SiO4] (d) Zr[SiO4]
SCRA-2015
325. The highest magnetic moment is shown by the Ans. (c): Mg2[SiO4] silicate, the central metal has
transition metal ion with outer electronic coordination number 6.
configuration 329. The coordination number and oxidation
(a) d9 (b) d7 number X in the following compound
(c) d5 (d) d3  X ( SO 4 )( NH 3 )5  CI will be
JCECE - 2016
(a) 10 and 3 (b) 02 and 6
Ans. (c) : More be the number of unpaired electrons,
more be the value of magnetic moment. (c) 06 and 3 (d) 06 and 4
5 CG PET- 2015
d - configuration contains maximum number of
unpaired electrons i.e. (5). Ans. (c) : In complex [X(SO 4 )(NH 3 )5 ]Cl or
Thus, show maximum dipole moment as per the [X(SO 4 )(NH 3 )5 ] +
following relation. Let, oxidation of metal = x
Magnetic moment (µeff ) = n ( n + 2 ) BM x + 1×(−2) + 5× 0 = +1
where, n = no. of unpaired electrons. x −2 =1
326. Coordination number of Co in [Co(NH3)6]Cl3 x = +3
(a) + 2 (b) + 3 Coordination number is the number of monodentate
(c) + 6 (d) + 8 ligands attach with central metal.
VITEEE-2016 Here, coordination number = 6
Objective Chemistry Volume-II 568 YCT
330. What is the value of x on the [Ni(CN)4]x 334. The complex showing a spin-only magnetic
complex ion? moment of 2.82 B.M. is:
(a) +2 (b) −2 (a) Ni(CO)4 (b) [NiCl4]2–
(c) 0(zero) (d) +4 (c) Ni(PPh3)4 (d) [Ni(CN)4]2–
CG PET- 2015 IIT JEE-2010
Ans. (b) : If any complex follows E.A.N. rule then that Ans. (b) : Given– µs = 2.82B.M.
complex is stable and exist. (a) Ni(CO)4, O.S. of Ni = 0
According to E.A.N. rule, complex having total electron 8
(28)Ni = [Ar]3d 4s
2
is equal to atomic number of inert gases. Unpaired electron (n) = 0
E.A.N. = atomic number of metal + 2 × number of µs = 0
ligand – oxidation state 2−
(b) [NiCl 4 ]
[Ni(CN) 4 ]x is a square planar complex and its E.A.N.
Oxidation state of Ni = +2
value is 34. 2+
28 Ni = [Ar]3d 8
34 = 28 + 4×(−1) − Oxidation state
Unpaired electron (n) = 2
Oxidation state = 2
2 + 4(−1) = x ∴ µs = n ( n + 2 )

x = −2 µs = 2 ( 2 + 2 ) = 8 = 2.82BM
331. Select the coloured and paramagnetic ions (c) Ni(PPh3)4, O.S. of Ni = 0
(a) Cu+, Zn2+, Ca2+ (b) Sc3+, Ti4+, V5+
28 Ni = [Ar]3d 4s
8 2
2+ + 2+
(c) Cu , Cr , Mn (d) Ni2+, Cu+, Hg2+
Unpaired electron = 0
JCECE - 2015
∴ µS = 0
Ans. (c) : Ion is coloured if there are electrons in d-
suborbit. Paramagnetic nature is also due to unpaired (d) [Ni(CN) 4 ]2− , O.S. of Ni = +2
electrons. Thus, every coloured ion is also Ni +2 = [Ar]3d 8
28
paramagnetic. Unpaired electron (n) = 0
Cu2+ = [Ar]3d9, one unpaired electron in 3d.
Cr+ = [Ar]3d5, five unpaired electrons in 3d. 335. Amongst Ni(CO)4, [Ni(CN)4]2– and  NiCl 42 − 
Mn2+ = [Ar]3d5 five unpaired electrons in 3d. (a) Ni(CO)4 and  NiCl 24−  are diamagnetic but
332. The compound which does not show
[Ni(CN)4]2– is paramagnetic
paramagnetism, is
(b) Ni(CO)4 and [Ni(CN)4]2– are diamagnetic but
(a) [Cu(NH3)4]Cl2 (b) [Ag (NH3)4]Cl
(c) NO (d) NO2 NiCl24− is paramagnetic
MPPET- 2009 (c) NiCl24− and [Ni(CN)4]2– are diamagnetic but
Ans. (b) : [Ag (NH3)4]Cl → [Ag (NH3)4] +Cl-
+
Ni(CO)4 is paramagnetic
↓ (d) Ni(CO)4 is diamagnetic but NiCl24− and
x+4×0=+1 [Ni(CN)4]2– is paramagnetic
x=+1 VITEEE-2011
Ag = [Kr] 4d105s1 Ans. (b) : Electronic configuration of 28Ni in [Ni(CO)4]
Ag+ = [Kr] 4d10 = [Ar] 3d8 4s2
= 0 unpaired electron (diamagnetic)
Electronic configuration of 28Ni+2 in [Ni(CN)4]2–
The no. of unpaired electron is zero its show
diamagnetic not paramagnetic. = [Ar] 3d8(square planar)
= 0 unpaired electron (diamagnetic)
333. Which of the following forms with an excess of
Electronic configuration of 28Ni+2 in [Ni(Cl)4]2–
CN– ion, a complex having coordination
number two : = [Ar] 3d8
(a) Cu2+ (b) Ag+ = 2 unpaired electron (paramagnetic)
(c) Ni 2+
(d) Fe2+ 336. Coordination number of Ni in [Ni(C2O4)3]4– is
(a) 3 (b) 6
MPPET-2013
(c) 4 (d) 5
Ans. (b) : The given metal ions only Ag+ forms VITEEE- 2010
complex with CN– having co-ordination number 2.
Ans. (b) : Coordination number of Ni in [Ni(C2O4)3]4–
Ag+ + 2CN– → [Ag(CN)2]–
Coordination number of metals is defined as the number is 6 because C 2 O 24− (oxalate) is a bidentate ligand and
of σ bond by which ligands are attached to the metal each have two sites to coordinated with the central
atom. atom.
Objective Chemistry Volume-II 569 YCT
337. The magnetic moment of a salt containing Zn2+ 340. Which of the following pair of transition metal
ion is ions, have the same calculated values of
(a) 0 (b) 1.87 magnetic moment?
(c) 5.92 (d) 2 (a) Ti2+ and V2+ (b) Fe2+ and Cu2+
VITEEE- 2008 2+
(c) Cr and Fe 2+
(d) Co2+ and Ti2+
Ans. (a) : Magnetic moment of a salt depends upon the AP EAMCET (Engg.) -2007
number of unpaired d-electrons. In Zn2+ salt
Ans. (c) :
configuration of cation is 4s03d10. Hence total no. of
unpaired electron, n is zero. So magnetic moment i.e. (a) Ti2+= d3
µ = n ( n + 2) = 0
Where, n is the number of unpaired electron. V2+ = d3
338. Among the following the compound that is both (b) Fe2+= d6
paramagnetic and coloured is
(a) K2Cr2O7 (b) (NH4)2[TiCl6] Cu2+ = d9
(c) VOSO4 (d) K3Cu(CN)4
VITEEE- 2007 (c) Cr2+ = d4
Ans. (c) : (c) In VOSO4, V is in +4 oxidation state. Fe2+ = d6
hence has one unpaired electron, thus it is coloured and
paramagnetic. So, both Cr2+ and Fe2+ having same magnetic moment.
(b) In (NH4)[TiCl6] Ti is in + 4 oxidation state. hence (d) Co2+ = d7
has no unpaired electron hence colourless and
diamagnetic. Ti2+ = d2
(d) In K4[Fe(CN)6] Cu is in + 1 oxidation state hence
has no unpaired electron hence colourless and 341. In cuprammonium sulphate, copper is
diamagnetic. (a) diamagnetic (b) paramagnetic
(a) In K2Cr2O7, Cr is in +6 oxidation hence has no (c) ferromagnetic (d) non magnetic
unpaired electron and thus it is diamagnetic. Though COMEDK-2011
K2Cr2O7 has no unpaired electron but it is coloured.
Ans. (b) : The chemical formula of Complex is
This is due to charge transfer spectrum.
339. Identify the order in which the spin only Cu ( NH 3 ) 4  SO 4 The oxidation state of the copper is
magnetic moment (in BM) increases for the +2. electronic configuration =[Ar] 3d9 (square planer)
following four ions unpaired electron ⇒ n = 1
(I) Fe 2+ (II) Ti 2+ Hence, the complex is paramagnetic.
(III) Cu 2+
(IV) V 2+
342. Coordination number of nickel in [Ni(C2O4)3]4–
(a) I, II, IV, III (b) IV, I, II, III is
(c) III, IV, I, II (d) III, II, IV, I (a) 3 (b) 12
AP EAMCET- (Engg.)-2011 (c) 6 (d) 4
COMEDK-2012
Ans. (d) : (i) Ans. (c) : The co-ordination number of nickel in
Magnetic moment (µ) = n ( n + 2 ) [Ni(C 2 O 4 )3 ]4– is 6 because oxalato is the bidentate
Where n = no. of unpaired electron ligand which have two site of donation.
343. Which is diamagnetic?
µ = 4 ( 4 + 2 ) = 24 = 4.90 BM
(a) [Co(F)6]3– (b) [Ni(CN)4]2–
2–
(ii) (c) [NiCl4] (d) [Fe(CN)6]3–
(At. No Co=27, Mo=42, Pt=78)
n = 2, µ = 2.83 BM
Karnataka NEET-2013
(iii) Ans. (b) :
Complex Oxidati Electronic No. of
n = 1, µ = 1.732 BM on state configuratio unpaired
n electron
(iv) a 3− +3 [Ar]3d6 n=4
Co ( F6 ) 
n = 3, µ=3.87 BM. (paramagn
etic)
Hence, the increasing order of magnetic moment is – 8
b [Ni(CN) 4 ] 2– +2 [Ar]3d n=0
Cu 2+ < Ti 2+ < V 2+ < Fe +2 (diamagnet
Hence, III < II < IV < I ic)
Objective Chemistry Volume-II 570 YCT
c [NiCl4 ]2 – +2 [Ar]3d8 n=2
(paramagn
etic)
3– 5
d [Fe(CN)6] +3 [Ar]3d n=1
(paramagn Therefore, Number of unpaired electron(n)=0
etic) Spin magnetic moment = n(n + 2) = 0
344. Which among the following is a paramagnetic 347. The effective atomic number of Cr in
complex? [Cr(NH3)6]Cl3 is:
(a) [Co(NH)3)6]3+ (b) [Pt(en)Cl2]2– (a) 35 (b) 36
(c) [CoBr4]2– (d) Mo(CO)6 (c) 27 (d) 33
Karnataka NEET-2013 AP EAMCET (Medical), 2006
Ans. (c) : Ans. (d) : The complex compound is [Cr(NH3)6]Cl3.
Complex Oxidati Electronic No. of unpaired The oxidation state of chromium in the compound is +3.
on configuratio electron EAN= atomic number–O.N. + 2 (Co-ordination
state n number)
a 3 + +3 [Ar]3d n=0 EAN = 24 – 3 + 2 × 6
Co ( NH 3 )6 
(low spin) (diamagnetic) EAN = 24 – 3 +12
8
b [Pt(en)Cl 2 ] +2 [Ar]5d n=0 EAN = 33
(diamagnetic) 348. AB is an ionic solid. If the ratio of ionic radii of
c [CoBr4 ]2– +2 [Ar]3d7 n=3 A+ and B– is 0.52, what is the coordination
(paramagnetic) number of B– ?
5 1
d Mo(CO)6 +3 [Ar]4d 5s n=0 (a) 2 (b) 3
(diamagnetic) (c) 6 (d) 8
345. Which one of the following sets correctly AP EAMCET (Medical), 2008
represents the increases in the paramagnetic Ans. (c) : For the ionic solids, we can write the
property of the ions? following radius ratio’s of the ions which is given
2+
(a) Cu > V > Cr > Mn2+ 2+ 2+ below-
Radius ratio r+/r– Co-ordination number
(b) Cu 2 + < Cr 2 + < V 2 + < Mn 2 +
0.155 – 0.225 3
(c) Cu 2 + < V 2 + < Cr 2 + < Mn 2 +
0.225 – 0.414 4
(d) V 2 + < Cu 2 + < Cr 2 + < Mn 2 + 0.414 – 0.732 6
AP EAMCET (Engg.)-2009 0.732 – 1.00 8
Ans. (c) : As the number of unpaired electron increases Hence , if the ratio of ionic radii is 0.52, then the co-
magnetic moment also increase which further increase ordination number of B– is 6.
the paramagnetic property of the ions. 349. Experimental value of magnetic momentum of
V ⇒d
2+ 3
3 unpaired electrons Mn+2 complex is 5.96 B.M. This indicates .......
(a) Axial and orbital motion of electron in same
Cr2+ ⇒d4 4 unpaired electrons direction.
(b) Axial and orbital motion of electron in
Mn2+ ⇒d5 5 unpaired electrons opposite direction.
Cu2+ ⇒d9 1 unpaired electrons (c) Electron does not exhibit orbital motion, it
only exhibit axial motion.
Paramagnetic property:–
(d) Electron does not exhibit axial motion, it only
Cu2+ < V2+ < Cr2+ < Mn2+ exhibits orbital motion.
346. The spin only magnetic moment value (in B.M. GUJCET-2011
unit) of Cr(CO)6 is Ans. (a): Experimental value of magnetic momentum of
(a) Zero (b) 2.84 Mn2+ complex is 5.96 B.M. This indicates that axial and
(c) 4.90 (d) 5.92 orbital motion of electron in same direction.
AMU-2014 350. What will be the theoretical value of magnetic
Ans. (a): In Cr (CO)6, the oxidation state of chromium moment (µ), when CN– ligands join Fe 3+ ion to
is zero. yield complex.
Cr= 3d5 4s1 (a) 2.83 BM (b) 3.87 BM
CO ligand act as strong field ligand hence pairing of (c) 5.92 BM (d) 1.73 BM
electrons occurs. GUJCET-2007

Objective Chemistry Volume-II 571 YCT


Ans. (d) : When CN – ligands join Fe3+ ion to yield the Number of unpaired electron = 4
3−
complex ion  Fe ( CN )6  . Electronic configuration of µ = n(n + 2) = 4(4 + 2) = 4.5 BM.
353. A compound of a metal ion Mx+(Z = 24) has a
Fe3+ is [ Ar ] 3d 5 .
spin only magnetic moment of 15 Bohr
Magnetons.The number of unpaired electrons
As we know CN – is the strong field ligand. in the compound are
Number of unpaired electron (n) = 1 (a) 2 (b) 4
(c) 5 (d) 3
Magnetic moment ( µs ) = n ( n + 2 )BM
[AIIMS-2013]
µs = 1(1 + 2 )BM Ans. (d): Spin only magnetic moment =
µ = 1.73 BM. n(n + 2) B.M. For given metal ion, µ = 15 B.M.
s

351. Which of the following is paramagnetic? 15 = n(n + 2)


(a) [ Fe(CN)6 ]
4−
[ Ni(CO)4 ]
2
(b) n + 2n = 15
n=3
(c) [ Ni(CO)4 ]2− (d) [ CoF6 ]
3−
Hence, number of unpaired electron = 3
AIIMS-2014 354. Maximum value of paramagnetism is shown by
Ans. (d): Paramagnetic species contains unpaired (a) [Fe(CN)6]3- (b) [Cr(CN)6]3-
electron. (c) [Co(CN)6] 3-
(d) [Sc(CN)6]3-
(a) [Fe(CN)6]4– ⇒ Fe2+ : d6 AMU-2012
CN act as strong field ligand cause pairing of metal d Ans. (b) : a) [Fe(CN)6] ⇒ Low spin complex pairing
3–
electron.
occur
Fe3+ : d5 (low spin)
Fe2+
Number of unpaired electron = 1
Number of unpaired electron = 0 b) [Cr(CN)6]3– ⇒
(b) [Ni(CO)4], CO act as strong field ligand cause Cr3+ : d3
pairing of metal d electron. Number of unpaired electron = 3
[Ni(CO)4] ⇒ Ni :- 3d84s2 c) [Co(CN)6]3– ⇒ Low spin complex, pairing occurs
Co3+ : d6
Number of unpaired electron = 0
Number of unpaired electron = 0 d) [Sc(CN)6]3–
(c) [Ni (CO)4]2– Sc3+ : d0
Number of unpaired electron = 0
Ni2– ⇒ 3d10 4s2 Hence, [Cr(CN)6]3– contains maximum unpaired
electrons due to this it has maximum paramagnetic in
Number of unpaired electron = 0 nature.
(d) [CoF6]3– Cobalt is present in +3 oxidation state, and
F– act as weak field ligand so no pairing of metal d 355. What is the magnetic moment of Fe3+ ion in
electron. [Fe(CN)6]3– ?
Co3+ ⇒ d6 (a) 1.73 B.M. (b) 5.9 B.M.
(c) Diamagnetic (d) None of these
Number of unpaired electron = 4
AMU – 2010
Hence, [CoF6]3– is act as paramagnetic.
Ans. (a) : Fe3+ is a d5 system and presence of strong
352. A complex [CoL6]n+ where L is neutral ligand field ligand CN–, causes pairing up of all electrons.
has a magnetic moment µ = 4.5 B. M. Hence,
(a) Co must be in +2 oxidation state. [Fe(CN)6]3–→
(b) L must be af strong ligand.
(c) The complex must be highly destabilised. Thus no. of unpaired electrons n = 1
(d) Co must be in +3 oxidation state. µ = n(n + 2)
AIIMS-2010
Ans. (d): [CoL6]n+ has a magnetic moment µ = 4.5 BM. µ= 3 = 1.73B.M.
It means cobalt having 4 unpaired electron. This is 356. Which of the following coordination entities is
possible when metal is present in +3 oxidation state and paramagnetic?
ligand (L) must be weak field ligand. (a) [Ni(CN)4]2– (b) [NiCl4]2–
Co3+ ⇒ d6 (weak field ligand) (c) [Fe(CN)6] 4–
(d) [Co(NH3)6] 3+
d6 AMU – 2007

Objective Chemistry Volume-II 572 YCT


Ans. (b) : The complex is paramagnetic because of the (b) [Co(NH 3 )6 2+
presence of unpaired electrons. NH3 act as weak field ligand with Co2+ and pairing of
Number of unpaired electron in [NiCl4]2– is 2 and in metal d- electron not occurs.
[Ni(CN)4]2–, [Fe(CN)6]4, [Co(NH3)6]3+, no unpaired
electrons are present. Thus the coordination entry which
is paramagnetic is [NiCl4]2. Number of unpaired electron = 3
357. Which of the following metal oxides is anti- Hence, [Co(NH 3 )6 ]2+ shows paramagnetic behavior.
ferromagnetic in nature?
(a) MnO2 (b) TiO2 (c) [Ni(CN)4 ]2−
(c) VO2 (d) CrO2 CN− Act as strong field ligand and do pairing of metal
BITSAT-2006 d- electron.
Ans. (a) : Anti-ferromagnetic substances are those in
which magnetic moment of domains arranged in anti-
parallel direction due to which they have zero magnetic Number of unpaired electron = 0
moment. (d) [NiCl 4 ]2−
MnO 2 is an example of anti-ferromagnetic substance.
Cl− act as weak field ligand and pairing of metal d-
358. In which of the following complex compounds electron not occurs.
the oxidation number of the metal is zero?
(a) [Pt(NH3)2C12] (b) [Cr(CO)6]
(c) [Cr(NH3)3C13] (d) [Cr(en)2C12] Number of unpaired = 2
CG PET -2008 Hence, [Ni(Cl) 4 ]2− act shows paramagnetic behavior.
Ans. (b) : [Cr(CO)6 ] complex has zero oxidation of 361. Which of the following ligand possess only one
metal because carbonyl group (CO) is an neutral ligand. co-ordination site?
Let, oxidation state of metal = x (a) O2– (b) CO32−
x + 6 (0 ) = 0 (c) SO 24− (d) [ox]2–
x=0 GUJCET-2014
359. Coordination number of Fe in the complexes 2−
Ans. (a) : O is only one co-ordination site while
4− 3−
 Fe ( CN )6  ,  Fe ( CN )6  and [ FeC14 ]

[ox]2− is bidentate. CO32− and SO 24− ligand act as
would be respectively flexidentate ligand i.e. with transition metal it act as
(a) 6,3,4 (b) 6,6,4 mono-dentate ligand and with inner transition metal it
act as bidentate ligand.
(c) 2,3,3 (d) 6,4,6
362. The spin only magnetic moment of
CG PET -2007
[ CrF6 ] (atomic number for Cr is 24) is
4−
Ans. (b) : Coordination number is the number of
monodentate ligands attached with central metal atom. (a) 0 (b) 1.73 BM
4−
[Fe(CN)6 ] having six monodentate CN ligand attach – (c) 2.83 BM (d) 4.9 BM
with Fe. Then coordination number of this complex is 6. J & K CET-(2012)
[Fe(CN)6 ] also having coordination number is 6. Ans. (d) : F act as weak field ligand , ∆0 < P then
3− –

pairing of d-electrons are not possible.


[FeCl 4 ]− having 4 monodentate Cl− ligand attached
with Fe atom. That’s why it coordination number is 4.
360. Which of the following is paramagnetic?
2+ 2+
No. of unpaired electron = 4
(a)  Ni ( CO )4  (b) Co ( NH 3 )6  Magnetic moment = 4(4 + 2) = 24 = 4.9 B.M.
2−
(c)  Ni ( CN ) 4  [ NiC14 ]
2−
(d) 363. Specify the co-ordination number of cobalt in
[Co(CN)(H2O) (en)2]2+
CG PET- 2010 (a) 6 (b) 4
Ans. (b,d) : Paramagnetic substance having unpaired (c) 0 (d) 3
electron. J & K CET-(2013)
(a)[Ni(CO) 4 ]2+ , Ans. (a) : Ethylene dimaine act as bidentate ligand and
CO ligand act as strong field ligand which do pairing of has two donor site while CN– and H2O is monodentate
metal d- electrons. ligand.
Ni 2+ : d 8 Coordination number (C.No.) is the number of ligands
attached with central metal.
In [Co(CN)(H2O)(en)2]2+ complex coordination number
Number of unpaired electron = 0 is 6 ( 2 × 2 + 1 + 1)

Objective Chemistry Volume-II 573 YCT


364. Among the following, which one is
paramagnetic and has tetrahedral geometry?
No. of unpaired electron = 5
(a) [Ni(CN)4]2– (b) [NiCl4]2–
(c) [Ni(CO)4] (d) [CoCl2(en)2]+ Magnetic moment = 5 ( 5 + 2 ) = 35 = 5.92B.M.
J & K CET-(2011) 367. The ion of least magnetic moment among the
Ans. (b) : CN– and CO act as strong field ligand and do following is
pairing of electrons. (a) Ti3+ (b) Ni2+
[Ni(CN)4]2– and [Ni(CO)4] are low spin complex and (c) Co 2+
(d) Mn2+
have no unpaired electron. Therefore, both are
diamagnetic in nature. J & K CET-(2009)
In [CoCl2(en)2]+ , cobalt is present in +3 oxidation state Ans. (a) : When ion contains least number of unpaired
and with Co3+ ethylene diamine (en) act as strong field electron then its value of magnetic moment is also least.
ligand. ( ∆ o > p ) (a)
No. of unpaired electron = 1
Magnetic moment = 1(1 + 2 ) = 3 = 1.73 B.M.
No. of unpaired electron is zero so it behaves as
diamagnetic. (b)
[NiCl4]2− Cl− act as weak field ligand and pairing of No. of unpaired electron = 2
electron are not possible. Magnetic moment = 2.84 B.M.
Ni2+ :d8 (c)
This complex [NiCl4]2− has two unpaired electron so it No. of unpaired electron = 3
behaves as paramagnetic and its hybridization is sp3 Magnetic moment = 3.8 B.M.
therefore [NiCl4]2− is tetrahedral complex.
(d)
365. Which one of the following complex ions has
the highest magnetic moment? No. of unpaired electron = 5
(a) [Cr(NH3)6]3+ (b) [Fe(CN)6]3– Magnetic moment = 5.9 B.M.
4–
(c) [Fe(CN)6] (d) [Zn(NH3)6]2+ 368. The secondary valency of platinum in tetra
J & K CET-(2009) amminedichloroplatinum (IV) chloride is
Ans. (a) : When Complex having maximum number of (a) + 2 (b) 3
unpaired electron then its magnetic moment is also (c) 6 (d) +4
maximum. Karnataka-CET-2012
(a) Ans. (c) : The name of the compound is tetraammine
No. of unpaired electron = 3 dichloroplatinum (IV) chloride. The chemical formula
Magnetic moment = 3 ( 3 + 2 ) = 3.8 B.M. of the complex will be  Pt ( NH 3 )4 Cl 2  Cl2 . As we
know that secondary valency is satisfied with co-
(b) [Fe(CN)6]3−, CN act as strong field ligand
ordination number of the compound. Hence the co-
( ∆ o > p ) then pairing of electrons occur. ordination number of complex is 6 and thus the
Fe3+ : d5 secondary valency of the complex will be 6.
369. The spin only magnetic moment of Fe2+ ion (in
BM) is approximately
No. of unpaired electron = 1 (a) 4 (b) 7
Magnetic moment = 1.73 B.M. (c) 5 (d) 6
(c) [Fe(CN)6]2− ⇒Fe2+ : d6 Karnataka-CET-2011
Ans. (c) : Fe2+ — [Ar] 3d6
Magnetic moment = 0 B.M. d 6 = ↑↓ ↑ ↑ ↑ ↑
(d) [Zn(NH3)6]2+: Zn2+ = d10 4 unpaired e −
No. of unpaired electron = 0 It has 4 unpaired electrons
Magnetic moment = 0 B.M.
µ = n (n + 2) ∵ n= Number of unpaired electron
Hence, [Cr(NH3)6]2+ complex has highest magnetic
moment. = 4 (4 + 2)
366. The transition metal ion that has ‘spin-only’
magnetic moment value of 5.96 is = 24
(a) Mn 2+
(b) Fe 2+
µ = 4.89 BM ≈ 5BM
(c) V2+ (d) Cu2+ 370. The number of unidentate ligands in the
J & K CET-(2011) complex ion is called
Ans. (a) : When spin−only magnetic moment of (a) oxidation number (b) primary valency
complex is 5.96 B.M. the complex contains 5 unpaired (c) coordination number (d) EAN
electron. This is only possible in Mn2+ ion from the given . Karnataka-CET-2007
Objective Chemistry Volume-II 574 YCT
Ans. (c) : The number of unidentate ligand in the 372. The complex ion which has the highest
complex ion is called coordination number. magnetic moment among the following is
(a) [CoF6]3– (b) [Co(NH3)6]3+
371. (A) K4 [Fe (CN)6] (B) K3 [Cr (CN)6] 2+
(c) [Ni(NH3)4] (d) [Ni(CN)4]2–
(C) K3 [Co (CN)6] (D) K2 [Ni (CN)4] 4–
(e) [Fe(CN)6]
Select the complexes which are diamagnetic :
Kerala-CEE-2011
(a) (A), (B) and (C) (b) (B), (C) and (D)
Ans. (a) : The complex ion which have the maximum
(c) (A), (C) and (D) (d) (A), (B) and (D) number of unpaired electron shows the highest
(e) (B) and (D) magnetic moment.
Kerala-CEE-2005
[ CoF6 ]
−3
(a)
Ans. (c) : Diamagnetic character shows such type of
compound which has zero dipole moment. x + 6 ( −1) = −3
(a) K 4  Fe ( CN )6  x = +3
Let, x be the oxidation state of Fe.
∴ 4 (1) + x + 6 ( −1) = 0
Or x = +2
∵ CN is a strong field ligand due to which they
form inner orbital complex. Unpaired electron(n) = 4
∴ µs = 4 ( 4 + 2 ) = 4.89BM
3+
(b) Co ( NH 3 )6 
x + 6 ( O ) = +3
(b) K 3 Cr ( CN )6 
x = +3
3 (1) + x + 6 ( −1) = 0 ∵ Cobalt metal form low spin complex with NH 3
x = +3 and in this case the oxidation state of Co must be +3
∵ CN is a strong field ligand, it from the low spin
complex.

2+
(c)  Ni ( NH 3 ) 4 
(c) K 3 Co ( CN )6 
x + 4 ( 0 ) = +2
3 (1) + x + 6 ( −1) = 0 x = +2
x = +3
∵ CN is a strong field ligand, it form the low spin
complex.

∵ Unpaired electron ( n ) = 2
∴ µs = n ( n + 2 )
(d) K 2  M ( CN )4 
2 (1) + x + 4 ( −1) = 0 µs = 2 ( 2 + 2 ) = 8 BM
4−
x = +2 (d)  Fe ( CN )6 
∵ CN is a strong field ligand and it forms the
square planar complex x + 6 ( −1) = −4
x = +2
∵ CN is a strong field ligand and it forms the
inner orbital complex.

Hence, compound (A), (C) and (D) are diamagnetic in


nature.
Objective Chemistry Volume-II 575 YCT
∵ Unpaired electron (n) = 0 µs = 2.82 BM
∴ Magnetic moment ( µs ) = 0 (c) Fe3+ = [ Ar ] 3d 5
Hence, [ CoF6 ]
3−
has the highest magnetic moment
373. Molecules/ions and their magnetic properties
are given below Unpaired electron (n) = 5
Molecule/Ion Magnetic property µs = 5 ( 5 + 2 )
(i) C6H6 1. Antiferromagnetic
(ii) CrO2 2. Ferrimagnetic µs = 5.91 BM
(iii) MnO 3. Ferromagnetic (d) Ag + = [ Kr ] 4d10
(iv) Fe3O4 4. Paramagnetic
(v) Fe3+ 5. Diamagnetic
The correctly matched pairs in the above is Unpaired electron (n) = 0
(a) i-5 ii-3 iii-2 iv-1 v-4
∴ µs = OBM.
(b) i-3 ii-5 iii-1 iv-4 v-2
(c) i-5 ii-3 iii-1 iv-2 v-4 Thus, Fe3+ has maximum magnetic moment.
(d) i-5 ii-3 iii-1 iv-4 v-2 375. [Cr(H2O)6]Cl3 (At. no. of Cr = 24) has a
(e) i-4 ii-5 iii-1 iv-2 v-3 magnetic moment of 3.83 B.M. The correct
Kerala-CEE-2012 distribution of 3d electrons in the chromium of
Ans. (c) : (I) As we know that benzene is dimagnetic in the complex is
nature because it has zero unpaired electron. (a) 3d1xy ,3d1yz ,3d1z2 (b) 3d1x 2 − y2 ,3d1z2 ,3d1xz
( )
(II) CrO 2
1 1 1
(c) 3d xy ,3d x 2 − y2 ,3d yz (d) 3d xy ,3d1yz ,3d1xz
1
Cr 4+ = [ Ar ] 3d1 ( )
It has one unpair electron and it is a rare AIPMT-2006
example of ferromagnetic. Ans. (d) : [Cr (H2O)6]Cl3
(III) MnO compound is antiferromagnetic because Or [Cr (H2O)6]3+
the orientations of the electrons of Mn and O in MnO is Let, x be the oxidation state of Cr.
such that they cancel their effect and so the compound x + 6 (0) = +3
is antiferromagnetic. x = +3
(IV) Fe3O 4 is ferromagnetic in nature because ∴Cr3+ = [Ar]3d3
ferromagnetic materials have populations of atoms with Ground state −
opposite magnetic moments.
(V) Fe3+ = [ Ar ] 3d5 ( n = 5, Paramagnetic )
374. Which of the following species shows the Hence the correct option is (d).
maximum magnetic moment? 376. Which of the following complexes exhibits the
(a) Mn6+ (b) Ni2+ highest paramagnetic behaviour?
(c) Fe3+ (d) Ag+ (a) [Co(ox)2(OH)2]–
MHT CET-2012 (b) [Ti(NH3)6]3+
Ans. (c) : The metal ions which have maximum number (c) [V(gly)2(OH)2(NH3)2]+
of unpaired electron, having the maximum magnetic (d) [Fe(en)(bpy)(NH3)2]2+
moment. Where gly = glycine, en = ethylenediamine and
(a) Mn 6+ = [ Ar ] 3d1 bpy = bipyridyl moities. (At. no. Ti = 22, V =
23, Fe = 26, Co = 27)
(AIPMT -2008)

Unpaired electron (n) = 1 Ans. (a) : [Co(ox)2(OH)2] is octahedral complex. The
oxidation state of Co is −
µs = n ( n + 2 )
x+2(−2)+2(−1) = −1
µs = 1(1 + 2 ) x = +5
Co5+ = [Ar] 3d4
µs = 1.732BM
(b) Ni 2+ = [ Ar ] 3d8
Unpaired electron (n) = 4
• [Ti(NH3)6]3+
Unpaired electron (n) = 2 x + 6(O) = +3
x = +3
µs = 2 ( 2 + 2 )
Ti3+ = [Ar] 3d1

Objective Chemistry Volume-II 576 YCT


Unpaired electron (n) = 0
Unpaired electron (n) = 1 ∵ [Cr(NH3)6]3+ has the maximum no. of unpaired
• [V(gly)2(OH)2(NH3)2]+ electron.
x + 2(−1) + 2(−1) + 2(0) = +1 Hence, is it more paramagnetic in nature.
x = +5 378. Coordination number on Ni in [Ni(C2O4)3]4– is
V5+ = [Ar] 3do (a) 3 (b) 6
Unpaired electron (n) = 0 (c) 4 (d) 2
• [Fe(en) (bpy)(NH3)2]2+ VITEEE-2010
x + 1(0) +1(0) +2(0) = +2 Ans. (b) : [Ni(C2O4)3]4− contains the three bidentate
x = +2 ligand. The co-ordination number of the compound is 6.
Fe2+ = [Ar] 3d6 379. The coordination number and oxidation state
of central atom for [TiF9 ]6 − are respectively
Unpaired electron (n) = 4 (a) 9, 6 (b) 9, + 3
Since, [Co(ox)2(OH)2]− has the maximum no. of (c) 6, +3 (d) 3, + 6
unpaired electrons that’s why it has the highest UP CPMT-2014
paramagnetic character. Ans. (b) : Coordination no. = no. of monodentate ligand
Note:– [Fe(en)(bpy)(NH3)2]2+ also has 4 unpaired =9
electrons but it has one paired electrons. Hence, it has Oxidation state; x+ (–1) 9= –6

lower paramagnetic behavior than [Co(ox)2(OH)2] . x = –6 + 9 = +3
377. Which of the following complex compounds 380. Among [Ni(CO)4], [Ni(CN)4]2– and [Ni(Cl4)]2–
will exhibit highest paramagnetic behaviour? (a) [Ni(CO)4] and [Ni(Cl4)]2– are diamagnetic
(a) [Ti(NH3)6]3+ (b) [Cr(NH3)6]3+ and [Ni(CN)4]2– is paramagnetic
(c) [Co(NH3)6] 3+
(d) [Zn(NH3)6]2+ (b) [NiCl4]2– and [Ni(CN)4)]2– are diamagnetic
(At. No. Ti = 22, Cr = 24, Co = 27, Zn = 30) and [Ni(CO)4] is paramagnetic
(AIPMT -2011) (c) [Ni(CO)4] and [Ni(CN)4)]2– are diamagnetic
and [NiCl4]2– is paramagnetic
Ans. (b) : The compound having the more unpaired
(d) [Ni(CO)4] is diamagnetic and [Ni(CN)4)]2–
electrons, are exhibit highest paramagnetic behavior.
and [NiCl4]2– are paramagnetic
(a) [Ti(NH3)6]3+
UP CPMT-2013
x + 6(0) = +3
x = +3 Ans. (c) : (i) In [Ni(CO)4], Ni has zero oxidation state
and exist as Ni (O). Four ligands (CO) are attached to
Ti3+ = [Ar] 3d1 central metal atom Ni and require four orbitals.
Electronic configuration of Ni atom = 3d8, 4s2.
Unpaired electron (n) = 1
(b) [Cr(NH3)6]3+
x + 6(0) = +3 In presence of four CO ligands, which are strong
x = +3 ligands, the electrons pair up against Hund's rule.
Cr3+ = [Ar] 3d3 Ni atom in Ni(CO)4 = 3d10 4s0

Unpaired electron (n) = 3


(c) [Co(NH3)6]3+
x + 6(0) = +3
x = +3
∵ NH3 act as strong field ligand with Co3+.Thus
pairing of electron take place in inner orbital.
For sp3 hybridisation, orbitals are arranged
tetrahedrally. So it is tetrahedral complex. All the
electrons are paired so it is diamagnetic.
(ii) In [NiCl4]2–, Ni is in (II) oxidation state and its
electronic arrangement is as follows
Unpaired electron (n) = 0 Ni-atom : 3d8 4s2
(d) [Zn(NH3)6]2+
x + 6(0) = +2
x = +2
Zn2+ = [Ar] 3d10 Ni2+ ion : 3d8 4s0

Objective Chemistry Volume-II 577 YCT


Ti4+ = [Ar] 3d0
Unpaired electron (n) = 0, Diamagnetic
(d) K3[Cu(CN)6]
3(1) +x +6(−1) = 0
x = +3
Ni2+ ion in [NiCl4]2– Cu3+ = [Ar] 3d8
CN is the strong field ligand so the pairing of electron
occurred.

Geometry is tetrahedral and it will be paramagnetic in


nature because there are two unpaired electrons.
(iii) [Ni(CN)4]2– Ni is in (II) oxidation state and Unpaired electron (n) = 0, Diamagnetic.
electronic arrangement is as follow 382. Amongst [Ni(H2O)6]2+, [NiCl4]2-

8
Ni-atom : 3d 4s 2 Ni(PPh3)2Cl2],[Ni(CO)4] and [Ni(CN4)]2 the
paramagnetic species are
(a) [NiCl4]2-, [Ni(H2O)6]2+, [Ni(PPh3)2Cl2]
(b) [Ni(CO)4],[Ni(PPh3)2Cl2], [NiCl4]2-
Ni2+ ion : 3d8 4s0 (c) [Ni(CN)4]2-, [Ni(H2O)6]2+, [NiCl4]2-
(d) [Ni(PPh3)2Cl2],[Ni(CO)4],[Ni(CN)4]2-
WB-JEE-2013
2+
Ans. (a) : [Ni(H 2 O) 6 ]
Ni2+ ion in [Ni(CN)4]2– x +6(0) = +2
x = +2
Ni2+ = [Ar] 3d8

Unpaired electron (n) = 2, paramagnetic


[Ni(PPh3)2Cl2]
x +2(0) +2(−1) = 0
CN– is strong ligand so it pairs up 3d-electrons against x = +2
Hund's rule. Due to dsp2 hybridisation, the geometry of Ni2+ = [Ar] 3d8
[Ni(CN)4]2– is square planar and the complex is
diamagnetic because all the electrons are paired.
381. Among the following compounds both coloured Unpaired electron (n) = 2, paramagnetic
and paramagnetic one is [Ni(CO)4]
(a) K2Cr2O7 (b) VOSO4 x +4(0) = 0
(c) (NH4)2[TiCl6] (d) K3[Cu(CN)4] x=0
UPTU/UPSEE-2013 Ni0 = [Ar] 3d8 4s2 4p0
Ans. (b) : The complex compound shows the colour
because of availability of unpaired electron due to
which transition of electron take place.
(a) K2Cr2O7
2(1) +2(x) +7(−2) = 0 Unpaired electron (n) = 0, Diamagnetic
x = +6 [Ni(CN)4]2−
Cr6+ = [Ar] 3d0, diamagnetic x +4(−1) = −2
(b) VOSO4 x = +2
x + 1(−2)+1(6)+4(−2) = 0
Ni2+ = [Ar] 3d8 (∵ pairing occurred because
x −2+6−8 = 0
CN is strong field ligand or SFL)
V=4
4+ 1
V = [Ar] 3d
Unpaired electron (n) = 1, Paramagnetic
(c) (NH4)2 [TiCl6]
2(1) +x +6(−1) = 0
x = +4 Unpaired electron (n) = 0, Diamagnetic

Objective Chemistry Volume-II 578 YCT


4. Hybridisation
383. Match List-I with List-II
List-I List-II
(Complex) (Hybridization)
A. Ni(CO)4 I. sp3
B. [Ni (CN)4]2– II. sp3d2
3–
C. [Co (CN)6] III. d2sp3
D. [CoF6]3– IV. dsp2
Choose the correct answer from the options
given below: The hybridization of metals are d2sp3 and sp3d2.
(a) A-IV, B-I, C-III, D-II 385. Which of the following has square planar
(b) A-I, B-IV, C-III, D-II structure ?
(a) [Ni(CO)4] (b) [NiCl4]2–
(c) A-I, B-IV, C-II, D-III 2–
(c) [Ni(CN)4] (d) [Ni(NH3)6]2+
(d) A-IV, B-I, C-II, D-III
Jharkhand-2019
JEE Main 28.07.2022, Shift-II
J & K CET-(2019, 2013, 2002)
Ans. (b) : [BITSAT-2017], BCECE-2007
List − I(Complex) List − II(Hybridisation) Ans. (c) : • [Ni(CN)4]2– has square planar structure–
A Ni(CO) 4 I sp3
[ Ni(CN)4 ]
2–
B IV dsp 2

[ Co(CN)6 ]
3–
C III d 2sp3 • Ni(CO)4 is tetrahedral structure.
• [NiCl4]2– is tetrahedral structure.
[CoF6 ]
3− 3 2
D II sp d
• [Ni(NH3)6]2+ is octahedral structure.
384. The hybridization of metals involved in the 386. Among the complexes of Ni listed below, the
following complexes, respectively are following is the only correct statement.
3− 3− I [Ni(CO)4]
[Mn(CN)6 ] ,[CoF6 ]
II [Ni(CN)4]2–
(a) sp3d2, sp3d2 (b) sp3d2, d2sp3 III [NiCl4]2–
2 3 2 3 2 3 3 2
(c) d sp , d sp (d) d sp , sp d (a) All of them have tetrahedral geometry.
AP-EAMCET-07.07.2022, Shift-II (b) I and III are tetrahedral and II has square
Ans. (d) : The complex compounds are- planar geometry.
(c) I and II are tetrahedral and III is square planar.
[Mn(CN)6]3–
(d) All of them have square planar geometry.
Let, x be the oxidation state of Mn COMEDK-2015, AMU-2011
x + 6(–1) = –3 (AIPMT -2004)
x = +3 Ans. (b) :
Mn3+ = [Ar]3d4 (Paring occure because CN is SFL)
Ground state- Compound Oxidation hybridisation
number
(I) Ni(CO)4 0 sp 3 – Tetrahedral

(II) [Ni(CN)4]2– +2 dsp2–


Excited state- squareplaner
(III) [Ni(Cl)4]2– +2 sp3–tetrahedral
387. The number of unpaired electrons are present
in [CoF6 ]3 − is
(a) 4 (b) 0
[CoF6]3– = x + 6 (–1) = –3 (c) 1 (d) 3
x = +3 TS EAMCET 10.08.2021, Shift-II
Co3+ = [Ar] 3d6 (AIPMT -2003)
Objective Chemistry Volume-II 579 YCT
Ans. (a) : The compound is – 2−
Ans. (b) : Hybridisation of Ni atom in  Ni ( CN )4  and
[ ]
3–
CoF6 Ni (CO)4 are dsp2 and sp3 respectively. In [Ni(CN4)]2–,
there is Ni2+ ion for which the electronic configuration in
Co +3 = 1s 2 2s 2 2p 6 3s 2 3p 6 3d 6 the valence shell is 3d84s0. In presence of strong field
CN– ions, all the electrons are paired up. The empty
orbitals (One 3d, one 4s and two 4p orbitals) undergo
dsp2 hybridization to make bonds with four CN ligands
which result in square planar geometry. Since all the
electrons are paired, [Ni(CN4)]2– is diamagnetic. In
2-
388. The hybridisation of [Ni(CN)4] and [Ni(CO)4], the valence shell electronic configuration of
[Ni(NH3)6]4+ ions are respectively. ground state Ni atom is 3d84s2. All of these 10 electrons
(a) sp3d2, sp3 (b) dsp2, sp3d2 one pushed into 3d orbitals and get paired up when strong
(c) dsp2, d2sp3 (d) d2 spd2, sp3 field CO ligands approach Ni atom. The empty 4s and
three 4p orbitals are undergo sp3 hybridization and form
CG PET -2019, AIPMT-2015, 2004 bonds with four CO ligands to give Ni(CO) with
4
Ans. (c) : [Ni(CN) 4 ]2 tetrahedral geometry, since all the electrons are paired,
Let, x be the oxidation state of Ni Ni(CO4) is diamagnetic.
x + 4(−1) = −2 390. The hybridization and magnetic nature of
Mn ( CN )6  , respectively are :
4–
Pairing of electron occurred because CN is SFL  
Ni 2+ = [ Ar ]3d8 4s 0 4p0 (a) d2sp3 and paramagnetic
(b) sp3d and paramagnetic
(c) d2sp3 and diamagnetic
(d) sp3d2 and diamagnetic
JEE Main 25-02-2021, Shift-I
Ans. (a) : [Mn(CN)6]4–
Let, O.N. of Mn = x
∴ x + (–1) × 6 = – 4
x – 6 = –4
x = +2
Mn2+ : [Ar] 3d5
Since, co-ordination no. is 6. So it is an octahedral
Hence, hybridization of [Ni(CN) 4 ]2− is dsp2 . complex and CN– is a strong field ligand so
[Ni(NH 3 )6 ]4+ hybridization will be d2sp3 ie., inner orbital complex
and low spin.
x + 6 ( 0 ) = +4
x = +4 2+ 5
Mn : [Ar] 3d
Ni 4+ = [ Ar ]3d 6 4s 0 4p0 (Pairing occur because Ni is in
+4 oxidation state).
391. What is the hybridization present in the
3+
complex Co ( NH 3 )6 
(a) d2sp3 (b) sp3d2
3
(c) dsp (d) sp3d
TS-EAMCET 09.08.2021, Shift-I
Ans. (a) : Given complex : [Co(NH3)6]3+
Oxidation state of metal = +3
Geometry = octahedral
Electronic configuration of Co3+ = [Ar]3d6
The hybridization of [Ni(CN) 4 ]2− and [Ni(NH 3 )6 ]4+ are
dsp2 and d2sp3 respectively.
389. Both Ni(CO)4 and [Ni(CN)4]2 and diamagnetic.
The hybridization of Nickel in these complexes
are respectively
(a) sp3, sp3 (b) sp3, dsp2
2
(c) dsp , sp 3
(d) dsp2, dsp2 [Co(NH3)6]+3 = d2sp3 Hybridized (Octahedral and
Assam CEE-2018, GUJCET-2016, 2015 diamagnetic]
Objective Chemistry Volume-II 580 YCT
392. The hybridisation and magnetic nature of Ni 2+ = [ Ar ]3d8 4s 0 4p0
[Mn(CN)6]4– and [Fe(CN)6]3–, respectively are
(a) d2sp3 and paramagnetic
(b) sp2d2 and diamagnetic
(c) d2sp3 and diamagnetic
(d) sp3d2 and paramagnetic
[JEE Main 2021, 25 Feb Shift-I]
Ans. (a) : [Mn(CN)6 ]4−
x + 6(−1) = −4
x = +2 394. The type of hybridisation and magnetic
property of the complex[MnCl6]3-, respectively,
Mn 2+ = [ Ar ]3d 5 (Pairing of electron occur because CN are
is SFL). (a) sp3d2 and diamagnetic
(b) d2sp3 and diamagnetic
(c) d2sp3 and paramagnetic
(d) sp3d2 and paramagnetic
[JEE Main 2021, 27 July Shift-I]
Ans. (d) : [MnCl6 ]3−
Let, x be the oxidation state of Mn.
x + 6(−1) = −3
x = +3
Mn 3+ = [ Ar ]3d 5

There is one unpaired electron.


Hence, [Mn(CN)6 ]4− is paramagnetic
And, [Fe(CN)6 ]3−
x + 6(−1) = −3
x = +3
Fe3+ = [ Ar ]3d 5
The same method also apply for [Fe(CN)6 ]3− because it
contain [ Ar ]3d 5 electronic configuration which is It contains the five unpaired electron hence this is a
similar to [Mn(CN)6 ]4− paramagnetic complex.
2 3
Thus, both are paramagnetic with d sp hybridization. 395. The hybridizations of N-orbitals
- - +
in NO 3 , NO 2 and NH 4 respectively are _____
393. According to the valence bond theory the
2
hybridisation of central metal atom is dsp for (a) sp2, sp2, sp3 (b) sp, sp3, sp2
2 3
which one of the following compound? (c) sp, sp , sp (d) sp2, sp, sp3
(a) NiCl2.6H2O (b) K2[Ni(CN)4] AP EAPCET 23-08-2021 Shift-I
(c) [Ni(CO)4] (d) Na2[NiCl4] Ans. (a) : Given molecules–
[JEE Main 2021, 20 July Shift-I]
Ans. (b) : The hybridization of central atom is dsp2 that
means the complex is square planar in geometry.
K 2 [Ni(CN) 4 ] is only complex which will form the
square planar geometry because it has the strong field
ligand due to which pairing of electron occurred in ∵ Hybridization = Lone pair + Bond pair
inner orbitals.
K 2 [Ni(CN) 4 ] ∴ Hybridization in NO3− = 0 + 3 = 3 (sp2)

2(+1) + x + 4(−1) = 0 Hybridization in NO −2 = 1 + 2 = 3 (sp2)

x = +2 Hybridization in NH 4+ = 0 + 4 = 4 (sp3)

Objective Chemistry Volume-II 581 YCT


396. Match the following ions with the 398. Which of the following outer octahedral
corresponding hybridization of their central complexes have same number of unpaired
atoms? electrons ?
Ions Hybridization 1. (MnCl6)3– 2. (FeF6)3–
3 3–
(i) NO3− (a) sp 3. (CoF6) 4. [Ni(NH3)6]2+
(a) 1 and 3 (b) 1 and 2
(ii) NH −2 (b) sp
(c) 3 and 4 (d) 2 and 3
(iii) SCN − (c) sp3d AP EAMCET (Engg.) 17.09.2020, Shift-II
(d) sp 2 Ans. (a) : All four complexes given are outer orbital or
(iv) ICl3 high spin octahedral complexes. So, d-orbital electrons
(a) (i – d), (ii – a), (iii – b), (iv – c) of central metal ions of the complexes remain
(b) (i – c), (ii – b), (iii – a), (iv – d) unaffected by weak field ligands.
(c) (i – a), (ii – d), (iii – c), (iv – b) 3−
 III 
(d) (i – b), (ii – c), (iii – d), (iv – a) 1.  M nCl 6 → Mn 3+ ⇒ d 4 , n = 4

AP-EAMCET 25-08-2021 Shift - I
[ ∵ n = Number of unpaired electrons]
Ans. (a) : 3−
Ions Hybridization  III 
2.  FeF6  → Fe3+ ⇒ d 5 ,n = 5
(i) NO −  
3
3−
 III 
(d) 3. CoF6  → Co3+ ⇒ d 6 , n = 4
 
2+
sp2
4.  Ni ( NH 3 )6  → Ni 2+ ⇒ d8 , n = 2
II

(ii) NH −2  
(a) So, complexes 1 and 3 have same number of unpaired
sp3 electrons (n = 4).
S–C≡N– 399. The ion that has sp3d2 hybridization for the
(iii) SCN − (b)
central atom,
sp
(a) [ICl4]– (b) [ICl2]–
(iv) ICl3 –
(c) [IF6] (d) [BrF2]–
(c) AIIMS 25 May 2019 (Morning)
Ans. (a) : (a) [ICl4 ]−
sp3d Hybridization = Bond pair + lone pair
397. Match the following based on valence bond = 4+2
theory? = 6(sp 3d 2 )
Hybridi- Geometry Complex
zation structure (b) [ICl2 ]−
(A) sp3 (i) Square (p) [Fe(CN)6]3– Hybridization = 2 + 3
planar = 5(sp3d )
(B) d2sp3 (ii) Tetrahedral (q) [ZnCl4]2– (c) [IF6 ]−
(C) dsp2 (iii) (r) [ Ni (NH3)4]2+ Hybridization = 6 + 1
Octahedral
= 7 (sp 3d 3 )
(s) [Ag(CN)2]–
(a) (A-ii-q), (B-iii-p), (C-i-r) (d) [BrF2 ]−
(b) (A-ii-q), (B-iii-r), (C-i-s) Hybridization = 2 + 3
(c) (A-i-q), (B-iii-p), (C-ii-r) = 5(sp3d )
(d) (A-ii-r), (B-iii-s), (C-i-q)
3 2
AP EAPCET 25.08.2021, Shift-II Hence, [ICl4 ]− has the sp d hybridization.
Ans. (a) : 400. Which of the following system has maximum
Hybridization Geometry Complex Structure number of unpaired electrons?
sp3 Tetrahedral [ZnCl4]2– (a) d6 (tetrahedral, high spin)
d2sp3 Octahedral [Fe(CN)6]3– (b) d9 (octahedral)
dsp2 Square planar [ Ni (NH3)4]2+ (c) d4 (octahedral, low spin)
Note: [Ag(CN)2]– has linear compound and (d) d7 (octahedral, high spin)
hybridization is sp. AMU-2019
Objective Chemistry Volume-II 582 YCT
Ans. (a)

Hence, the complexes [Ni(CO) 4 ] and [Ni(CN) 4 ]2− are


tetrahedral and square planar respectively
402. According to Werner's theory the geometry of
the complex is determined by
(a) only from the primary valence in space
(b) number and position of the primary valences
in space
(c) number and position of the secondary valency
(d) only from the position of secondary valence
in space
Hence, d6 contains the maximum number of electron. MHT CET-02.05.2019, SHIFT-II
401. The complexes [Ni(CO)4] and [Ni(CN)4]2– are, Ans. (c) : According to the Werner’s theory the primary
respectively valency is satisfy by the oxidation state of the complex
whereas secondary valency is satisfy by the co-
(a) tetrahedral and square planar ordination number of the complex. Thus the geometry
(b) tetrahedral and tetrahedral of the complex is determined by number and position of
(c) square planar and square planar the secondary valency.
(d) square planar and tetrahedral. 403. Aluminium chloride in acidified aqueous
AMU-2019 solution forms a complex 'A', in which
hybridisation state of Al is 'B'. What are 'A'
Ans. (a) : [Ni(CO) 4 ] and 'B', respectively?
(a) [Al(H2O)6]3+, sp3d2 (b) [Al(H2O)4]3+, sp3
x + 4 ( 0) = 0
(c) [Al(H2O)4]3+, dsp2 (d) [Al(H2O)6]3+, d2sp3
x=0 (Odisha NEET 2019)
Pairing of electron takes place because CO is SFL. Ans. (a) : Alumunium chloride is a salt and when
treated with water forms a complex. The complex
Ni = [ Ar ] 3d 8 4s 2 4p 0
formed will be [Al(H 2O)6 ]3+ . The oxidation state of Al
is +3. The hybridization of Al in complex is sp 3d 2 .
404. Regarding [Ni(CN)4]2– complex ion which of
the following statements is true?
(a) Ni+2 ion is sp3-hybridized and complex ion is
paramagnetic.
(b) Ni+2 ion is dsp2-hybridized and complex ion is
diamagnetic
(c) Ni+2 ion is sp3-hybridized and complex ion is
diamagnetic
(d) Ni+2 ion is dsp2-hybridized and complex ion
is paramagnetic.
Tripura JEE-2019
2−
And, [Ni(CN) 4 ]2− Ans. (b) : (Ni(CN)4 ]
Let, x be the oxidation state of Ni.
x + 4 (−1) = −2
x + 4 (−1) = −2
x = +2
x = +2
Pairing of electron takes place because CN is SFL. 2+
Ni = [ Ar ]3d 8 (Pairing of electron occur because CN
Ni 2+ = [ Ar ]3d 8 4s 0 4p 0 is SFL).

Objective Chemistry Volume-II 583 YCT


Ans. (d) : K 3 [Fe(CN)6 ]
Let, x be the oxidation state of Fe.
3(+1) + x + 6 (−1) = 0
x = +3
Fe3+ = [ Ar ]3d 5 (∵ Pairing occur becauseCN is SFL)

Unpaired e− = 0 (diamagnetic)
Hybridization = dsp2 (square planar)
405. Which of the given options are correct for
[Fe(CN)6]3– complex?
(a) d2sp2 Hybridization (b) sp3d2 Hybridization
(c) Paramagnetic (d) Diamagnetic 2 3
HP CET-2018 Hence, the hybridization of K 3 [Fe(CN)6 ] is d sp .
Ans. (c) : [Fe(CN)6 ]3− 408. The number of d-electrons in [Cr (H2O)6]3+ is
(a) 2 (b) 3
x + 6 (−1) = −3 (c) 4 (d) 5
x = +3 MPPET- 2009
Fe3+ = [ Ar ]3d 5 (Pairing occur because CN is SFL) Ans. (b) : [Cr (H2O)6]3+
The oxidation state of Cr is +3.
Cr = [Ar] 4s1 3d5
Cr+3 = [Ar ] 4s0 3d3
∴ The d-electron in [Cr(H2O)4]3+ is 3.
409. Among [Fe(H2O)6]3+, [Fe(CN)6]3-, [Fe(Cl)6]3-
There is one unpaired electron present. species, the hybridization state of the Fe atom
Hence, complex ion is paramagnetic. are, respectively.
406. Which of the following is the correct (a) d 2sp3 ,d 2sp3 ,sp3d 2 (b) sp 3d 2 ,d 2sp3 ,d 2sp3
configuration of the complex [RhCl6]3–?
(a) High spin t2g 4e2g (b) Low spin t2g 6e0g (c) sp 3d 2 ,d 2sp3 ,sp3d 2 (d) None of the above
3 3 5 1
(c) Low spin t2g e g (d) High spin t2g e g VITEEE-2014
J & K CET-(2018) Ans. (c) :
Ans. (b) : [RhCl6 ]3−
Let, x be the oxidation state of Rh.
x + 6 (−1) = −3
x = +3
Rh = [ Kr ] 4d 7 5s 2
Rh 3+ = [ Kr ]4d 6 ( ∵ Pairing occur because Rh belongs
to 4d series)

410. Which one of the following complexes is not


It forms the low spin complex because electron expected to exhibit isomerism?
6 0 (a) [Ni(NH3)4(H2O)2]2+ (b) [Pt(NH3)2Cl2]
fill with forcefully in t2g orbital i.e. low spin t t 2 g eg
(c) [Ni(NH3)2Cl2] (d) [Ni(en)3]2+
407. The hybridization of Fe in K3Fe(CN)6 is VITEEE- 2012
(a) sp3 (b) dsp3 Ans. (c) : Compounds having tetrahedral geometry does
(c) sp3d2 (d) d2sp3 not exhibit isomerism due to presence of symmetry
[BITSAT-2016] elements. Here, [Ni(NH3)2Cl2] has tetrahedral geometry.

Objective Chemistry Volume-II 584 YCT


411. The number of unpaired electrons calculated in Ans. (c) : The complex is [PtCl3 (C 2 H 4 )]− . The
3−
Co ( NH 3 )6  and Co ( F6 ) 
3+
platinum metal connected with the four ligand. The
complex formed the square planar geometry because Pt
(a) 4 and 4 (b) 0 and 2
belongs to the 5d series.
(c) 2 and 4 (d) 0 and 4
VITEEE- 2008 [Pt Cl3 (C 2 H 4 )]−
3+ 3− x + 3(−1) + 0 = −1
Ans. (d) : In Co ( NH 3 )6  and Co ( F6 )  both the
oxidation state of cobalt ion is +3. In first case NH3 is x = +2
2+
the neutral ligand which is a strong field ligand. Pt = [ Xe]5d8 6s 0 6p 0 (Pairing take place)
Hence, the electrons in Co (+3) i.e. 4s03d6get paired to
form inner orbital complex.
Hence, no unpaired electron.
On the other hand F– is a weak field ligand hence it
forms an outer orbital complex with 4 unpaired
electrons.
412. In TeCl4 the central atom tellurium involves
(a) sp3 hybridization (b) sp3d hybridization
(c) sp d hybridization (d) dsp2 hybridization
3 2

VITEEE- 2007
1 2
Ans. (b) : Hybridisation = [Number of valence Hence, shape of compound is square planar with dsp
2 hybridization.
electrons of central atom + no. of monovalent atoms
415. In [NiCl4]2-, the type of hybridization for Ni is
attached to it + negative charge if any – positive charge
if any] (a) sp3d2 (b) dsp3
3
(c) sp (d) dsp2
1
= [ 6 + 4 + 0 − 0] = 5 = sp d 3
AMU-2012
2 Ans. (c) : For the complex–
∵ Te ( 52 ) = [ Kr ] 4d10 5s 2 5p 4 
[NiCl4 ]2−
413. The hybridization of nickel in nickel Let, x be the oxidation state of Ni
tetracarbonyl is ∴ x + 4(−1) = −2
(a) sp3 (b) sp2
(c) dsp 2 2 3
(d) d sp x = +2
COMEDK 2011 Ni 2+ = [ Ar ]3d8
Ans. (a) : The chemical formula of nickeltetracarbonyl Cl is a weak field ligand thus the pairing is not occur.
is  Ni ( CO )4  . The oxidation number of the complex is
0.
28 Ni = [ Ar ] 3d 4s
8 2

Hence, the hybridization of [Ni(Cl) 4 ]2− is sp3.


416. Which of the following is an outer orbital
complex ?
(a) [Cr(NH3)6]3+ (b) [Ni(NH3)6]2+
3–
(c) [Fe(CN)5] (d) [Mn(CN)6]4–
AMU – 2010
Ans. (b) :
414. The shape of [PtCl3(C2H4)]– and the
hybridisation of Pt respectively are
(a) tetrahedral, sp3 [Cr(NH3)6]3+:
(b) trigonal pyramidal, sp3
(c) square planar, dsp2 [Ni(NH3)6]2+:
(d) square planar, d2sp3
UPTU/UPSEE-2013
Objective Chemistry Volume-II 585 YCT
[Fe(CN)6]3– : Select the correct answer using the codes given
below:
(a) 1 and 2 (b) 1 and 3
[Mn(CN)6]4– : (c) 2 and 4 (d) 1, 3 and 4
(e) 2, 3 and 4
when outer 4d orbitals are involved in hybridisation, it Kerala-CEE-2006
is called an outer orbital complex. Ans. (e) :
417. Which of the following has square planar
geometry ?
(a) [PtCl4]2– (b) [NiCl4]2–
2–
(c) [ZnCl4] (d) [CoCl4]2–
[BITSAT-2010]
Ans. (a) : All the compounds have four co-ordination
number but [PtCl 4 ]2− which contains the 5d-series metal
rather than the other compound. Thus [PtCl 4 ]2− form
the square planar complex. Hence, compound (2), (3) and (4) are not correctly
418. The number of unpaired electrons in the matched.
square planar [Pt(CN)4]2– ion are 421. In Cu-ammonia complex, the state of
(a) 2 (b) 1 hybridization of Cu2+ is
(c) 0 (d) 3 (a) sp3 (b) d3s
J & K CET-(2008) 2
(c) sp f (d) dsp2
2−
Ans. (c) : [Pt(CN) 4 ] WB-JEE-2009
Let, x be the oxidation state of Pt. Ans. (d) : Copper form complex with NH3 in +2 and +3
x + 4(−1) = −2 oxidation state. Cu 2+ make the square planar complex
x = +2 with NH3 because NH3 act as strong field ligand.
2+ 8
Pt = [Xe] 5d (Pairing occur because Pt belongs to Cu 2+ = [ Ar ]3d 9 4s° 4p°
the 5d series metal)

Hence, the hybridization of Cu 2+ is dsp 2 .


There is no unpaired electron that’s why it is
diamagnetic.
419. Potassium ferrocyanide is an example of 5. Spectrochemical Series,
(a) tetrahedral (b) octahedral
(c) square planar (d) linear
Complex Stability
J & K CET-(2007) 3+
Ans. (b) : The chemical formula of potassium 422. Fe cation gives a prussian blue precipitate on
ferrocyanide is K 4 [Fe(CN)6 ] . Here, the central metal addition of potassium ferrocyanide solution
due to the formation of:
attached with the six cyanide ligand, hence it form the
octahedral geometry. (a) [Fe(H2O)6]2 [Fe(CN)6] (b) Fe2[Fe(CN)6]2
(c) Fe3[Fe(OH)2(CN)4]2 (d) Fe4[Fe(CN)6]3
420. Which of the following complexes are not
correctly matched with hybridization of their JEE Main 27.07.2022, Shift-II
central metal ion? Ans. (d) : The reaction is–
(1) [Ni(CO)4] sp3 4Fe3+ + 3[Fe(CN)6 ]  → Fe 4 [Fe(CN)6 ]3
2– 3
(2) [Ni(CN4)] sp Potassium
3– 2 3
(3) [CoF6] d sp Ferrocynide
(4) [Fe(CN)6]3− sp3d2 (Pr ussian Blue)

Objective Chemistry Volume-II 586 YCT


423. The correct order about the number of Ans. (d) : [Ni(NH3)6]2+ is an outer orbital complex
unpaired electrons present in the following because nd orbital is used in hybridisation.
complexes is Oxidation of Ni in [Ni(NH3)6]2+ complex is +2
[Fe(CN)6]4- [Fe(H2O)6]2+ [Co(H2O)6]2+ Ni2+:d8
I II III
(a) II > III > I (b) II > I > III
(c) I > II > III (d) III > II > I
AP-EAMCET-06.07.2022, Shift-II
Ans. (a) : (i) [Fe(CN)6]4– = 26Fe2+ = [Ar] 4s2 3d6
4s0, 3d6

CN is strong field ligands so it pair d orbital electron


Hence, it is an outer orbital complex.
427. The most stable ion is
No unpaired electron
(a) [Fe(OH)3]3– (b) [FeCl6]3–
(ii) [Fe(H2O)6]2+ = 26Fe2+ = [Ar] 4s2, 3d6 3–
(c) [Fe(CN)6] (d) [Fe(H2O)6]3+
4s0, 3d6
[BITSAT-2011], AIEEE 2002
Ans. (c) : [Fe(CN)6]3– Complex forms stable ion
No. of unpaired electron = 4 because it used inner d-orbital (n–1) d for formation of
(iii) [CO(H2O)6]2+ = 27CO2+ = [Ar] 4s2, 3d7 complex.
4s0, 3d7 In [Fe (CN)6]3– complex, Fe is present in +3 oxidation
state. CN– is a strong field ligand and do pairing of
No. of unpaired electron = 3 metal d– electrons.
So, maximum number of unpaired in II > III > I Here complex formed by using (n–1) d orbital. So the
complex is called inner orbital complex.
424. An element 'X' with atomic number 13 forms a Fe3+ : d5
2-
complex of the type  XCl ( H 2 O )5  . The
covalency and oxidation state of X in it are
respectively
(a) 5,+2 (b) 6,+2
(c) 5,+3 (d) 6,+3
AP-EAMCET-05.07.2022, Shift-I
Ans. (d) : For the complex, [XCl(H2O)5]2+
Let, oxidation state of X is x
∴ x + (–1) + 5(0) = 2 428. The correct representation of complex ion is
2+
x=+3 (a) Co ( H 2 O )( NH 3 )4 Cl 
Hence, the oxidation state of element (X) is +3. The
2+
element (X) has the following electronic configuration – (b) CoCI ( H 2 O )( NH 3 ) 4 
X = 2, 8, 3
2+
Thus, the covalence of the element is 6 because it (c) Co ( NH 3 )4 CI ( H 2 O ) 
acquire the stable configuration by six bond. 2+
425. The correct order of decreasing field strength (d) Co ( NH 3 ) 4 ( H 3O ) Cl 
of the below given ligands is Manipal-2020, CG PET- 2015
NCS− S2 − en SCN − Ans. (d) : IUPAC naming of complex is done in
I II III IV alphabetical order of ligands. Then it should be written
(a) I > II > IV > III (b) III > II > IV > I in alphabetical order.
(c) III > I > IV > II (d) III > IV > I > II IUPAC name of complex is tetraamine aqua chloro
cobalt (II)
TS-EAMCET-18.07.2022, Shift-I
i.e., [Co (NH3)4(H2O) Cl]2+
Ans. (c) : According to the petrochemical series, the
decreasing order of ligand field will be – 429. Which of the following spectrochemical series
is true?
en ( III ) > NCS ( I ) > SCN (IV) > S (II)
− − 2−
(a) SCN − < NH 3 < F− < en < CO
426. Which one of the following complexes is outer
orbital complex? (b) SCN − < F− < NH 3 < en < CO
(a) [Co(NH3)6]3+ (b) [Mn(CN)6]4- (c) SCN − < F− < en < NH 3 < CO
4- 2+
(c) [Fe(CN)6] (d) [Ni(NH3)6]
(d) SCN − < F− < en < CO < NH 3
Manipal-2020, 2017, UPTU/UPSEE-2012
JIPMER-2011 GUJCET-2016, 2015
Objective Chemistry Volume-II 587 YCT
Ans. (c) : Correct spectrochemical series is :-
SCN– < F–< en < NH3 < CO
On moving left to right in above series strength of
ligand increase and that’s why CO act as strong field
ligand.
430. Homoleptic complex among the following are Hence, two equivalents of ethylene diamine required to
replace the neutral ligand (NH3).
(A) K3 [Al(C2O4)3] (B) [CoCl2(en)2]+
433. Which one of the following metal complexes is
(C) K2[Zn(OH)4]
most stable?
(a) A only (b) A and B only
(a) [Co(en)(NH3)4]Cl2 (b) [Co(en)3]Cl2
(c) A and C only (d) C only (c) [Co(en)2(NH3)2]Cl2 (d) [Co(NH3)6]Cl2
Karnataka-CET-2021 [JEE Main 2021, 25 July Shift-II]
Kerala-CEE-29.08.2021 Ans. (b) : As the given complexes of cobalt, cobalt
Ans. (c) : Homoleptic complex contains only are type having the same +2 oxidation state. But [Co(en)3]Cl2
of ligands which is directly attached with metal. complex is most stable due to presence of more number
Hence, K3[Al(C2O4)3] and K2[Zn(OH)4] complex of chelating ligands.
contains only one type of lignad C2O42– and OH– ion If chelating ligand is present in complex then stability
respectively. Therefore these complexes are known as of complex will be increased.
homoleptic complex. Hence [Co(en)3]Cl2 has 3 chelating ligand ethylene
431. The correct pair(s) of the ambident diamine so it has most stable metal complex.
nucleophiles is (are) 434. Three moles of AgCl get precipitated when one
(A) AgCN/KCN (B) RCOOAg/RCOOK mole of an octahedral co-ordination compound
(C) AgNO2/KNO2 (D) AgI/KI with empirical formula CrCl3. 3NH3. 3H2O
reacts with excess of silver nitrate. The number
(a) B and C (b) Only A
of chloride ions satisfying the secondary
(c) A and C (d) Only B valency of the metal ion is………
[JEE Main 2021, 17 March Shift-II] [JEE Main 2021, 25 July Shift-I]
Ans. (c) : Ambidentate ligands- Those ligands which Ans. (0) : When Cl– is present outside the coordination
have two different donor atoms but only one donor sphere then it reacts with AgNO3 and forms precipitate
atom is attached to the metal. These ligands are called of AgCl. For formation of 3 moles of AgCl. 3Cl– ligand
ambidentate ligands. should be present outside the coordination sphere.
For example:- AgCN is covalent and KCN is ionic Cr ( NH 3 )3 ( H 2 O )3  Cl3 + AgCl 
→ 3AgCl ↓
compound. (ppt)
KCN  → K + + CN − Zero chloride ion satisfying the secondary valency i.e.,
M ←  CN ⇒ Cyanido no chloride ion will directly attached with metal ion.
AgCN  → Ag. + CN. 435. An organic compound is subjected to
chlorination to get compound A using 5.0 g of
M ←  NC ⇒ Isocyanido chlorine. When 0.5 g of compound A is reacted
CN has two donor atoms N and C but at a time only one with AgNO3[Carius method], the percentage of
chlorine in compound A is…….when it forms
atom is donated lone pair of electron to metal in AgNO2
0.3849 g of AgCl. (Round off to the nearest
and KNO2, NO2 has also two donor atom N and O but integer) (Atomic masses of Ag and Cl are
at a time only one atom is attached with metal . 107.87 and 35.5 respectively)
[JEE Main 2021, 27 July Shift-II]
Ans. (19) :
Organic compound + Cl2  →A
5gm
A + AgNO3  → AgCl
0.5gm 0.3849g
432. The equivalents of ethylene diamine required 1mole AgCl contains 1 mole of Cl
to replace the neutral ligands from the Mole of AgCl = Moles of Cl
coordination sphere of the trans-complex of weight of AgCl
COCl3. 4NH3 is……..(Round off to the nearest Mole of AgCl =
Molar mass of AgCl
integer).
[JEE Main 2021, 16 March Shift-I] Molar mass of AgCl = 107.87+35.5
= 143.37 g/mol
Ans. (2) : Ethylene diamine is a bidentate ligand so it
replaced two mono-dentate ligands. The complex is 0.3849
Moles of AgCl = mol = Moles of Cl
trans - [CO (NH3)4Cl2]. Cl 143.37

Objective Chemistry Volume-II 588 YCT


Mass of Cl = Moles of Cl × Atomic Mass of Cl 1
∵ wavelength of light absorbed ∝ by
0.3849 Strength of ligand
= × 35.5 = 0.0953gm
143.37 complex
0.0953 Order for wavelength of light absorbed in the complex
% of Cl in compound A = × 100 ions are:-
5
= 19% [COCl(NH3)5]2+ > [Co(NH3)6]3+ > [Co(CN)6]3–
436. 1 mol of an octahedral metal complex with 440. Which of the following pairs has both the ions
formula MCl3 . 2L on reaction with excess of coloured in aqueous solution ?
AgNO3 gives 1 mol of AgCl. The denticity of [Atomic numbers of Sc = 21, Tl = 22, Ni = 28,
ligand L is …….. (Integer answer) Cu = 29 and Mn = 25]
[JEE Main 2021, 27 Aug Shift-I] (a) Sc3+, Mn2+ (b) Ni2+, Tl4+
Ans. (2) : Coordination number of an octahedral 3+
(c) Tl , Cu +
(d) Mn2+, Tl3+
complex is 6. Kerala-CEE-29.08.2021
When Cl– ion present outside the coordination sphere
then it reacts with AgNO3 and forms precipitate of Ans. (d) : The ions get coloured in aqueous solution
AgCl. 1 mole of complex [M L2Cl2]. Cl reacts with Ag only when. The metal ions have the unpaired electron.
NO3 gives 1 mole of AgCl. Mn2+ and Ti3+ both have the 5 and 1 unpaired electron
respectively due to which transition occur between
[M L 2 Cl 2 ]. Cl+ Ag NO3  →1Ag Cl ↓
(ppt.) lower energy state to higher energy state. Thus, Mn2+
Hence, to satisfying the coordination number of and Ti3+ both the ions coloured in aqueous solution.
octahedral complex, the denticity of ligand is 2 i.e. 441. For the crystal field splitting in octahedral
bidentate ligand. complexes,
437. 3 moles of metal complex with formula Co(en)2 (a) the energy of the eg orbitals will decrease by
Cl3 gives 3 moles of silver chloride on treatment ( 3/ 5 ) ∆° and that of the t2g will increase by
with excess of silver nitrate. The secondary
valency of Co in the complex is ….. (Round off ( 2 / 5) ∆°
to the nearest integer) (b) the energy of the eg orbitals will increase by
[JEE Main 2021, 31 July Shift-II] ( 3/ 5 ) ∆° and that of the t2g will decrease by
Ans. (6) : 3 Co ( en ) 2 Cl2  .Cl + AgNO3 
→ 3AgCl ( 2 / 5) ∆°
Secondary valency of metal is also known as (c) the energy of the eg orbitals will increase by
coordination number.
Ethylene diamine is bidentate ligand and chloride is ( 3/ 5 ) ∆° , and that of the t2g will increase by
monodentate. Hence, secondary valency of Co is 6. ( 2 / 5) ∆°
438. The denticity of an organic ligand, biuret is (d) the energy of the eg orbitals will decrease by
(a) 2 (b) 4
(c) 3 (d) 6 ( 3/ 5 ) ∆° , and that of the t2g will decrease by
[JEE Main 2021, 31 Aug Shift-I] ( 2 / 5) ∆°
Ans. (a) : The denticity of an organic ligand buriet is 2 Kerala-CEE-29.08.2021
Ans. (b) : In octahedral complexes, the d-orbital
⇒ Biuret ligand splitted in two degenerate set i.e. t2g and eg. t2g has the
lower energy than the eg orbital.

439. The correct order for wavelengths of light


absorbed in the complex ions
[CoCl(NH3)5]2+, [Co(NH3)6]3+ and [Co(CN) 6]3-
is
(a) [CoCl(NH3)5]2+ > [Co(NH3)6]3+ > [Co(CN)6]3-
(b) [Co(NH3)6]3+> [Co(CN)6]3– > [CoCl(NH3)5]2+
(c) [Co(CN)6]3– > [CoCl(CN3)5]2+ > [Co(CN)6]3–
(d) [Co(NH3)6]3+ > [CoCl(NH3)5]2+ > [Co(CN)6]3-
Karnataka-CET-2021 442. Ethylene diaminetetraacetate (EDTA) ion is
Ans. (a) : Wavelength of light absorbed (a) tridentate ligand with three "N" donor atoms
1 (b) hexadentate ligand with four "O" and two "N"
∝ in the complex.
Splitting energy (∆) donor atoms
As the strength of ligand in crease splitting energy also (c) unidentate ligand
increase. (d) bidentate ligand with two "N" donor atoms.
Strength of ligand :- Cl– < NH3 < CN– (NEET 2021)

Objective Chemistry Volume-II 589 YCT


Ans. (b) : Ethylene diaminetetra acetate (EDTA) ion is In [Ni (CN)6]2– complex, nickel is present is +4
6 donor atoms so it is called hexadentate ligand with oxidation state.
four ''O'' and two ''N'' donor atoms. CN– is strong field ligand and do pairing of metal d-
electrons.
Ni4+ : d6

443. The stepwise formation of Number of unpaired electron = 0


[Cu(NH3)4]2+ is given below The total change in number of unpaired electrons on
metal centre = 2 – 0 = 2
Cu 2+ + NH 3 ↽ 1 ⇀[Cu ( NH 3 )]2+
k
446. The overall stability constant of the complex
2+ 2+
ion [Cu(NH3)4]2+ is 2.1×1013. The overall
Cu ( NH 3 )  + NH 3 ↽ ⇀ Cu ( NH 3 ) 2 
k2

dissociations constant is y×10–14. Then, y is ....


2+ 2+
Cu ( NH 3 )2  + NH 3 ↽ 3 ⇀ Cu ( NH 3 ) 
k
(Nearest integer)
2+ 2+
[JEE Main 2021, 26 Aug Shift-II]
Cu ( NH 3 )3  + NH 3 ↽ 4 ⇀ Cu ( NH 3 ) 4 
k
1
Ans. (5) : Dissociation constant =
The value of stability constant K1, K2, K3 and stability constant
K4 are 104, 1.58×103, 5×102 and 102 respectively. Dissociation constant of [Cu (NH3)4]2+
The overall equilibrium constants for
dissociation of [Cu(NH3)]2+] is x×10–12. The 1
= 2+
value of x is .......... stability constant of  Cu ( NH 3 )4 
(Rounded off to the nearest integer)
[JEE Main 2021, 24 Feb Shift-I] 1
Ans. (1.26) : Dissociation of [Cu(NH3)4]2+ is =
2+
2.1× 1013
Cu 2+ ( NH 3 ) 4  ↽
K 'eq
⇀ Cu 2+ + NH 3 = 5 × 10 −14
2+ 447. Which is not act as ligand?
⇀  Cu ( NH 3 ) 
K eq
Overall reaction:. Cu 2+ NH 3 ↽  4
(a) NO (b) H2NCH2CH2NH2
Keq = K1 × K2 × K3 × K4 (c) NH 4 +
(d) CO
= 104 × 1.58 × 103 × 5 × 102 × 102
GUJCET-2021
= 7.90 × 1011
1 1 Ans. (c) : NH +4 is not act as ligand because NH +4 have
K'eq = =
K eq 7.90 ×1011 not any empty orbital and lone pair of electron for
donation.
= 1.26 × 10 –12
448. The correct order of field strength of the
444. On complete reaction of FeCl3 with oxalic acid following ligands is
in aqueous solution containing KOH, resulted
in the formation of product A. The secondary (a) N 3− > C 2 O 42− > NH 3 > CO
valency of Fe in the product A is ...... (Round
(b) N 3− > NH3 > C 2 O 24− > CO
off to the nearest integer).
[JEE Main 2021, 17 March Shift-II] (c) CO > NH3 > C 2 O 24 − > N3−
Ans. (6) : Secondary valency is the coordination (d) CO > N 3− > NH 3 > C2 O 24 −
number of complex i.e., number of ligands attached
with central metal. TS-EAMCET (Engg.), 06.08.2021
FeCl3 + 3H 2 C2 O 4 + 6KOH  → K 3  Fe ( C2 O 4 )3  + 3KCl + 6H 2O Ans. (c): According spectrchemical series –
Coordination number is 6 because oxalate ion is Strength order of following ligands –
bidentate ligand. Hence , secondary valency is also CO > CN − > NO 2− > NH3 > O −2 > C 2 O 4−2 > N 3− > Cl−
equal to 6. Therefore, the field strength of CO is maximum and N3–
445. An aqueous solution of NiCl2 was heated with is minimum field strength.
excess sodium cyanide in presence of strong −2 −3
oxidising agent to form [Ni(CN)6]2–. The total So, CO > NH3 > C 2 O 4 > N is correct option.
change in number of unpaired electrons on 449. The correct pair(s) of the ambident
metal centre is....... nucleophiles is (are):
[JEE Main 2021, 20 July Shift-II] (i) AgCN/KCN (ii) RCOOAg/RCOOK
2−
Ans. (2) : NiCl 2 + NaCN  Oxidising
agent
→  Ni ( CN )6  (iii) AgNO 2 /KNO 2 (iv) AgI/KI
(a) (ii) and (iii) only (b) (ii) only
NiCl2 ⇒ Νi2+ : d8 (c) (i) and (iii) only (d) (i) only
Number of unpaired electron = 2 JEE Main 17.03.2021, Shift-II

Objective Chemistry Volume-II 590 YCT


Ans. (c) : The nucleophile that can attack through two Ans. (d) : [Ni(CN)4]2+ complex form dsp2 hybridisation
different sites are known as ambident nucleophiles and square planar geometry.
AgCN/KCN and AgNO2/KNO2 are both ambident CN– act as strong field ligand and do pairing of metal
nucleophile. d-electrons.
450. The number of bridged CO ligands present in [Ni (CN)4 ]2+ → Ni 2+ : d 8
Fe2(CO)9 and Co2(CO)8, respectively, are
(a) 2, 1 (b) 2, 2
(c) 2, 3 (d) 3, 2
TS-EAMCET (Engg.), 05.08.2021 Shift-II
Ans. (d) : No. of bridged CO ligands present in –
Fe 2 (CO)9 = 3
Co 2 (CO)8 = 2
O
OC C OC Hence, square planar complex is formed by using only
one d-orbital of the central atom.
OC Fe Fe OC 453. For octahedral Mn(II) and tetrahedral Ni(II)
C complexes, consider the following statements,
O OC (I) Both the complexes can be high spin.
OC (II) Ni(II) complex can very rarely be low spin.
C (III) With strong field ligands, Mn(II)
O complexes can be low spin.
Fe2(CO)9 (IV) Aqueous solution of Mn(II) ions is yellow
Bridged CO = 3 in colour.
The correct statements is
O (a) (I) and (II) only
CO CO (b) (II), (III) and (IV) only
C
(c) (I), (II) and (III) only
(d) (I), (III) and (IV) only
OC Co Co
[JEE Main 2020, 2 sep Shift-I]
Ans. (c) : Tetrahedral complexes mostly form high spin
CO C CO
complexes because metal form bonds with weak field
O ligand. Hence, Ni (II) tetrahedral complexes can be high
spin.
Co2(CO)8 Mn(II) Octahedral complex form high spin when it form
No. of bridging CO ligands = 2 bonds with weak field ligand due to which outer d-
451. Which of the following represents the correct orbital's are used.
order of increasing electron gain enthalpy with Mn(II) : d5
negative sign for the elements?
(A) Nitrogen (B) Phosphorous
(C) Chlorine (D) Fluorine
(a) P < N < F < Cl (b) N < P < F < Cl
(c) Cl < F < P < N (d) L < Cl < N < P
AP EAPCET 19-08-2021 Shift-I
Ans. (b) : As a general rule, electron gain enthalpy
becomes more negative with increase in the atomic Ni (II) : d8
number across a period Nitrogen and Fluorine are the
element of second period and phosphorus and chlorine
are prom the third period.
The size of nitrogen are less to the phosphorous atom Mn (II) complexes with strong field ligands form low
As we know halogens have more negative electron gain spin complex i.e., pairing of d-electron occurs, and
enthalpy. So the order will be – inner d-orbitals are used for hybridization
N< P < F < Cl. Mn2+ : d5 (low spin)
452. The molecule in which hybrid MOs involve
only one d-orbital of the central atom is
(a) XeF4 (b) [CrF6]3–
(c) BrF5 (d) [Ni(CN)4]2–
[JEE Main 2020, 4 Sep Shift-II]
Objective Chemistry Volume-II 591 YCT
Tetrahedral complexes of NI (II) mostly forms high 457. The number of EDTA (Ethylene Diamine
spin but it can very rare to form low spin complex. Tetracetic Acid) molecules to make an
Aqueous solution of Mn (II) ions is pink in colour. octahedral complex with a Ca2+ ion is
Hence, correct statement is (I), (II) and (III). (a) 1 (b) 2
454. Complex(A) has a composition of H12O6Cl3Cr. (c) 6 (d) 3
If the complex on treatment with conc.H2SO4 Assam CEE-2020
loses 13.5% of its original mass, the correct Ans. (a) : An octahedral complex has 6 bonds around
molecular formula of (A) is[Given: atomic mass the central atom. One EDTA molecule complexes with
of Cr = 52 amu and Cl = 35 amu] one molecule of calcium cation to form octahedral
(a) [Cr(H2O)4Cl2]Cl2H2O complex.
(b) [Cr(H2O)6]Cl3 458. Which one follow 18 electron octet rule?
(c) [Cr(H2O)5Cl]Cl2.H2O
(a) [Mn(CO)5] (b) [Cr(CO)5]
(d) [Cr(H2O)3Cl3].3H2O
(c) [Fe(CO)5] (d) None of these
[JEE Main 2020, 3 Sep Shift-II]
JIPMER-2019
Ans. (a) : Molecular weight of H12O6Cl3Cr
Ans. (c) : Valence electron of Fe = 3d6 4s2
= 52 + 3 × 35 + 6 × 16 +12 × 1
Lone pair of electron donated by 1 CO = 2 e–
= 52 + 105 + 96 +12
= 265 By 5 CO = 2×5 = 10e–
% loses of ligand when it reacts with H2SO4 = 13.5% Number of electron in [Fe(CO)5] = 8+10
= 18e–
13.5
Molecular weight of lose ligand = × 265 [Fe(CO)5] is follow the 18octet rule.
100 459. Which among the following is the strongest
= 35.7 ≈ 36 ligand?

Two ligands are present Cl and H2O. Out of these only (a) CN– (b) NH3
H2O will reacts with H2SO4 (c) CO (d) En
Molecular weight of 1H2O = 18 Karnataka-CET-2019
But M.wt of lose H2O is 36 then 2 mole of H2O is lose.
Ans. (c) : CO is strongest ligand among the following
Hence, the correct molecular formula of (A) is
because both σ and π bond is formed due to synergic
[Cr(H2O)4Cl2 ]. Cl2H2O effect.
455. For Cu2Cl2 and CuCl2 in aqueous medium,
which of the following statement is correct? 460. The donor atom in EDTA are
(a) CuCl2 is more stable than Cu2Cl2 (a) Two N and two O
(b) Stability of Cu2Cl2is equal to stability of (b) Two N and Four O
CuCl2 (c) Four N and two O
(c) Both are unstable (d) Three N and three O
(d) Cu2Cl2 is more stable than CuCl2 (e) Two N and three O
Karnataka-CET-2020 Kerala-CEE-2019
Ans. (a) : CuCl2 is more stable than Cu2 Cl2 in aqueous Ans. (b) : The donar atom of EDTA is –
medium because stability in aqueous medium depends Two N atom and four O atom.
and the hydration enthalpy of ion. Hydration enthalpy
depends upon the size of ions.
1
Hydration enthalpy ∝
Size of ions
In CuCl2 copper is present in +2 oxidation and in
Cu2Cl2 oxidation state is +1 size of Cu2+ is smaller than
Cu+ therefore hydration energy of Cu2+ is higher than
Cu+. Hence, CuCl2 is more stable than Cu2Cl2.
456. What is the value of x on the [Ni(CN)4]x
complex ion? 461. The effective atomic number of Iron (Z = 26) in
(a) +2 (b) –2 [Fe(CN)6]–3 is
(c) 0 (zero) (d) +4 (a) 36 (b) 33
Manipal-2020 (c) 35 (d) 34
Ans. (b) : Nickel forms square planar complex with MHT CET-03.05.2019, SHIFT-I
coordination number 4 and strong field ligand with Ans. (c) : Atomic number of Fe = 26
oxidation state is +2. Oxidation state of Fe in [Fe(CN)6]–3 = +3
[Ni(CN)4]x Number of donor atom attached with Fe = 6
2+4×(–1) = x EAN= Atomic No. – oxidation state + 2x(No. of donor
2–4 = x atom attached with metal)
x = –2 EAN of [Fe(CN)6]–3 =26–3+2×6
The complex is [Ni(CN)4]2– = 35
Objective Chemistry Volume-II 592 YCT
462. Which among the following is a neutral 465. Which of the following carbonyls will have the
complex? strongest C–O bond?
(a)  Fe ( H 2 O )6  Cl3 (b)  Ni ( NH 3 )6  Cl 2 (a) [Mn(CO)6]+ (b) [Cr(CO)6]
(c) [V(CO)6]− (d) [Fe(CO)5]
(c)  Pt ( NH 3 )2 Cl2  (d) K  Ag ( CN )2 
[BITSAT-2019]
MHT CET-02.05.2019, SHIFT-II Ans. (a) : Bond strength of C.O bond
Ans. (c) : Complexes which does not contain any 1
charge or the coordination sphere is at as neutral ligand. ∝
Back − bonding
3+
( )  ( 2 )6  3
a  Fe H O  C l →  ( 2 )6 
 Fe H O  + 3C l −
Back bonding ∝ Electron density of metal
2+ Negative Charge on metal
( b )  Ni ( NH 3 )6  Cl2 
→  Ni ( NH 3 )6  + 2Cl − ∝
Positive Charge on metal
( c ) pt ( NH3 )2 Cl2   → No charge or coordiantion sphere If metal having more electron density then it
− donates more lone pair of electron to carbon atom of
( d ) K  Ag ( CN )2   → K +  Ag ( CN ) 2 
+

Hence, [Pt(NH3)2Cl2] complex is a neutral complex. CO group. Metal donate from filled orbital to vacant π*
orbital of carbon due to which bond strength decreases.
463. In Fe (CO)5, the Fe – C bond possesses
Hence, [Mn(CO)6] will have the strongest C–O bond
(a) ionic character
because here tendency of back bonding is less.
(b) σ – character only
(c) π– character 466. Which of the following ligands can act as an
ambidentate ligand?
(d) both σ and π characters
AIIMS 25 May 2019 (Evening) (a) NO −2 (b) Cl −
Ans. (d) : In Fe (CO)5, the Fe −C bond possesses both σ (c) C 2 O 24 − (d) CN −
and π character. J & K CET-(2019)
Ans. (a,d) : Those ligands which has two different
donor atoms but at a time only one donor atom is
attached with metal.

CN is an example of ambidentate ligand.
M ←  CN M ←  NC
Cyanido Isocyanido
467. The ligand called π - acid is
464. The molar ionic conductances of the octahedral
complexes: (a) CO (b) NH3
(i) PtCl4 ⋅ 5NH3 (ii) PtCl4 ⋅ 4NH3 (c) C 2 O 24 − (d) ethylene diamine
(iii) PtCl4 ⋅ 3NH3 (iv) PtCl4 ⋅ 2NH3 COMEDK-2019
Follow the order
Ans. (a) : An example of a π – acid ligand is carbon
(a) i < ii < iii < iv (b) iv < iii < ii < i
monoxide (CO). It is a good pi acceptor (lewis acid) due
(c) iii < iv < ii < i (d) iv < iii < i < ii
to empty pi orbital’s and good sigma donor.
AIIMS 26 May 2019 (Morning)
Ans. (b) : Molar ionic conductance of the octahedral 468. The solution of the complex [Cu(NH3)4] SO4 in
complex depends on the number of ions formed. Pt in + water will
Cl oxidation state form complex of coordination (a) give the test for Cu2+ ion
number of 6 (b) give the test for NH +4 ion
(i)
3+ (c) give the test for SO 24 − ion
Pt Cl4 ⋅ 5NH 3 :  Pt ( NH 3 )5 Cl  Cl3 
→  Pt ( NH 3 )5 Cl  + 3Cl −
(d) not give the test for any of the ions.
No of ions = 4 COMEDK-2019
(ii)
2+
Ans. (c) : [Cu(NH3)4]SO4 in onises in solution to give
PtCl4 ⋅ 4NH3 :  Pt ( NH 3 )4 Cl2  Cl2 
→  Pt ( NH3 )4 Cl 2  + 2Cl−
simple SO 2– 4 ions which can be identified by adding
No of ions = 3 BaCl2 Solution.
(iii) 2+
+ Cu ( NH 3 )  SO 4 
→  Cu ( NH 3 )4  + SO 24−
PtCl4 ⋅ 3NH 3 :  Pt ( NH 3 )3 Cl3  Cl 
→  Pt ( NH 3 )3 Cl3  + Cl −  4 

No of ions = 2 469. Number of donating atoms present in EDTA


(a) 6 (b) 5
(iv) PtCl4 ⋅ 2NH 3 :  Pt ( NH 3 )3 Cl 4 
(c) 2 (d) 4
No of ions = 0 JCECE - 2018
Objective Chemistry Volume-II 593 YCT
Ans. (a) : Number of donating atoms present in EDTA [Co(NH3 )6 ]3+ ↿⇂ ↿⇂ ↿⇂ ××
 ×× ×× ×× ×× ××
are 6 (six). It is a hexadentate ligand. It can bind
(inner orbitalor
through two nitrogen and four oxygen atom to a central low spin complex
Six pairs of electrons
from six NH3 molecule
metal ion.
Six pairs of electrons one from each NH3 molecule,
occupy the six hybrid orbitals. Thus, complex has
octahedral (d2sp3) geometry.
[Ni(CO)4] and [PtCl4]2– are tetrahedral and square
planar respectively.
472. What is the number of donar atoms in
dimethylglyoximato ligand?
(a) 1 (b) 2
(c) 3 (d) 4
MHT CET-2018
470. Colour of V4+ ion is Ans. (b) : Dimethylglyoximato ligand has two donor
(a) green (b) violet atom which is called bidentate ligand,
(c) pink (d) blue
JCECE - 2018
Ans. (d) : Colour of V4+ ion is blue. The colour
observed corresponds to the complementary colour of 473. Iron carbonyl, Fe(CO)5 is
the light absorbed. (a) tetranuclear (b) mononuclear
The frequency of light absorbed is determined by the (c) trinuclear (d) dinuclear
nature of the ligand. Thus, most of the compounds of (NEET 2018)
transition metals are coloured due to d-d transition. Ans. (b) : Iron carbonyl, [Fe(CO)5] is mononuclear
471. Octahedral complex among the following complex because in Fe (CO)5 only one central metal is
(a) Ni(CO)4 (b) [PtCl4]2– involved in complex.
(c) [Fe(CN)6] 4–
(d) [Co(NH3)6] 3+ 474. Which of the following give good synergic bond
with metal?
JCECE - 2018
4–
(a) CO (b) NH3
Ans. (c,d) : (i) [Fe(CN)6] (c) H 2 O (d) Cl-
4– 2+
In [Fe(CN)6] , Fe is present as Fe . [AIIMS-27 May, 2018 (M)]
3d6 4s0 Ans. (a) : Synergic bonding is also known as
Fe2+ = ↿⇂ ↿ ↿ ↿ ↿ backbonding generally ligand donate lone pair into the
vacant orbital of metal. But in case of synergic bond
[CN]– being strong field ligand, paired up the d- ligand to metal donation occur as well as donation of
electrons of the metal]. lone pair from filled orbital of metal to vacant orbital of
[Fe(CN6)]4– = ligand is occurred. CO ligand give good synergic
bonding among the following.
3d 4s 4p
475. Cr 3+ in aquous medium forms green colored
↿⇂ ↿⇂ ↿⇂  ×× ×× ×× ×× ×× ××
complex with NH3 ligand. How many ligands
CN− CN− CN− CN− CN− CN− are associated with it ?
2 3
(a) 3 (b) 4
d sp − hybridisation
(c) 5 (d) 6
Shape Octahedral. [AIIMS-26 May, 2018 (E)]
Among the given options [Fe(CN)6]4– and [Co(NH3)6]3+
Ans. (d) :
are the octahedral complex. 3+ 3+
(ii) [Co(NH3)6] In this complex, oxidation state of Cr ( H 2 O )6  + 6NH 3 
3+
→ Cr ( NH 3 )6  + 6H 2 O
cobalt is +3 and has electronic configuration 3d6. Its Green
hybridisation scheme can be shown as : colour
3d 4s 4p 3+
Co − atom Coordination number of product  − Cr ( NH 3 )6 
Z = 27 ↿⇂ ↿⇂ ↿ ↿ ↿ ↿⇂
complex is 6 because Cr 3+ attached with six NH3
Orbitals of ligands.
3+
Co
2
ion ↿⇂ ↿ ↿ ↿ ↿ ↿ 476. The number of donor sites in dimethyl
3
d sp glyoxime, glycinato, diethylene triamine and
− hybridised EDTA are respectively:
orbitals of Co3+ ↿⇂ ↿⇂ ↿⇂ (a) 2, 2, 3 and 4 (b) 2, 2, 3 and 6
Six empty orbitals = 2d,
(c) 2, 2, 2 and 6 (d) 2, 3, 3 and 6
1s and 3p-orbitals [AIIMS-26 May, 2018 (M)]

Objective Chemistry Volume-II 594 YCT


Ans. (b) : Demethyl glyoxine, Contribution by (σ – C6H6) = 1e– contribution by
(π − C5 H 5 ) = 5e–
EAN = Atomic No. – Oxidation state + Number of
electrons donated by ligands.
EAN of [Ti (σ – C6 H 6 ) 2 (π − C5 H5 ) 2 ] = 22 − 2 + 2 × 1 + 2 × 5 = 34
3+ +3
479. Assertion : Co ( NH 3 )6  and Co ( en )3  are
stable complex.
Reason: They are low spin complex
has 2 donor sites
(a) If both Assertion and Reason are correct and
Glycinato, the Reason is the correct explanation of
Assertion.
(b) If both Assertion and Reason are correct, but
Reason is not the correct explanation of
has two donor sites Assertion.
Diethylene triamine, (c) If Assertion is correct but Reason is incorrect.
(d) If both the Assertion and Reason are
incorrect.
[AIIMS-27 May, 2018(M)]
Ans. (b) : Coordination complexes are stable when it
follows EAN rule.
3+
CO ( NH 3 )6 
has 3 donor sites  
EDTA4− , EAN = 27 + 6 × 2 − 3 = 36
3+
CO ( Cn )3 
 
EAN = 27 + 6 × 2 − 3 = 36
Both complexes have EAN value is equal to atomic
number of inert gases. So they are stable. NH3 and
ethylene dia amine behaves as strong field ligand with
cobalt in +3 oxidation, due to which pairing of metal d
electron configuration and complex is known as low
spin complex.
has 6 donor sites. Hence, both Assertion and Reason are correct and the
477. Which of the following can be reduced easily? reason is not the correct explanation of Assertion.
(a) V ( CO )6 (b) Mo ( CO )6 480. How many bridging carbonyl (s) is / are
present in the compound Fe3 (CO)12

(c) Co ( CO )4  (d) Fe ( CO )5 compounds?
(a) Three (b) Four
[AIIMS-26 May, 2018 (M)] (c) Two (d) Five
Ans. (a) : If complex follows 18 e– rule than it is stable. J & K CET-(2017)
V ( CO )6 ⇒ One CO group donate 2e– Ans. (c) :
V = [ Ar ] 3d3 4s 2
Electron count in V ( CO )6 complex = 5 + 2 × 6 = 17e–
So, V ( CO )6 doesn't follow 18e– rule. To follow 18e–
rule V ( CO )6 gain 1e– i.e., it is reduced easily and form

 V ( CO )6  complex. Two bridging carbonyls are present in Fe3(CO)12
 
compound.
478. The EAN value y [Ti (σ–C6H5)2 (π – C5H5)2]0 is 481. The correct order of electrical conductivity of
(a) 32 (b) 33 the given complexes is
(c) 34 (d) 35 (a) [Ni(NH3)4Cl2]<K2[PtCl6]<K4[Fe(CN)6]<
AMU-2018 [Co(NH3)4Cl2)Cl
Ans. (c) : Atomic Number of Ti = 22 (b) K2 [PtCl6] < K4 [Fe(CN)6]<[Co(NH3)4Cl2]Cl
Oxidation state of Ti = x <[Ni(NH3)4Cl2]
x + 2 × 0 + 2 × (–1) = 0 (c) K4[Fe(CN)6]<[Co(NH3)4Cl2]Cl<
x=+2 [Ni(NH3)4Cl2] < K2 [PtCl6]

Objective Chemistry Volume-II 595 YCT


(d) [Ni(NH3)4Cl2]<[Co(NH3)4Cl2]Cl 485.
The effective atomic number (EAN) of platinum
< [K2[PtCl6]<K4[Fe(CN)6] in potassium hexachloro palatinate (IV) is
J & K CET-(2017) (a) 78 (b) 74
Ans. (d) : Electric conductivity depends on the number (c) 84 (d) 86
ions formed during ionization of complex compounds CG PET -2017
[Ni(NH 3 )4 Cl2 ]  →[Ni(NH 3 ) 4 Cl 2 ] Ans. (d) : ∵ EAN (effective atomic number
No. of ions = 1 = [Z− oxidation number]+ [2×number of ligands]
K 2 [PtCl6 ]  → 2K + + [PtCl6 ]2– For Pt (platinum) Z = 78
No. of ions = 3 Oxidation number = 4
∴ EAN = (78−4)+(2×6)
K 4 [Fe(CN)6 ]  → 4K + + [Fe(CN)4 ]4− = 74+12 = 86 (Atomic number Rn)
No. of ions = 5 486. Which of the following sequence is correct
[Co(NH 3 ) 4 Cl2 ]Cl →[Co(NH 3 )4 Cl2 ]+ + Cl – regarding field strength of ligands as per
No. of ions = 2 spectrochemical series?
Order of electric conductivity:- (a) SCN– < F– < CN– < CO
[Ni(NH3)4Cl]<[Co(NH3)4Cl2]Cl<K2[PtCl6]<K4[Fe(CN)6] (b) F– < SCN– < CN– < CO
482. An example of a sigma bonded organometallic (c) CN– < F– < CO– < SCN–
compound is (d) SCN– < CN– < F– < CN–
(a) Grignard's reagent (b) ferrocene Karnataka-CET-2016
(c) cobaltocene (d) ruthenocene Ans. (a) : According to spectrochemical series, the
arrangement of ligands in increasing order of their field
(NEET 2017)
Ans. (a) : Grignard's reagent is a sigma bonded organ strength can be written as-
metallic compound. SCN– < F– < CN– < CO
R–Mg x 487. CO is a stronger ligand than Cl–, because
Grignard's reagent (a) CO is neutral molecule
(b) CO has π bonds
(c) CO is poisonous
(d) CO is more reactive
Karnataka-CET-2016
Ans. (b) : According to spectrochemical series, CO is
strongest ligand than Cl– because CO forms both σ and
π bond due to synergic effect.
488. The molecular formula of Wilkinson's catalyst
used in the hydrogenation of alkenes is
(a) Co(CO)8 (b) (Ph3P)3 RhCl
(c) [Pt(NH3)2Cl2] (d) K[Ag(CN)2]
483. Which of the following chelating legands can be MHT CET-2016
used to remove the excess of copper in the Ans. (b) : Wilkinson's catalyst, [Rh(PPh)3Cl] is used for
biosystems? the hydrogenation of alkenes.
(a) D-pencillamine (b) Desferrioxamine B
(c) EDTA (d) Dimercaprol
AMU-2017
Ans. (a) : The chelating ligand D-pencillamine can be
used to remove the excess of copper in the biosystem. 489. Which of the following has longest C–O bond
484. The Magnetic moment (spin only) of [NiCl4]2– is- length? (Free C–O bond length in CO is 1.128
(a) 1.82 BM (b) 5.46 BM Å.)
(c) 2.82 BM (d) 1.41 BM (a) [Fe(CO)4]2– (b) [Mn(CO)6]+
BCECE-2017 (c) Ni(CO)4 (d) [Co(CO)4]–
2–
Ans. (c) : In [NiCl4] complex, Ni metal is present in (NEET 2016)
+2 oxidation state Cl– is weak field ligand and do not do Ans. (a) : As metals having more electron density then
pairing of metal 3d–electrons. tendency of backbonding increase where metal donates
Ni2+ : 3d8 lone pair of electrons to empty antibonding orbital of
8 CO ligand. Due to which C–O bond strength decrease
d and C–O bond length will increase.
No, if unpaired election = 2 Hence, in [Fe(Co)4]2– complex iron is present in –2
oxidation state therefore more electron density on it So,
Magnetic moment = n (n+2) = 2(2 + 2) = 8 B.M. C–O bond length in [Fe(Co) ]2– complex is longest
4
= 2.82 B.M. among the following.

Objective Chemistry Volume-II 596 YCT


490. The correct increasing order of trans-effect of (a) N O-3 < OH– < Pyridine < PPh3
the following species is
(a) NH3 > CN– > Br– > C6H5– (b) OH– < N O- <PPh < pyridine
3 3
(b) CN– > C6H5– > Br– > NH3 (c) OH < N O-3 <Pyridine < PPh3

(c) Br– > CN– > NH3 > C6H5–
(d) CN– > Br– > C6H5– > NH3 (d) N O-3 < OH– < PPh3 < pyridine.
[NEET-II 2016] J & K CET-(2015)
Ans. (b) : Trans effect increase the rate of substitution Ans. (a) : Ligands are arranged in increasing order of
reaction in square planar complex. The group having field strength of ligands in spectro chemical series.
more trans effect will allow easier substitution at its Increasing field strength is :-
trans position. NO3– < OH – < Pyridine < PPh 3
Increasing order of trans effect:-
CN − > C 6 H 5 > Br − > NH 3 496. In the complexes [Fe(H2O)6]3+, [Fe(CN)6]3–,
[Fe(C2O4)3]3– and [FeCl6]3–, more stability is
491. The ligand shown by
(OOC – CH2)2NCH2 – CH2 – N (CH2COO–)2 is (a) [FeCl6]3– (b) [Fe(C2O4)3]3–
3+
(a) Didentate (b) Tirdentate (c) [Fe(H2O)6] (d) [Fe(CN)6]3–
(c) Hexadentate (d) Tetradentate JIPMER-2015
J & K CET-(2016) Ans. (b) : If complex containing chelating ligand then
Ans. (c) : complex is more stable then other complex stability of
complex ∝ Number of chelating ligand.
[Fe(C2O4)3]3– is more stable among the following
complexes because C2O42- or oxalate ion is a chelating
ligand.
497. Trien ligand is:
This ligand is EDTA4– has 6 donor atoms hence it is
known as hexadentate ligand. (a) Hexadentate, mono anionic
(b) Tetradentate, neutral
492. In which one of the following four pairs of (c) Tetradentate, dianion
oxides are both oxides amphoteric?
(d) Monodentate, anion
(a) Cr2O3 and V2O5 (b) CrO3 and V2O5
[AIIMS-26 May, 2015 (M)]
(c) CrO and V2O3 (d) CrO and V2V4
J & K CET-(2016) Ans. (b) : Trien ligand
Ans. (a) : Amphoteric oxides are those oxide
compounds which reacts with both acids and bases
Cr2O3 and V2 O5 reacts with both acid and bases.
493. Mohr’s salt is
(a) FeSO4(NH4)2SO4.6H2O is tetradentate and neutral.
(b) FeSO4.7H2O 498. Which of the following carbonyls will have the
(c) Fe2(SO4)3(NH4)2SO4.6H2O strongest C – O bond?
(d) MgSO4.7H2O
(a) Mn(CO)+6 (b) Cr(CO)6
COMEDK-2016
(c) V(CO)−6 (d) Fe(CO)5
Ans. (a) : Mohr's salt ( FeSO 4 ( NH 4 )2 SO 4 .6H 2 O ) is
VITEEE- 2012
double salt which lose his identity in water and break in
Ans. (a) : Due to positive oxidation state of Mn back
the cation and anion.
donation in π* (ABMO) of CO is minimum. Therefore,
494. Which of the following represents hexadentate C – O bond is strongest in Mn (CO)6+.
ligand?
499. The anion of acetylacetone (acac) forms
(a) EDTA (b) DMG
Co(acac)3 chelate with Co3+. The rings of the
(c) Ethylenediamine (d) None of the above chelate are
[BITSAT-2015] (a) five membered (b) four membered
Ans. (a) : EDTA is a hexdentate ligand because it has 6 (c) six membered (d) three membered
donor sites. Karnataka NEET 2013
Ans. (c) : The stability of a metal chelate also depends
upon the size of the ring. It has been observed that metal
cheats with 5-membered ring are more stable. 6-
membered chelates having double bond or unsturation
in the ring are more stable than corresponding 5-
495. The correct order of the ligands, OH–, NO 3– , membered chelates. Acetylacetonato (acac) complexes
PPh3, pyridine, according to their increasing containing 6-membered ring and double bonds in the
field strength is ring form stable complexes.

Objective Chemistry Volume-II 597 YCT


500. Which of the following is an outer orbital (a) If both Assertion and Reason are correct and
complex? the Reason is the correct explanation of
(a) [Fe(CN)6]4- (b) [FeF6]3- Assertion.
3+
(c) [Co(NH3)6] (d) [Co(CN)6]3- (b) If both Assertion and Reason are correct, but
AMU-2011 Reason is not the correct explanation of
Assertion.
Ans. (b) : [ FeF6 ] is an outer orbital complex F− act as
3−
(c) If Assertion is correct but Reason is incorrect.
weak field ligand so pairing of metal d - electrons not (d) If both the Assertion and Reason are
possible. incorrect.
In [ FeF6 ] , Fe is in +3 oxidation state.
3−
[AIIMS-2014]

Fe3+ : [Ar] 3d5 Ans. (a) :  M ( en )( CH 3COO )4  complex has co-
ordination number 6 because it has 6 donor site ( 4 from
four CH3 COO− and 2 from one ethylene diamine (en)
Here, the complex formed involved the outer orbitals because en act as bidentate ligand) if complex having
for hybridization. So the complex is called outer orbital coordination number 6 then it forms octahedral
complex. complex.
Hence both Assertion and Reason are correct and the
Reason is the correct explanation of Assertion.
503. Assertion : A chelating ligand must possess two
or more lone pairs at such a distance that it may
form suitable strain free rings at the metal ion.
Reason: H 2 N − NH 2 is a chelating ligand.
501. Which one of the following pairs of complexes (a) If both Assertion and Reason are correct and
has the effective atomic number equal to 36 for the Reason is the correct explanation of
the transition element? Assertion.
(a) [Co(NH3)6]Cl3, K3[Fe(CN)6] (b) If both Assertion and Reason are correct, but
(b) [Co(NH3)6Cl3, [Cr(H2O)6]Cl Reason is not the correct explanation of
(c) [Fe(CO)5], K4[Fe(CN)6] Assertion.
(d) [Fe(CO)5], K3[Fe(CN)6] (c) If Assertion is correct but Reason is incorrect.
AP-EAMCET (Medical), 2008 (d) If both the Assertion and Reason are
Ans. (c) :(a) Co ( NH 3 )6  Cl3 K 3  Fe ( CN )6  incorrect.
3+
[AIIMS-2010]
Co ( NH 3 )6 
3–
 Fe ( CN )6  Ans. (c) : A chelating lig and must possess twe or more
O.S. of Co = +3 O.S. of Fe = +3 lone pairs which donate to metal and form 5 or 6
mempered ring generally.
EAN= Z– O.N.+C.N. × No. of e– donated by each
ligand. Example of chelating ligand is EDTA4− , acac etc H2N -
∴ EAN of Co = 27–3+6×2 EAN of Fe = 26–3+6×2 NH2 is not a chelating lig and because it from 3
membered & train ring with metal ion which is
= 36 = 26–3+12
unstable.
= 35
(b) Cr ( H 2 O )6  Cl3
3+
or  Cr ( H 2 O )6 
Hence, Assertion is correct but reason is incorrect.
O.S. of Cr = +3
504. Assertion : [ FeF6 ] is a low spin complex.
3−
∴ EAN = 24 – 3 + 6× 2 = 33
(c)  Fe ( CO )5  K 4  Fe ( CN )6  Reason: Low spin complexes have lesser number
of unpaired electrons.
 Fe ( CO )5   Fe ( CN )6 
4–
(a) If both Assertion and Reason are correct and
the Reason is the correct explanation of
O.S. of Fe = 0 O.S. of Fe = +2 Assertion.
∴ EAN of Fe = 26–0+5×2 EAN of Fe =26–2+6×2 (b) If both Assertion and Reason are correct, but
= 36 =36 Reason is not the correct explanation of
Hence, the correct option is C. Assertion.
502. Assertion: Ethylenediaminetetraacetate ion forms (c) If Assertion is correct but Reason is incorrect.
an octahedral complex with the metal ion. (d) If both the Assertion and Reason are
Reason: It has six donor atoms which coordinate incorrect.
simultaneously to the metal ion. [AIIMS-2013]
Objective Chemistry Volume-II 598 YCT
(c) linked only through 3 CO molecules as
Ans. (d) : In [ FeF6 ]
3−
Complex, oxidation of iron is + 3 bridging ligands.
F− is a weak field ligand which not do pairing of metal d (d) Joined through one CO group as bridging
- electrons. Therefore, [ FeF6 ] ligands.
3−
act as high spin
[AIIMS-2011]
complex.
Ans. (b) : In Fe2 (CO)9, linked directly along with the 3
In low spin complex pairing of metal d - orbital occurs. CO molecules as bridging ligands.
Hence, both Assertion and Reason is incorrect.
505. In the isoelectronic series of metal carbonyl, the
CO bond strength is expected to increase in the
order:
(a) [Mn(CO)6]+ < [Cr(CO)6]< [V(CO)6]-
(b) [V(CO)6]- < [Cr(CO)6]< [Mn(CO)6]+
(c) [V (CO)6]- < [Mn(CO)6]+< [Cr(CO)6]
(d) [Cr(CO)6] < [Mn(CO)6]+ < [V(CO)6]- Structure of Fe2 (CO)9
[AIIMS-2014] 508. What is incorrect about homoleptic metal
1 carbonyls?
Ans. (b) : Bond strength of CO ∝ (a) M - C, bond is formed by donation of lone
Back bonding pair of electron from CO.
Metal having negative oxidation state (b) M - C, π-bond is formed by back donation of
Back bonding ∝ electron from filled d-orbital of metal to
Metal having positive oxidation state
vacant p-orbital of carbon.

 V ( CO )6  ⇒ Oxidation stateof V is −1 (c) M - CO bonding produce synergic effect.
 
(d) Metal carbonyl contain only σ-bonds.
Cr ( CO )6  ⇒ Oxidation stateof Cr is 0 [AIIMS-2010]
 
+ Ans. (c) : In homoleptic metal carbonyls , M − CO
 Mn ( CO )6  ⇒ Oxidation stateof Mn is + 1. bonding produce synergic effect due to formation of σ
 
Back bonding tendency − and π − backbonding, σ bond is formed by donation of
− +
lone pair of electrons from carbon of CO group to
 V ( CO )6  > Cr ( CO )6  >  Mn ( CO )6  vacant orbital of metal and π − bond is formed by
      backdonation of lone pair of electron from filled orbital
CO bond strength– of metal to vacant p-orbital of carbon.
− +
 V ( CO )6  < Cr ( CO )6  <  Mn ( CO )6 
     
506. Which method can be used to distinguish
[Co(NH3)6][Cr(NO2)6] and
[Cr(NH3)6][Co(NO2)6]?
(a) By measurement of their conductivity.
(b) By titration method. 509. Which statement is true for ferrocene?
(c) By precipitation method with Ag NO3. (a) all Fe-C are of equal length.
(d) By electrolysis of their aqueous solutions. (b) It has sandwich type structure.
[AIIMS-2011] (c) It was the first discovered organometallic
Ans. (d) : When aqueous solution of compound.
Co ( NH 3 )6  Cr ( NO 2 )6  and (d) All of these.
   [AIIMS-2007]
Cr ( NH 3 )6   Co ( NO 2 )6  are electrolysed then Ans. (d) : In Ferrocene all Fe – C bond lengths are
  
equal and here one Fe atom sandwich between two
different types of cations and anions are formed in cyclopentadienyl ring. Ferrocene was the first
cathode and anode. discovered organometallic compounds. Its molecular
3+ 3−
Co ( NH 3 )6   Cr ( NO 2 )6  
electrolysis
→  Co ( NH 3 )6  +  Cr ( NO 2 )6  formula is Fe(C5H5)2.
  

3+ 3−
Cr ( NH 3 )6   Co ( NO 2 )6  
electrolysis
→  Cr ( NH 3 )6  +  Co ( NO 2 )6 
  

510. The ligands in anti - cancer drue cis - platin


507. In Fe2(CO)9, the two iron atoms are are:
(a) linked only dirctly. (a) NH3,Cl (b) NH3,H2O
(b) linked directly along with 3 CO molecules as (c) Cl,H2O (d) NO,Cl
bridging ligands. [AIIMS-2006]
Objective Chemistry Volume-II 599 YCT
Ans. (a) : Cis-platin is used as anti-cancer drug for 514. In the silver plating of copper, K[Ag(CN)2 ] is
treatment of cancer. used instead of AgNO3. The reasion is
(a) a thin layer of Ag is formed on Cu
(b) more voltage is required
(c) Ag+ ions are completely removed from
Cis -[PtCl2(NH3)2] solution
The ligand NH3 and Cl– is present in cis-platin. (d) less availability of Ag+ ions, as Cu cannot
511. The only cations present in a slightly acidic displace Ag from [Ag(CN)2]– ion
solutions are Fe3+,Zn2+ and Cu2+ . The reagent BCECE-2012
that when added in excess to this solution Ans. (d) : In the silver plating of copper K[Ag (CN)2] is
would identify and separate Fe3+ in one step is used because Ag is present inside the coordination
(a) 2 M HCl (b) 6 M NH3 sphere due to which rate of dissociation is slow and Cu
(c) 6M NaOH (d) H2S gas cannot displace Ag from [Ag (CN)2]– ion.
[AIIMS-2010] While in case of AgNO3 compound rate of dissociation
is faster because it is ionic compound due to which Cu
Ans. (b) : For separation and identification of Fe3+ easily displace Ag from AgNO .
3
from solution of Fe3+, Cu2+ and Zn2+ that reagent is used Hence, K[Ag(CN) ] is used for silver plating of copper.
3+ 2
in which Fe is not soluble and forms precipitate with it
while other two Cu2+ and Zn2+ are soluble in that 515. CuSO4 when reacts with KCN forms CuCN
reagent. The only reagent from the given is 6M NH3 which is insoluble in water. It is soluble in
3+ excess of KCN due to the formation of the
which form precipitate with Fe ion.
following complex
3+
Fe + dil.NH 3  → Fe(OH)3 ↓ + NH+4 (a) K 2 [Cu(CN)4 ] (b) K 3 [Cu(CN)4 ]
(ppt.)
(c) CuCN 2 (d) Cu[KCu(CN)4 ]
Cu 2+ + dil . NH 3 → [Cu(NH 3 ) 4 ]2+ BCECE-2012
Zn 2+ + dil . NH 3 → [Zn(NH 3 )4 ]2+ Ans. (b) :
512. The coordination number of a central metal CuSO 4 + 2KCN  → Cu(CN) 2 + K 2SO 4
atom in a complex is determined by– Insoluble
(a) the number of ligands around a metal ion 2Cu(CN)  → 2CuCN + (CN) 2
2
bonded by σ-bonds
(b) the number of ligands around a metal ion Cyanogen
bonded by π-bonds CuCN + 3KCN  → K 3[Cu(CN)4 ]
(c) the number of ligands around a metal ion Soluble
bonded by σ and π-bonds both 4–
516. [EDTA] is a:
(d) the number of only anionic ligands bonded to
(a) monodentate ligand
the metal ion
(b) bidentate ligand
BCECE-2010
(c) quadridentate ligand
Ans. (a) : The coordination number of a central metal (d) hexadentate ligand
atom in a complex is determined by the number of [BITSAT-2009]
ligands around a metal ion bonded by σ − bond through UPTU/UPSEE-2004
metal.
Ans. (d) : [EDTA]4- is a hexadentate ligand having six
513. The hypothetical complex chloro donor sites.
diaquatriammine cobalt (III) chloride can be
represented as
(a) [CoCl(NH 3 )3 (H 2 O) 2 ]Cl 2
(b) [Co(NH 3 )3 (H 2 O)Cl3 ]
(c) [Co(NH 3 )3 (H 2 O) 2 Cl]
(d) [Co(NH 3 )3 (H 2 O)3 ]Cl3
BCECE-2012
Ans. (a) : The complex chlorodiaquatriammine cobalt 517. Number of ions for the complex
(III) chloride can be represented as Co ( CH 3 ) Cl  Cl 2 shown by the conductivity
 4 
Oxidation state of Co = x measurement, is
[CoCl(NH 3 )3 (H 2 O) 2 ]Cl 2 (a) 2 (b) 4
x + (–1) + 0 + 0 = +2 (c) 3 (d) 1
x = +3 CG PET -2006
Objective Chemistry Volume-II 600 YCT
Ans. (c) : [Co(CH3)4Cl]Cl2 will give 3 ions on EAN of [Rh(H2O)6]3+ =Atomic no. – Oxidation + 2×
ionization. Number of donor atom attached with metal.
− 2+ = 45 – 3 + 2 × 6 = 54
Co ( CH 3 )4 Cl  Cl 2 
→  Co ( CH 3 ) 4 Cl  + 2Cl −

521. EDTA can form complex with how many


No. of ions = 3 number of donor atoms?
518. Which of the following facts about the complex (a) One (b) Two
[Cr(NH3)6]Cl3 is wrong? (c) Six (d) Three
(a) The complex involves d2sp3 hybridization and J & K CET-(2014)
is octahedral in shape Ans. (b) : EDTA has 6 donor atoms hence it is also
(b) The complex is paramagnetic called hexdentate ligand
(c) The complex is an outer orbital complex
(d) The complex gives white precipitate with
silver nitrate solution
AIEEE 2011
Ans. (c) : [Cr (NH3)6]Cl3 complex, Cr is present in +3
oxidation. 522. Ethylenediaminetetraacetate ion is a
3+
In Cr always inner d-orbital i.e., (n–1) d is used for (a) unidentate ligand (b) bidentate ligand
2 3
hybridisation and forms d sp hybridisation with (c) pentadentate ligand (d) hexadentate ligand.
octahedral geometry. J & K CET-(2013)
3+
Cr : d 3 Ans. (d) : Ethylene diaamine tetra acetate ion (EDTA4–)
is a hexadentate ligand i.e, having six donor atoms.

523. Among the following the ambidentate ligand is


(a) H2NCH2CH2NH2 (b) CO2–

[Cr (NH3)6] Cl3 contains three unpaired electrons (c) NO 2– (d) C 2 O 42–
therefore it shows paramagnetic behavior. J & K CET-(2011)
Cr ( NH 3 )6  .Cl3 + AgNO3 
→ 3AgCl ↓ Ans. (c) : Those ligands which has two different donor
atom but at a time only one donor atom is attached with
(ppt.)
metal.
519. The equation which is balanced and represents
the correct product(s) is
(a) Li 2 O + 2KCl → 2LiCl + K 2 O
+
(b) CoCl ( NH 3 )5  + 5H 
+
→ CO 2+
+ 5NH +4 + Cl− NO 2– has two donor sites N and O.
2+
(c)  Mg ( H 2 O6 )  ( EDTA )
4−
524. The metal ion in complex A has EAN identical
to the atomic number of krypton A is
2+

excess NaOH
→  Mg ( EDTA )  + 6H 2 O (At. No. of Cr = 24, Fe = 26, Pd = 46)
(a) [pd(NH3)6]Cl4 (b) [Cr(NH3)5Cl]SO4
CuSO 4 + 4KCN → (c) Na4[Fe(CN)6] (d) K3[Fe(CN)6]
(d) K 2  Cu ( CN )4  + K 2SO 4 J & K CET-(2006)
JEE Main 2013 Ans. (c) : Atomic number of krypton = 36
Ans. (b) : The balanced and correct representation of E.AN of complex A = Atomic No of Krypton
product is Atomic number of Fe = 26
Oxidation state of Fe in Na4[Fe(CN)6] complex
+
CoCl ( NH 3 )5  + 5H + 
→ CO 2+ + 5NH 4+ + Cl − x + 6 (–1) = – 4
x=2
520. The effective atomic number for [Rh (H2O)6]3+ Number of donor sites attached with Fe = 6
(atomic number of Rh is 45) is E.AN of [Fe (CN)6]4– = Z – (ON) +2 (CN)
(a) 42 (b) 45 Where, ON = Oxidation no.
(c) 48 (d) 54 CN = coordination number
J & K CET-(2012) Z = atomic number
Ans. (d) : Atomic Number of Rh = 45 = 26 – 2 + 2 (6)
Oxidation state of Rh = +3 = 36
No. of donor site = 6 Hence, complex A is Na4[Fe(CN)]6
Objective Chemistry Volume-II 601 YCT
525. Finely divided iron combines with CO to give: Ans. (d) : Dimethyl glyoximato is a negatively charged
(a) Fe (CO)5 (b) Fe2(CO)9 bidentate ligand.
(c) Fe3 (CO)12 (d) Fe(CO)6
JCECE - 2006
Ans. (a) : Finely divided iron combines with CO ligand
and forms Fe(CO)5.
Fe + 5CO  → Fe(CO)5
531. A ligand can also be regarded as
526. Which does not give a precipitate with AgNO3 (a) Lewis acid (b) Bronsted base
solution? (c) Lewis base (d) Bronsted acid
(a) [Co(NH3)6]Cl3 (b) [Co(NH3)5Cl]Cl2 Karnataka-CET, 2008
(c) [Co(NH3)4Cl2]Cl (d) [Co(NH3)3Cl3] Ans. (c) : A ligand is a species that is capable of
JCECE - 2007 donating an electron pair (s) to the central metal ion.

Ans. (d) : When Cl is present outside the complex then The substances which are capable of donating an
it reacts with AgNO3 and give precipitate of AgCl. electron pair are called Lewis base, so a ligand is also a
[CO(NH3)3Cl3]+AgNO3 →NO ppt. Lewis base.
527. Which of the following ions forms most stable 532. Which complex cannot ionize is solution?
complex compound? (a) [CoCl3 (NH3)3] (b) K4[Fe(CN)6]
(a) Fe3+ (b) Mn2+ (c) K2[Pt(F6)] (d) [Pt(NH3)6]Cl4
(c) Ni2+ (d) Cu2+ Karnataka-CET-2007
JCECE - 2011 Ans. (a) : [CoCl3 (NH3)3] cannot ionize in solution
Ans. (a) : Greater the charge on the central metal ion, because three chloride ions satisfy primary and
greater is the stability of complex. secondary valencies. It will not be precipitated by
528. The π-bonded organometallic compound which addition of AgNO3.
has ethene as one of its component is 533. Which among the following statements are true
(a) Zeise's salt (b) ferrocene for the complex [Co(NH3)6] [Cr(CN)6]?
(c) dibenzene chromium (d) tetraethyltin 1. It is a non-electrolyte
JCECE - 2012 2. The magnitude of the charge on each
complex ion is 3
Ans. (a) : Zeise's salt K[PtCl3(C2H4)] is a π-bonded
organometallic compound. Its structure is 3. The complex will not conduct current
_ 4. The complex will exhibit coordination
H isomerism
5. The magnitude of the charge on each
H C complex ion is 1
K+ Cl (a) 1 and 4 (b) 1 and 2
C H (c) 1 and 3 (d) 3 and 5
Pt (e) 2 and 4
H
Cl Cl Kerala-CEE-2010
Ans. (e) : Let oxidation state of Co = x
529. The complex ion which has no d-electrons in Cr = y
the central metal atom is x +o+ y +6 ×(–1) = 0
(a) [MnO4]– (b) [Co(NH3)6]3+ x+y=6
3–
(c) [Fe(CN)6] (d) [Cr(H2O)6]3+ x = y = +3
JIPMER-2010 Oxidation state of both metal Cr and Co is same i.e. +3
Ans. (a) : In [MnO4]– complex no d-electrons is used Therefore.
because oxidation of Mn is +7. In +7 oxidation state Mn [Co(NH3)6][Cr(CN)6]
does not contain any d-electron. 
Ionisation
→ [Co(NH3)6]3++[Cr(CN)6]3–
Oxidation state of Mn = x
On ionisation two ions are present therefore, is complex
x + 2x(–4) = –1 will conduct current.
x = +7
2 2 6 2 6 5 2 [Co(NH3)6][Cr(CN)6] will exhibit coordination
Mn = 1s 2s 2p 3s 3p 3d 4s isomerism,
Mn7+ 2 2 6 2 6
= 1s 2s 2p 3s 3p
Co ( NH 3 )6  Cr ( CN )6  
530. Which of the following is a negatively charged  

bidentate ligand? Co ( NH 3 )5 ( CN )  Cr ( CN )5 ( NH 3 )   Coordination isomers
(a) Cyano (b) Ethylene diamine 
Co ( NH 3 ) 4 ( CN )2  Cr ( CN ) 4 ( NH 3 ) 2  
(c) Acetato (d) Dimethyl glyoximato  
Karnataka-CET-2012 Hence, statement 2 and 4 are true.
Objective Chemistry Volume-II 602 YCT
534. An example of ambidentate ligand is (a) CoCl3.4NH3 and PtCl4. 4NH3
(a) ammine (b) aquo (b) CoCl3.3NH3 and PtCl4. 5NH3
(c) chloro (d) oxalato (c) CoCl3.6NH3 and PtCl4. 5NH3
(e) Thiocyanato (d) CoCl3.4NH3 and PtCl4. 3NH3
Kerala-CEE-2010 (e) CoCl3.5NH3 and PtCl4. 6NH3
Ans. (e) : Those ligands which has two donor atom but Kerala-CEE-2007
at a time only are donor atom is attached to control
metal this is called ambidentate ligand. Ans. (c) : If complexes contains same electrical
In thiocyanato two donor atom are S and N. conductance when the number of ions formed during
ionisation is same.
Θ Θ
M ← S−C ≡ N M ←  N ≡ C−S CoCl36NH3 forms [Co(NH3)6]Cl3 complex and PtCl4.
Thiocyanato Isothiocyanato 5NH3 forms [pt(NH3)5Cl] Cl3 complex has equal
535. Which of the following statements is not number of ions on ionisation
correct? [Co(NH 3 )6 ]Cl3  →[Co(NH 3 )6 ]3+ + 3Cl−
(a) The complexes [NiCl4]2– and [Ni(CN)4]2–
differ in the state of hybridization of nickel →[Pt(NH 3 )5 Cl]3+ + 3Cl−
[Pt(NH 3 )5 Cl].Cl3 
(b) The complexes [NiCl4]2– and [Ni(CN)4]2– 537. The colour of the solution/precipitate obtained
differ in the magnetic properties in the elemental analysis of an organic
(c) The complexes [NiCl4]2– and [Ni(CN)4]2– compound and the molecule/ion responsible for
differ in geometry the colour are given below. Choose the
(d) The complexes [NiCl4]2– and [Ni(CN)4]2– of incorrectly matched pair.
differ in primary valencies (a) Prussian blue - Fe4[Fe(CN)6]3. xH2O
(e) Nickel ion has the same secondary valency in (b) Black - pbS
the complexes [NiCl4]2– and [Ni(CN)4]2–
(c) Violet colour - [Fe(CN)5NOS]4-
Kerala-CEE-2007
(d) Blood red colour - [Fe(SCN)]2+
Ans. (d) : In [NiCl4]2– and [Ni(CN)4]2– complex nickel
is present in +2 oxidation state. (e) Yellow - (NH4)2 MoO4
Cl– is weak field ligand then pairing of metal d-electron Kerala-CEE-2013
is not possible [NiCl4]2– Ans. (e) : Incorrectly matched pair is (NH4)2MoO4 of
Ni2+ : d8 yellow colour.
While (NH4)3PO4. 12MoO3 is yellow in colour.
538. Which of the following carbonyls will have the
strongest C – O bond?
(a) Mn ( CO )6
+
(b) Cr(CO)6
Geometry of [NiCl4] ⇒Tetrahedral
2–

(c) V ( CO )6

Number of unpaired electron = 2 (d) Fe(CO)5
Magnetic moment = 2 ( 2 + 2 ) = 8 = 2.84 B.M. (AIPMT -2011)
Primary valence is satisfied by oxidation state of metal. Ans. (a) : When electron density on metal is high then
Here primary valency is +2 tendency of back bonding is also high. Due to
Secondary valency is satisfied by coordination numbers backbonding lone pair of electron from filled metal
of complex. Here secondary valency is 6. orbital is donated to empty antibonding (π*) orbital of
[Ni(CN)4]2– CO lignad. So, bond strength of C–O decrease and bond
Ni2+ : d8 length increase. Hence, [Mn(CO)6]+ has strength C–O
bond because metal has low electron density due to
which backbonding tendency decreases.
539. Which one of the following is an inner orbital
complex as well as diamagnetic in behaviour?
Geometry ⇒ Square planar (a) [Zn(NH3)6]2+ (b) [Cr(NH3)6]3+
3+
Magnetic moment = oxidation state = +2 (c) [Co(NH3)6] (d) [Ni(NH3)6]2+
Secondary valency = Coordination No. = 6 (Atomic number : Zn = 30, Cr = 24, Co = 27, Ni
Hence, the complexes [NiCl4]2– and [NiCl4]2– of same in = 28)
primary valency of nickel. (AIPMT -2005)
536. Both Co3+ and Pt4+ have a coordination number Ans. (c) : In [Co(NH3)6]3+ cobalt has oxidation state +3
of six. Which of the following pairs of
NH3 is strong field ligand with oxidation number of
complexes will show approximately the same
electrical conductance for their 0.001M metal +3. So, pairing of metal d-electrons are possible.
aqueous solutions? Co3+ : d6

Objective Chemistry Volume-II 603 YCT


Ans. (b) : The following are some of the advantages of
crystal field theory:–
• This theory can be used to describe the stability of
Here, inner d orbital is used for hybridisation then complexes. The greater the crystal field splitting
complex is known as inner orbital complex. energy, the greater the stability.
No unpaired electron is present in [Co(NH3)6]3+ • Complexes colour and spectra can be explains
complex so it behave like diamagnetic substance. complexes magnetic properties.
• This theory explains complexes magnetic properties.
6. Crystal Field Theory 543. According to crystal field theory, when ligands
approach the metal atom or ion in an
540. [Fe(CN)6]3- should be an inner orbital complex octahedral field, the d orbits that undergo
Ignoring the pairing the pairing energy, the increase in energy are
value of crystal field stabilization energy for (a) d xy , d z 2 (b) d yz , d z 2
this complex is (-) ______ ∆o. (Nearest integer)
(c) d x 2 − y2 ,d z2 (d) d xz , d x 2 − y2
JEE Main 29.07.2022, Shift-I
Ans. (2) : [Fe (CN)6]3– AP-EAPCET-12.07.2022, Shift-I
CN– is strong field ligand, Ans. (c) : In octahedral symmetry the -orbitals split into
Fe+3 = 3d5  t 52g eog  two sets with an energy difference, where the dxy, dyz,
dzx orbitals will be lower in energy than the d x 2 − y2 ,d z2
which will have higher energy because the former group
is farther from the ligands than the latter therefore
experience less repletion.
CFSE = (–0.4∆0) = –2∆0 544. The crystal field splitting energy (CFSE) for
541. Octahedral complexes of copper (II) undergo [CoCl6]4- is about 18000 cm–1. What would be
structural distortion (Jahn-Teller). Which one the CFSE value for [CoCl4]2-?
of the given copper (II) complexes will how the (a) 1800 cm–1 (b) 8000 cm–1
maximum structural distortion ? (c) 16000 cm –1
(d) 2000 cm–1
(en-ethylenediamine; H 2 N − CH 2 − CH 2 − NH 2 )
Karnataka CET-17.06.2022, Shift-II
(a) Cu ( H 2O )6  SO 4 (Odisha NEET 2019)
(b) Cu ( en )( H 2 O 4 )  SO 4 AMU-2018
Ans. (b) : [CoCl6]4– is a octahedral complex. The CFSE
(c) cis − Cu ( en )2 Cl2  of the compound is 18000 cm–1 i.e. ∆o = 18000 cm–1
[COCl4]2– is a tetrahedral complex.
(d) trans −  Cu ( en ) 2 Cl 2 
Now, from relation between ∆oh and ∆t is –
JEE Main 29.07.2022, Shift-II 4
Ans. (a) : There is unsymmetric filling of eg subset of ∆ t = ∆o
9
Cu+2 ion, while there is symmetrical distribution in t2g
set, if the complex has same ligand there will be equal 4
∆t = ×18000cm−1
repulsion which leads to symmetrical bond length along 9
t2g, but due to uneven filling of electron in eg subset, ∆t = 8000cm−1
either octahedral will be elongated or compressed.
t2g → (dxz, dyz, dxy) 545. For the crystal field splitting in octahedral
complexes,
( )
eg → d z 2 and d x 2 − y2 (a) the energy of the eg orbitals will decrease by
(3/5) ∆0 and of the t2g will increase by (2/5) ∆0
542. The crystal filed theory is successful in
explaining which of the following? (b) the energy of the eg orbitals will increase by
I. Ligands as point charges; (3/5) ∆0 and that of the t2g will decrease by
II. formation and structures of complexes. (2/5) ∆0
III. colour, (c) the energy of the eg orbitals will increase by
IV. Magnetic properties (3/5) ∆0 and that of the t2g will increase by
V. covalent character of metal-ligand bonding (2/5) ∆0
(a) I, II, III, only (d) the energy of the eg orbitals will decrease by
(b) II, III, IV only (3/5) ∆0 and that of the t2g will decrease by
(c) III, IV, V only (2/5) ∆0
(d) II, IV, V only Karnataka-CET-2021
AP-EAMCET-04.07.2022, Shift-II Kerala-CEE-29.08.2021
Objective Chemistry Volume-II 604 YCT
Ans. (b) : For the crystal filed splitting in octahedral c = speed of light (3 × 108ms−1)
complexes, the energy of the eg orbital’s will increase λ = Wavelength of absorption
by (3/5)∆o ≈ 0.6∆o and that of the t2g will decrease by 6.6 × 10−34 × 3 × 108 (Js)(ms −1 )
(2\5) ∆o ≈ 0.4∆o ∴ ∆o =
498 ×10−9 m
or ∆o = 3.97 × 1019 J
or ∆o ≈ 4 × 10−19 J
548. Arrange the following cobalt complexes in the
order of increasing crystal field stabilization
546. Among the following complexes the one which energy (CFSE) value.
shows zero crystal field stabilisation energy [CoF6]3–, [Co(H2O)6]3+,
(CFSE) is (A) (B)
(a) [Mn(H2O)6]3+ (b) [Fe(H2O)6]3+ [Co(NH3)6]3+ and [Co(en)3]3+
2+
(c) [Co(H2O)6] (d) [Co(H2O)6]3+ (C) (D)
Kerala-CEE-29.08.2021 Choose the correct option.
(NEET-2014) (a) A < B < C < D (b) B < A < C < D
Ans. (b) : (c) B < C < D < A (d) C < D < B < A
[JEE Main 2021 26 Aug Shift-II]
(a) [Mn(H2O)6]3+ (b) [Fe(H2O)6]3+
x + 6 (0) = +3 x + 6 (0) = +3 Ans. (a) : The complex contains –F, −H2O, − NH3 and
en ligands. The CFSE of complex depends upon the
x = +3 x = +3
nature of ligand. Higher the field of ligand, more will be
∴ Mn3+ = [Ar]3d4 ∴ Fe3+ = [Ar]3d5 CFSE. The order of field of ligand are F < H2O < NH3 <
en.
Thus, the order of CFSE will be –
A<B<C<D
549.The crystal field stabilization energy (CFSE) and
magnetic moment (spin-only) of an octahedral
CFSE = [−0.4 × 3 + CFSE = [−0.4 × 3 + 0.6 × aqua complex of a metal ion (M+) are –0.8∆0
0.6 × 1]∆o 2]∆o and 3.87 BM, respectively.
CFSE = −0.6 ∆o CFSE = [−1.2 + 1.2 ]∆o= identify (M2+).
CFSE = 0 (a) V3+ (b) Cr3+
4+
(c) [Co(H2O)6]2+ (d) [Co(H2O)6]3+ (c) Mn (d) Co2+
x + 6 (0) = +2 x + 6(0) = +3 [JEE Main 2021, 1 sep, Shift-II]
x = +2 x = +3 Ans. (d) : The metal ion is Co2+ because it form the
∴Co2+ = [Ar]3d7 ∴ Co3+ = [Ar]3d6 aqua complex with +2 oxidation state.
Co2+ = [Ar]3d7 (Hight spin)

CFSE = [−0.4 × 5 + 0.6 CFSE = [−0.4 × 4 + 0.6 × Unpaired electron (n) = 3


× 2]∆o = −0.8∆o 2]∆o ∴ µs = n(n + 2)
CFSE = −0.4 ∆o
Hence, (Fe(H2O)6)3+ has zero CFSE. µs = 3(3 + 2)
3+
547. [Ti(H2O)6] absorbs light of wavelength µs = 3.87 BM
498 nm during a d-d transition. The octahedral CFSC = [−0.4 × Nt2g + 0.6 × N eg ]∆o
splitting energy for the above complex is ....... × CFSE = [–0.4 × 5 + 0.6 × 2] ∆
10–19 J. o
CFSE = −0.8∆o
(Round off to the nearest integer).
[h = 6.626×10–34 Js, c = 3×108 ms–1] 550. The correct order for wavelengths of light
absorbed in the complex ions CoCl ( NH 3 )5  ,
2+
[JEE Main 2021, 16 March Shift-II]
−9
Ans. (4) : Given, λ = 498nm = 498 × 10 m
Co ( NH 3 )6  and Co ( CN )6  is
3+ 3+
∆o = ?
Now, (a) [CoCl(NH3)5]2+ > [Co(NH3)6]3+ > [Co(CN)6]3–
hc (b) [Co(NH3)6]3+ > [Co(CN)6]3– >[CoCl(NH3)5]2+
∆o or E =
λ (c) [Co(CN)6]3– >[CoCl(NH3)5]2+ >[Co(NH3)6]3+
Where - ∆o = octahedral splitting energy (d) [Co(NH3)6]3+ >[CoCl(NH3)5]2+ >[Co(CN)6]3–
h = Planck’s constant Kerala-CEE-29.08.2021

Objective Chemistry Volume-II 605 YCT


2–
Ans. (a) : The absorption of light in the complex ions 554. [Pd(F)(Cl)(Br)(I)] has n number of
depends upon the nature of metal, oxidation number and geometrical isomers. Then, the spin-only
ligand (spectrochemical series). The compounds have magnetic moment and crystal field stabilization
the same metal as well as same oxidation state but energy [CFSE] of [Fe(CN)6]n–6 respectively, are
different in ligands. Weak field ligands (WFL) have the [Note-Ignore the pairing energy]
lower energy gap between t2g and eg orbital. So, low (a) 1.73 BM and –2.0∆0
amount of energy required and high wavelength of light (b) 2.84 BM and –1.6∆0
used for absorption. Hence the correct order of (c) 0 BM and –2.4∆0
wavelength of light absorbed are that (d) 5.92 BM and 0∆0
2+ 3+ 3+
CoCl ( NH 3 )5  > Co ( NH 3 )6  > Co ( CN )6  . [JEE Main 2020, 9 Jan Shift-II]
Ans. (a) : [Pt(F)(Cl)(Br)(I)]2– is square planar type of
551. Consider the elements Mg, Al, S, P and Si the [MABCD].[MABCD] type complex exist in three
correct increasing order of their first ionization isomeric forms. Hence, the value of n will be 3.
enthalpy is : n−6
 Fe (CN ) 
(a) Al < Mg < Si < S < P  6 

(b) Mg < Al < Si < P < S 3−6


or  Fe (CN )6 
(c) Mg < Al < Si < S < P
−3
(d) Al < Mg < S < Si < P or  Fe (CN )6 
JEE Main 24.02.2021, Shift-I
Let, x be the oxidation state of Fe.
Ans. (a) : The first ionization energy of an element is x + 6 (–1)= –3
the energy needed to remove the outermost or highest x = +3
energy electron from a neutral atom in the gas phase. 3+
As we know, the first ionization energy in the periodic Fe = [ Ar ]3d 5 (Pairing occure because CN is SFL)
table is decreases down the group and increases in the
left to right in period. Al has less ionization energy than
other because the removal of one electron from 3p-
orbital it goes to stable configuration.
The same reason occure between S and P i.e. sulphur Unpaired electron (n) = 1
has less ionization energy than phosphorus.
Hence, the correct order will be– Al < Mg < Si < S < P. ∴ µs = n ( n + 2)BM
552. On complete reaction of FeCl3 with oxalic acid µs = 1(1 + 2)BM
in aqueous solution containing KOH, resulted
in the formation of product A. The secondary µs = 1.732BM
valency of Fe in the product A is ______.
CFSE = −0.4× N t 2 g + 0.6× N eg  ∆0
(Round off to the Nearest Integer).  
JEE Main 17.03.2021, Shift-II CFSE = [−0.4×5 + 0.6× 0]∆0
Ans. (6) : CFSE = −2.0∆0
When FeCl3 reacted with oxalic acid and KOH then the
following product formed- 555. Among the statements (A)-(D), the incorrect
ones are
FeCl3+3H2C2O4+6KOH→K3[Fe(C2O4)3]+3KCl+6H2O
(A) Octahedral Co (III) complexes with strong,
Secondary valency= 6 field ligands have very high magnetic
As we know that secondary valency refers to the co- moments
ordination number. (B) When ∆0<P, the d-electron configuration of
553. Accordingly to the crystal field theory, for an 4
Co(III) in an octahedral complex is t eg ,eg2
octahedral field, the energy of two eg ordinals
will be increased by (C) Wavelength of light absorbed by [Co(en)3]3+
is lower than that of [CoF6]3–
1 2 (D) If the ∆0 for an octahedral complex of Co(III)
(a)   ∆ 0 (b)   ∆ 0
5 5 is 18,000 cm–1, the ∆t for its tetrahedral
3 4 complex with the same ligand will be 16,000
(c)   ∆ 0 (d)   ∆ 0 cm–1
5 5 (a) B and C only (b) A and B only
TS EAMCET 10.08.2021, Shift-I (c) A and D only (d) C and D only
Ans. (c) : According to the crystal field theory for [JEE Main 2020, 7 Jan Shift-II]
octahedral field the energy levels of eg are higher, 0.6∆o
Ans. (c) : (a) CO = [ Ar ] 3d 6
3+

3 2 Pairing occurred because ligand is strong field in


or   ∆o while t2g is lower 0.4∆o. or   ∆o
5 5 nature.

Objective Chemistry Volume-II 606 YCT


CFSE =  −0.4 × N t 2 g + 0.6 × N eg  ∆ o
 
CFSE = [ −0.4 × 4 + 0.6 × 2] ∆ o
CFSE = [ −1.6 + 1.2] ∆ o
CFSE = −0.4∆ o
Here, n = 0 that means it forms the diamagnetic 557. The values of the crystal field 6
stabilization
character. Thus, it's magnetic moment will be zero. energies for a high spin d metal ion in
octahedral and tetrahedral fields respectively,
are
(b) If crystal field stabilization energy of octahedral
complex is less than the pairing energy then following (a) –0.4 ∆0 and –0.27 ∆t (b) –1.6 ∆0 and –0.4 ∆t
condition take place- (c) –2.4 ∆0 and –0.6 ∆t (d) –0.4 ∆0 and –0.6 ∆t
[JEE Main 2020, 5 Sep Shift-II]
6
Ans. (d) : For d (High spin)→ splitting of d-orbital in
octahedral field-

t 42g eg2
Hence, statement is true.
(c) Higher the field of ligand, higher the CFSE of the
CFSE =  −0.4 × N t 2 g + 0.6 × N eg  ∆ o
compound and have lower the wavelength of  
absorption. Here, [Co(en)3]3+ contains the strong field CFSE = [ −0.4 × 4 + 0.6 × 2] ∆ o
ligand due to which it has the more CFSE than the
[Co(F)6]3– and it absorbs the minimum wavelength of CFSE = −0.4∆ o
light. Hence, statement is true. For d6 → splitting of d-orbital in tetrahedral field-
(d) For C3O+ −
∆ o = 18,000cm −1
4
∴ ∆ t = ∆o
9
4
∆ t = ×18000
9 CFSE =  −0.6 × N e + 0.4 × N t 2  ∆ t
−1
∆ t = 8000cm CFSE = [ −0.6 × 3 + 0.4 × 3] ∆ t
Hence, statement is false. CFSE = [ −1.8 + 1.2] ∆ t
Thus, statement (a) and (d) are incorrect.
556. The crystal field stabilisation energy (CFSE) of CFSE = −0.6∆ t
[CoF3(H2O)3] (∆0 < P) is 558. Consider that a d6 metal ion [M2+] forms a
(a) –0.8 ∆0 (b) –0.8 ∆0 +2P complex with aqua ligands and the spin only
(c) –0.4 ∆0 (d) –0.4 ∆0 + P magnetic moment of the complex is 4.90 BM.
[JEE Main 2020, 4 Sep Shift-II] The geometry and the crystal field stabilisation
Ans. (c) : [CoF3(H2O)3] energy of the complex is
Let, X be the oxidation state of Co. (a) tetrahedral and –1.6∆t + 1P
x + 3 (–1) + 3(0) = 0 (b) octahedral and–2.4∆0 + 2P
x = +3 (c) octahedral and –1.6∆0
CO = [ Ar ] 3d
3+ 6 (d) tetrahedral and –0.6∆t
[JEE Main 2020, 2 Sep Shift-I]
Ans. (d) : The metal ion (M2+) forms the complex with
aqua ligands that means it contain the weak field ligand.
It is a iron metal which has the following electronic
configuration-
Fe 2t = [ Ar ] 3d 6
If metal forms the tetrahedral complex then it contains
the four unpaired electron.
Objective Chemistry Volume-II 607 YCT
∵ Pairing occure because
'Ru' belongs to the 4d Fe2+ = [Ar]3d6 (High spin)
-series metal.
Ru2+[Kr]4d655o

CFSE =  −0.6 × N e + 0.4 × N t 2  ∆ t


CFSE = [ −0.6 × 3 + 0.4 × 3] ∆ t
CFSE = [ −1.8 + 1.2] ∆ t
CFSE = −0.6∆ t
Hence, the geometry and the crystal field stabilisation
energy of the complex is tetrahedral and –0.6∆t
respectively.
559. The electronic spectrum of [Ti(H2O)6]3+ shows
a single broad peak with a maximum at 20,300
cm–1. The crystal field stabilization energy 561. The theory that can completely/properly
(CFSE) of the complex ion, in kJ mol–1, is explain the nature of bonding in [Ni(CO)4] is:
(a) 145.5 (b) 242.5 (a) Werner's theory
(c) 83.7 (d) 97 (b) Crystal field theory
[JEE Main 2020, 3 Sep Shift-I] (c) Valence bond theory
Ans. (d) : Given: ∆o = 20,300 cm–1 (d) Molecular orbital theory
CFSE = ? [JEE Main 2020, 7 Jan Shift-I]
∵ 1Kj/mol = 83.7 cm–1 Ans. (d) : [Ni(CO)4] is a tetrahedral complex in which
[Ti(H2O)6]3+ bonding in complex properly explain by the ‘Molecular
orbital theory’ (MOT).
Let, x be the oxidation state of Ti.
∴ x+6(0) = +3 562. Which of the following is the correct order of
increasing field strength of ligands to form
x = +3 coordination compounds?
(a) SCN– < F– < C 2 O 24− < CN–
(b) SCN– < F– < CN– < C 2 O 24−
(c) F– < SCN– < C 2 O 24− < CN–
(d) CN– < C 2 O 24− < SCN– < F–
(NEET 2020)
CFSE =  −0.4 × N t 2 g + 0.6 × N eg  ∆ o
  Ans. (a) : The spectrochemical series is given as-
CFSE = [ −0.4 ×1 + 0.6 × 0] ∆ o I− < Br− < S− < SCN− < Cl− < N 3− < F− < Urea < OH− <
Or etOH < ox < O2− < H2O < EDTA < NCS− < Py, < NH3
CFSE = 0.4∆ o (ignoring the negtive sign ) < en < bpy, phen < NO −2 < PPh3 < CH 3− < C6 H 5− < CN−
CFSE = 0.4 × 20,300cm −1 CO.
Thus, option (a) has the correct order
1
CFSE = 0.4 × 20,300 × kJ / mol 563. Crystal field splitting in octahedral
83.7 coordination entities, 'eg' set of orbital's are
CFSE = 97 kJ mol–1 (a) dxy, dyz, dzx (b) d x 2 − y2 , , d z 2 , dyz
560. The d-electron configuration of [Ru(en)3]Cl2
and [Fe(H2O)6]Cl2, respectively are (c) d x 2 − y2 ,d z2 (d) d z 2 , dxy
(a) t 62g e0g and t 62g e0g (b) t 62g e0g and t 42g eg2
JHARKHAND – 2019
(c) t 42g eg2 and t 62g eg0 (d) t 42g eg2 and t 2g
4 2
eg
Ans. (c) : Crystal field splitting in octahedral
[JEE Main 2020, 3 sep Shift-II] coordination entities, 'eg' set of orbital's are d x 2 − y2 ,d z 2 
Ans. (b) :
[Ru(en)3]Cl2 [Fe(H2O)6]Cl2
Let, x be the oxidation Let, x be the oxidation
state of Ru. state of Fe.
∴ x+3(0)+2(–1)=0 ∴ x+3(0)+2(–1)=0
x = +2 x = +2

Objective Chemistry Volume-II 608 YCT


564. The metal d-orbitals that are directly facing the Let, x be the oxidation state of Fe.
ligands in K3[Co(CN)6] are x + 6(0) + 2 (–1) = 0
(a) d xz , d yz and d z 2 (b) d x 2 − y2 , and d z2 x = +2
2+
(c) d , d and d (d) d and d 2 2 Fe = [ Ar ]3d 6
xy xz yz xz x −y

[JEE Main 2019, 12 Jan Shift-I]


Ans. (b) : K3[Co(CN)6]
Let, x be the oxidation state of Co.
3(+1) + x + 6 (–1) = 0
x = +3 CFSE = −0.4× N t 2 g + 0.6× N eg  ∆0
 
Co3+ = [Ar]3d6
Pairing occure because CN is SFL. CFSE = [−0.4× 4 + 0.6× 2]∆0
CFSE = −0.4∆0
and, for K 2  Ni (Cl)4 
2(+1) + x + 4(−1) = 0
x = +2
2+
Here, d x 2 –y2 and d z 2 are directly facing to the metal. Ni = [ Ar ]3d8
565. The complex that has highest crystal field
splitting energy (∆), is
(a) [Co(NH3)5Cl]Cl2
(b) [Co(NH3)5(H2O)]Cl3
(c) K3[Co(CN)6] CFSE = −0.6× N e + 0.6× N t 2  ∆t
(d) K2(CoCl4)
[JEE Main 2019, 9 Jan Shift-II] CFSE = [−0.6× 4 + 0.6× 4]∆t
Ans. (c) : It is clear that the CFSE of tetrahedral CFSE = −0.8∆ t
complex are always less than the octahedral complex.
567. The complex ion that will lose its crystal field
Thus, option (d) has less CFSE than option (a) , (b) and
(c) because it is a tetrahedral complex. K3[Co(CN)6] stabilisation energy upon oxidation of its metal
forms the low spin complex because it contains the to +3 state is
SFL.
K 3 Co (CN )6 
3(+1) + x + 6(−1) = 0
x = +3
(a) [Co(phen)3]2+ (b) [Ni(phen)3]2+
Co3+ = [ Ar ]3d 6 2+
(c) [Zn(phen)3] (d) [Fe(phen)3]2+
[JEE Main 2019, 12 April Shift-I]
Ans. (d) : Here, phenanthroline is a strong field ligand.
−e−2+ +3
  →  Fe (phen )3 
CFSE = −0.4× N t 2 g + 0.6× N eg  ∆o  Fe (phen )3   
  Oxidation state =+2 Oxidation state =+3
CFSE = [−0.4× 6 + 0.6× 0]∆o
E.C. = [ Ar ]3d 6 E.C. = [ Ar ]3d5
CFSE = −2.4∆o
Hence, K 3 Co (CN )6  has highest crystal field splitting
energy.
566. The crystal field stabilization energy (CFSE) of
[Fe(H2O)6]Cl2 and K2[Ni(Cl)4], respectively, are
(a) –0.4 ∆0 and –1.2 ∆t (b) –0.4 ∆0 and –0.8 ∆t
(c) –2.4 ∆0 and –1.2 ∆t (d) –0.6 ∆0 and –0.8 ∆t Hence, CFSE decreases after the oxidation.
[JEE Main 2019, 10 April Shift-II] 568. Complete removal of both the axial ligands
Ans. (b) : [Fe(H2O)6] Cl2 and K2[NiCl4] are octahedral (along the z-axis) from an octahedral complex
and tetrahedral complex respectively. leads to which of the following splitting
[Fe(H2O)6] Cl2 patterns? (relative orbital energies not on scale)

Objective Chemistry Volume-II 609 YCT


Ans. (a) : The relation between crystal field
stabilisation energy for octahedral (∆o) and tetrahedral
(∆t) complexes are –
4
∆ t ≈ ∆o
9
571. Which combines with Fe2+ to form brown
complex?
(a) NO (b) N2O
(c) N2O3 (d) N2O5
CG PET -2019
Ans. (a) : A common nitrate test is also known as the
[JEE Main 2019, 12 April Shift-II] brown ring test.
2+ 2+
Ans. (a) : Octahedral complex contains the two axial  Fe ( H 2 O)  + NO  →  Fe (H 2O)5 NO + H 2 O
 6 
ligands, if we remove the both axial ligand then it Thus, NO combines with Fe2+ to form the brown
converted into square planar field then following energy complex.
diagram take place–
572. [Co(C2O4)3]3– is a:
(a) Low spin complex (b) Paramagnetic
(c) High Spin (d) sp3 d2 hybridized
AIIMS 25 May 2019 (Evening)
3−
Ans. (a) : Co (C 2 O 4 )3 
Let, x be the oxidation state of Co.
x = 3 (–2) = –3
x = +3
C2O4 is a bidentate ligand and this ligand form the
octahedral complex. The complex exhibit the low spin
complex because of bidentate nature of the ligand that's
why pairing of electron occure in the inner orbital.
569. What is the correct electronic configuration of
the central atom in K4[Fe(CN)6] based on
crystal field theory?
(a) e 4 t 22 (b) t 42g eg2
Hence, it form the low spin complex and it is
(c) t 62g e0g (d) e3 t 32 diamagnetic in nature. The hybridisation of the
2 3
(NEET 2019) compound is d sp .
Ans. (c) : K4 (Fe(CN)6) 573. Which of the following complexes has
Let, maximum CFSE?
X be the oxidation state of fe (a) K 3  Fe ( CN )6  (b) K 3 [ CoCl6 ]
∴ 4(+1) + x + 6(−1) = 0
x = +2 (c) K 3 [ CoF6 ] (d) K 3 Co ( CN )6 
Fe2+ = [Ar]3d6 AIIMS 26 May 2019 (Evening)
Pairing occure because CN is strong field ligand (SFL) Ans. (d) : (a) K3[Fe(CN)6] (Low spin complex)
3 (+1) + x + 6 (–1) = 0
x = +3

Here, t 62g eg0 is the correct configuration K4[Fe(CN)6].


CFSE = −0.4 + N t 2 g + 0.6N eg  ∆0
570. Crystal field splitting energy for octahedral  
(∆o) and tetrahedral (∆t) complexes is related as CFSE = [−0.4×5 + 0.6× 0]∆0
4 1 CFSE = −2.0∆0
(a) ∆ t ≈ ∆ o (b) ∆ t ≈ ∆ o
9 2 Similarly,
4 (b) K3[CoCl6] (High spin complex)
(c) ∆ o ≈ 2∆ t (d) ∆ o ≈ ∆ t
9 3(+1) + x + 6 (–1) = 0
AMU-2019 x = +3

Objective Chemistry Volume-II 610 YCT


576. Which one of the following statement is correct
for d4 ions [P = pairing energy]
(a) When ∆o > P, low-spin complex form
(b) When ∆o < P, low-spin complex form
CFSE = [–0.4 × 4 + 0.6 × 2] ∆0 (c) When ∆o > P, high-spin complex form
CFSE = [–1.6 + 1.2] ∆0 (d) When ∆o < P, both high-spin and low-spin
CFSE = – 0.4 ∆0 complexes form
(c) K3[CoFb] High spin complex) TS EAMCET-2017
3 (+1) x + 6 (–1) = 0 Ans. (a) : For
x = +3
(pairing energy)
When value of ∆o is greater than pairing energy pairing
of electron takes place hence low-spin complexes are
formed.
CFSE = [–0.4 × 4 + 0.6 × 2] ∆0 577. The correct electronic configuration of d4 in
CFSE = –0.4 ∆0 low spin state according to Crystal Field
Splitting theory is
(d) K 3 Co (CN )6  (Low spin complex)
(a) t 42g eog (b) t 32g e1g
3 (+1) x + 6 (–1) = 0
x = +3 (c) t 22g eg2 (d) t12g e3g
J & K CET-(2017)
Ans. (a) : For d4, low spin complex –

CFSE = [–0.4 × 6 + 0.6 × 0] ∆o


CFSE = –2.4 ∆o
Here, – and + signs indicate decrease and increase in the
energy. Hence, K 3 Co (CN )6  has maximum CFSE. 4
d-electronic configuration = t 2g ego
574. Which of the complexes is expected to have 578. According to crystal field theory, the M–L
lowest ∆ 0 value? bond in a complex is
3+ 3+
(a) [Co(NH 3 )6 ] (b) [Rh(NH 3 )6 ] (a) partially covalent (b) purely ionic
3+ 3− (c) purely covalent (d) purely coordinate
(c) [Ir(NH 3 )6 ] (d) [CoF6 ]
Karnataka-CET-2017
BCECE-2018
Ans. (b) : According to crystal field theory (CFT), the
3−
Ans. (d) : [CoF6 ] has lowest ∆o value because it bonding between control metal cation and is not
contains the weak field ligand and weak field ligand covalent but it is regarded as purely electrostatic or
forms the high spin complex with 3d-series metal. As purely ionic.
we know that the high spin complex has lower CFSE 579. Correct increasing order for the wavelengths of
(∆o) than low spin complex. absorption in the visible region for the
575. Identify the correct increasing order of crystal complexes of Co3+ is
field stabilization energy value for the given (a) [Co(H2O)6]3+, [Co(en)3]3+, [Co(NH3)6]3+
complexes. (b) [Co(H2O)6]3+, [Co(NH3)6]3+, [Co(en)3]3+
3+ 3+ 3+
(a) [Ir(NH3)6] < [Rh(NH3)6] < [Co(NH3)6] (c) [Co(NH3)6]3+, [Co(en)3]3+, [Co(H2O)6]3+
3+ 3+ 3+
(b) [Rh(NH3)6] < [Co(NH3)6] < [Ir(NH3)6] (d) [Co(en)3]3+, [Co(NH3)6]3+, [Co(H2O)6]3+
3+ 3+ 3+
(c) [Co(NH3)6] < [Ir(NH3)6] < [Rh(NH3)6] (NEET 2017)
(d) [Co(NH3)6]3+ < [Rh(NH3)6]3+ < [Ir(NH3)6]3+ Ans. (d) : The wavelength of absorption in the visible
J & K CET-(2018) range for the complex are depends upon the ligand field,
Ans. (d) : The crystal field stabilisation energy depends higher the field of ligand, lower will be wavelength of
upon the following factor- absorption the ligand field ordere are H2O < NH3 < en.
(1) Charge on metal, (2) Size of metal Thus, the order of wavelength of absorption will be
(3) Nature of ligand [Co(en)3]3+ < [Co(NH3)6]3+ < [Co(H2O)6]3+
Here, charge of metal and nature of ligand are same i.e. 580. [Ti(H2O)6]2+ is a 3d2 system, the value of crystal
+3 and NH3. The unsimilarities arises in the size of field stabilization energy decreases by
metal, as we know that as well as size of metal incrases,
(a) 4×10–1 ∆ 0 (b) 4.O ∆ 0
the CFSE of complex is also increases. Hence, the
correct order or CFSE will be- (c) 8.O ∆ 0 (d) 8×10–1 ∆ 0
3+ 3+ 3+
[Co(NH3)6] < [Rh(NH3)6] < [Ir(NH3)6] COMEDK 2015
Objective Chemistry Volume-II 611 YCT
2+ (c) Transition metal in Transition metal in
Ans. (d) : The given compound is Ti ( H 2 O )6  . The
+3 state +2 state
oxidation number of metal is +2. The electronic (d) II A element Transition metal in
configuration of Ti2+ is [ Ar ] 3d 2 . Hence the splitting of +2 state
d-orbital can be written as- AP - EAMCET(MEDICAL) - 2009
Ans. (a) : The general formula of the spinal is AB2O4
where, A represents divalent metal ion such as Mg, Fe,
Ni, Mn and B represents trivalent metal ion such as Al,
Fe, Cr, Ti, etc. Generally MgAl2O4 as spinal is mostly
occur in nature.
585. Crystal field splitting energy for high spin d4
CFSE = – 0.4N +2g + 0.6Neg octahedral complex is
CFSE = – 0.4 × 2 + 0.6 × 0 (a) –1.2 ∆ o (b) –0.6 ∆ o
CFSE= – 0.8 (c) –0.8 ∆ o (d) –1.6 ∆ o
or CFSE = 8 × 10 –1 ∆° Karnataka NEET 2013
4
581. The complex ion having minimum magnitude Ans. (b) : The given d=orbital is d for high spin.
of ∆0 (CFSE) is
(a) [Cr(CN)6]3– (b) [Co(CN)6]3+
3–
(c) [Co(Cl)6] (d) [Cr(H2O)6]3+
Karnataka-CET-2015
Ans. (c) : The CFSE of complex depends upon the
nature of ligands, lower the field of ligand , less will be
CFSE. Here, Cl has the less field of energy due to
CFSE = – 0.4 ∆ ° N t 2 g + 0.6∆ ° N eg
which it has the minimum magnitude of ∆o(CFSE).
582. The correct ascending order of ligand field CFSE = –0.4∆ ° × 3 + 0.6∆ ° × 1
strength of the given ligands is CFSE = –1.2∆ ° + 0.6∆ °
(a) F– < I– < CN– < H2O < CO
CFSE = – 0.6∆ °
(b) I– < F– < H2O– < CO < CN–
– – – – 586. Assertion : Copper sulphate solution is not
(c) I < F < H2O < CN < CO
(d) F– < H2O– < I– < CN– < CO stored in zinc vessel.
– –
(e) F < I < CO < H2O < CN – Reason: Zinc forms complex with CuSO4.
Kerala-CEE-2015 (a) If both Assertion and Reason are correct and
the Reason is a correct explanation of the
Ans. (c) : According to the spectrochemical series, the Assertion.
ascending order of ligand field strength of the ligands
(b) If both Assertion and Reason are correct but
are-
Reason is not a correct explanation of the
I– < F– < H2O– < CN– < CO Assertion.
583. In which of the following octahedral complexes (c) I the Assertion is correct but Reason is
of Co (At. No. 27), will the magnitude of ∆0 be incorrect.
the highest? (d) I both the Assertion and Reason are incorrect.
(a) [Co(CN)6]3– (b) [Co(C2O4)3]3– (e) If the Assertion is incorrect but the Reason is
(c) [Co(H2O)6]3+ (d) [Co(NH3)6]3+ correct.
VITEEE- 2011 [AIIMS-2007]
Ans. (a) : The given compounds are : Ans. (a) : Copper sulphate solution is not stored in zinc
(i) [Co(CN)6]3– (iii) [Co(H2O)6]3+ vessel because zinc is more reactive than copper,
3–
(ii) [Co(C2O4)3] (iv) [Co(NH3)6]3+ therefore reaction will be take place and forms the zinc
The crystal field stabilisation is also depends upon the sulphate.
spectrochemical series i.e. which type of ligand attached The reaction is illustrated below –
to the central metal atom. As we know that CN is the Zn + CuSO 4  → ZnSO 4 + Cu
strong field ligand and extra pairing occurred in the t2g
orbital. So, the CFSE will be more. 587. A chemist wants to determine the molecular
geometry of the [CoCl4]2- ion. Which of the
584. The formula of 'spinal' is AB2O4, where A is following gives the best suggestion for a
......., and B is ......... . measurement and for the interpretation of that
A B measurement?
(a) II A element III A metal (a) Using aborption spectroscopy, measure λmax
(b) III A metal II A element calculate ∆0 for octahedral geometry.
Objective Chemistry Volume-II 612 YCT
(b) Measure the molecule's magnetic moment and 589. Which complex of Co2+ will have the weakest
use the result to estimate the number of crystal field splitting ?
unpaired spins in the molecule. If this number (a) [Co(CN)6]4– (b) [CoCl6]4–
is low, geometry is likely to be square planar; (c) [Co(en)3] 2+
(d) [Co(H2O)6]2+
otherwise, it is likely to be tetrahedral.
(c) Measure the molecule's magnetic moment and AMU – 2010
use the result to estimate the number of Ans. (b) : Spectrochemical series:-
I– < SCN– < Cl– < S2– < F– < OH– < C2 O 24− < H2O <
unpaired spins in the molcule. If this number is
low, the geometry is likely to be tetrahedral;
NCS– < EDTA4– < NH3 < en < CN– < CO (strongest)
otherwise, it is likely to be square planar.
(d) Measure the molecule's magnetic moment and 590. The number of unpaired electrons in Ni(CO)4
use the result to estimate the number of is–
unpaired spins in the molecule. if this number (a) 0 (b) 1
is low, the geometry is likely to be tetrahedral; (c) 3 (d) 4
otherwise, it is likely to be octahedral. BCECE-2007
[AIIMS-2009]
2−
 
Ans. (a) :  Ni (CO)4 
Ans. (b) : The complex ion is [CoCl4 ] . The oxidation
Let, x be the oxidation state of Ni.
state of cobalt metal is +2.
x + 4 (0) = 0
Co 2+ = [ Ar ]3d 7 x=0
CO is a strong field ligand thus pairing occur in inner
orbitals.
Ni = [Ar] 3d8 4s2

The square planar complex has one unpaired electron


whereas tetrahedral compound has 3 unpaired electrons.
Hence, it is the best method of determining of the
molecular geometry of [CoCl4]2–.
588. The correct order for the wavlelength of
absorption in the visible region is: The number of unpaired electron is 0.
(a) [Ni(NO2)6]4- < [Ni(NH3)6]2+ 591. Potassium ferricyanide on ionization produces
< [Ni(H2O)6]2+ (a) 2 ions (b) 1 ions
(b) [Ni(NO2)6] < Ni (Ni(H2O)6]2+
4- (c) 3 ions (d) 4 ions
< Ni(NH3)6]2+ BCECE-2010
(c) [Ni(H2O)6] < [Ni(NH3)6]2+
2+
Ans. (d) : The chemical formula of potassium
< [Ni(NO2)6]4- ferricyanide is K3[Fe(CN)6]. The ionization of the
(d) [Ni(NH3)6] <[Ni(H2O)6]4-
2+ compound is given as –
<[Ni(NO2)6]4-
[AIIMS-2005]
Ans. (a) : As we know that the crystal field stabilisation
energy is inversely proportional to the wavelength of
absorption of light. The compounds are – 592. The example of σ−complex is
[Ni(NO2)6]4– [Ni(NH3)6]2+ [Ni(H2O)6]2+
+2 +2 +2 (a) Fe ( η5 − C5 H 5 ) (b) Cr ( CO )6 
2
All the compounds have same metal with same
oxidation state then we move to the nature of ligand, (c) Al 2 ( CH 3 )6 (d) Ziesse salt
according to the spectrochemical series, the placing CG PET- 2013
order of ligands are H2O < NH3 < NO2. Thus, the CFSE Ans. (c) : Complexes in which a sigma bond acts as a
order of the compounds are two -electron donor to the metal center are called sigma
2+ +2
 Ni (H 2 O)  <  Ni ( NH 3 )  <  Ni ( NO 2 ) 4− complexes. In Al2(CH3)6, all –CH3 group attached with
 6   6   
the A1 through the sigma bond.
Hence, the order of wavelength of absorption will be
4− +2 2+ Remaining complex are connect with the metal by co-
 Ni ( NO 2 )  <  Ni ( NH 3 )  <  Ni (H 2O)  . ordinate bond.
 6   6   6 

Objective Chemistry Volume-II 613 YCT


593. Which one of the following will give a white Ans. (b) : For octahedral d6 (low spin)-
precipitate with AgNO3 in aqueous medium?
(a) Co( NH3 )5 C1 ( NO2 )2 (b)  Pt ( NH 3 ) 2 C12 
(c)  Pt ( en ) C12  (d)  Pt ( NH 3 )4  C12
CG PET -2005
Ans. (d) : If the complex ionises the Cl– ion then they CFSE = [−0.4 × N t 2 g + 0.6 × N eg ]∆o + mp
form the precipitate with AgCl. In the given option, Where - N = no. of electron in t orbital
2g
only option (d) ionises own Cl– ion. t2g

 Pt ( NH 3 )  Cl2 → 2AgNO3  Pt ( NH 3 )  NO3 + 2AgCl N eg = no. of electron in t eg orbital


 4   4 
PPt mp = no. of forcefully paired electron.
594. Elements Se, Cl and S have been arranged in CFSE = [−0.4 × 6 + 0.6 × 0] ∆o + 3P
the order of increasing ionisation energies. CFSE = −2.4 ∆ + 3P
o
Identify the correct order.
(a) S < Se < Cl (b) Se < S < Cl
(c) Cl < S < Se (d) Se = S < Cl 7. Oxidation State of
JCECE - 2013 Coordination Compounds
Ans. (b) : Ionisation energy generally increases from
left to right across a row of the Periodic Table and 598. In complex CrCl3.6H2O, the correct primary
decreases from top to bottom down a group. Hence, IE valency, secondary valency, and coordination
of Cl > S and IE of Se < S. So, the order is Se < S < Cl. number are respectively as x, y and z. What are
595. In which of the following coordination entities x, y and z?
the magnitude of ∆0 (CFSE in octahedral field) (a) x = 6 y = 6 z = 3
will be maximum? (b) x=6 y=3 z=6
(a) [Co(CN)6]3– (b) [Co(C2O4)3]3– (c) x=3 y=6 z=6
(c) [Co(H2O)6]3+ (d) [Co(NH3)6]3+ (d) x = 3 y = 6 z = 4
(At. No. Co = 27) AP-EAPCET-12.07.2022, Shift-II
(AIPMT -2008) Ans. (c) : Primary valency= Oxidation state
Ans. (a) : The complex which has the high ligand field Secondary valency = No. of molecule co–ordinate to
according to the spectrochemical series, having the central atom.
greater CFSE in octahedral field. Here, CN ligand has In the molecule CrCl .6H O-
3 2
more field due to Which [CO(CN)6]3− has more CFSE.
Cr is +3 State.
596. Crystal field stabilization energy for high spin Hence Primary valency (x) = 3
d4 octahedral complex is 6H2O is CO–ordinate with central atom. Hence,
(a) –1.8 ∆0 (b) –1.6 ∆0 + P secondary valency (y) = 6
(c) –1.2 ∆0 (d) – 0.6 ∆0 And, coordination number of CrCl3.6H2O is (z) = 6
AIPMT-2010 599. Which of the following has least oxidation state
Ans. (d) : For d4 (High spin)- of Fe?
(a) K3[Fe(OH)6] (b) K2[FeO4]
(c) FeSO4(NH4)2SO4.6H2O (d) [Fe(CN)6]3–
UPTU/UPSEE-2018
BCECE-2004
Ans. (c) : (a) K3 [Fe (OH)6]
CFSE = [−0.4× N t 2 g + 0.6 × N eg ]∆o 3(+1) + x + 6(–1) = 0
CFSE = [−0.4 × 3 + 0.6 × 1] ∆o x = +3
CFSE = [−1.2 + 0.6] ∆o (b) K 2[FeO4]

CFSE = −0.6∆o 2(+1) + x + 4 (–2) = 0


6 x=+6
597. Low spin complex of d –cation in an octahedral
field will have the following energy (c) FeSO4(NH4)2 SO4.6H2O
x + 1 (–2) + 2(1) + 1(–2) = 0
−12 −12
(a) ∆0 + P (b) ∆ 0 + 3P x – 2 + 2 –2 = 0
5 5 x=+2
−2 −2 (d) [Fe(CN) 3–
(c) ∆ 0 + 2P (d) ∆0 + P 6]
5 5 x+ 6 (–1) = – 3
(∆0 = crystal field splitting energy in an x=+3
octahedral field, P= Electron pairing energy) So, FeSO4 (NH4)2SO4.6H2O has the least oxidation
AIPMT-2012 state.
Objective Chemistry Volume-II 614 YCT
600. Which of the following is wrong statement?
(a) Ni(CO)4 has oxidation number +4 for Ni
(b) Ni (CO)4 has zero oxidation number for Ni
(c) Ni is metal
(d) CO is gas
[BITSAT-2008]
JIPMER-2005
Ans. (a) : (a) Ni(CO)4
Let, x be the oxidation sate of Ni.
x + 4(0) = 0
x=0
∴oxidation state of Ni is 0
(c) Ni is a metal which is belongs to the 3d-series of
transition metal.
(d) CO is a poisons gas. The name of the gas is carbon
monoxide.
So, statement (a) is false.
601. The oxidation number of central metal in
Hence, Co2+ has the difference in oxidation state in low
[Pt(NH3)2Cl(NO2)] and [CoCl2(en)2]⊕, spin and spin, is +2
respectively, are
603. The oxidation number of Mn is maximum in
(a) +2; + 1 (b) +2; +2 (a) MnO4 (b) K2MnO4
(c) +2; +3 (d) +3; +2 (c) MnO4 (d) KMnO4
TS EAMCET 04.08.2021, Shift-I Assam CEE-2020
Ans. (c) : The oxidation no. of central metal Pt and Co Ans. (d) : Let x be the oxidation number of Mn
are–
MnO2 ⇒ x + 2 (–2) = 0
[Pt (NH3)2 Cl(NO2)] [CoCl2(en)2]+
x = +4
x + 2(0) + (–1) + (–1) x + 2 (–1) + 2(0) = +1
K2MnO4 ⇒ 2(1) + x + 4(–2) = 0
x + (–1) + (–1) = 0 x – 2 + 0 = +1
x = +6
x = +2 x = +3
Mn2O3 = 2x + 3 (–2) = 0
602. The difference in the number of unpaired x = +3
electrons of a metal ion in its high–spin and KMnO4 = 1 + x + 4 (–2) = 0
low-spin octahedral complexes is two. The
x=7
metal ion is:
So, the option D is correct.6
(a) Ni2+ (b) Fe2+
(c) Co 2+
(d) Mn2+ 604. The oxidation states of Pt in [Pt(NH3)4] [PtCl4]
is
[BITSAT-2021]
(a) +1 and +1 (b) +1 and +2
Ans. (c) : The compound is octahedral type of complex. (c) +2 and +2 (d) +2 and +1
Here, d-orbital split into two energy level i.e. t2g and eg. AMU-2019
Ans. (c) : The co- ordination compound is [P+ (NH3)4]
[P+Cl4]
Let, x be the oxidation state of P+
x + 4 (0) + x + 4 (–1) = 0
2x – 4 = 0
x=+2
So, oxidation state of P+ is +2
605. Which of the following complex ion is the most
stable?
3+
(a) Co ( NH 3 )6  (b) [CoCl6 ]
3−

3+
(c) [CoF6 ] (d) Co ( H 2 O )6 
3−

GUJCET-2019
Ans. (a) : The complex compound which has the strong
field ligand, are more stable in nature. Here,
[Co(NH3)6]3+ has the NH3 ligand which is act as SFL in
Co3+.

Objective Chemistry Volume-II 615 YCT


606. The oxidation states of Cr, in [Cr(H2O)6]Cl3, 610. What should be possible d-orbital energy levels
[Cr(C6H6)2], and K2[Cr(CN)2(O)2(O2)(NH3)] of Ni in [Ni(CN)4]2-?
respectively are (a) dz2 < dxy < dxy = dyz , dx2 – y2
(a) +3, +4 and +6 (b) +3, +2 and +4 (b) dx2 – y2 < dxy < dz2 < dxy – dyz
(c) +3, 0 and +6 (d) +3, 0 and +4 (c) dxz = dyz < dz2 < dxy < dx2 – y2
JEE Main 2018 (d) dz2 < dxy = dyz < dxy < dx2 – y2
Ans. (c) : Let x be the oxidation state of Cr in J & K CET-(2017)
complexes. Ans. (c) : [Ni(CN4)]2– forms the square planar geometry
(i) [Cr(H2O)6]Cl3 with CN ligand. The d-orbital energy levels illustrated
x+ 6 (0) + 3 (–1) = 0 below–
x = +3 d x2 −y2
(ii) [Cr(C6H6)2]
x + 2(0) = 0
x=0
(iii) K2 [Cr(CN)2 (O)2 (O)2 (NH3)]
2(+ 1) + x + 2 (–1) + 2 (–2) + 1 (–2) + 0 = 0 d xz ,d yz
2 + x –2 –4 –2 = 0
x=+6 Hence, the increasing order of energy can be written as
So, the oxidation state of compounds are +3, O and +6. –
607. Oxidation state of manganese in Mn(CO)5 is d xz = d yz < d z 2 < d xy < d x 2 −y2
(a) +5 (b) +1 611. The correct charge on and co-ordination
(c) +6 (d) 0 number of 'Fe' in K3[Fe(CN)6] is
JCECE - 2018 (a) +2, 4 (b) +3, 6
Ans. (d) : Oxidation state of manganese in Mn(CO)5 is (c) +2, 6 (d) +3, 3
0. Here CO is neutral ligand. MHT CET-2017
Mn(CO)5 Ans. (b) : K 3  Fe (CN )6 
⇒ x + 5(0) = 0
Let, x be the oxidation state of Fe.
⇒ x=0
3(+1) + x + 6 (–1) = 0
608. In the complex, [Cr(NH3)4Cl2]+, the oxidation x = +3
number of Cr is The oxidation state of Fe is +3 and the co-ordination
(a) +4 (b) +3 number of the compound is 6.
(c) +2 (d) +6 612. The number of unpaired electrons in [NiCI4]2-,
JIPMER-2018 Ni(CO)4 and [Cu(NH3)4]2+ respectively are
+ (a) 2,2,1 (b) 2,0,1
Ans. (b) : For the complex, Cr ( NH 3 )4 Cl 2  –
(c) 0,2,1 (d) 2,2,0
Let, x be the oxidation state of Cr. WB-JEE-2017
∴ x + 4 (0) + 2 (–1) = +1 2–
Ans. (b) : In [NiCl4] Ni is in +2 oxidation state
X – 2 = +1 3d
x = +3 Ni2+ = [Ar] 3d8 4s0
Thus, the oxidation state of Cr is +3. Cl– is a weak field ligand, thus pairing of electrons does

609. In case of ring rest of NO3 ion, the complex not takes place.
formed has formula [Fe(H 2 O)5 NO]SO 4 . The
[NiCl4]2– =
oxidation state of iron in the complex is
(a) +1 (b) +2
(c) +3 (d) +6 3
sp hybridisation
CG PET -2017
Unpaired electrons = 2
Ans. (a) : The oxidation state of Fe in Brown ring is +1. In Ni(Co) Ni is in zero oxidation state.
4,
The complex formed is [Fe(H2O)5NO]SO4. The charge
Ni = [Ar] 3d8 4s2
on anion is−2 Hence, charge on cation complex has to
3d
be +2 as ring is electrically neutral. The charge on Fe on
calculation comes as +2. But as the electron transfer
takes place from NO to Fe. Hence, NO and Fe acquire a Co is strong field ligand and cause pairing of electron
charge of +1 each. of 4s to 3d.

Objective Chemistry Volume-II 616 YCT


Cu2+:[Ar]3d94s0 (ii) K4[Fe(CN)6]
3d 4 × (+1) + x + 6 (–1) = 0
[Cu(NH3)4] 2+ = x=+2
(iii) Fe (CO)5
x+5×0=0
3 x=0
sp hybridisation
(iv) [Fe (H2O)6] Cl2
3d x + 6 × (0) + 2 (–1) = 0
Ni(CO)4 = x=+2
So, the Fe(CO)5 has zero oxidation state in central
metal.
sp3 hybridisation 616. Which of the following compounds has same
oxidation state of the central metal atom in the
613. In the complex, K2 Fe[Fe(CN)6] cationic and anionic part?
(a) the complex is high spin complex (a) [Pt(NH3)4][PtCl6]
(b) both Fe atoms are in the same oxidation state (b) [Pt(py)4][PtCl4]
(c) the coordination number of iron is 4 (c) [Pt(NH3)4Cl2][PtCl4]
(d) both Fe atoms are in different oxidation states (d) K4[Ni(CN)6]
[BITSAT-2016] JCECE - 2015
Ans. (b) : K2Fe[Fe(CN)6] Ans. (b) : Let the oxidation state of Pt is x.
Let, x be the oxidation state of Fe. [Pt(py)4] [PtCl4]
2(+ 1) + x + x + 6 (–1) = 0 x + 4 × 0 + x + (–1) × 4 = 0
2x = + 4 2x = 4
x=+2 x = +2
(a) The complex will be low spin because CN is strong Thus, it is the complex in which the central metal atom,
filled ligand due to which pairing take place in inner Pt in cationic and anionic parts has same oxidation state.
orbital’s. 617. The sum of coordination number and oxidation
(b) Both Fe metal are in same oxidation state i.e. +2. number of the metal M in the complex
Thus, statement is true. [M(en)2(C2O4)]Cl (where en is
(c) The co-ordination number of iron is 6 because metal ethylenediamine) is
connected with six CN ligand. (a) 6 (b) 7
(d) Both Fe is in same oxidation state. (c) 8 (d) 9
614. The coordination number and the oxidation (NEET-2015)
state of the central metal ion in the complex ion  
[CrCl2(ox)2]3– are respectively, Ans. (d) :  M (en )2 (C2 O 4 ) Cl
(a) 4 and 3– (b) 6 and 3+ Let, x be the oxidation state of M.
(c) 4 and 3+ (d) 6 and 3– ∴ x + 2(0) + 1 (–2) – 1 = 0
J & K CET-(2016) x=3
Ans. (b) : The complex contains the two bidentate The co-ordination number of the compound is 6. Hence,
ligand and two chlorine ligand. Thus, the metal the sum of co-ordination number and oxidation of the
connected with the six donor sites. Hence, the metal M is 6 + 3 = 9.
coordination number is 6. 618. The oxidation state and covalency of Al in [Al
Let, x be the oxidation state of Cr. Cl(H2O)5]2+ are respectively
∴ x + 2(–1) + 2 (–2) = –3 (a) +6, 6 (b) +3, 6
x – 2 – 4 = –3 (c) +2, 6 (d) +3, 3
x = +3 AP-EAMCET (Engg.) - 2014
The oxidation state of Cr is +3 Ans. (b) : Given, [Al Cl(H2O)5]2+
615. In which of the following coordination Let the O.S. of Al is x.
compounds, the central metal ion is in zero ∴ x+(–1)+5×0 = +2
oxidation state? x = +3
(a) [Fe(H2O)6]Cl3 (b) K4[Fe(CN)6] Covalency of Cl = 1 and H2O = 5
(c) Fe(CO)5 (d) [Fe(H2O)6]Cl2 ∴ Total covalency = 1+5 = 6
AP-EAMCET (Engg.) 2015 619. The ratio of anion radius to cation radius of a
Ans. (c) : The oxidation state of all the central metal ion crystal is 10 : 9.3 . Then, the coordination
are– number of the cation in the crystal is
(i) [Fe(H2O)6] Cl3 (a) 2 (b) 4
x + 6 × 0 + 3(–1) = 0 (c) 6 (d) 8
x=+3 AP-EAMCET- (Engg.) - 2010
Objective Chemistry Volume-II 617 YCT
Ans. (d) : Given that, the ratio of anion to cation radius Ans. (d) : [E (en)2 (C2O4)] NO2
= 10:9.3 As we know that ‘en’ and C2O4 are bidentate ligand.
9.3 Thus, central metal connect with the six donor site and
∴ The ratio of cation to anion radius is = = 0.93
10 Hence coordination number is 6.
Since, the ratio of cation to anion radius lie between [E(en) 2 (C2 O 4 )]NO 2
0.732 to 1.00, thus the co-ordination number of cation is
8. Let, x be the oxidation state of E.
620. The oxidation state and effective Atomic x + 2 (0) + 1 (–2) –1 = 0
Number (EAN) of cobalt in [CoF6]2– are x=3
respectively Co- ordination number and oxidation state of the
(a) 3 and 36 (b) 4 and 35 compound are +6 and +3 respectively.
(c) 4 and 37 (d) 2 and 35 625. What is the oxidation state of iron in
MPPET-2008 [Fe(H 2 O)5 NO]2 + ?
Ans. (b) : [CoF6]2– O.S. of Co ⇒ x + 6 (–1) = –2 (a) 0 (b) + 1
⇒ x = +4 (c) + 2 (d) + 3
EAN = Atomic number – oxidation state + 2 C.N BCECE-2013
2+
= 27 – 4 + 2 × 6 Ans. (b) : [Fe(H2O)5NO]
= 35 Let, x be the oxidation state of Fe.
621. Aluminium reacts with NaOH and forms x + 5 (0) + 1(+1) = + 2 (∵NO is linear)
compound ‘X’. If the coordination number of
aluminium is 6, the correct formula of X is x=+1
(a) [Al(H2O)4 (OH)2]+ (b) [Al(H2O)3 (OH)3] So, the oxidation state of Fe is + 1
(c) [Al(H2O)2 (OH)4]– (d) [Al(H2O)6 ] (OH)3 626. The coordination number and oxidation state
AP EAMCET (Engg.)-2009 of Cr in K 3 Cr ( C2 O 4 )3  are respectively
Ans. (c) : Aluminum reacts with NaOH and forms the (a) 6 and +3 (b) 3 and 0
sodium meta aluminates (NaAlO3). The complex is (c) 4 and +2 (d) 3 and +3

soluble in water and forms the [Al(H2O)2 (OH)4] CG PET- 2011
Number of Al in complex is 6. Ans. (a) : Coordination number of Cr is 6 (oxatate is
622. The oxidation state and covalency of Al in bidentate ligand)
[AlCl(H2O)5]2+ are respectively K 3 [Cr(C2 O 4 )] is calculated as
(a) + 6,6 (b) + 3, 6
(c) + 2, 6 (d) + 3, 3 3(1) + x + 3(−2) = 0
AP EAMCET (Engg.) -2014 3 + x + (−6) = 0
Ans. (b) : [Al Cl (H2O)5]2+ x=6–3
Let, x be the oxidation state of Al. x = +3
x + 1(–1) + 5 (0) = + 2
x=+3 627. The oxidation number, d-orbital occupation
and coordination number of Cr in the complex
The oxidation state of Al is +3. The covalency of Al is
cis[Cr(en)2 Cl2]Cl are respectively
+6
(a) +3, 3d and 4 (b) +3, 4d and 6
623. The effective atomic number (EAN) of central (c) +3, 3d and 6 (d) +2, 3d and 6
metal atom in [Co(NH3)6]Cl3 is [at. No. of
Co=27] J & K CET-(2010)
(a) 34 (b) 35 Ans. (c) : In the complex cis  Cr (en )2 Cl 2  Cl is -
(c) 36 (d) 32 Let, x be the oxidation state of Cr.
COMEDK 2012
x + 2(0) + 2(−1) −1 = 0
Ans. (c) :The given complex is [Co(NH 3 )6 ]Cl3 .
x = +3
Effective atomic number (EAN)= Z-oxidation number
+2×co-ordinationa number The oxidation state of Cr is +3.
EAN = 27–3+2×6 Cr 3+ = [ Ar ]3d 3
= 27–3+12 The oxidation number, d-orbital occupation and co-
=36 ordination number of Cr in the complex cis –
624. The coordination number and the oxidation (Cr (en ) ) Cl  Cl are +3, 3d and 6.
state of the element 'E' in the complex  2 2 

[E(en)2(C2O4)]NO2 (where (en) is ethylene 628. The tetrahedral complexes have coordination
diamine ) are respectively, number
(a) 6 and 2 (b) 4 and 2 (a) 3 (b) 6
(c) 4 and 3 (d) 6 and 3 (c) 4 (d) 8
[AIIMS-2014] J & K CET-(2007)
Objective Chemistry Volume-II 618 YCT
Ans. (c) : The tetrahedral complexs have co-ordination Ans. (d) (A) [Ni(H2O)2(en)2]2+
number 4 because the central metal attacted with the (B) [Ni(H2O)4(en)]2+
four donor atom or ligands. (C) [Ni(en)3]2+
Here en is a strong field ligand (SFL). When the
number or en (strong ligand is increases) then splitting
is also increases.
So, ∆0 increases.
It means maximum energy will be absorbed in case of
option c.
629. The oxidation number of cobalt in K[Co(CO)4] Hence the order is C > A > B
is 633. The correct order of energy of absorption for
(a) +1 (b) +3 the following metal complexes is
(c) –1 (d) –3 A : [Ni(en)3]2+, B: [Ni(NH3)6]2+, C: [Ni(H2O)6]2+
J & K CET-(2005) (a) C < B < A
Ans. (c) : K  Co (CO)4  (b) B < C < A
(c) C < A < B
Let, x be the oxidation state of Co. (d) A < C < B
∴ +1 + x + 4 ×0 = 0 JEE Main 25.07.2022, Shift-II
x = –1 Ans. (a) : The correct order is
Thus, the oxidation state of the Co is –1 [Ni(en)3]2+ > [Ni(NH3)6]+2 > [Ni(H2O)6]2+
(A) (B) (C)
630. The oxidation state of chromium in chromium
trioxide is Stronger the ligand , larger the spliting and higher the
energy of absorption.
(a) +3 (b) +4
(c) +5 (d) +6 634. Match List I with List II and select the correct
answer using the code given below the lists :
J & K CET-(2007)
List I List II
Ans. (d) : The chemical formula of chromium oxide is (Metallic oxide) (Colour imparted)
CrO3. The oxidation state of Cr is –
∴ x + 3 (–2) = 0 A. Uranium oxide 1. Red
x = +6 B. Cuprous oxide 2. Blue
Hence, the oxidation state of Cr is +6. C. Cobalt oxide 3. Green
631. What is the oxidation number of Fe in D. Chromium oxide 4. Yellow
Fe(CO)5? Code
(a) +3 (b) zero A B C D
(c) +2 (d) +5 (a) 4 1 2 3
UP CPMT-2009 (b) 3 2 1 4
Ans. (b): Let the oxidation state of Fe in Fe (CO)5 is x. (c) 4 2 1 3
Fe (CO)5 (d) 3 1 2 4
x+0×5=0 NDA (II)-2011
∴ x=0 Ans. (a) :
In metal carbonyls, oxidation state of metal is always List I List II
zero. (Metallic oxide) (Colour imported)
A. Uranium oxide Yellow
B. Cuprous oxide Red
8. Spectra and Colour of Co- C. Cobalt oxide Blue
ordination Compounds D. Chromium Green
635. A metal complex absorbed orange light. The
632. The order of energy absorbed which is color in which it appears is
responsible for the color of complexes (a) Yellow
(1) [Ni(H2O)2(en)2]2+ (b) Yellow-Green
(2) [Ni(H2O)4(en)]2+ and (c) Red
(3) [Ni(en)3]2+ (d) Green-Blue
is TS-EAMCET-19.07.2022, Shift-II
(a) (B) > (A) > (C) (b) (A) > (B) > (C) Ans. (d) : Here, metal complex absorbed orange light,
(c) (C) > (B) > (A) (d) (C) > (A) > (B) we cannot detect the orange light and we will see the
NEET-17.06.2022 complementary color as the colour of complex.

Objective Chemistry Volume-II 619 YCT


We can determine colour of complex with the help of Ans. (d) : Color of salts is a property of partially filled
colour wheel. d-orbitals. since TiF62– has completely empty and
Cu2Cl2 has completely filled d-subshells hence, these
are colourless salts.
2+
639. The reaction of aqueous  Ni ( H 2O )6  is
performed by the addition of a bidentate ligand
ethane-1, 2, diamine (en)
Match the following
2+
The complementary colour is simply the colour is the Ni ( H2O )6  : en Colour of
opposite sector of the colour wheel. Here, metal  
complex absorb orange colour and blue is present in molar ratio the product
opposite sector of colour wheel. (A) 1 : 1 (I) Pale blue
So, The complex will appear in blue colour (B) 1 : 2 (II) Blue/Purple
636. Match the following (C) 1 : 3 (III) Violet
Column-I Column-II (IV) Green
(Reaction) (colour of the The correct match is
product or nature ) A B C
(A) FeCl3(aq) + NH3(aq) → (I) Green ppt (a) I II III
(B) AgCl(aq) + NH3(aq) → (II) Deep blue (b) II III IV
(C) Cu2+(aq) + NH3 (aq) → (III) Brown ppt
(c) III I II
(V) Colourless
(d) IV I III
The correct match
A B C TS-EAMCET (Engg.), 05.08.2021 Shift-II
(a) I II III Ans. (a) : The influence of the ligands on the colour of
A B C a complex may be illustrated by considering the
(b) I III IV [ Ni(H 2 O)6 ]2+ complex, which forms when NiCl2 is
A B C
(c) III IV II dissolved in water. If the bidentate ligand, ethylene
A B C diamine (en) is progressively added in the aqueous
(d) III I IV medium in which colour changes occur. Which is given
TS-EAMCET-20.07.2022, Shift-II below:-
Ans. (c) : FeCl3 (aq) + NH3 (aq) → Brown p.pt [ Ni(H2O)6 ]2+ (aq) →
en (aq)
[ Ni(H2O)4 (en)]2+ (aq) + 2H2O
AgCl (aq) + NH3 (aq) → Colourless Pale blue
Cu2+ (aq) + NH3 (aq) → Deep blue 2+ 2+
[ Ni(H2O)4 (en)] (aq) →[ Ni(H2O)2 (en)2 ]
en (aq)
+ 2H 2O
637. Which of the following ions in aqueous solution
in colourless ? Blue/Purple
2+ 2+
(a) Ti 3+ [ Ni(H2O)2 (en)2 ] (aq) →[ Ni(en)3 ] (aq) + 2H2O
en (aq)

(b) Ti 4+ Violet
(c) Co 2+ Hence, the correct option is (a).
(d) Cu 2+ 640. The potassium ferrocyanide solution gives a
Assam-CEE-31.07.2022 prussian blue colour, when added to
Ans. (b) : Electronic configuration of Ti4+ is [Ar] 3d0 (a) CoCl3 (b) FeCl2
4s0. (c) CoCl2 (d) FeCl3
∴ Ti4+ ions is ions in aqeous solution in colourless. [JEE Main 2021, 1 sep Shift-II]
638. Amongst TiF62- ,CoF63- , Cu 2 Cl 2 and NiCl 42- (at. Ans. (d) : FeCl3 + K4 [Fe(CN)6]→ Fe4[Fe(CN)6]3
no. Ti=22, Co=27, Cu = 29, Ni = 28). The Prussian blue
colourless species are 641. The correct order for wavelengths of
(a) CoF63− and NiCl42− absorption in the visible region for the
(b) TiF62− and CoF63− following complexes will be
(a) [Ni(NH3)6]2+ < [Ni(H2O)6]2+ < [Ni(NO2)6]4-
(c) Cu 2 Cl2 and NiCl 24− (b) [Ni(NO2)6]4- < [Ni(NH3)6]2+ < [Ni(H2O)6]2+
(d) TiF62− and Cu 2 Cl2 (c) [Ni(NH3)6]2+ < [Ni(NO2)6]4- < [Ni(H2O)6]2+
J & K CET-(2003) (d) [Ni(NO2)6]4- < [Ni(H2O)6]2+ < [Ni(NH3)6]2+
(AIPMT -1995) AP EAMCET (Engg.) 17.09.2020 Shift-I
Objective Chemistry Volume-II 620 YCT
Ans. (b) : Complexes, (I) [Ni(NH3)6]2+, (II) 645. Homoleptic octahedral complexes of a metal
[Ni(H2O)6] and ligands —NH3, —H2O and — NO −2
2+ ion M3+, with three monodentate ligands L1, L2
respectively. The field strength order of the ligands is and L3 absorb wavelengths in the region of
green, blue and red respectively. The
H2O < NH3 < NO −2 increasing order of the ligand strength is
So, the order CFSE values : △0 II < △0 I < △0 III (a) L1 < L2 < L3 (b) L2 < L1 < L3
hc 1 (c) L 3 < L 1 < L 2 (d) L3 < L2 < L1
We know, E = or E ∝ [JEE Main 2019, 9 Jan Shift-II]
λ λ
So, the order of wavelength of absorption in visible Ans. (c) : Stronger the ligand absorption of light having
region will be lower wavelength
2+ 2+ 4− As
[ Ni(H 2O)6 ] >  Ni ( NH3 )6  > [ Ni(NO2 )6 ] λ L3 > λ L1 > λ L2
642. For the ions Zn2+, Ni2+ and Cr3+ which among Hence, ligand strength is L < L < L
3 1 2
the following statement is correct?
(Atomic number of Zn = 30, Ni = 28 and Cr = 646. Two complexes [Cr(H 2 O)6]Cl3(A) and
24) [Cr(NH3)6]Cl3 (B) are violet and yellow
(a) All these are colourless coloured, respectively. The incorrect statement
regarding them is
(b) All these are coloured
2+
(c) Only Ni is coloured and Zn and Cr are 2+ 3+ (a) ∆0 value for (A) is less than that of (B)
colourless (b) both absorb energies corresponding to their
2+
(d) Only Zn is colourless and Ni and Cr are 2+ 3+ complementary colours
coloured (c) ∆0 values of (A) and (B) are calculated from
AIIMS 26 May 2019 (Morning) the energies of violet and yellow light,
respectively
Ans. (d) : Electronic configuration of
2+ 10 0 2+
Zn [Ar] 3d 4s , Ni [Ar]3d , 4s 8 0 (d) both are paramagnetic with three unpaired
3 3 0 electrons
Cr +[Ar] 3d , 4s
[JEE Main 2019, 9 Jan Shift-I]
Zn2+ colourless (no unpaired electron)
2+ 3+
Ni and Cr coloured due to unpaired electrons. There Ans. (c) : Since H 2 O is a weak field ligand but NH3 is a
is d-d transition of electron, so the solution of ions will strong field ligand. Strong field ligands cause greater ∆0
be coloured. value. Both the complexes have 3d3 configuration and
643. Correct increasing order for the wavelength of hence paramagnetic with three unpaired electrons.
absorption in the visible region for the 647. Assertion:
complexes of Co3+ is: Co ( NH 3 )6  → Co ( NH 3 )5 H 2O 
3+ 3+
3– 3+ 3+
(a) [Co(CN)6] ,[Co(NH3)6] ,[Co(NH3)5(H2O)] ,
[Co(NH3)5Cl]2+ color continuously changes.
3– Reason: Larger wavelength will be absorbed.
(b) [Co(CN)6]
,[Co(NH3)5(H2O)]3+[Co(NH3)5Cl]+2 (a) If both Assertion and Reason are correct and
[Co(NH3)6]3+ the Reason is the correct explanation of
(c) [Co(CH3)6]3+,[Co(CN)6]3–,[Co(NH3)5(H2O)]3+ Assertion.
[Co(NH3)5Cl]+2 (b) If both Assertion and Reason are correct, but
(d) [Co(NH3)5Cl]2+, [Co(NH3)5(H2O)]3+, Reason is not the correct explanation of
3+ 3–
[Co(NH3)6] , [Co(CN)6] Assertion.
AIIMS 26 May 2019 (Morning) (c) If Assertion is correct but Reason is incorrect.
Ans. (a) : Strength of ligands attached with Co3+ ion is (d) If both the Assertion and Reason are
in the order of CN– > NH3 > H2O > Cl–. So, order of incorrect.
crystal field splitting energy (values of ∆o) will be in the [AIIMS-27 May, 2018(E)]
same order. Ans. (a) : H2O is a weaker ligand than NH3. So, larger
1 wavelength will be absorbed.
∴ Wavelength of absorbing light ∝
∆o 648. Assertion : Cr ( H 2O )6  Cr ( H 2O )6  while
2+ 3+

644. Which give colored carbonate precipitate? converting, color continuously changes.
(a) Hg 22+ (b) Sr+2 Reason: CFSE increases during change.
(c) Bi+3 (d) Li+ (a) If both Assertion and Reason are correct and
AIIMS 26 May 2019 (Morning) the Reason is the correct explanation of
Ans. (c) : Among the given ions Hg2+, Sr+2 and Li+ do Assertion.
not have unpaired electrons while Bi2+ has one unpaired (b) If both Assertion and Reason are correct, but
electron. Thus, carbonate of Bi2+ show coloured Reason is not the correct explanation of
precipitate. Assertion.
Objective Chemistry Volume-II 621 YCT
(c) If Assertion is correct but Reason is incorrect. 652. Which of the following pairs of complexes
(d) If both the Assertion and Reason are whose Aqueous solutions gives pale yellow and
incorrect. white Precipates respectively with 0.1 M
[AIIMS-26 May, 2018(E)] AgNO3 ?
Ans. (a) [Cr(H 2 O)6 ]2+ → [Cr(H 2 O)6 ]
3+
(a)  Pt ( NH 3 ) 4 Br2  Cl2 and  Pt ( NH 3 )4 Cl2  Br2
Oxidation stateof Oxidation stateof
Cr = + 2 Cr = + 3
(b) Co ( NH3 )5 NO3  Br and Co ( NH3 )5 Br  NO3
2+ 4 3+
∵ Cr = [Ar]3d ∵ Cr = [Ar]3d 3
(c)  Pt ( NH 3 ) 4 Cl2  Br2 and  Pt ( NH 3 )4 Br2  Cl2
(d) Co ( NH 3 )5 NO3  Cl and Co ( NH 3 )5 Cl  NO3
GUJCET-2017
CFSE = [−0.4N t 2g + 0.6N eg ] ∆o CFSE = [−0.4×3 + 0.6× 0]∆o
Ans. (c) : The complex having Br and Cl out of
coordination sphere only gives AgBr (Pale yellow) and
CFSE = [−0.4× 3 + 0.6×1] ∆o CFSE =[−1.2]∆o AgCl (White ppt.) respectively upon reaction with
CFSE = [−1.2 + 0.6]∆o CFSE = −1.2 ∆o AgNO3.
CFSE = −0.6 ∆o 653. Which of the following compound is not
Hence, CFSE increases during the changing from coloured?
[Cr(H2O)6]2+ to [Cr(H2O)6]3+. (a) Na2CuCl (b) Na2Cd.Cl4
(c) FeSO4 (d) VI3
649. In acidic medium which of the following does
not change its colour? JIPMER-2017
Ans. (b) : Na2[Cd.Cl4] has no unpaired electron due to
(a) MnO −4 (b) MnO 2-
4 presence of d10 electrons. Cd2+ is a colourless
(c) CrO 2-
4 (d) FeO 24 − compound.
[AIIMS-26 May, 2018 (E)] 654. The deep blue color produced on adding excess
of ammonia to copper sulphate due to presence
Ans. (a) : MnO −4 is stable in acidic medium of
MnO 24 − disproportionate. CrO 24 − converts into (a) Cu2+ (b) [ Cu(NH 3 ) 4 ]
2+

2− 2−
Cr2 O and FeO decomposed.
(c) [ Cu(NH 3 )6 ] (d) [ Cu(NH 3 ) 2 ]
7 4 2+ 2+

650. A brown ring is formed in the ring test for


NO3– ion. It is due to the formation of: [AIIMS-2016]
(a) [Fe(H2O)5NO]2+ (b) FeSO4NO2 Ans. (b) : CuSO4 react with excess NH3 to form
(c) [Fe(H2O)4(NO)2]2+ (d) FeSO4HNO3 [Cu(NH3)4] (OH)2 with deep colour.
HP CET-2018 CuSO 4 + 4NH 3 →[Cu(NH 3 ) 4 ]SO 4
Blue complex due to [ Cu(NH 3 ) 4 ]
2+
Ans. (a) : When freshly prepared solution of FeSO4 is
added in a solution containing NO−3 ion, it lead to
655. The purple colour of KMnO4 can be attributed
formation of a brown coloured complex. to
NO3− + 3Fe 2 + + 4H + 
→ NO + 3Fe3+ + 2H 2 O (a) d-d transitions
(b) charge transfer transition
[Fe(H 2 O)6 ]2 + + NO 
→ Fe(H 2 O)5 (NO) 2 + + H 2 O
(Brown) (c) n-π* transitions
(d) none of these
This is called brown ring test of nitrate.
AMU-2016
651. Which of the following complex ions absorbs Ans. (b) : In KMnO4, Mn is in +7 oxidation state and
the light of minimum wavelength? has all the 3d-orbitals vacant. Mn7+ ion is surrounded.
3+
(a) Co ( H 2O6 )6  (b) [CoF6 ]
3−
Tetrahedrally by four oxide ions. All oxide ions have
filled 2p-orbitals. There is a transfer of an electron from
3− 3+
(c) Co ( CN )6  (d) Co ( NH 3 )6  filled 2p-orbitals of oxide ion to vacant d-orbital’s of
Mn+7 ion. So there is a charge transfer transition which
GUJCET-2017 results in intense purple colour of KMnO4.
Ans. (c) : Wavelength of light absorbed ∝ 656. Coordination compounds have great
1 importance in biological systems.In this context
which of the following statements is incorrect?
Stability of complex
(a) Cyanocobalamin is B12 and contains cobalt.
In all given complexes Co is present in +3 state and (b) Haemoglobin is the red pigment of blood and
among the given ligands (CN)– is the strongest ligand. contains iron.
[Co(CN)6]3– is most stable, thus it absorbs light of (c) Chlorophylls are green pigments in plants and
minimum wavelength. contain calcium.
Objective Chemistry Volume-II 622 YCT
(d) Carboxypeptidase-A is an enzyme
and Ans. (d) : Ferric ion forms a prussian blue precipitate
contains zinc. due to the formation of Fe4[Fe(CN)6]3. This complex is
[AIIMS-2015] nitrogen in the given sample. In this method, to portion
Ans. (c) : Chlorophylls are green pigments in plants and of sodium fusion extract, freshly prepared ferrous
contain Magnesium and does not contain Calcium. sulphate, FeSO4 solution is added and warmed. Then
about 2 to 3 drops of FeCl3 solution are added and
657. The color of CoCl3. 5NH3. H2O is
acidified with conc. HCl. The appearance of a Prussian
(a) red (b) orange blue colour indicate the presence of nitrogen.
(c) orange-yellow (d) pink
FeSO 4 + 2NaOH → Fe(OH) 2 + Na 2SO 4
[BITSAT-2015]
Ans. (d) : CoCl3 5NH3 H2O is pink in colour. It has 6NaCN + Fe(OH) 2 → Na 4 [Fe(CN) 6 ]+ 2NaOH
Sodium ferrocyanide
Co3+ ion with [Ar] 3d6 electronic configuration. If has 4
unpaired electrons. 3Na 4 [Fe(CN)6 ] + 4FeCl3→ Fe 4 [Fe(CN)6 ]3 + 12NaCl
Ferricferrocyanide(prussian blue)
658. Which of the following compounds is not
662. Assertion (A) Cu and Cd2+ are separated by
2+
coloured yellow?
first adding KCN solution and then passing
(a) Zn2 [Fe(CN)6] (b) K3 [Co(NO2)6] H2S gas.
(c) (NH4)3[As (Mo3O10)4] (d) BaCrO4 Reason (R) KCN reduces Cu2+ to Cu+ and
JEE Main 2015 forms a complex with it.
Ans. (a) : The correct answer is
(NH4)3[As(Mo3O10)4] : Yellow (a) Both (A) and (R) are true and (R) is the
BaCrO43 : Yellow correct explanation of (A)
Zn2 [Fe(CN)6] : White (b) Both (A) and (R) are true but (R) is not the
K2 [Co(NO2)6] : Yellow correct explanation of (A)
659. When H2O2 is added to ice cold solution of (c) (A) is true but (R) is not true
acidified potassium dichromate in ether and (d) (A) is not true but (R) is true
the contents are shaken and allowed to stand VITEEE- 2011
(a) a blue colour is obtained in ether because of Ans. (b) : Addition of KCN in CuSO4 solution first
formation of CrO5 causes reduction and then precipitates CuCN (Curprous
(b) a blue colour is obtained in ether because of cyanide).
formation of CrO3 This reacts with excess CN, forming a soluble four co-
(c) a blue colour is obtained in ether because of ordinate complex.
formation of Cr2(SO4)3 [Cu(CN)4]–3 which is tetrahedral in shape.
(d) chromyl chloride is formed 2Cu2– + 4CN– —→ 2Cu'CN + (CN)2
JIPMER-2015 CuCN + 3CN  → [Cu + (CN) 4 ]3−
Ans. (a) : When H2O2 is added to acidified potassium colourlesscomplex
dichromate in others solution potassium dichromate is Thus, if Cd2+ and Cu2+ both are present, Cu2+ and Cd2+
– 2+
oxidized to blue perioxide of chromium (CrO5) which is both are complexed by CN if H2S gas is passed Cd
2–
soluble in ether and produces blue coloured solution. (in unstable) [Cd(CN) 4 ] gives yellow precipitate of
CdS.
663. Which of the following compounds is not
coloured?
660.Cold ferrous sulphate solution on absorption of (a) Na2[Cu(Cl)4] (b) Na[Cd(Cl)4]
NO develops brown colour due to the (c) K4[Fe(CN)6] (d) K3[Fe(CN)6]
formation of VITEEE- 2010
(a) paramagnetic [Fe(H2O)5(NO)]SO4
Ans. (c) : We know that complex compound having no
(b) diamagnetic [Fe(H2O)5(N3)]SO4 unpaired electron is colourless.
(c) paramagnetic [Fe(H2O)5(NO3)][SO4]2 Among the given complexes, K4[Fe(CN)6] has no
(d) diamagnetic [Fe(H2O)4(SO4)]NO3 unpaired electron as CN– is a strong field lignad and
WB-JEE-2015 causes pairing of electrons.
Ans. (a) : Ferrous sulphate is a type of iron compound So, it is colourless.
its formula is FeSO4. When cold ferrous sulphate absorb 664. The purple colour of KMnO4 is due to the
NO, it develops a brown colour. This occurs due to the transition
formation of paramagnetic [Fe(H2O)5 (NO)]SO4. It is a (a) C.T.(L→M) (b) C.T.(L→M)
paramagnetic.
(c) d – d (d) p – d
661. Ferric ions form prussian blue coloured
VITEEE- 2007
precipitate due to ............ .
(a) KMnO4 (b) K4Fe(CN)6 Ans. (a) : The permanganate ion has an intense purple
colour. Mn (+ VII) has a d0 configuration. So the colour
(c) Fe(OH)3 (d) Fe4[Fe(CN)6]3 arises from charge transfer and not from d–d spectra. In
– – – –
MPPET-2013 MnO4 an electron is momentarily changing O to O
Objective Chemistry Volume-II 623 YCT
and reducing the oxidation state of the metal from (a) ferroferricyanide
Mn(VII) to Mn(VI). Charge transfer requires that the (b) ferriferrocyanide
energy levels on the two different atoms are fairly close. (c) potassium ferricyanide
O = ( 8 ) = 2K ,6L (d) potassium ferrocyanide
Mn ( 25) = 2K ,8L ,15M COMEDK 2012
Hence, the charge transfer occurs from L → M. Ans. (b) : The blue colour is due to the formation of
ferriferrocyanide, Fe 4 [Fe(CN)6 ]3 .
665. Which of the following is a correct statement?
(a) Aqueous solutions of Cu+ and Zn2+ are
colourless
(b) Aqueous solutions of Cu2+ and Zn2+ are
colourless
(c) Aqueous solution of Fe3+ is green in colour
670. Nitrogen detection in an organic compound is
(d) Aqueous solution of MnO −4 is colourless carried out by Lassaigne’s test. The blue colour
AP-EAMCET- (Engg.) - 2010 formed corresponds to which of the following
formulae?
Ans. (a) : (a) Fe3[Fe(CN)6]2 (b) Fe4[Fe(CN)6]3
(It is colourless) (c) Fe4[Fe(CN)6]2 (d) Fe3[Fe(CN)6]3
Karnataka NEET 2013
Ans. (b) :
(It is colourless)
666. The orange coloured compound formed when
H2O2 is added to TiO2 solution acidified with
conc. H2SO4 is
(a) Ti2O3 (b) H2Ti2O8
(c) H2TiO3 (d) H2TiO4
AP-EAMCET- (Engg.) - 2010 671. When dil. H2SO4 is added to aqueous solution
Ans. (d) : When an acidified solution of TiO2 is reacted of Potassium chromate, yellow colour of
with H2O2, an intense yellow orange color is obtained solution turns to orange colour. It
due to the formation of H2TiO4. indicates........
TiO 2 + H 2 O 2 → H 2 TiO 4
H 2SO4 (a) Chromate ions are reduced.
(b) Chromate ions are oxidized.
Yellow-orange
(c) Mono centric complex is converted into
667. Which one of the following gives Prussian blue dicentric complex.
colour? (d) Oxygen gets removed from chromate ions.
(a) Fe 2  Fe ( CN )6  (b) Na 4  Fe ( CN )6  GUJCET-2011
(c) Fe3  Fe ( CN )6  (d) Fe 4  Fe ( CN )6  Ans. (c) : When dil. H 2 SO 4 is added to the yellow
3 3 solution of K2CrO4 yellow colour changes to orange red
AP-EAMCET (Engg.) 2013 due to the formation of K2Cr2O7
Ans. (d) : Ferric salt (FeCl3) form Prussian blue with 2K2CrO4 + H2SO4 → K2Cr2O7 + K2SO4 + H2O
potassium Ferrocyanide. The above reaction shows that the monocentric complex
4FeCl3 + 3K 4 [Fe(CN)6 ] → Fe4 [Fe(CN)6 ]3 + 12KCl (K2CrO4) is converted into dicentric (K2Cr2O7)
Prussian blue
(Ferricferrocyanide)
complex.
668. A compound absorbs light in the wavelength 672. In which of the following ions, d-d transition is
region 490–500 nm. Its complementary colour not possible?
is (a) Ti 4+ (b) Cr 3+
(a) Red (b) Blue (c) Mn 2+ (d) Cu 2+
(c) Orange (d) Blue-green GUJCET-2007
AP-EAMCET (Engg.) 2013 2+ 3+ 2+
Ans. (a) : Mn ,Cr and Cu having the electron in
Ans. (a) : When a compound absorbs a specific
wavelength from the visible light of any colour then the outer d -orbital which is available for d-d transition but
perceived colour of light is the colour complementary to in the case of Ti 4+ , there are no electron present in
it. The wavelength region 490–500 nm corresponds to outer orbital.
blue– green colour. Its complementary colour is red. 673. Assertion : When NO reacts with FeSO4, a
669. In Lassaigne’s test for nitrogen in an organic brown colored complex is formed.
compound, the blue colouration is due to the Reason: In the complex, the coordination
formation of number of Fe is 6.
Objective Chemistry Volume-II 624 YCT
(a) If both Assertion and Reason are correct and 677. An aqueous solution of CoCl2 on addition of
the Reason is the correct explanation of excess of concentrated HCl turns blue due to
Assertion. the formation of:
(b) If both Assertion and Reason are correct, but (a) [Co(H2O)4Cl2] (b) [Co(H2O)2Cl4]2-
Reason is not the correct explanation of (c) [CoCl4] 2-
(d) [Co(H2O)2Cl2]
Assertion. [AIIMS-2005]
(c) If Assertion is correct but Reason is incorrect.
Ans. (c) : CoCl2 is a weak Lewis acids, reacting with
(d) If both the Assertion and Reason are incorrect.
chloride ion to produce salt containing the tetrahedral,
[AIIMS-2009] and a deep magenta colour [CoCl4]2– ion. [CoCl4]2– is
Ans. (b) : FeSO4, when reacts with NO, a brown colour blue when anhydrous and a deep magenta colour when
complex called ferrous nitro forms sulphate. hydrated, therfore it is widely used as an indicator for
FeSO 4 + NO  H2 O
→[Fe(OH)5 NO]SO 4 water the reaction occurs as.
In this complex, the coordination number of Fe is 6. [Co(H 2 O)6 ]Cl 2 + 2HCl  →[CoCl4 ]2 −
674. In the change [Cu(H2O6)]2+ →HCl Pink solution Blue colour
+
[CuCl(H2O)5] , the colour changes from 678. Cuprous ion is colourless while cupric ion is
(a) blue to green (b) blue to pink coloured because:
(c) pink to green (d) pink to blue (a) Both have half filled p-and d-orbitals.
[AIIMS-2007] (b) Cuprous ion has incomplete d-orbitaland
Ans. (b): In the change [Cu(H2O6)]2+ → HCl cupric ion has a complete d-orbital.
+
[CuCl(H2O)5] , the colour changes from blue to pink. (c) Both have unpaired electrons in the d-orbitals.
(d) Cuprous ion has complete d-orbtal and cupric
675. Wavelength of red light is absorbed by the
complex ion has an incomplete d-orbital.
(a) [Cu(CN)4]2- (b) [Cu(NH3)4]2+ [AIIMS-2012]
(c) CuSO4 (d) Cu(CN)2 Ans. (d) : Cu+ ion has (3d10) configuration while Cu2+
[AIIMS-2007] ion has (3d9) configuration. In Cu+ ion there is no d-d
transition while in Cu2+, d-d transition is possible.
Ans. (b) : When NH4OH is added to the solution
containing Cu2+ ions, deep blue solution is obtained. 679. White silver surface turns black when O3 is
Cu 2+ + 4NH 4 OH  →[Cu(NH 3 ) 4 ]2 + H 2 O passed over it. This is due to the formaton of
(a) silver hydroxide.
Hydrated cupric compounds absorb radiations (b) freshly reduced silver which is black in
corresponding to red light and transmitted colour is
colour.
greenish blue (which is complementary to red color).
Thus, cupric compounds give greenish blue colour. (c) silver oxide.
(d) a complex compound of silver and ozone.
676. During estimation of nickel, we prepare nickel
dimethylglyoxime, a scarlet red solid. This [AIIMS-2011]
compound is  . Ans. (b) : 2Ag + O3 → Ag2O + O2
(a) ionic (b) covalent Ag2O + O3 → 2Ag + 2O2
(c) metallic (d) non-ionic complex (Black)
680. The colour of copper sulphide is
[AIIMS-2007]
Ans. (d) : (a) Blue (b) Black
(c) Red (d) Green
[AIIMS-2010]
Ans. (a) : Crystal of copper sulphide is blue in colour
because Cu2+ absorbs the wavelength of orange light
and emits blue colour.
681. A metal X on heating in nitrogen gas gives Y. Y
on treatment with H2O gives a colourless gas
which when passed through CuSO4 solution
gives a blue colour. Y is
(a) Mg(NO3)2 (b) Mg3N2
(c) NH3 (d) MgO
Assam CEE-2014
Ans. (b) : When magnesium metal is heated in nitrogen,
it forms magnesium nitride.
3Mg + N2 → Mg3 N2
Bis -(dimethylglyoximato) nickel (II) chelated complex Hydrolysis of magnesium nitride gives magnesium
(red PPt). hydroxide and ammonia.

Objective Chemistry Volume-II 625 YCT


CuSO 4 + 4NH 3 → [Cu ( NH 3 )4 ]SO 4 Ans. (b) : Since Cr3+ in the complex has unpaired
deep blue solution electrons in the d-orbital hence will be coloured.
Mg3N2  Water
→ Mg(OH)2 +NH3 Ti = [Ar] 3d2 4s2 : Ti4+ = 3d0
When ammonia gas is dissolved in the copper sulphate Cr = [Ar] 3d5 4s1; Cr3+ = 3d3
solution blue colour is obtained. Zn = [Ar] 3d10 4s2; Zn2+ = 3d10
Sc = [Ar] 3d1 4s2; Sc3+ = 3d0
682. The octahedral complex of a metal ion M3+
with four monodentate ligands L1, L2, L3 and 686. The aqueous solution containing which one of
L4 absorb wavelengths in the region of red, the following ions will be coluorless?
green, yellow and blue, respectively. The (Atomic number : Sc = 21,l Fe = 26, Ti = 22,
inreasing order of ligand strength of the four Mn = 25)
ligands is (a) Sc3+ (b) Fe2+
3+
(a) L4 < L3 < L2 < L1 (b) L1 < L3 < L2 < L4 (c) Ti (d) Mn2+
(c) L3 < L2 < L4 < L1 (d) L1 < L2 < L4 < L3 (AIPMT -2005)
JEE Main 2014 Ans. (a) : 21Sc3+ = 1s2, 2s2 2p6, 3s2 3p6
Ans. (b) : Due to absence of unpaired electrons in d-subshell, it is
Ligand : L1 L2 L3 L4 colourless. It will not undergoes d-d transition and does
not show colour.
λ abs Re d Green Yellow Blue 687. In which of the following pairs are both the
The order of Wavelength of absorption is Blue < Green ions coloured in aqueous solution?
< Yellow < Red (At. no. : Sc = 21, Ti = 22, Ni = 28, Cu = 29, Co
The order of splitting energy levels  =27)
L4 > L2 > L3 > L1 (a) Ni3+ (b) Ni2, Ti3+
3+ 3+
Increasing order of ligand strength is  (c) Sc , Ti (d) Sc3+, Co2+
L1 < L3 < L2 < L4 (AIPMT -2006)
683. The colour in the coordination compounds Can Ans. (b) :
be readily explained in terms of Sc → [Ar] 3d1 4s2
(a) spectrochemical (b) chelate effect Sc3+ → [Ar]
(c) crystal field theory (d) none of the above. Ti → [Ar] 3d2 4s2 Ti3+ –––– [Ar]3d1
J & K CET-(2014) Ni → [Ar] 3d8 4s2 Ni2+ –––– [Ar]3d8
Ans. (c) : The Crystal Field Theory (CFT) is successful Cu → [Ar] 3d10 4s1 Cu+ –––– [Ar]3d10
in explaining the formation structure colour and Co → [Ar] 3d7 4s2 Co2+ –––– [Ar]3d7
magnetic properties of coordination compounds to a Ti , Ni and Co2+ are coloured due to presence of
3+ 2+

large extent. unpaired electrons.


684. Out of TiF62 − , CoF63 − , Cu2Cl2 and NiCl 42− (Z of 688. Which of the following ions will exhibits colour
Ti = 22, Co = 27, Cu = 29, Ni = 28) the in aqueous solutions?
colourless species are (a) La3+ (Z = 57) (b) Ti3+ (Z = 22)
3+
(a) Cu2 Cl2 and NiCl42– (b) TiF22– and Cu2 Cl2 (c) Lu (Z = 71) (d) Sc3+ (Z = 21)
(c) CoF63– and NiCl42– (d) TiF62– and CoF63– (AIPMT -2010)
(AIPMT -2009) Ans. (b) : Ti3+ (Z = 22) 1s2 2s2 2p6 3s2 3p6 4s1 3d0
presence of an unpaired electron in the s-orbital of Ti3+
Ans. (b) : A species is coloured when it contain
causes it to exhibit color in aqueous solution.
unpaired d-electron which are capable of undergoing-
d-d transition on absorption of light of a particular 689.For which one of the following ions, the colour is
wavelength. not due to a d-d transition?
In TiF62– Ti4+ : 3d0 colourless (a) CrO 24 − (b) [Cu(NH3)4]2+
3+
CoF63– Co3+ : 3d6 coloured (c) [Ti(H2O)6] (d) [CoF6]3–
Cu2Cl2 Cu+ : 3d10 coloured UP CPMT-2013
NiCl42– Ni2+ : 3d8 colourless Ans. (a) d-d transition is a process of shifting of
TiF6 (3d0) and Cu2Cl2 (3d10) with empty and fully
2–
electrons between the lower energy d-orbital to a higher
filled d-orbitals appear colourless as they are not energy d-orbital by absorption of energy, So the colour
capable by undergoing d-d transition. is due to charge transfer not due to d-d transition.
685. Which of the following complex ions is Here, in CrO 24− Cr is in +6 oxidation state which has d0
expected to absorb visible light? configuration.
(a) [Ti(en)2(NH3)2]4+ (b) [Cr(NH3)6]3+ 690.Which compound is expected to be coloured ?
2+
(c) [Zn(NH3)6] (d) [Sc(H2O)3(NH3)3]3+ (a) CaCl (b) CuF2
(At. nos. Zn = 30, Sc = 21, Ti = 22, Cr = 24) (c) Ag2SO4 (d) MgF2
(AIPMT -2009) UP CPMT-2009
Objective Chemistry Volume-II 626 YCT
Ans. (b): The compound of transition metals show Ans. (d) : Transition metal ions have the property to
colours mainly due to electronic transition between absorb certain radiation from the visible region and
partially filled d-orbitals. exhibit the complementary colour.
In CuF2, Cu is present as Cu2+.
Cu2+ = [Ar] 3d9, 4s0 696. Red precipitate is obtained when ethanol
Since, in it one unpaired electron is present, it is solution of dimethylglyoxime is added to
coloured. ammoniacal Ni (II).
691.A violet colour compound is forms in detection of Which of the following statements is not true?
'S' in a compound (a) Red complex has a square planar geometry.
(a) Na4 [Fe(CN)5NOS] (b) Na3 [Fe(CN)5NOS] (b) Complex has symmetrical H-bonding.
(c) Na2 [Fe(CN)5NOS] (d) Na2 [Fe(CN)5NO] (c) Red complex has a tetrahedral geometry.
UP CPMT-2006 (d) Dimethylglyoxime functions as bidentate
Ans. (a) : When Na2S is mixed with freshly prepared ligand.
sodium nitroprusside, a deep violet colour indicates the
presence of sulphur.
Na 2S + Na 2 [Fe(NO) (CN)5 ] → Na 4 [(Fe(CN)5 )NOS]
Sodium nitroprusside Sodium thionitroprusside
violet colouredcomplex

692.Which of the following metal ions is not AIPMT -Mains 2012


coloured? +2 8
Ans. (c) : Ni = [Ar] 3d
(a) Ti3+ (b) Fe3+
2+
(c) V (d) Zn2+
UPTU/UPSEE-2006 Dmg is strong ligand so pairing possible
Ans. (d) : Zn2+ : (3d10) has no unpaired electron. So it is
colourless.
Zn2+ is a divalent metal cation, a zinc cation and a
monoatomic dication. It has a role as a human dsp2 , square planar
metabolite and cofactor.
693.Copper sulphate solution, When added to an
excess of amonium hydroxide, forms a complex
compound due to:
(a) [Cu(NH3)2]2+ (b) [Cu(NH3)4]2+
2+
(c) [Cu(NH3)6] (d) Cu2+
UPTU/UPSEE-2005
Ans. (b) :
Dimethyl glyoxime acts as bidentate ligands.
CuSO 4 + NH 4 OH → Cu [(NH 3 )4 ]SO 4 + 4H 2O
2+
CuSO 4 + NH 4 OH → Cu [(NH 3 )4 ] + 4H 2 O
9. Werner's Theory
694.Manganese salt + PbO2 + conc. HNO3 → the
solution acquires purple colour. The colour is
due to. 697. Given below are two statements.
(a) Mn(NO3)2 (b) Pb(NO3)2 Statement-I: In CuSO 4 .5H 2O,Cu − O bonds
(c) HMnO4 (d) MnO are present.
UPTU/UPSEE-2013 Statements-II: In CuSO 4 5H 2O, ligands
Ans. (c) : The manganese salt is MnO2
coordinating with Cu (II) ion are O-and S-
based legends.
2MnO 2 + 4HNO3  → 2Mn(NO3 ) 2 + 2H 2 O + O 2

In the light of the above statements, choose the
2Mn(NO3 ) 2 + 5PbO2 + 6HNO3  → 2HMnO 4 + 5Pb(NO3 ) + 2H 2 O
correct answer from the options given below:
Permangaic acid (Niolet pink)
(a) Both Statement-1 and Statement-II are correct
695.Formation of coloured ions by transition metals (b) Both Statement-1 and Statement-II are
signifies;
(a) absorption of light from UV range incorrect
(b) emission of light (c) Statement-1 is correct but statement-II is
(c) presence of unpaired electrons in s and p incorrect
orbitals (d) Statement-1 is incorrect but Statement-II is
(d) complimentary colours to the absorbed light correct.
UPTU/UPSEE-2009 JEE Main-29.06.2022, Shift-II
Objective Chemistry Volume-II 627 YCT
Ans. (c) : In CuSO4.5H2O, Cu – O bonds are present,
10. Bonding in Metal carbonyl
700.Number of bridging CO ligands in [Mn2(CO)10] is
_______.
JEE Main 26.02.2021, Shift-I
Ans. (0) : The structure of [ Mn 2 (CO)10 ] is-

Only oxygen atom forms coordinate bond with Cu2+ in


CuSO4.5H2O.
698. The primary valency and secondary valency of There is no bridging CO ligand in [ Mn 2 (CO)10 ]
central metal ion and the number of total ions
produced in aqueous solution for
K Co ( OX ) 2 ( NH 3 ) 2  complex respectively is 11. Organometallic compound
_________.
(a) 3, 4, 2 (b) 4, 4, 2 701. Number of complexes which will exhibit
synergic bonding amongst, [Cr(CO)6],
(c) 3, 6, 2 (d) 3, 6, 1
[Mn(CO)5] and [Mn2(CO)10] is ––––––
GUJCET-2019
JEE Main-28.06.2022, Shift-I
Ans. (c) : Primary valency is define as the oxidation
state of central metal atom. Ans. (3) : The metal – carbon bond is formed by the
donation of a pair of electron from a filled d – orbitats
Secondary valency is define as the coordination
of metal into the vacant antibonding π* orbital of
number
carbon monoxide. Thus carbon monoxide acts as σ
In K[Co(OX)2(NH3)2]
donor (OC → M) and a π acceptor (OC ← M).]
+1 +x +2(–2) +2(0) = 0
Hence, interactions creating a synergic effect.
1 + x –4 + 0 = 0
x = +3
702. The number of bridging CO ligand(s) and Co-
and K[Co(OX)2(NH3)2] → K+ + [Co(OX)2(NH3)2] Co bond(s) in Co2(CO)8 respectively are
Therefore, the primary valency and secondary valency (a) 2 and 0 (b) 0 and 2
of central atom and the number of total ions are 3, 6, 2
(c) 4 and 0 (d) 2 and 1
respectively.
JEE Main-24.06.2022, Shift-I
699. The primary and secondary valencies of [JEE Main-2019, 11 Jan Shift-II]
chromium, in the complex ion,
dichlorodioxalatochromium (III), are Ans. (d) : The compound is Co2(CO)8. It is a low
respectively nudearity carbonyl cluster.
(a) 3, 4 (b) 4, 3 Formula for M –M Bond: –
(c) 3, 6 (d) 6, 3 A = no. of valence e– of metal + no. of e– from
each ligand ± charge
(e) 4, 4
B = (18 × n – A) (n = no. of metal atom) total
Kerala-CEE-2008
B
Ans. (c): The formula of dichlorodioxalaochromium number of M – M bond (c) =
(III) is [Cr Cl2 (OX)2]3– 2
Primary valency of a metal Cr in the complex = ∴ For Co2 (CO)8 –
oxidation number of that metal A = 2(9) + 3 (2) = 18 + 16 = 34
In [Cr Cl2 (OX)2]3– B = 2 × 18 – 34 = 2
x + 2 (–1) +2 (–2) = –3 B 2
C = = =1
x – 2 – 4 = –3 2 2
x – 6 = –3
x = +3
Primary valency is 3.
Secondary valency of chromium = coordination number
(ligand attached to the central metal ion and oxalate ion Number of bridging CO = 2
is a bidentate ligand) = +6 Number of M – M bond = 1
Objective Chemistry Volume-II 628 YCT

You might also like